Bài giảng "Lý thuyết xác xuất và thống kê toán"

Bài giảng "Lý thuyết xác xuất và thống kê toán" giúp sinh viên củng cố kiến thức và đạt điểm cao trong bài thi kết thúc học phần.

ĐẠI HỌC DUY TÂN
KHOA KHOA HỌC TỰ NHIÊN
BỘ MÔN C SUẤT THỐNG
BÀI GIẢNG
LÝ THUYẾT C SUẤT VÀ THỐNG TOÁN
(Lưu hành nội bộ)
Đà Nẵng, năm 2018
Downloaded by Di?p DN - Chuyên Viên R&D (diepdn@bibabo.vn)
lOMoARcPSD|36212343
Li m đầu
Trong khoa hc cũng như trong đời sng hàng ngày, chúng ta rt thường gp các
hin tượng ngu nhiên (toán hc gi là biến c ngu nhiên). Đó các biến c mà ta
không th d báo mt cách chc chn rng chúng xy ra hay không xy ra.
thuyết xác sut là b môn toán hc nghiên cu nhm tìm ra các quy lut chi phi
đưa ra các phương pháp nh toán xác sut ca các hin tượng ngu nhiên. Ngày
nay thuyết xác sut đã tr thành mt ngành toán hc quan trng c v phương
din lý thuyết ng dng. Nó công c không th thiếu được mi khi ta nói đến
d báo, bo him, mi khi cn đánh giá các cơ may, các nguy cơ ri ro. Ntoán
hc Pháp Laplace thế k 19 đã tiên đoán rng: ‘Môn khoa hc này ha hn tr
thành mt trong nhng đi tượng quan trng nht ca tri thc nhân loi. Rt
nhiu nhng vn đề quan trng nht ca đi sng thc tế thuc v nhng bài
toán ca lý thuyết xác sut’.
Lí thuyết xác sut và thng kê toán hc là môn hc cơ bn được ging dy hu hết
các trưng Đại hc.
Ngoài tp bài ging này ra, ging viên khuyến khích sinh viên khi hc n hc c
sut và thng nên ít nht 1 tài liu khác để đọc thêm, bt c cun sách nào v
xác sut thng kê có trên th trường đều tt. Nó s b sung kiến thc cho bn.
Trong quá trình son i ging này, ging viên đã tham kho nhiu ý kiến ca các
đồng nghip, ging viên cũng c gng rt ln trong quá trình biên son nhưng do
hn chế v nhiu mt nên không th tránh được sai sót. Rt mong nhn đưc s phê
bình và s đóng góp ý kiến ca các đồng nghip và các bn sinh viên.
Xin chân thành cm ơn.
Biên son: Nguyn Quang Thi
Mc lc
Li m đầu ....................................................................................................... 3
Mc lc ............................................................................................................. v
Chương I.
Các khái nim cơ bn trong lí thuyết xác sut. ...................... 1
1. Nhc li mt sng thc gii tích t hp. ..........................................................1
1.1. Quy tc cng và quy tc nhân........................................................................1
1.2. Hoán v. ........................................................................................................2
1.3. Chnh hp (chnh hp không lp)..................................................................2
1.4. Chnh hp lp................................................................................................2
1.5. T hp...........................................................................................................3
1.6. Công thc nh thc Newton...........................................................................3
1.7. Bài tp...........................................................................................................3
2. Biến c và các phép toán trên biến c. .................................................................4
2.1. Phép th và biến c.......................................................................................4
2.2. Các loi biến c.............................................................................................4
2.3. Biến c bng nhau (biến c tương đương).....................................................5
2.4. Các phép toán trên biến c. ...........................................................................5
2.5. Nhóm đầy đ các biến c. .............................................................................6
2.6. Bài tp...........................................................................................................6
3. Đnh nghĩa xác sut..............................................................................................7
3.1. Các định nghĩa xác sut.................................................................................7
3.2. Các định lí v xác sut...................................................................................9
3.3. Công thc xác sut đầy đủ. Công thc Bayes. .............................................13
3.4. Bài tp.........................................................................................................15
4. Dãy phép th Bernoulli. Công thc Bernoulli. ...................................................15
4.1. Dãy phép th Bernoulli. ..............................................................................15
4.2. S có kh năng nht. ...................................................................................16
5. Bài tp chương...................................................................................................19
Đáp s và hướng dn..........................................................................................21
Chương II. Đại lượng ngu nhiên. Hàm phân phi xác sut. ..................... 25
1. Khái nim. Phân loi đại lượng ngu nhiên. .......................................................25
1.1. Đi lượng ngu nhiên ri rc.......................................................................26
1.2. Đi lượng ngu nhiên liên tc......................................................................26
1.3. Hàm phân phi ca đại lượng ngu nhiên....................................................26
2. Đi lượng ngu nhiên ri rc..............................................................................27
2.1. Bng phân phi xác sut..............................................................................27
2.2. Hàm phân phi xác sut. .............................................................................28
2.3. Phép toán đại lượng ngu nhiên...................................................................31
3. Đi lượng ngu nhiên liên tc. ...........................................................................32
4. Các đặc trưng ca đại lượng ngu nhiên.............................................................34
4.1. Kì vng. ......................................................................................................34
4.2. Phương sai. .................................................................................................36
4.3. Mt, trung v và moment trung tâm.............................................................37
5. Hàm ca mt đại lượng ngu nhiên....................................................................41
vi
5.1. Đi lượng ngu nhiên ri rc. ..................................................................... 41
6.2. Đi lượng ngu nhiên liên tc..................................................................... 42
6. Bài tp chương. ................................................................................................. 45
Đáp s và hướng dn......................................................................................... 45
Chương III. Các quy lut phân phi thường gp......................................... 47
1. Quy lut phân phi ri rc. ................................................................................ 47
1.1. Phân phi nh thc...................................................................................... 47
1.2. Phân phi siêu bi. ..................................................................................... 48
1.3. Phân phi Poisson....................................................................................... 50
2. Quy lut phân phi liên tc................................................................................ 52
2.1. Phân phi đều. ............................................................................................ 52
2.2. Phân phi mũ.............................................................................................. 52
2.3. Phân phi chun. Phân phi chun tc. ....................................................... 54
2.4. Phân phi Chi bình phương. ....................................................................... 60
2.5. Phân phi Student....................................................................................... 61
2.6. Công thc tính gn đúng............................................................................. 61
3. Đi lượng ngu nhiên nhiu chiu. .................................................................... 63
3.1. Khái nim................................................................................................... 63
3.2. Quy lut phân phi xác sut ca đại lượng ngu nhiên hai chiu................. 63
3.3. Hàm phân phi ca đại lượng ngu nhiên hai chiu. ................................... 64
4. Bài tp chương. ................................................................................................. 65
Đáp s và hướng dn......................................................................................... 67
Chương IV. Lí thuyết mu ............................................................................ 71
1. Tng th và mu................................................................................................ 71
1.1. M đầu. ...................................................................................................... 71
1.2. Mu ngu nhiên, mu c th. ...................................................................... 72
1.3. Bng phân phi tn s................................................................................. 73
1.4. Hàm phân phi mu.................................................................................... 76
2. Các tham s đc trưng ca mu ......................................................................... 76
2.1. T l mu. ................................................................................................... 76
2.2. S mt (Mode) ca mu.............................................................................. 79
2.3. S trung v (Median) ca mu..................................................................... 79
2.4. Các quy lut phân phi mu........................................................................ 81
3. Bài tp chương. ................................................................................................. 83
Chương V. Lí thuyết ước lượng .................................................................... 85
1. Ước lượng đim. ............................................................................................... 85
2. Ước lượng khong............................................................................................. 86
2.1. Ước lượng khong tin cy cho kì vng ....................................................... 87
2.2. Ước lượng khong tin cy cho phương sai.................................................. 90
2.3. Ước lượng khong tin cy cho t l. ............................................................ 92
2.4. Ước lượng kích thước mu. ........................................................................ 94
3. Bài tp chương. ................................................................................................. 95
Đáp s và hướng dn......................................................................................... 97
Chương VI. Kim định gi thiết thng kê.................................................... 99
1. Các khái nim cơ bn ........................................................................................ 99
1.1. Đt vn đề: ................................................................................................. 99
1.2. Phương pháp kim định gi thiết thng ................................................ 101
2. Kim định gi thiết v tham s.........................................................................101
2.1. Các loi kim định và phương pháp kim định gi thiết v các tham s. ...101
2.2. Kim định gi thiết v trung bình ca ĐLNN X~N(µ; σ
2
). ........................102
2.3. Kim định gi thiết v phương sai ca ĐLNN X~N(µ; σ
2
). .......................106
2.4. Kim định gi thiết v t l các phn t có tính cht nào đó trong tng th.108
2.5. Kim định gi thiết v hai kì vng ca hai ĐLNN chun độc lp...............110
2.6. Kim định gi thiết thng kê v hai t l ca hai ĐLNN. ...........................113
2.7. Kim định gi thiết thng kê v quy lut phân phi...................................115
2.8. Kim định gi thiết thng kê v tính độc lp. ............................................120
3. Bài tp chương.................................................................................................122
Các bng s................................................................................................... 125
Bng 1. Bng phân phi Poisson:.........................................................................125
Bng 2. Giá tr tích phân Laplace:........................................................................126
Bng 3. Phân v α ca phân phi Student .............................................................127
Bng 4. Phân v α ca phân phi Chi bình phương...............................................128
Chương I.
Các khái nim cơ bn trong lí thuyết xác
sut.
A. Mc tiêu
- Ôn li các kiến thc v Tp hp Gii tích t hp như: tp hp, các phép toán v tp hp,
qui tc nhân, hoán v, chnh hp, t hp . . .
- Rèn luyn cách gii mt s bài tp liên quan.
- Gii thiu các khái nim v phép th, biến c và phép toán gia các biến c.
- Nm vng khái nim v các biến c xung và các biến c độc lp.
- Xây dng mt s định nghĩa xác sut (định nghĩa c đin, định nghĩa theo hình hc định
nghĩa theo thng kê) và tìm công thc th hin định nghĩa đó.
- Nm được các công thc cng, công thc nhân xác sut.
- Hiu được c công thc tính xác sut điu kin, công thc c sut đầy đủ, công thc
Bayes.
- Gii thiu v dãy phép th Bernoulli và công thc Bernoulli.
B. Ni dung.
1. Nhc li mt s công thc gii tích t hp.
1.1. Quy tc cng và quy tc nhân.
1.1.1. Quy tc cng.
Nếu mt công vic được chia làm
k
trường hp để thc hin, trường hp
1
1
n
cách thc hin xong công vic, trường hp
2
2
n
cách thc hin xong công vic,
…, trường hp
k
k
n
cách thc hin xong công vic và không có bt kì mi cách
thc hin nào các trường hp nào li trùng vi mt cách thc hin các trường
hp khác, thì có
k
nnn
+
+
+
L
21
cách thc hin xong công vic.
1.1.2. Quy tc nhân.
Nếu mt công vic được chia làm
k
giai đon, giai đon
1
1
n
cách thc hin
xong công vic, giai đon
2
2
n
cách thc hin xong công vic, …, giai đon
k
k
n
cách thc hin xong công vic, thì
k
nnn L
32
cách thc hin xong công
vic.
Bài ging
2
1.2. Hoán v.
Mt hoán v t
n
phn t là mt b th k th t gm
n
phn t khác nhau đã
cho.
S các hoán v t
phn t kí hiu là
n
P
.
Công thc tính:
!
nP
n
=
.
Ví d 1.1.
4
sinh viên
4
cái ghế được sp xếp theo mt hàng ngang. Sp xếp mi sinh
viên ngi mt ghế. Có bao nhiu cách sp xếp khác nhau?
Rõ ràng mi kiu sp xếp là mt hoán v ca
4
phn t. S cách sp xếp ch ngi là
!4
4
=
P
.
1.3. Chnh hp (chnh hp không lp).
Mt chnh hp chp
k
(
nk
1
) t
n
phn t mt b th k th t gm
k
phn t khác nhau ly t
phn t đã cho
S các chnh hp chp
k
t
phn t kí hiu là
k
n
A
.
Công thc tính:
( ) ( )
( )
!
!
11
kn
n
knnnA
k
n
=+= K
Nhn xét.
S các chnh hp chp
n
ca
n
phn t bng s các hoán v ca
n
phn t, nghĩa
là:
n
n
n
PA =
.
Ví d 1.2.
bao nhiêu s khác nhau gm
3
ch s phân bit được thiết lp t các ch s
1
,
2
,
3
,
4
,
5
?
Gii
Mt s gm
3
ch s phân bit được thiết lp t các ch s bng
( )
60
!35
!5
3
5
=
=A
.
1.4. Chnh hp lp.
Mt chnh hp lp chp
k
(
1
k
) t
phn t mt b th k th t gm
k
phn t không nht thiết khác nhau ly t
n
phn t đã cho
S các chnh hp lp chp
k
t
phn t kí hiu là
k
n
A
.
Công thc tính:
k
k
n
nA =
.
Ví d 1.3.
Gi s
{
}
3;2;1
=
A
tp hp gm
3
phn t. Khi đó, các dãy
11
,
21
hoc
33
nhng chnh hp lp
2
t
3
phn t ca
A
. Ta có th lit kê ra đây tt c các chnh
hp lp là:
11
,
12
,
13
,
21
,
22
,
23
,
31
,
32
,
33
. Và s chnh hp đó là
93
2
2
3
==
A
.
Chương I. Các khái nim cơ bn trong lí thuyết xác sut.
3
1.5. T hp.
Mt t hp chp
k
t
n
phn t là mt tp con gm
k
phn t khác nhau đã cho.
S các t hp chp
k
t
phn t kí hiu là
k
n
C
.
Công thc tính:
(
)
(
)
( )
!!
!
!
11
knk
n
k
knnn
C
k
n
=
+
=
K
Nhn xét:
1
0
=
n
C
,
k
n
kn
n
CC
=
, vi mi
nk ;0=
.
Ví d 1.4.
bao nhiêu cách phân công
5
sinh viên đi lao động ca mt lp gm
45
sinh
viên?
Gii
Mi cách chn ngu nhiên
5
người trong
50
sinh viên là mt t hp chp
5
ca
50
.
Vy s cách phân công khác nhau
5
sinh viên trong
50
sinh viên đi lao động là
( )
2118760
!550!5
50
5
50
=
=
C
.
Ví d 1.5.
bao nhiêu cách phân công
50
sinh viên thành
3
nhóm
I
,
II
,
III
sao cho nhóm
I
đúng
30
sinh viên.
Gii
Ta thy
30
50
C
cách phân công
30
sinh viên vào nhóm
I
. S ch phân công
(
)
3050
sinh viên còn li vào nhóm
II
III
bng s các chnh hp lp chp
20
ca
2
, nghĩa bng
20
2
. Vy, s cách phân công
50
sinh viên thành
3
nhóm
I
,
II
,
III
sao cho nhóm
I
đúng
30
sinh viên là
2030
50
2×
C
1.6. Công thc nh thc Newton.
Công th
c:
( )
kkn
n
k
k
n
n
baCba
=
=+
0
Nhn xét:
a)
( )
nn
nnn
n
xCxCCx
+++=+
L
10
1
b)
( ) ( )
kkn
n
k
k
n
kn
baCba
0
1
=
=
1.7. Bài tp.
1.
Tìm
t
các ph
ươ
ng trình:
a)
45
2
=
n
C
,
b)
60
3
1
4
=
n
n
C
A
,
Bài ging
4
c)
128
nn
CC
=
2.
Trên m
t ph
ng có 20
đ
i
m (không có 3
đ
i
m này cùng n
m trên m
t
đườ
ng th
ng).
Qua m
i c
p
đ
i
m, ta v
m
t
đườ
ng th
ng. H
i có bao nhiêu
đườ
ng th
ng nh
ư
v
y.
3.
T
thành ph
A
có
3
con
đườ
ng
đ
i
đế
n thành ph
B
t
B
2
con
đườ
ng
đ
i t
i
thành ph
C
. H
i có m
y cách
đ
i t
A
đế
n
C
(ph
i qua
B
)?
4.
Trên m
t
đườ
ng tròn
12
đ
i
m. Có m
y cách v
dây cung có các mút là các
đ
i
m
đ
ã
cho. Có m
y tam giác nh
n các
đ
i
m là
đỉ
nh.
2. Biến c các phép toán trên biến c.
2.1. Phép thbiến c.
Phép th (phép th ngu nhiên) là s thc hin mt nhóm các điu kin xác định và
có th được lp li nhiu ln. Kết qu ca nó, ta không đoán trước được.
Mt kết qu ca phép th gi là mt biến c.
Ví d 2.1.
a) Để nghiên cu hin tượng ngu nhiên v s xut hin sp hay nga khi tung đồng
tin, ta tiến hành phép th: “tung mt đồng tin”. Kết qu nhn được s
S
(được
mt sp) hoc
N
(được mt nga).
S
N
là nhng biến c.
b) Chn ngu nhiên mt sinh viên trong lp, ta đưc các biến c, chng hn:
A
:
“sinh viên đó n”,
B
: “sinh viên đó nam”,
C
: “sinh viên đó sinh viên gii
Toán”.
2.2. Các loi biến c.
Biến c không th (hay biến c rng) biến c không bao gi xy ra khi phép
th thc hin. Kí hiu:
.
Biến c ngu nhiên biến c th xy ra hoc không xy ra tùy thuc vào tng
phép th.
Biến c sơ cpbiến c xy ra khi và ch khi có mt kết qu c th trong s nhng
kết qu loi tr nhau ca phép th. Kí hiu là
ω
.
Biến c chc chn biến c luôn luôn xy ra khi phép th thc hin. Kí hiu:
.
Biến c chc chn gm tt c các biến c sơ cp. Ta thường coi đó không gian
biến c sơ cp.
Ví d 2.2.
Trong d 2.1. a) Nếu đồng tin hai mt đu nga thì
S
biến c rng
N
là biến c chc chn.
Trong d 2.1. b) Nếu lp hc đó không có nam thì
A
là biến c chc chn
B
là biến c rng.
Ví d 2.3.
Gieo
1
mt ln
1
con xúc xc. Gi
i
B
biến c “Mt trên con c xc ca
i
chm”,
6;1=
i
. Khi đó
Chương I. Các khái nim cơ bn trong lí thuyết xác sut.
5
Không gian biến c sơ cp
{
}
654321
,,,,,
BBBBBB
=
. Các
1
B
,
2
B
,
K
,
6
B
là
nhng biến c sơ cp.
Chú ý:
Mi biến c sơ cp đều biến c ngu nhiên. Ngược li, biến c ngu nhiên nói
chung không là biến c sơ cp.
2.3. Biến c bng nhau (biến c tương đương).
Biến c
A
gi là kéo theo biến c
B
nếu
A
xy ra thì
B
xy ra. Kí hiu:
B
A
.
Nếu đồng thi
B
A
A
B
thì các biến c
A
B
gi bng nhau.
hiu:
B
A
=
.
2.4. Các phép toán trên biến c.
Cho hai biến c
A
B
. Khi đó, ta gi:
Tích ca
A
B
, hay
A
nhân
B
, biến c xy ra khi
A
B
đồng thi xy ra.
Kí hiu:
BA
.
( hoc
AB
hoc
B
A
).
Tng ca
A
B
, hay
A
cng
B
, biến c xy ra khi
A
xy ra hoc
B
hoc
BA
.
xy ra. Kí hiu:
B
A
+
(hoc
B
A
).
Cho mt biến c
A
. Khi đó, ta gi biến c đi lp ca biến c
A
biến c xy ra
nếu
A
không xy ra và không xy ra nếu
A
xy ra. Kí hiu:
A
.
Tính cht.
Vi các biến c
A
,
B
,
C
tùy ý, ta có các tính cht sau:
1)
A
B
B
A
+
=
+
,
BA
AB
=
.
2)
(
)
(
)
CBACBA ++=++ ,
(
)
(
)
BCACAB = .
3)
(
)
ACABCBA +=+ ,
(
)
(
)
(
)
CABABCA ++=+ .
4) N
ế
u
B
A
thì
B
B
A
=
+
,
A
AB
=
.
5)
A
A
=
.
6)
=
+
A
A
,
=
AA
.
7)
B
A
B
A
=
+
,
B
A
AB
+
=
(quy t
c
đố
i ng
u)
8) Vi các biến c
1
A
,
2
A
,
K
,
n
A
ta có
n
AAA
+
+
+
L
21
là biến c xy ra khi có ít nht mt biến c
i
A
xy ra (
ni
;1=
).
n
AAA ...
21
L
là biến c xy ra khi tt c các
i
A
đều xy ra (
ni
;1=
).
Ví d 2.4.
Bn
3
mũi tên vào mt tm bia. Gi
i
A
biến c “mũi tên th
i
trúng đích”
(
3;1=i
). Hãy biu din qua
1
A
,
2
A
,
3
A
các biến c:
A
: C
3
mũi tên đều trúng đích.
B
: Có đúng
1
mũi tên trúng đích.
C
: Có ít nht
1
mũi tên trúng đích.
D
: Không có mũi tên nào trúng đích.
Bài ging
6
Gii
Ta có:
321
AAAA
=
,
3
213
2
132
1
AAAAAAAAAB ++=
,
321
AAAC
+
+
=
,
321
AAAD =
.
2.5. Nhóm đầy đủ các biến c.
Hai biến c
A
B
gi là xung khc nếu
=
AB
.
Các biến c
1
A
,
2
A
,
K
,
n
A
gi là đôi mt xung khc nếu hai biến c khác nhau bt
kì trong đó đều xung khc, tc là
=
ji
AA
vi mi
j
i
.
Các biến c
1
A
,
2
A
,
K
,
n
A
gi mt nhóm đầy đủ các biến c nếu chúng đôi mt
xung khc ít nht mt trong chúng xy ra, tc
=
+
+
+
n
AAA
L
21
,
=
ji
AA
vi mi
j
i
, và
(
)
0
>
i
AP
vi mi
ni
;1
=
.
Ví d 2.5.
a) Gieo mt ln mt con xúc xc:
Đặt
i
B
biến c “mt trên ca con xúc xc
i
chm”,
6;1
=i
. Dãy
1
B
,
2
B
,
3
B
,
4
B
,
5
B
,
6
B
lp thành h đầy đủ các biến c. Vì nó có tính cht:
=
+
+
+
621
BBB
K
,
=
ji
BB
v
i m
i
j
i
, và
(
)
0
>
i
BP
, v
i m
i 6;1
=i
.
b) Gieo mt đồng tin mt ln:
Đặt
A
là biến c “xut hin mt sp”, khi đó
A
biến c “xut hin mt nga”. Ta
thy rng dãy
A
,
A
lp thành mt h đầy đủ
=AA
=
+
A
A
.
Chú ý.
Hai biến c đi lp nhau thì xung khc vi nhau. Điu ngược li nói chung là không
đúng.
2.6. Bài tp.
1.
Xét phép th
: gieo con xúc x
c
2
l
n. Mô t
không gian bi
ế
n c
s
ơ
c
p
ng v
i phép
th
trên. Tìm các bi
ế
n c
:
A
“t
ng s
ch
m chia h
ế
t cho
3
”;
B
“tr
tuy
t
đố
i c
a hi
u
s
ch
m là s
ch
n”.
2.
Ki
m tra theo th
t
m
t lô hàng g
m
N
s
n ph
m. Các s
n ph
m
đề
u thu
c m
t trong
2
lo
i: t
t ho
c x
u. Kí hi
u
k
A
(
Nk
;1
=
) là bi
ế
n c
ch
s
n ph
m ki
m tra th
k
thu
c lo
i x
u. Vi
ế
t b
ng kí hi
u các bi
ế
n c
d
ướ
i
đ
ây:
a)
C
N
s
n ph
m
đề
u x
u.
b)
Có ít nh
t
1
s
n ph
m x
u.
c)
m
s
n ph
m ki
m tra
đầ
u là t
t, các s
n ph
m còn l
i là x
u.
d)
Các s
n ph
m ki
m tra theo th
t
ch
n là x
u, còn các s
n ph
m ki
m tra theo th
t
l
là t
t.
e)
Không gian bi
ế
n c
s
ơ
c
p có bao nhiêu ph
n t
.
3.
B
n 3 viên
đạ
n vào m
t t
m bia. G
i
i
A
là bi
ế
n c
: “viên
đạ
n th
i
trúng bia”, 3;1
=i
.
B
là bi
ế
n c
: “có
đ
úng
1
viên
đạ
n trúng m
t t
m bia”,
C
là bi
ế
n c
“có ít nh
t
2
viên
đạ
n trúng bia” và
D
bi
ế
n c
“c
3
viên
đạ
n không trúng bia”. Hãy bi
u di
n các bi
ế
n
c
B
,
C
,
D
,
CB
+
qua các
i
A
i
A
.
Chương I. Các khái nim cơ bn trong lí thuyết xác sut.
7
4.
B
n không h
n ch
ế
vào m
t m
c tiêu cho
đế
n khi có viên
đạ
n trúng m
c tiêu thì thôi
b
n. Gi
s
m
i l
n b
n ch
2
kh
n
ă
ng trúng bia (g
i là bi
ế
n c
A
) ho
c ch
ch bia
(bi
ế
n c
A
).
a)
Hãy mô t
không gian bi
ế
n c
s
ơ
c
p.
b)
Hãy nêu m
t h
đầ
y
đủ
các bi
ế
n c
.
3. Định nghĩa xác sut.
3.1. Các định nghĩa xác sut.
3.1.1. Định nghĩa c đin.
Ta gi các trường hp đng kh năng các trường hp kh năng xy ra ca
chúng là ngang bng nhau.
Ta gi mt trường hp thun li cho biến c
A
nếu trưng hp này xy ra thì
A
xy ra.
Gi s tt c
(
)
n
trường hp đồng kh năng, trong s đó
(
)
An
trưng hp
thun li cho biến c
A
.
Khi đó, ta gi xác sut ca biến c
A
( )
(
)
( )
=
n
An
AP
.
Như vy, xác sut ca biến c là t s v kh năng biến c đó xut hin.
Ví d 3.1.
Gieo mt ln con xúc xc cân đối và đng cht. Tìm xác sut để
a) Mt trên ca nó có
1
chm.
b) Mt trên ca nó có s chm là s chn.
Gii
a) Đặt
i
B
là biến c “mt trên ca con xúc xc có
i
chm”,
6;1
=i
.
Đặt
A
biến c mt trên ca con xúc xc
1
chm. Do con xúc xc cân đối
đồng cht nên kh năng xut hin các mt
1
B
,
2
B
,
3
B
,
4
B
,
5
B
,
6
B
là như nhau
(
)
6
=
n
s kh năng thun li cho
A
là
1
. Vy xác sut cúa biến c
A
là
( )
6
1
=AP
.
a) Đt
B
biến c mt trên ca con xúc xc s chm s chn”. D thy
{
}
321
;; BBBB
=
và s kh năng thun li cho
B
3
. Vy
( )
2
1
6
3
==BP
.
Ví d 3.2.
Mt lp hc gm
N
sinh viên trong đó
M
nam
MN
n. Chn ngu nhiên
s
sinh viên. Tìm xác sut để trong
s
sinh viên đưc chn thì đúng
k
sinh viên
nam
Gii
Bài ging
8
S cách chn
s
sinh viên trong
N
sinh viên là
s
N
C
.
S cách chn được
k
sinh viên nam trong
M
sinh viên là
k
M
C
.
S cách chn được
s
sinh viên trong lp trong đó
k
sinh viên nam và
ks
sinh
viên n
ksk
M
MN
CC
×
.
Vy, xác sut cn tìm là
( )
s
N
ks
MN
k
M
C
CC
AP
×
=
.
3.1.2. Đnh nghĩa hình hc.
Gi s tp hp (vô hn) các trường hp đồng kh năng ca mt phép th có th biu
th bi mt min
(chng hn đon thng, mt phng, không gian ba chiu v.v…)
còn tp hp các kết qu thun li cho cho biến c
A
mt min con
S
ca
. Ta
ly ngu nhiên mt đim trong min
. Xác sut ca biến c
A
đưc xác đnh như
sau:
(
)
=
AP
(độ đo ca
S
)/(độ đo ca
).
Nếu min
đường cong hay đon thng thì “độ đo” ca
độ dài ca nó.
Nếu min
hình phng hay mt cong thì “độ đo” ca
là din tích ca nó.
Ví d 3.3.
Đường dây đin thoi ngm ni mt tng đài đến mt trm dài
1
km
. Tính xác sut
để dây đứt ti nơi cách tng đài không quá
100
m
biết rng dây đin thoi đồng
cht.
Gii.
Do dây đin thoi đồng cht nên kh năng b đứt ti mt đim bt như
nhau. Khi đó, tp hp các trường hp đồng kh năng th biu th bng đon
thng ni tng đài vi trm. Các trường hp thun li cho biến c
A
“dây b đứt ti
nơi ch tng đài không quá
100
m
đon thng độ dài
100
m
. Khi đó
( )
10
1
1000
100
==AP
.
Ví d 3.4.
Hai người bn hn gp nhau ti mt địa đim theo quy ước như sau:
Mi người độc lp đến đim hn trong khong t
7
gi đến
8
gi.
Mi người đến, nếu không gp người kia thì đợi
30
phút hoc đến
8
gi không đợi
na.
Tính xác sut hai người gp nhau, nếu biết rng mi người th đến ch hn trong
khong thi gian quy định mt cách ngu nhiên không tùy thuc vào người kia
đến lúc nào.
Gii
Chương I. Các khái nim cơ bn trong lí thuyết xác sut.
9
Gi
x
+
7
,
y
+
7
thi đim hai người này đến đim hn,
1,0
yx
. Các trưng
hp đồng kh năng tương ng vi các đim
(
)
yx;
to thành hình vuông cnh
bng
1
, có din tích (độ đo) bng
1
.
Các trường hp thun li cho biến c
A
(hai người gp nhau) tương ng vi các
đim
(
)
yx;
tha mãn
2
1
yx
.
Da vào hình v, ta có
Din tích hình
4
3
2
1
1
2
=
. T đó, ta có
( )
4
3
1
4
3
==AP
Ví d 3.5.
Tìm xác sut để đim
M
rơi vào hình tròn ni tiếp hình vuông có cnh
2
cm.
Gii
Hình tròn ni tiếp hình vuông có cnh
a2
đưng kính
a2
.
Vy din tích hình tròn đó là
22
aR
ππ
=
và din tích hình vuông là
2
422 aaaS =×=
.
Khi đó, xác sut phi tìm là
( )
4
4
2
2
ππ
==
a
a
AP
.
3.1.3. Định nghĩa thng kê.
Gi s trong
phép th vi điu kin như nhau, biến c
A
xut hin
k
ln. Khi đó
ta gi
( )
n
k
Af
n
=
tn sut xut hin biến c
A
trong
phép th. Khi
tăng lên
rt ln, ta thy rng
(
)
Af
n
dao động quanh mt s
p
c định và tiến dn v s
p
đó. Ta gi xác sut ca biến c
A
(
)
(
)
AfpAP
n
n
+∞
=
=
lim
.
3.2. Các định lí v xác sut.
3.2.1. Định lí cng xác sut.
Định lí 3.1.
Nếu
1
A
,
2
A
,
K
,
n
A
các biến c đôi mt xung khc thì
(
)
(
)
(
)
nn
APAPAAAP
+
+
=
+
+
+
LL
121
.
Định lí 3.2.
Vi các biến cy ý
A
B
, ta có
(
)
(
)
(
)
(
)
ABPBPAPBAP
+
=
+
.
Chng minh
Do
A
BA
nên
A
BA
A
=
+
. T đó
(
)
ABAABBAAAABABA +=++=++=+
.
Do
A
A
B
xung khc nên
(
)
(
)
(
)
ABPAPBAP +=+
.
Tương t, ta có:
Bài ging
10
A
B
BA
B
+
=
nên
(
)
(
)
(
)
ABPBAPBP +=
hay
(
)
(
)
(
)
ABPBPABP =
.
T các điu kin trên, ta suy ra:
(
)
(
)
(
)
(
)
ABPBPAPBAP
+
=
+
.
Áp dng Đnh lí 3.2. và áp dng nguyên lí quy np, ta có:
Định lí 3.3.
( ) ( )
( ) ( )
( ) ( )
n
n
kji
kji
ji
ji
i
n
AAAPAAAPAAPAPAAAP
KLL
21
1
1
121
1
<<<=
++=+++
Ví d 3.6.
Trong s
50
sinh viên ca lp
20
sinh viên gii Toán,
25
sinh viên gii Anh
10
hc sinh gii c Toán và Anh. Chn ngu nhiên mt sinh viên ca lp. Tính xác
sut để sinh viên này gii Toán hoc gii Anh.
Gii
Gi
A
B
ln lượt là biến c sinh viên được chn gii Toán và gii Anh.
Khi đó
B
A
+
biến c sinh viên được chn gii Toán hoc gii Anh. Áp dng
Định lí 3.2., ta có:
( ) ( ) ( ) ( )
10
7
50
10
50
25
50
20
=+=+=+ ABPBPAPBAP
Ví d 3.7.
Xếp ngu nhiên
n
bc thư vào
n
phong bì đã ghi sn địa ch (mi phong bì chì có
1
thư). Tìm xác sut để có ít nht
1
thư đến đúng địa ch.
Gii
Đặt
i
A
biến c “bc thư th
i
đến đúng người nhn”,
ni
;1
=
. Gi
A
biến c
“ít nht
1
thư đến đúng địa ch”. Khi đó, ta có:
n
AAAA
+
+
+
=
L
21
. Theo Định
3.3. ta suy ra
( ) ( )
( )
( ) ( )
( )
( )
<<<
<=
=
++=+++
niii
iii
k
n
n
ji
ji
i
n
k
k
AAAP
AAAPAAPAPAAAP
L
L
KLL
21
21
1
1
21
1
1
121
1
1
D thy
(
)
(
)
!
!
21
n
kn
AAAP
k
iii
=
L
các bc thư
1
i
,
2
i
,
K
,
k
i
đến đúng địa ch, còn
li
kn
khác có th đến đúng người nhn hoc không.
Ta có
( )
(
)
!
1
!
!
21
21
1
kn
kn
CAAAP
k
n
niii
iii
k
k
=
=
<<< L
L
.
Vy
( ) ( )
=
=
n
k
k
k
AP
1
1
!
1
1
.
Chương I. Các khái nim cơ bn trong lí thuyết xác sut.
11
3.2.2. Xác sut có điu kin.
Định nghĩa.
Cho hai biến c
A
B
. Ta gi xác sut ca biến c
A
khi biến c
B
đã xy ra
(
(
)
0
>
BP
) là xác sut ca
A
đối vi điu kin
B
. Kí hiu:
(
)
BAP /
.
Ngưi ta chng minh được công thc
( )
(
)
( )
BP
ABP
BAP =/
, trong đó
(
)
0
>
BP
.
Chng minh
Ta chng minh cho trường hp phép th
trường hp cùng kh năng. Gi s
trong
n
trường hp này
m
trưng hp thun li cho
B
k
trường hp thun
li cho
AB
. Vì
B
đã xy ra nên s trường hp cùng kh năng lúc y là
m
, s
trường hp thun li cho
A
trong đó chính s trường hp thun li cho
AB
, tc
k
. Vì vy
( )
( )
( )
BP
ABP
n
m
n
k
BAP ==/
.
Chú ý.
Định nghĩa trên mang tính cht thun túy toán hc. Tuy nhiên trong trong thc tế, ta
có th tính xác sut bng trc giác.
3.2.3. Định lí nhân xác sut. Tính độc lp ca các biến c.
Định lí 3.4.
Nếu các biến c tùy ý
A
B
cùng liên kết vi mt phép th (
(
)
AP
,
(
)
0
>
BP
), thì
ta có:
(
)
(
)
(
)
(
)
(
)
BAPBPABPAPABP //
=
=
.
Áp dng Định lí 3.4. và áp dng nguyên lí quy np, ta có:
(
)
(
)
(
)
(
)
12112121
//.
=
nnn
AAAAPAAPAPAAAP
KKK
Bây gi ta đưa điu kin để xác sut ca tích bng tích các xác sut.
Hai biến c
A
B
được gi độc lp nếu xác sut ca biến c y không ph
thuc vào s xy ra hay không xy ra ca biến c kia, tc là:
(
)
(
)
APBAP
=
/
hoc
(
)
(
)
BPABP
=
/
.
Chú ý rng ch cn tha mãn mt trong hai điu kin này thì s tha mãn điu kin
kia.
Các biến c
1
A
,
2
A
,
K
,
n
A
gi độc lp toàn th nếu xác sut ca mi biến c
trong đó không ph thuc vào s xy ra hay không xy ra ca mt t hp bt kì ca
các biến c khác.
Định lí 3.5.
a) Nếu
A
B
độc lp thì
(
)
(
)
(
)
BPAPABP .
=
.
b) Nếu các biến c
1
A
,
2
A
,
K
,
n
A
độc lp toàn th thì
(
)
(
)
(
)
(
)
nn
APAPAPAAAP
KK
2121
.
=
.
Bài ging
12
Tính cht
Nếu
A
B
là hai biến c độc lp thì các cp biến c
a)
A
B
độc lp.
b)
A
B
độc lp.
c)
A
B
độc lp.
Ví d 3.8.
Cho
3
hp bi, mi hp
10
bi. Trong hp th
i
i
bi đỏ
i
10
bi xanh
(
3;1=i
). Ly ngu nhiên mi hp ra
1
bi.
a) Tính xác sut c
3
bi ly ra đều đ.
b) Tính xác sut trong
3
bi ly ra có
2
đỏ và
1
xanh.
c) Biết trong
3
bi ly ra
2
đỏ
1
xanh. Tính xác sut bi ly ra t hp th
2
màu xanh.
Gii
Gi
i
A
biến c “ly ra t hp th
i
bi đỏ(
3;1=i
). D thy
1
A
,
2
A
,
3
A
độc lp
toàn th và
( )
10
1
1
=AP
,
( )
10
2
1
=AP
,
( )
10
3
1
=AP
.
a) Biến c “c
3
bi ly ra đều đ” là
321
AAA
.
Ta có
( ) ( ) ( ) ( )
1000
6
10
3
.
10
2
.
10
1
..
321321
=== APAPAPAAAP
.
b) Biến c “trong
3
bi ly ra có
2
đỏ
1
xanh”
321321321
AAAAAAAAAB
++=
.
Do
B
tng ca các biến c đôi mt xung khc nên
(
)
(
)
(
)
(
)
( ) ( )
( )
( )
( )
( ) ( ) ( )
( )
.
1000
92
10
3
.
10
2
.
10
9
10
3
.
10
8
.
10
1
10
7
.
10
2
.
10
1
321321321
321321321
=++=
++=
++=
APAPAPAPAPAPAPAPAP
AAAPAAAPAAAPBP
c) Ta có:
(
)
32132132132122
AAAAAAAAAAAAABA
=++=
. Khi đó xác sut bi ly ra t
hp th
2
có màu xanh là
( )
( )
( )
( )
( )
23
6
1000
92
1000
24
/
3212
2
====
BP
AAAP
BP
BAP
BAP
.
Ví d 3.9.
Mt hàng gm
10
sn phm, trong đó
3
phế phm. Ly ngu nhiên tng sn
phm ra kim tra đến khi gp đ
3
phế phm thì dng li.
a) Tính xác sut dng li ln kim tra th
3
.
b) Tính xác sut dng li ln kim tra th
4
.
Chương I. Các khái nim cơ bn trong lí thuyết xác sut.
13
c) Biết rng đã dng li ln kim tra th
4
, tính xác sut ln kim tra th
2
gp
phế phm.
Gii
Gi
i
A
là biến c “kim tra ln th
i
gp phế phm” (
10;1=i
).
a) Biến c “dng li ln kim tra th
3
321
AAA
. Ta
( ) ( ) ( ) ( )
120
1
8
1
.
9
2
.
10
3
/./.
213121321
=== AAAPAAPAPAAAP
b) Ta biến c “dng li ln kim tra th
4
là
432143214321
AAAAAAAAAAAAF
++=
.
Ta có
(
)
(
)
(
)
(
)
(
)
120
1
7
1
.
8
2
.
9
3
.
10
7
/././.
32142131214321
=== AAAAPAAAPAAPAPAAAAP
.
Tương t, ta có:
(
)
(
)
120
1
43214321
== AAAAPAAAAP
.
Do
F
tng ca các biến c đôi mt xung khc nhau nên
( )
40
1
120
1
.3 ==FP
.
c) Ta cn tính
(
)
FAP
/
2
. Tht vy, ta có
( )
( )
( )
( ) ( )
( )
3
2
120
1
.3
120
1
.2
/
432143212
2
==
+
==
FP
AAAAPAAAAP
FP
FAP
FAP
.
3.3. Công thc xác sut đầy đủ. Công thc Bayes.
Cho
1
A
,
2
A
,
K
,
n
A
là mt nhóm đầy đủ các biến c liên kết vi mt phép th.
F
biến c bt liên kết vi phép th đó, hay
F
xy ra khi mt trong các biến c
1
A
,
2
A
,
K
,
n
A
xy ra. Khi đó, ta Định lí sau đây
Định lí 3.6.
a) Vi mi biến c
F
, ta luôn
(
)
(
)
(
)
(
)
(
)
nn
AFPAPAFPAPFP /./.
11
+
+
=
L
.
Công thc này được gi là công thc xác sut đầy đủ.
c) Vi mi
k
(
nk ;1=
), ta có:
( )
(
)
(
)
( )
(
)
(
)
( ) ( )
=
==
n
i
ii
kkkk
k
AFPAP
AFPAP
FP
AFPAP
FAP
1
/.
/./.
/
.
Công thc này được gi là công thc Bayes.
Chng minh
a) Ta
(
)
nn
FAFAFAAAAFFF
+
+
+
=
+
+
+
=
=
LL
2121
.
.
Do
1
FA
,
2
FA
,
K
,
n
FA
đôi mt xung khc nên
Bài ging
14
(
)
(
)
(
)
(
)
( ) ( ) ( ) ( )
nn
n
AFPAPAFPAP
FAPFAPFAPFP
/./.
11
21
++=
+
+
+
=
L
L
b) D thy rng:
( )
(
)
( )
(
)
(
)
( )
FP
AFPAP
FP
FAP
FAP
kkk
k
/.
/ ==
ta suy ra điu phi chng
minh.
Ví d 3.10.
20
kin hàng, mi kin hàng
10
sn phm. Trong s đố
8
kin hàng loi
I
, mi kin hàng
1
phế phm;
7
kin loi
II
, mi kin
3
phế phm;
5
kin
loi
III
, mi kin
5
phế phm. Ly ngu nhiên mt kin, ri t kin đó ly ra
ngu nhiên mt sn phm.
a) Tính xác sut sn phm ly ra là phế phm.
b) Biết sn phm được ly là phế phm. Tính xác sut kin được ly là loi
II
.
Gii
Gi
i
A
biến c “ly được sn phm loi
i
”,
IIIIIIi ,,
=
. Khi đó,
1
A
,
2
A
,
3
A
nhóm đầy đ các biến c. Gi
F
biến c sn phm được ly t kin phế
phm”.
a) Theo công thc xác sut đầy đủ, ta có
(
)
(
)
(
)
(
)
(
)
(
)
(
)
27,0
10
5
.
20
5
10
3
.
20
7
10
1
.
20
8
/././.
332211
=++=
+
+
=
AFPAPAFPAPAFPAPFP
b) Theo công thc Bayes, ta có
( )
( )
( )
( ) ( )
( )
18
7
200
54
200
21
/.
/
22
2
====
FP
AFPAP
FP
FAP
FAP
k
.
Ví d 3.11.
5
bình đựng bi, trong đó
2
bình loi 1: mi bình đựng
3
bi đen và
4
bi đỏ,
mt bình loi 2: mi bình đựng
3
bi đen
2
bi đỏ. Bình loi 3: mi nh đng
4
bi đen
3
bi đỏ. Chn ngu nhiên mt bình và t bình đó, chn ngu nhiên mt bi.
a) Tính xác sut để bi ly ra là bi đen.
b) Biết bi ly ra là bi đen. Tính xác sut để bình ly ra là bình loi 3.
Gii
a) Gi
i
A
biến c “bình chn ra bình loi
i
”,
F
biến c bi chn ra bi
đen”.
Ta có
1
A
,
2
A
3
A
nhóm đầy đủ các biến c. Khi đó
Chương I. Các khái nim cơ bn trong lí thuyết xác sut.
15
(
)
(
)
(
)
(
)
(
)
(
)
(
)
52,0
7
4
.
5
2
5
3
.
5
1
7
3
.
5
2
///
332211
=++=
+
+
=
AFPAPAFPAPAFPAPFP
b) Đây là xác sut điu kin
( )
( ) ( )
( )
35
16
52,0
7
4
.
5
2
/.
/
33
3
===
FP
AFPAP
FAP
.
3.4. Bài tp.
1.
M
t lô hàng g
m có
150
s
n ph
m có ch
a
%6
ph
ế
ph
m. Ng
ườ
i ta dùng ph
ươ
ng pháp
ch
n m
u
để
ki
m tra lô hang và quy
ướ
c r
ng: Ki
m tra l
n l
ượ
t
6
s
n ph
m, n
ế
u có ít
nh
t
1
trong
6
s
n ph
m
đ
ó là ph
ế
ph
m thì lo
i lô hàng. Tìm xác su
t
để
ch
p nh
n lô
hàng.
2.
B
n liên ti
ế
p vào m
t m
c tiêu cho
đế
n khi nào có
1
viên
đạ
n
đầ
u tiên trúng m
c tiêu
thì ng
ng b
n. Tìm xác su
t sao cho ph
i b
n
đế
n viên
đạ
n th
6
bi
ế
t r
ng xác su
t
trúng
đ
ích c
a m
i viên
đạ
n là
2,0
và các l
n b
n là
độ
c l
p.
4. Dãy phép th Bernoulli. Công thc Bernoulli.
4.1. Dãy phép th Bernoulli.
Mt dãy
n
phép th gi mt dãy
n
phép th Bernoulli nếu tha mãn hai điu
kin sau đây:
- Dãy
n
phép th đó là độc lp vi nhau.
- Trong mi phép th xác sut ca biến c
A
ta quan tâm xác sut
(
)
pAP
=
không đổi.
Xác sut
p
gi là xác sut thành công, s ln
A
xut hin trong
phép th gi
s ln thành công trong dãy
n
phép th Bernoulli.
Kí hiu:
(
)
(
)
pkPkP
nn
,
=
xác sut để
k
ln thành công.
Định lí 4.1.
(
)
knkk
nn
qpCpkP
=,
,
nk ,1=
,
pq
=
1
.
Chng minh
hiu
i
A
biến c “phép th th
i
thành công”,
ni ;1=
. Gi
F
biến c “có
k
ln thành công” t
F
tng ca
k
n
C
biến c đôi mt xung khc dng
nkk
iiiii
AAAAA KK
121 +
trong đó
{
}
{
}
niii
n
;;2;1;;;
21
KK
=
.
Do tính độc lp nên ta có:
(
)
(
)
(
)
(
)
(
)
(
)
knk
iiiiiiiiii
qpAPAPAPAPAPAAAAAP
nkknkk
==
++
KKKK
121121
T đó, ta suy ra:
(
)
knkk
nn
qpCpkP
=,
(đpcm)
Ví d 4.1.
Mt lô hàng trong kho có
%20
phế phm.
Bài ging
16
a) Ly ngu nhiên
5
sn phm. Tính xác sut trong
5
sn phm này.
i) Có
2
phế phm.
ii) Có ít nht
1
phế phm.
b) Cn ly ít nht bao nhiêu sn phm để xác sut có ít nht mt phế phm không
nh hơn
99,0
.
Gii
a) S phế phm trong
5
sn phm ly ra s ln thành công trong y
5
phép th
Bernoulli vi xác sut thành công là
2,0
.
i) Ta có
(
)
(
)
(
)
2048.08,02,02,0;2
32
2
55
== CP
.
ii) Ta có
( ) ( ) ( ) ( )
67232,08,02,012.0;012,0;
50
0
55
5
1
5
====
=
CPkPP
k
.
b) Gi
n
s sn phm cn ly ra. Khi đó, xác sut ít nht mt phế phm là
( ) ( ) ( )
n
n
n
k
n
PkPP 8,012,0;012,0;
1
===
=
.
Ta cn tìm
n
nh nht sao cho
(
)
99,08,01
n
hay
64,20
8,0ln
01,0ln
=n
.
Vy, ít nht phi ly ra
21
=
n
sn phm.
4.2. S có kh năng nht.
Trong dãy
n
phép th Bernoulli, s
m
xác sut
(
)
mP
ln nht được gi s
kh năng nht.
Định lí 4.2.
S kh năng nht bng
q
np
nếu
q
np
nguyên; bng
[
]
qnp
hoc bng
[
]
1
+
qnp
nếu
q
np
không nguyên.
Chng minh
Ta có
(
)
knkk
nn
qpCpkP
=,
,
(
)
111
,1
++
=+
knkk
nn
qpCpkP
.
Khi đó
(
)
( )
(
)
( )
qk
pkn
qpC
qpC
pkP
pkP
knkk
n
knkk
n
n
n
1,
,1
111
+
==
+
++
.
Ta xét nhn xét sau:
(
)
( )
1
1
+
qk
pkn
hay
(
)
(
)
qkpkn 1
+
hay
qnpk
.
(
)
( )
1
1
<
+
qk
pkn
hay
(
)
(
)
qkpkn 1
+
<
hay
qnpk
>
.
Khi đó, ta suy ra:
Xác sut
(
)
pkP
n
,
tăng khi
k
tăng t
0
đến
q
np
giàm khi
k
tiếp tc tăng t
q
np
đến
n
. Vì
k
nhn giá tr nguyên nên ta có kết lun sau:
Chương I. Các khái nim cơ bn trong lí thuyết xác sut.
17
- Nếu
q
np
nguyên thì xác sut
(
)
pkP
n
,
đạt giá tr ln nht ti hai giá tr ca
k
qnpk
=
0
1
1
+
=
qnpk
(chú ý rng
(
)
(
)
pkPpkP
nn
,,
10
=
).
- Nếu
q
np
không nguyên thì xác sut
(
)
pkP
n
,
đạt giá tr ln nht ti mt giá tr
ca
k
[
]
1
0
+
=
qnpk
, trong đó
[
]
qnp
kí hiu phn nguyên ca
q
np
.
Ví d 4.2.
Gi s t l ngưi dân tham gia giao thông thành ph M hiu biết v lut giao
thông
%80
. Gi s, ta chn ngu nhiên
20
ngưi tham gia giao thông trên
đường. Hãy tính xác sut trong các trường hp sau:
a) Có
15
người hiu biết lut giao thông.
b) Có
9
người không hiu biết v lut giao thông.
c) S người không hiu biết v lut giao thông có kh năng nht.
Gii
Vic chn ngu nhiên
20
ngưi là dãy phép th Bernoulli, vi
H
biến c “người
được chn hiu biết lut giao thông” và
(
)
8,0%80
=
=
Hp
a) Gi
A
là biến c “có
15
người hiu biết lut giao thông”. Khi đó, ta có:
(
)
(
)
(
)
(
)
515
15
2020
2,0.8,08,0;15 CPAP ==
.
b) Gi
B
biến c “có
9
người không hiu biết lut giao thông”. Khi đó, ta có:
(
)
(
)
(
)
(
)
(
)
911
11
202020
2,0.8,02,0;98,0;920 CPPBP ===
.
c) Áp dng Định lí 3.8, ta có:
(
)
(
)
(
)
(
)
2,32,012,0.201.20 === HpHpqnp
không nguyên
Vy, s người được chn không hiu biết lut giao thông
[
]
41
0
=
+
=
qnpk
.
5. Bài tp chương.
1.
sinh viên. G
i
k
A là bi
ế
n c
sinh viên th
k
là nam. Hãy vi
ế
t b
ng kí hi
u các
bi
ế
n c
sau:
a)
T
t c
sinh viên là nam.
b)
Có ít nh
t
1
m
t sinh viên n
.
c)
đ
úng
1
m
t sinh viên n
.
d)
đ
úng
2
sinh viên là n
.
2.
Ch
n ng
u nhiên
1
ng nhân trong s
các công nhân có m
t
xí nghi
p. G
i
A
là bi
ế
n
c
x
y ra khi ng
ườ
i công nhân
đượ
c ch
n là nam và
B
bi
ế
n c
ng
ườ
i công nhân
đượ
c
ch
n
khu t
p th
;
C
là bi
ế
n c
ng
ườ
i công nhân
đượ
c không hút thu
c là.
a)
Hãy mô t
bi
ế
n c
CAB
.
b)
V
i
đ
i
u ki
n nào ta có
ACBA =
.
c)
Khi nào thì ta có AC = .
3.
Ch
ng minh r
ng:
a)
rn
n
r
n
CC
=
,
b)
r
n
r
n
r
n
CCC +=
+
1
1
,
c)
kr
m
n
k
k
mn
r
n
CCC
=
=
0
,
d)
( )
n
n
n
k
k
n
CC
2
0
2
2
=
=
,
e)
( )
=
+
n
k
k
n
n
kn
n
kn
CCC
0
2
222
.
4.
Cho các ch
s
0
,
1
,
2
,
3
,
4
,
5
. H
i t
các ch
s
này:
a)
L
p
đượ
c bao nhiêu s
4
ch
s
khác nhau trong
đ
ó nh
t thi
ế
t ph
i có m
t ch
s
5
.
b)
L
p
đượ
c bao nhiêu s
7
ch
s
trong
đ
ó ch
s
5
có m
t
đ
úng
3
l
n còn các ch
s
khác có m
t không quá
1
l
n?
5.
Các s
1
,
2
,
K
,
n
l
p thành m
t hang ngang. H
i có m
y cách s
p x
ế
p sao cho:
a)
Hai ch
s
1
2
đứ
ng c
nh nhau.
b)
Ba ch
s
1
,
2
3
đứ
ng c
nh nhau.
6.
Rút
2
bài t
b
bài có
52
lá. G
i
A
bi
ế
n c
đượ
c
2
lá c
ơ
”,
B
là bi
ế
n c
đượ
c
2
10
” và
C
là bi
ế
n c
đượ
c
2
đỏ
”.
a)
Các c
p bi
ế
n c
sau, c
p nào xung kh
c:
A
B
,
A
C
,
B
C
.
b)
Tính các xác su
t:
(
)
BAP
+
,
(
)
CBP
+
(
)
CAP
+
.
c)
Tính các xác su
t:
(
)
ABP
,
(
)
BCP
(
)
ACP
.
7.
M
t bàn dài g
m
2
dãy gh
ế
đố
i di
n nhau, m
i dãy g
m
6
gh
ế
. Ng
ườ
i ta mu
n s
p x
ế
p
6
ch
ng
i cho
6
sinh viên l
p
A
6
sinh viên l
p
B
vào bàn nói trên. H
i có bao
nhiêu cách s
p x
ế
p trong m
i tr
ườ
ng h
p sau:
a)
B
t c
hai sinh viên nào ng
i c
nh nhau ho
c
đố
i di
n nhau thì khác l
p v
i nhau.
b)
B
t c
hai sinh viên nào ng
i
đố
i di
n nhau thì khác l
p v
i nhau.
8.
bao nhiêu cách s
p x
ế
p
10
ng
ườ
i ng
i thành ngang sao cho
2
hai ng
ườ
i
A
,
B
ng
i
c
nh nhau và
2
ng
ườ
i
C
,
D
không ng
i c
nh nhau.
9.
bao nhiêu ng
ườ
i tham gia vào cu
c
đấ
u c
, n
ế
u bi
ế
t r
ng cu
c
đấ
u
đ
ó có t
t c
10
ván c
và m
i
đấ
u th
ph
i
đấ
u m
i m
i
đấ
u th
khác m
t ván?
10.
Gieo
đồ
ng th
i
2
con xúc s
c. Tìm xác su
t
để
:
a)
T
ng s
ch
m xu
t hi
n trên
2
con xúc s
c là 7 .
b)
T
ng s
ch
m xu
t hi
n trên
2
con xúc s
c là 8.
Bài ging
20
c)
T
ng s
ch
m xu
t hi
n trên
2
con xúc s
c h
ơ
n kém nhau
2
.
11.
B
ng
u nhiên
5
lá th
ư
vào
5
phong bì
đ
ã
đề
đị
a ch
tr
ướ
c (m
i phong bì ch
ch
a
đ
úng m
t lá th
ư
). Tìm xác su
t
để
:
a)
C
5
lá th
ư
đề
u
đ
úng ng
ườ
i nh
n.
b)
Lá th
ư
th
nh
t
đ
úng ng
ườ
i nh
n.
c)
Lá th
ư
th
nh
t và lá th
ư
th
hai
đ
úng ng
ườ
i nh
n.
12.
X
ế
p ng
u nhiên
5
ng
ườ
i lên
7
toa tàu
đượ
c
đ
ánh s
(m
i toa tàu có th
ch
a nhi
u
ng
ườ
i). Tìm xác su
t các bi
ế
n c
sau:
a)
5
ng
ườ
i cùng lên m
t toa.
b)
5
ng
ườ
i lên
5
toa
đầ
u.
c)
5
ng
ườ
i lên
5
toa khác nhau.
d)
Hai ng
ườ
i
A
B
cùng lên toa
đầ
u.
e)
Hai ng
ườ
i
A
B
cùng lên m
t toa.
f)
Hai ng
ườ
i
A
B
cùng lên m
t toa, ngoài ra không có ai khác lên toa này.
13.
Ba kh
u súng
độ
c l
p cùng b
n vào m
t m
c tiêu. Xác su
t
để
kh
u th
nh
t b
n trúng
7,0
,
đề
kh
u th
hai b
n trúng là
8,0
,
để
kh
u th
ba b
n trúng là
5,0
. M
i kh
u b
n
m
t viên. Tính xác su
t
để
:
a)
1
kh
u b
n trúng.
b)
2
kh
u b
n trúng.
c)
C
3
kh
u b
n tr
t.
d)
Ít nh
t
1
kh
u b
n trúng.
e)
Kh
u th
nh
t b
n trúng bi
ế
t r
ng
đ
ã có
2
hai kh
u b
n trúng.
14.
M
t h
p
đự
ng 15 qu
bóng bàn trong
đ
ó có 9 qu
n m
i. L
n
đầ
u ng
ườ
i ta l
y ng
u
nhiên
3
qu
để
thi
đấ
u, sau
đ
ó l
i tr
vào h
p. L
n
2
l
y ng
u nhiên
3
qu
. Tìm xác
su
t
để
3
qu
l
y ra l
n sau
đề
u m
i.
15.
Có hai h
p
A
B
. H
p
A
đự
ng
8
bi tr
ng và
2
bi
đ
en. H
p
B
đự
ng
9
bi tr
ng và
1
bi
đ
en. L
y ng
u nhiên
2
bi t
h
p
A
b
sang h
p
B
r
i sau
đ
ó rút ng
u nhiên
3
bi t
h
p
B
. Tìm xác su
t
để
trong
3
bi l
y t
h
p
B
2
bi tr
ng.
16.
M
t h
p ch
a
5
t
vé s
, trong
đ
ó có
đ
úng
1
t
vé s
trúng th
ưở
ng.
5
b
n Tr
ườ
ng,
Đạ
i,
H
c, Duy, Tân l
n l
ượ
t rút ng
u nhiên m
i ng
ườ
i
1
t
vé s
. H
i rút tr
ướ
c hay rút sau
có l
i h
ơ
n (xác su
t
đượ
c t
vé s
trúng th
ưở
ng cao h
ơ
n)? Hãy t
ng quát bài toán này
cho
(
1
n
) t
vé s
mà ch
đ
úng
1
t
trúng th
ưở
ng.
17.
Trong m
t lô hàng g
m có
100
s
n ph
m, trong
đ
ó có
30
s
n ph
m lo
i t
t, l
y ng
u
nhiên l
n l
ượ
t
4
s
n ph
m không tr
l
i. Tìm xác su
t
để
:
a)
l
n th
2
l
y
đượ
c s
n ph
m lo
i t
t.
b)
l
n th
3
l
y
đượ
c s
n ph
m lo
i t
t.
c)
2
l
n
đầ
u l
y
đượ
c s
n ph
m lo
i t
t.
18.
M
t s
đ
i
n tho
i có
7
s
. Ng
ườ
i g
i quên ch
s
cu
i cùng nh
ư
ng anh ta bi
ế
t r
ng s
đ
ó khác
0
, và anh ta quay s
đ
ó m
t cách ng
u nhiên. Tìm xác su
t
để
anh ta th
c hi
n
đượ
c cu
c liên l
c mà không ph
i quay q3 l
n.
19.
Trong gi
bài t
p, giáo viên cho m
t bài toán. L
p có 30 sinh viên nh
ư
ng ch
6 b
n
gi
i
đượ
c bài toán này. Giáo viên g
i ng
u nhiên m
t sinh viên cho
đế
n khi có m
t sinh
viên gi
i
đượ
c bài toán này. Tính xác su
t giáo viên g
i
đế
n sinh viên th
4
.
20.
M
t ng
ườ
i b
n l
n l
ượ
t
2
viên
đạ
n vào m
t t
m bia. Xác su
t trúng bia c
a viên
đạ
n
th
nh
t là
8,0
và c
a viên
đạ
n th
hai là
6,0
.
a)
Tìm xác su
t
để
đ
úng
1
viên
đạ
n trúng
đ
ích.
b)
Bi
ế
t r
ng có
1
viên trúng
đ
ích. Tìm xác su
t
để
đ
ó là viên
đạ
n th
hai.
21.
M
t c
a hàng bán m
t lo
i s
n ph
m trong
đ
ó có
%40
là do x
ưở
ng
A
s
n xu
t, còn l
i
do x
ưở
ng
B
s
n xu
t. T
l
s
n ph
m lo
i
I
do x
ưở
ng
A
s
n xu
t là 8,0 và c
a x
ưở
ng
B
s
n xu
t là 9,0 .
Chương I. Các khái nim cơ bn trong lí thuyết xác sut.
21
a)
Mua ng
u nhiên m
t s
n ph
m. Tìm xác su
t
để
mua
đượ
c s
n ph
m lo
i
I
.
b)
Mua m
t s
n ph
m t
c
a hàng và th
y
đ
ó không ph
i là s
n ph
m lo
i
I
. H
i s
n
ph
m
đ
ó có kh
n
ă
ng do x
ưở
ng nào s
n xu
t nhi
u h
ơ
n.
22.
B
n
3
viên
đạ
n
độ
c l
p vào m
t m
c tiêu. Xác su
t trúng
đ
ích c
a m
i viên t
ươ
ng
ng
3,0
;
4,0
;
5,0
. N
ế
u ch
1
trúng thì m
c ti
u b
phá h
y v
ơ
i xác su
t là
2,0
. N
ế
u ít
nh
t
2
viên trúng thì m
c tiêu ch
c ch
n b
phá h
y. Hãy tìm xác su
t
để
m
c tiêu b
phá h
y khi b
n
3
viên trên.
Đáp s và hướng dn.
1.
a)
n
AAA
K
21
, b)
n
AAA
+
+
+
K
21
,
c)
nnn
AAAAAAAAA KKKK
212121
+++
,
d)
nnnnn
AAAAAAAAAAAA
122132121
+++ KKKK
2.
a)
CAB
là bi
ế
n c
“ng
ườ
i công nhân
đượ
c ch
n là nam và
trong khu t
p th
không
hút thu
c”.
b) Khi
B
A
,
CA
thì
ACBA =
.
3.
Dùng các công th
c:
( )
!!
!
knk
n
C
k
n
=
,
( )
=
=+
n
k
kknk
n
n
baCba
0
(
)
(
)
(
)
nnn
xxx
2
111 +=++
.
4.
a)
204
, b)
3720
.
5.
a)
(
)
!12
n
, b)
(
)
!2
n
.
6.
a)
=
AB
,
BC
,
AC
, b)
( )
2
52
2
4
2
13
C
CC
BAP
+
=+
,
( )
2
52
2
26
C
C
CAP =+
,
( ) ( ) ( ) ( )
2
52
2
26
2
4
1
C
CC
BCPCPBPCBP
+
=+=+
,
c)
(
)
0
=
ABP ,
( )
2
52
2
13
C
C
ACP =
,
( )
2
52
1
C
BCP =
.
7.
a)
( )
2
2. 6!
, b)
( )
2
6
2 . 6!
.
8.
2
2.8! 2 .6!
.
9.
2
10 5
n
C n
=
=
.
10.
a)
6
1
, b)
36
5
, c)
9
2
.
11.
a)
1 1
5! 120
=
, b)
1.4! 1
5! 5
=
, c)
1.1.3! 1
5! 20
=
.
12.
a)
5 4
7 1
7 7
= , b)
5
5!
7
, c)
5
7
5
7
A
, d)
25
3
7
1
7
7
= , e)
7
1
7
7
5
4
= , f)
4
3
5
3
7
6
7
6.7
=
13.
a) 22,0 , b) 47,0 , c) 03,0 , d) 97,0 , e)
47
35
.
14.
G
i
A
là bi
ế
n c
“c
3 qu
bóng l
y
đượ
c l
n sau
đề
u m
i”. G
i
i
B là bi
ế
n c
“trong
3qu
l
y ra thi
đấ
u có
i
qu
m
i”, 3;0=i . Khi
đ
ó
( ) ( ) ( )
=
=
3
0
/
i
ii
BAPBPAP
hay
( )
3
15
3
6
3
15
3
9
3
15
3
7
3
15
1
6
2
9
3
15
3
8
3
15
2
6
1
9
3
15
3
9
3
15
3
6
..
.
.
.
.
C
C
C
C
C
C
C
CC
C
C
C
CC
C
C
C
C
AP +++=
Bài ging
22
15.
T
ươ
ng t
bài 14, ta
đượ
c
( )
3
12
1
1
2
11
2
10
2
8
3
12
1
2
2
10
2
10
1
2
1
8
3
12
1
3
2
9
2
10
2
2
.
.
.
.
..
.
C
CC
C
C
C
CC
C
CC
C
CC
C
C
AP ++=
16.
Các xác su
t trúng th
ưở
ng c
a
5
b
n Tr
ườ
ng,
Đạ
i, H
c, Duy, Tân là nh
ư
nhau và
đề
u
b
ng
5
1
.
17.
G
i
i
A là bi
ế
n c
“l
n th
i
l
y
đượ
c s
n ph
m lo
i t
t”,
100;1=i
. Khi
đ
ó
a)
Chú ý r
ng:
21212
AAAAA +=
n
(
)
(
)
(
)
(
)
(
)
1211212
/./. AAPAPAAPAPAP +=
.
b)
321321321321
. AAAAAAAAAAAAA +++=
.
c)
D
th
y:
( )
330
29
2
100
2
30
21
==
A
A
AAP
.
18.
G
i
A
là bi
ế
n c
“g
i
đ
úng
đượ
c s
cu
i cùng” và
i
A là bi
ế
n c
“g
i
đ
úng
đượ
c s
cu
i cùng
l
n th
i
”, 3;1=i . Khi
đ
ó
321211
... AAAAAAA ++=
và chú ý r
ng:
(
)
(
)
(
)
(
)
(
)
(
)
(
)
3
1
7
1
.
8
7
.
9
8
8
1
.
9
8
9
1
.///
213121
2
1211
=++=
++= AAAPAAPAPAAPAPAPAP
19. Gi
i
A là biến c “sinh viên được gi ln th
i
gii được bài toán”, 30;1=i .
Ta có:
(
)
(
)
(
)
(
)
(
)
(
)
27
6
.
28
22
.
29
23
.
30
24
///
32142131214321
=
== AAAAPAAAPAAPAPAAAAPAP
20. Gi
A
là biến c “có đúng
1
viên trúng đích”.
i
A là biến c “viên đạn th
i
trúng
đích”, 2;1=i .
a)
2121
AAAAA +=
. T đó suy ra
(
)
44,0
=
AP
.
b)
( )
(
)
(
)
( )
44,0
12,0
/.
/
2121
21
==
AP
AAAPAAP
AAAP
21. Gi
M
là biến c “Sn phm mua được loi
I
”.
N
,
Q
ln lượt là biến c “Sn phm
mua được do xưởng
A
sn xut”, “Sn phm mua được do xưởng
B
sn xut”.
a) Ta có:
(
)
(
)
(
)
(
)
(
)
9,0%.608,0%.40/./.
+
=
+
=
QMPQPNMPNPMP .
b) Ta có:
(
)
14
8
/ =NMP
(
)
14
6
/ =QMP
.
22. Gi
A
là biến c “mc tiêu b phá hy”.
i
B là biến c “có
i
viên đạn bn trúng mc
tiêu”, 3;1=i .
j
C là biến c “viên đạn th
j
bn trúng mc tiêu”, 3;1=j .
Ta có:
( ) ( ) ( )
438,0/
3
1
==
=i
ii
BAPBPAP
,
trong đó
(
)
2,0/
1
=
BAP
,
(
)
(
)
1//
32
=
=
BAPBAP
(
)
(
)
(
)
(
)
(
)
(
)
(
)
(
)
(
)
(
)
3213213211
...... CPCPCPCPCPCPCPCPCPBP ++=
. Tương t, ta
tính được
(
)
2
BP
,
(
)
3
BP .
C. Phương pháp ging dy.
- Vn đáp và làm bài tp.
Chương I. Các khái nim cơ bn trong lí thuyết xác sut.
23
- Đưa ra các d thường gp trong thc tin để to động cơ hướng đích to nên hng thú
hc tp cho sinh viên.
- Kim tra, đánh giá vic làm bài tp ca SV.
- Gi m t trc quan sinh đng đến tư duy tru tượng gii quyết vn đề.
- Phi hp phương pháp thuyết trình và vn đáp gii quyết vn đề và làm bài tp.
- Yêu cu SV đọc bài ging trước khi lên lp.
- Kim tra, đánh giá vic làm bài tp ca SV.
- S dng phương tin dy hc hin đại như Mic, Projector.
D. Tài liu tham kho
[1] Đậu Thế Cp, Xác sut thng kê: thuyết các bài tp (Chương 1), NXB
Giáo dc, 2006.
[2] Đinh Văn Gng, Bài tp xác sut thng (Chương 1), NXB Giáo dc,
2007.
[3] PGS. TS. Phm Xuân Kiu, Giáo Trình xác sut thng (Chương 1), NXB
Giáo dc, 2005.
[4] Đặng Công Hanh, Đặng Ngc Dc, Giáo trình thuyết xác sut Thng
toán (Chương 1), trường Đại hc Duy Tân, 1996.
Chương II.
Đại lượng ngu nhiên. Hàm phân phi xác
sut.
A. Mc tiêu.
- Gii thiu biến ngu nhiên và m phân phi xác sut.: biến ngu nhiên ri rc cùng
vi bng phân phi xác sut ca nó, biến ngu nhiên liên tc ng vi hàm mt độ ca
nó.
- Nm các đặc trưng ca biến ngu nhiên: vng, phương sai, Mod, Med,hiu được ý
nghĩa ca chúng.
B. Ni dung.
1. Khái nim. Phân loi đại lượng ngu nhiên.
Định nghĩa.
Cho mt phép th
không gian các biến c sơ cp ca nó. Mt ánh x t
R
hay mt quy tc cho tương ng mi kết qu ca phép th vi mi mt s
thc nào đó được gi là mt đại lượng ngu nhiên (biến ngu nhiên) liên kết vi phép
th nào đó.
Ta thường hiu đại lưng ngu nhiên bng ch in hoa
X
,
Y
,
Z
,
K
Giá tr ca
được kí hiu bng ch in thường
x
,
y
,
z
,
K
Ví d 1.1.
a)
X
s con gái trong mt ln sinh (
1
con).
X
đại lượng ngu nhiên. Giá tr ca
nó có th nhn
0
,
1
.
b)
X
s viên đạn trúng đích khi bn liên tiếp
n
viên đạn độc lp vào mt mc tiêu.
Giá tr ca nó có th nhn là
0
,
1
,
K
,
.
c)
X
s sn phm tt trong
10
sn phm được chn ngu nhiên t lô sn phm có
100
sn phm tt
50
phế phm.
X
cũng đại lượng ngu nhiên. Giá tr ca
th nhn là
0
,
1
,
K
,
10
.
d)
X
s ln tung mt đồng tin cho đến khi được mt nga thì dng. Khi đó
X
đại lượng ngu nhiên và giá tr ca nó có th nhn là
1
,
2
,
K
,
,
K
e)
X
độ cao ca mt cây ti thi gian
t
nào đó.
X
đại lượng ngu nhiên.
Bài ging
26
Trong d này, xét a):
X
s con gái trong
1
ln sinh con. Ta thy
X
tha mãn
định nghĩa đi lượng ngu nhiên trên. Tht vy, ta có không gian đại lưng c sơ cp
{
}
GT;
=
, và
X
có th nhn
2
giá tr
0
hoc
1
.
Vi mi
Rx
, ta s chng minh tp hp
{
}
xX
<
là biến c ngu nhiên.
D dàng ta có:
{ } { }
>
<
=<
1,
10,
0,
x
xG
x
xX
. Ba tp
,
{
}
G
đều biến c ngu
nhiên. Vy
{
}
xX
<
biến c ngu nhiên.
Ta quan m nghiên cu đến hai loi đại lượng: đại lượng ngu nhiên ri rc đại
lượng ngu nhiên liên tc.
1.1. Đại lượng ngu nhiên ri rc.
Định nghĩa:
Đại lượng ngu nhiên ri rc đại lượng ngu nhiên mà các giá tr có th nhn ca nó
là tp hp hu hn hoc vô hn đếm được. Trong d 1.1. Các ví d a), b), c), d) đều
đại lượng ngu nhiên ri rc.
1.2. Đại lượng ngu nhiên liên tc.
Định nghĩa.
Đại lượng ngu nhiên liên tc đại lưng ngu nhiên các giá tr th nhn ca
lp đầy khong
(
)
ba;
(hoc đon
[
]
ba;
) nào đó,
th bng
,
b
th
bng
+
.
1.3. Hàm phân phi ca đại lượng ngu nhiên.
Ta nhn thy tp hp
{
}
xX
<
,
Rx
thay đổi nếu
x
thay đổi. Do đó
{
}
(
)
xP
<
X
cũng
thay đổi, tc là xác sut này ph thuc vào
x
. Nó là hàm ca
x
.
Định nghĩa.
Cho
X
đại lượng ngu nhiên. Ánh x
[
]
1;0: RF
xác định bi
[
]
( ) ( )
xXPxFx
RF
<=
1;0:
được gi là hàm phân phi xác sut ca đại lượng ngu nhiên
X
.
Ví d 1.2.
Tìm hàm phân phi ca đại lượng ngu nhiên
X
ch s ln xut hin mt sp khi gieo
mt đồng tin cân đối và đồng cht.
Gii
Không gian biến c sơ cp tương ng vi phép th “gieo đồng tin
{
}
NS;
=
. Vì
X
có th nhn
2
giá tr
0
hoc
1
.
Chương II. Đại lượng ngu nhiên. Hàm phân phi xác sut
27
Vì vy:
{ } { }
>
<
=<
1,
10,
0,
x
xS
x
xX
.
Khi đó, hàm phân phi ca đại lượng ngu nhiên
X
là:
( ) ( )
(
)
{ }( )
( )
>
<
=<=
1,
10,
0,
xP
xSP
xP
xXPxF
hay
( )
>
<
=
1,1
10,
2
1
0,0
x
x
x
xF
Các tính cht ca hàm phân phi
a) Hàm phân phi
(
)
xF
hàm không gim.
b)
(
)
(
)
(
)
aFbFbXaP
=
<
.
c)
(
)
(
)
(
)
aFbFaXP
ab
=
=
+
lim
.
d)
(
)
0
=
F
(
)
1
=
+
F
.
Nhn xét.
T a) d), ta
(
)
10
xF
. Tính cht a) d) được gi tính cht đặc trưng ca
hàm phân phi xác sut. Mt hàm
(
)
xF
xác định trên
R
nh cht a) d) đều
phân phi xác sut ca mt đại lượng ngu nhiên nào đó
Ví d 1.3.
Cho hàm s
( )
2
1
arctan
1
+= xxF
π
.
Chng minh rng
(
)
xF
là hàm phân phi xác sut ca đi lượng ngu nhiên.
Gii.
Do
( )
0
1
1
.
1
'
2
>
+
=
x
xF
π
nên
(
)
xF
tăng.
Mt khác
( )
0
2
1
2
1
2
1
arctan
1
limlim =+
=
+=
−∞−∞
π
ππ
xxF
xx
( )
1
2
1
2
1
2
1
arctan
1
limlim =+
=
+=
+∞+∞
π
ππ
xxF
xx
nên
(
)
xF
là hàm phân phi xác sut ca đi lượng ngu nhiên (đpcm).
2. Đại lượng ngu nhiên ri rc
2.1. Bng phân phi xác sut.
Gi s
X
đại lượng ngu nhiên ri rc. Nó nhn các giá tr
1
x
,
2
x
,
K
,
n
x
,
K
th
vi các xác sut tương ng
(
)
0
=
=
ii
pxXP
.
Ta lp bng sau đây
Bài ging
28
X
1
x
2
x
n
x
(
)
i
xXP
=
1
p
2
p
n
p
Vi
1
1
=
=
n
i
i
p
. Bng này có thhn khi
n
nhn giá tr
+
.
Bng trên được gi là bng phân phi xác sut ca đi lượng ngu nhiên
X
.
2.2. Hàm phân phi xác sut.
Nếu ta sp xếp các giá tr
1
x
,
2
x
,
K
,
n
x
,
K
theo th t tăng dn, d
LL
<
<
<
<
n
xxx
21
thì hàm phân phi ca
X
có th viết dưới dng:
( )
>
<+++
<+
<
=
n
nnn
xx
xxxppp
xxxpp
xxxp
xx
xF
,1
,
,
,
,0
1121
3221
211
1
L
KKK
Nếu các giá tr v trí bt thì ta th viết hàm phân phi dưới dng:
(
)
<
=
xx
i
i
pxF
,
Rx
.
Ví d 2.1.
Trong mt hàng gm
4
sn phm tt
6
sn phm xu. Ly ngu nhiên
4
sn
phm. Gi
X
s sn phm xu ly được.
Lp bng phân phi xác sut ca
X
. Viết hàm phân phi ca
X
và tính xác sut
(
)
30
<
XP
.
Gii
Ly ngu nhiên
4
sn phm thì vi
X
s sn phm tt ly được, ta
X
th
nhn các giá tr
0
,
1
,
2
,
3
,
4
.
Ta có:
( )
210
1
.
0
4
10
4
4
0
6
===
C
CC
XP
,
( )
35
4
.
1
4
10
3
4
1
6
===
C
CC
XP
,
( )
7
3
.
2
4
10
2
4
2
6
===
C
CC
XP
,
( )
21
8
.
3
4
10
1
4
3
6
===
C
CC
XP
,
( )
14
1
.
4
4
10
0
4
4
6
===
C
CC
XP
.
T đó, ta có bng phân phi xác sut như sau:
X
0
1
2
3
4
P
210
1
35
4
7
3
21
8
14
1
T đó, ta có hàm phân phi xác sut
Chương II. Đại lượng ngu nhiên. Hàm phân phi xác sut
29
( )
>
<
<
<
<
=
4,1
43,
14
13
32,
42
23
21,
42
5
10,
210
1
0,0
x
x
x
x
x
x
xF
Khi đó, ta
( ) ( ) ( )
42
23
0
42
23
0330 ===< FFXP
hoc ta th tính như sau
( ) ( ) ( ) ( )
42
23
21030 ==+=+==< XPXPXPXP
Ví d 2.2.
Bn liên tiếp
3
viên đn độc lp vào mt mc tiêu. Xác sut trúng đích ca mi viên
đạn
5,0
. Gi
X
là s viên đạn trúng đích trong
3
viên. Tìm hàm phân phi xác sut
ca
X
. Viết hàm phân phi ca
X
. Tính xác sut
(
)
1
XP
.
Gii
Ta xem vic bn
3
viên đạn độc lp vào mt mc tiêu tiến hành dãy
3
phép th
Bernoulli. Xác sut bn trúng đích ca mi viên đạn là
2
1
=p
.
Theo công thc c sut, ta có:
( )
kk
k
CkXP
==
3
3
2
1
1
2
1
,
3;0=k
hay
( )
3
3
2
1
==
k
CkXP
,
3;0=k
, phân phi xác sut ca
X
. Ta th viết dưới dng
bng sau:
X
0
1
2
3
P
8
1
8
3
8
3
8
1
Hàm phân phi ca
X
Bài ging
30
( )
>
<
<
<
=
3,1
32,
8
7
21,
8
5
10,
8
1
0,0
x
x
x
x
x
xF
Xác sut
( ) ( ) ( )
8
7
8
1
101111 ===<= PXPXP
.
Ví d 2.3.
Trong mt ng gm
10
y vi tính mi thì
3
chiếc b li, ly ngu nhiên
4
máy trong
10
máy tính này. Gi
X
là s máy tính b li trong
4
máy ly ra. Hãy:
a) Lp bng phân phi xác sut ca
X
.
b) Khi ly
4
máy thì có my máy b li là có kh năng xy ra cao nht.
c) Tìm xác sut khi ly ra
4
máy s có ít nht mt máy b li.
d) Nếu người nào đó ly ngu nhiên ra
3
máynh để kim tra thy không có máyo
b li thì s chp nhn c hàng. Tìm xác sut người mua chp nhn ng xác
sut người mua bác b lô hàng.
Gii.
a) Ta có
{
}
3;2;1;0
X
( )
4
10
4
7
0
3
.
0
C
CC
XP ==
,
( )
4
10
3
7
1
3
.
1
C
CC
XP ==
,
( )
4
10
2
7
2
3
.
2
C
CC
XP ==
,
( )
4
10
1
7
3
3
.
3
C
CC
XP ==
T đó ta có bng phân phi
X
0
1
2
3
(
)
i
xXP
=
4
10
4
7
0
3
.
C
CC
4
10
3
7
1
3
.
C
CC
4
10
2
7
2
3
.
C
CC
4
10
1
7
3
3
.
C
CC
b) Da vào bng xác sut, ta
( )
5,0
.
1
4
10
3
7
1
3
===
C
CC
XP
cao nht nên trong
4
máy
tính ly ra thì b
1
máy tính b li là có kh năng cao nht.
c)
( ) ( )
833,0167,01
.
1011
4
10
4
7
0
3
====
C
CC
PXP
.
d)
2917,0
.
4
10
1
7
3
3
==
C
CC
p
xác sut để người mua chp nhn hàng. Xác sut để người
mua bác b lô hàng
7083,02917,011
=
=
p
.
Chương II. Đại lượng ngu nhiên. Hàm phân phi xác sut
31
2.3. Phép toán đại lượng ngu nhiên.
Cho
X
Y
là các đại lượng ngu nhiên có bng phân phi xác sut
X
1
x
2
x
n
x
(
)
i
xXP
=
1
p
2
p
n
p
Y
1
y
2
y
n
y
(
)
i
yYP
=
1
q
2
q
n
q
hiu:
(
)
jiij
yYxXPp === ;
để cho ĐLNN
X
nhn giá tr
i
x
ĐLNN
Y
nhn giá
tr
j
y
. Gi s
1
z
,
2
z
,
K
,
S
z
là các giá tr khác nhau ca tng
ji
yx
+
, đặt
=+
+
=
kji
zyx
ijk
pp
.
Ta gi tng ca
X
Y
đại lượng ngu nhiên
Y
X
+
có bng phân phi xác sut là
Y
X
+
1
z
2
z
n
z
(
)
i
zYXP
=
+
+
1
p
+
2
p
+
n
p
Tương t, gi s
1
z
,
2
z
,
K
,
T
z
là các giá tr khác nhau ca tích
ji
yx .
, đặt
=
=
kji
zyx
ijk
pp
.
Ta gi tích ca
X
Y
đại lượng ngu nhiên
YX.
(hoc
XY
) bng phân phi
xác sut là
YX.
1
z
2
z
n
z
(
)
i
zYXP
=
.
1
p
2
p
n
p
Đại lượng ngu nhiên
X
Y
gi độc lp nếu
(
)
jijiij
qpyYxXPp ====
;
.
Ví d 2.4.
Cho
X
Y
độc lp có bng phân phi xác sut
X
0
1
2
(
)
i
xXP
=
2,0
3,0
5,0
Y
1
0
1
(
)
i
yyP
=
4,0
3,0
3,0
Tìm phân phi xác sut ca
Y
X
+
,
YX
.
.
Gii
Bài ging
32
Theo định nghĩa, ta có:
(
)
(
)
(
)
(
)
08,04.0.2,01.01;043
=
=
=
=
=
=
=
=
=
+
YPXPYXPYXP
.
(
)
(
)
(
)
( ) ( ) ( ) ( )
18,04,0.3,03,0.2,0
1.10.0
1;10;033
=+=
==+===
=
=
+
=
=
=
=
+
YPXPYPXP
YXPYXPYXP
Tương t,
(
)
(
)
(
)
(
)
1;20;11;023
=
=
+
=
=
+
=
=
=
=
+
YXPYXPYXPYXP
(
)
(
)
(
)
0;21;113
=
=
+
=
=
=
=
+
YXPYXPYXP
(
)
(
)
1;203
=
=
=
=
+
YXPYXP
Khi đó, ta có bng phân phi ca đại lượng
3
+
YX
3
+
YX
4
3
2
1
0
(
)
i
zYXP
=
+
3
08,0
18,0
35,0
24,0
15,0
Tương t, ta có bng phân phi ca
XY
YX.
2
1
0
1
2
(
)
i
zYXP
=
.
20,0
12,0
44,0
09,0
15,0
3. Đại lượng ngu nhiên liên tc.
Định nghĩa.
Đại lượng ngu nhiên liên tc
X
(
)
xF
m phân phi xác sut ca nó. Nếu tn
ti hàm s
(
)
xf
xác định và không âm trên
R
sao cho
( ) ( )
=
x
dttfxF
thì m s
(
)
xf
được gi hàm mt độ ca
X
.
(
)
xF
chính din tích gii hn bi đường cong ca hàm mt độ
(
)
xf
phn trc
hoành bên trái đim
x
.
Ví d 3.1.
( )
=
x
t
dtexF
2
2
2
1
π
được gi hàm phân phi chun. Đó din tích gii hn bi
đường cong
( )
2
2
2
1
x
exf
=
π
trc hoành bên trái
x
.
T tính cht ca hàm phân phi, ta suy ra tính cht ca hàm mt độ
+
(
)
0
xf
,
( ) ( )
=
x
dttfxF
.
Chương II. Đại lượng ngu nhiên. Hàm phân phi xác sut
33
+
( )
1=
+∞
dxxf
(
)
0
=
F
(
)
1
=
+
F
.
+
( ) ( )
=<
b
a
dxxfbXaP
. Tht vy
( ) ( ) ( ) ( ) ( ) ( )
===<
b
a
ab
dxxfdxxfdxxfaFbFbXaP
Ví d 3.2.
Gi s hàm phân phi ca đại lượng ngu nhiên
( )
2
1
arctan
1
+= xxF
π
. Tìm m mt
độ ca
X
và tính xác sut
(
)
11
<
XP
.
Gii.
Ta có hàm mt độ
( ) ( )
( )
2
1
1
'
x
xFxf
+
==
π
và xác sut
( ) ( ) ( )
2
1
44
1
1111
=
==<
ππ
π
FFXP
.
Định lí.
Nếu hàm phân phi
(
)
xF
ca đi lượng ngu nhiên
X
liên tc ti
a
x
=
thì
(
)
0
=
=
axP
.
Chng minh
Do
(
)
(
)
(
)
aFbFbXaP
=
<
liên tc ti
a
nên cho
+
ab
, ta
(
)
(
)
(
)
(
)
(
)
0lim
=
=
=
=
+
aFaFaFbFaxP
ab
.
Nhn xét.
Theo Đnh lí, nếu
(
)
xF
liên tc ti
a
b
thì
(
)
(
)
(
)
(
)
bXaPbxaPbXaPbXaP
=
<
=
<
<
=
<
.
Ví d 3.3.
Gi s hàm mt độ ca đại lượng ngu nhiên
X
( )
>>
=
0,0
0;0,
x
xMe
xf
x
λ
λ
.
Tìm
M
. Tìm hàm phân phi ca
X
.
Gii
Theo tính cht ca hàm mt độ, ta có:
( )
1
=
+∞
dxxf
.
D thy
( )
λλ
λλ
M
eMMedxdxxf
xx
=
=+=
+∞
+∞
+∞
0
0
0
1
0
.
Vy
λ
=
M
.
Bài ging
34
Ta có hàm phân phi
(
)
xF
được xác định như sau:
+ Nếu
0
<
x
thì
( ) ( )
0==
x
dxxfxF
.
+ Nếu
0
x
thì
( ) ( ) ( ) ( )
x
x
x
xx
edxedxxfdxxfdxxfxF
λλ
λ
=+=+==
10
00
0
Vy
( )
<
=
0,1
0,0
xe
x
xF
x
λ
4. Các đặc trưng ca đại lượng ngu nhiên.
4.1. Kì vng.
Định nghĩa.
Kì vng ca đại lượng ngu nhiên
X
, kí hiu là:
(
)
XE
xác định bi:
+ Nếu
X
đại lượng ngu nhiên ri rc có bng phân phi xác sut
X
1
x
2
x
n
x
(
)
i
xXP
=
1
p
2
p
n
p
thì
( )
+∞
=
=++++=
1
2211
i
iinn
pxpxpxpxXE
LL
.
Trong trưng hp có vô hn
n
x
thì ta nói
X
kì vng và
(
)
XE
là kì vng ca nó nếu
chui
+∞
=1i
ii
px
hi t tuyt đối.
+ Nếu
X
đại lượng ngu nhiên liên tc hàm mt độ xác sut
(
)
xf
thì
( ) ( )
+∞
= dxxxfXE
.
Ý nghĩa ca kì vng.
vng ca đại lượng ngu nhiên trung nh theo xác sut các gtr th nhn
ca đại lượng ngu nhiên đó.
Tính cht
Vi mi đi lượng ngu nhiên
X
,
Y
, ta có:
a)
(
)
CCE
=
vi
C
đại lưng ngu nhiên hng s.
b)
(
)
(
)
(
)
YEXEYXE
+
=
+
.
c)
(
)
(
)
XEXE .
λ
λ
=
,
λ
mt s.
d)
(
)
(
)
(
)
YEXEXYE .
=
nếu
X
Y
độc lp.
Chương II. Đại lượng ngu nhiên. Hàm phân phi xác sut
35
Ví d 4.1.
Nghiên cu v đim thi môn Toán ca
400
sinh viên mt trưng Đi hc, ta được
bng s liu như sau
Đim
2
3
4
6
7
8
S sinh
viên
10
60
160
100
40
30
Gi
X
là s đim môn Toán ca sinh viên mt trường Đi hc.
a) Tính
(
)
XE
?
b) Tính tng s đim môn Toán ca
400
sinh viên. Như vy, đim trung bình môn
Toán ca mt sinh viên là bao nhiêu? So sánh giá tr đó vi
(
)
XE
?
Gii
a) Ta lp bng phân phi xác sut như sau
X
(Đim)
2
3
4
6
7
8
(
)
i
xXP
=
40
1
40
6
40
16
40
10
40
4
40
3
Khi đó, ta có
( )
400
1960
40
3
.8
40
4
.7
40
10
.6
40
16
.4
40
6
.3
40
1
.2 =+++++=XE
.
b) Ta tng s đim môn Toán ca
400
sinh viên là
196030.840.7100.6160.460.310.2
=
+
+
+
+
+
.
Suy ra đim trung bình môn Toán ca mt sinh viên là
400
1960
.
D thy
( )
400
1960
=XE
. Khi đó,
(
)
XE
đim trung bình môn Toán ca sinh viên.
Như vy, ta suy ra vng ca mt đại lưng ngu nhiên
X
giá tr trung bình ca
đại lượng ngu nhiên đó.
Ví d 4.2.
Trong mt cuc thi vn đáp, có hai hình thc thi như sau:
+ Hình thc thi th nht mi người phi tr li 2 câu hi, mi u tr li đúng thì
được 5 đim.
+ Hình thc thi th hai là nếu tr li đúng câu th nht thì mi được tr li câu th hai.
Câu th nht tr li đúng đưc 5 đim, câu th hai tr li đúng đưc 10 đim.
Trong c hai hình thc thi này, các câu tr li sai đều không được đim. Gi s xác
sut tr li đúng mi câu
4
3
vic tr li mi câu độc lp vi nhau. Theo bn,
nên chn hình thc nào để s đim trung bình đạt được nhiu hơn.
Gii.
Bài ging
36
Gi
i
A
là biến c “tr li đúng câu hi th
i
”,
2;1=i
. Ta có:
( ) ( )
4
3
21
== APAP
.
Gi
1
X
,
2
X
là s đim đt được tương ng vi hai hình thc thi trên. Theo yêu cu bài
toán, ta cn so sánh
(
)
1
XE
(
)
2
XE
.
Ta có bng phân phi xác sut ca
1
X
như sau
1
X
0
5
10
(
)
i
xXP
=
1
16
1
16
6
16
9
Khi đó, đim trung bình trong hình thc thi th nht là
(
)
5,7
1
=
XE
.
Ta có bng phân phi xác sut ca
2
X
2
X
0
5
15
(
)
i
xXP
=
2
4
1
16
3
16
9
Khi đó, đim trung bình trong hình thc thi th hai là
(
)
375,9
2
=
XE
.
Vy, ta có
(
)
(
)
21
XEXE
<
nên chn hình thc thi th hai.
4.2. Phương sai.
Định nghĩa.
Cho
X
mt đại lưng ngu nhiên vng
(
)
XE
. Khi đó, ta gi phương sai ca
X
là kì vng ca bình phương độ sai khác gia
X
(
)
XE
, kí hiu là
(
)
XD
.
Vy
(
)
(
)
(
)
(
)
(
)
XEXEXEXEXD
22
2
==
Ý nghĩa ca phương sai.
Phương sai trung bình ca bình phương sai s gia
X
EX
. Như vy, phương
sai càng nh thì các giá tr ca
X
càng tp trung quanh
EX
.
Do
(
)
0
XD
nên ta định nghĩa độ lch chun ca đại lượng ngu nhiên
X
như sau
Định nghĩa.
Độ lch chun ca đại lượng ngu nhiên
X
(
)
(
)
XDX
=
σ
.
Độ lch chun được dùng thưng xuyên hơn phương sai do ng đơn v đo vi đại
lượng ngu nhiên
X
.
Tính cht.
Vi mi đi lượng ngu nhiên
X
,
Y
, ta có:
a)
(
)
0
XD
.
(
)
0
=
XD
X
đại lượng ngu nhiên hng s.
b)
(
)
0
=
CD
vi
C
đại lượng ngu nhiên hng s.
c)
(
)
(
)
XDXD
2
λλ
=
,
λ
mt s.
Chương II. Đại lượng ngu nhiên. Hàm phân phi xác sut
37
d)
(
)
(
)
XDXD
=
+
λ
,
λ
là mt s.
e)
(
)
(
)
(
)
XEXEXD
22
=
.
f)
(
)
(
)
(
)
YDXDYXD
+
=
+
nếu
X
Y
độc lp.
Ví d 4.3.
Đim các môn Toán cao cp
1
A
,
2
A
,
3
A
, Xác sut thng (XSTK) Kinh tế lượng
(KTL) ca hai sinh viên An và Bình đưc cho theo bng sau
Môn
TCC
1
A
TCC
2
A
TCC
3
A
XSTK KTL
Đim ca
An
7
6
8
9
5
Đim ca
Bình
9
10
5
10
1
Gi
X
,
Y
ln lưt là đim môn Toán ca bn An và Bình.
a) Hãy tính
(
)
XE
,
(
)
YE
và so sánh
(
)
XE
,
(
)
YE
.
b) Tính
(
)
XD
,
(
)
YD
. So sánh các giá tr này.
Gii
a) Ta
( )
7
5
35
5
59867
==
+
+
+
+
=XE
,
( )
7
5
35
5
1105109
==
+
+
+
+
=YE
.
Vy
(
)
(
)
YEXE
=
.
b) Ta
( )
51
5
59867
22222
2
=
++++
=XE
.
Khi đó
(
)
(
)
(
)
6755
222
=== XEXEXD
.
( )
5
307
5
1105109
22222
2
=
++++
=
YE
.
Khi đó
( )
(
)
( )
4,127
5
307
222
=== YEYEYD
. Vy
(
)
(
)
XDYD
>
.
Ta thy rng An và Bình cùng có đim trung bình các môn Toán, tuy nhiên An là “hc
đều” hơn Bình.
4.3. Mt, trung v và moment trung tâm.
a) M
t (mod).
Định nghĩa.
Mtgiá tr ca đại lưng ngu nhiên
X
được kí hiu là
(
)
XMod
mà ti đó hàm mt
độ
(
)
xf
đạt giá tr ln nht.
Bài ging
38
Trường hp
X
đại lượng ngu nhiên ri rc,
(
)
XMod
là giá tr ca
X
mà ti đó xác
sut
(
)
(
)
XModXP
=
là ln nht.
Mt ca
X
còn gi là s có kh năng nht.
Chú ý.
a) Mt có th không tn ti và khi nó tn ti không nht thiết là giá tr duy nht.
b) Mt không phi luôn luôn tn ti, chng hn khi tt c các s liu trong mu s
ln xut hin bng nhau.
Ví d 4.4.
Cho đại lượng ngu nhiên
X
có bng phân phi
X
0
1
2
(
)
i
xXP
=
4
1
2
1
4
1
Ta có
(
)
1
=
XMod
( )
2
1
1 ==XP
là xác sut ln nht.
Ví d 4.5.
Cho
X
đại lượng ngu nhiên m mt độ
( )
>
=
0,
2
0,0
4
2
xe
x
x
xf
x
. Hãy xác định
(
)
XMod
.
Gii
Ta có:
+
(
)
0
=
xf
,
0
x
.
+
( )
4
2
2
x
e
x
xf
=
,
0
>
x
.
Ta có
( )
==
2
1
2
1
42
1
'
2
44
2
4
222
x
ee
x
exf
xxx
.
Khi đó
(
)
0'
=
xf
2=x
hoc
2=x
. Do
0
>
x
nên
2=x
.
Da vào bng biến thiên, ta được
( )
( )
2
1
2
2
2
= efxf
.
Vy
(
)
2=XMod
.
b) Phân v
.
Đim
0
x
được gi phân v vi xác sut
α
ca đại lượng ngu nhiên
X
nếu
(
)
α
=
>
0
xXP
(hoc
(
)
α
=
<
0
xXP
).
Hin nhiên
(
)
(
)
α
=
>
=
11
00
xXPxXP
.
Chương II. Đại lượng ngu nhiên. Hàm phân phi xác sut
39
Trong bài ging này, chúng ta dùng phân v
(
)
α
=
>
0
xXP
.
Nếu
2
1
=
α
thì đim
0
x
này được gi là trung v ca
X
. Khi đó, ta xác định như sau
c) Trung v
(median).
Định nghĩa.
Cho
X
là mt đại lượng ngu nhiên. S
m
gi là trung v ca
X
, kí hiu
(
)
XMed
nếu
( )( ) ( )( )
( )( ) ( )( )
=
=<
2
1
2
1
XMedFXMedXP
XMedFXMedXP
)1(
hoc
( )( )
( )( )
>
<
2
1
2
1
XMedXP
XMedXP
)2(
* Nếu
X
đại lượng ngu nhiên ri rc
(
)
XMed
là giá tr
k
x
sao cho
( ) ( ) ( )
( ) ( ) ( )
=++=+=
=++=+=
2
1
2
1
21
121
k
k
xXPxXPxXP
xXPxXPxXP
L
L
, trong đó
k
xxx
L
21
.
* Nếu
X
đại lượng ngu nhiên liên tc thì
(
)
XMed
tha
( )( )
2
1
=XMedF
.
Ví d 4.6.
Cho hàm phân phi ca đại lượng ngu nhiên
X
( )
>
<
=
1,1
10,
0,0
x
xx
x
xF
.
Ta có
( )
2
1
=xF
suy ra
( )
2
1
=XMed
Ví d 4.7.
Cho đi lượng ngu nhiên
X
bng phân phi
X
0
1
2
(
)
i
xXP
=
4
1
2
1
4
1
Ta có
( )
2
1
4
1
1 =<XP
( )
2
1
4
3
1 >=XP
nên
(
)
1
=
XMed
.
Chú ý.
Theo đnh nghĩa trên thì
X
th mt hoc nhiu trung v. Nếu có
1
m
,
2
m
cùng
tha
)1(
hoc
)2(
21
mm
<
thì vi
m
bt kì thuc
[
]
21
;mm
cũng là median ca
X
.
Ví d 4.8.
Gi
X
là s chm xut hin khi gieo con xúc xc. Khi đó
X
có bng phân phi
Bài ging
40
X
1
2
3
4
5
6
(
)
XP
6
1
6
1
6
1
6
1
6
1
6
1
Ta có
( ) ( ) ( )
2
1
3
1
6
1
.2213 ===+==< XPXPXP
( ) ( ) ( ) ( )
2
1
2
1
6
1
.33213 ===+=+== XPXPXPXP
.
Suy ra
3
1
=
m
.
Mt khác
( ) ( ) ( ) ( )
2
1
6
1
.33214 ===+=+==< XPXPXPXP
( ) ( ) ( ) ( ) ( )
2
1
3
2
6
1
.443214 ===+=+=+== XPXPXPXPXP
.
Suy ra
4
2
=
m
.
Khi đó,
(
)
mXMed
=
,
[
]
4;3
m
.
3
=
m
hoc
4
=
m
.
c) Moment trung tâm. Moment g
c.
Định nghĩa.
Cho
X
mt đại lượng ngu nhiên vng
(
)
aXE
=
. Ta gi moment trung tâm
cp
k
ca
X
(
)
(
)
k
kk
aXEX
==
µµ
.
Ta gi moment gc cp
k
(
)
k
k
XE=
γ
.
Ta có
a
=
1
γ
. Theo công thc nh thc Newton
( ) ( ) ( )
( )
( )
=
=
=
=
=
==
n
k
k
kn
k
k
n
n
k
kn
k
k
n
n
k
kn
k
k
n
n
n
C
XEaCXaCEaXE
0
1
00
1
γγ
µ
Vy
( )
=
=
n
k
k
kn
k
k
nn
C
0
1
1
γγµ
.
Ví d 4.9.
Đại lượng ngu nhiên
X
có bng phân phi xác sut như sau:
X
2
3
4
6
7
(
)
XP
1,0
2,0
3,0
2,0
2,0
Tính
(
)
XE
,
(
)
XD
,
(
)
X
σ
,
(
)
3
XE
,
(
)
XMed
,
(
)
XMod
,
(
)
2
<
EXXP
.
Gii
Chương II. Đại lượng ngu nhiên. Hàm phân phi xác sut
41
(
)
6.42,0.72,0.63,0.42,0.31,0.2
=
+
+
+
+
=
XE
.
(
)
242,0.72,0.63,0.42,0.31,0.2
222222
=++++=XE
.
(
)
(
)
(
)
84,26,424
222
=== XEXEXD
.
(
)
(
)
685,184,2
===
XDX
σ
.
(
)
2,1372,0.72,0.63,0.42,0.31,0.2
333333
=++++=XE
.
D thy
(
)
4
=
XMed
( )
2
1
3,04 =<XP
( )
2
1
6,04 =XP
.
(
)
4
=
XMod
(
)
(
)
3,04max
=
=
=
=
XPXXP
i
.
(
)
(
)
( ) ( ) ( ) ( )
7,02,03,02,06436,66,2
26,42
=++=++=<<=
<=<
PPPXP
XPEXXP
5. Hàm ca mt đại lượng ngu nhiên.
Nếu ta xác định
(
)
XgZ
=
mt hàm ca đại lượng ngu nhiên
X
thì
Z
tr thành đại
lượng ngu nhiên mi. Vn đề đặt ra tìm cách xác đnh lut phân phi ca
Z
qua
lut phân phi đã biết ca
X
. đây, ta ch xét các trường hp đơn gin khi m
g
không quá phc tp.
5.1. Đại lượng ngu nhiên ri rc.
Ví d 5.1.
Cho đi lượng ngu nhiên
X
lut phân phi
X
2
1
0
1
2
(
)
1
xXP
=
1,0
2,0
3,0
2,0
2,0
Xác định lut phân phi ca đại lượng ngu nhiên
2
X
Z
=
và tìm kì vng ca
Z
.
Gii.
D dàng ta có
(
)
(
)
3,000
=
=
=
=
XPZP ,
(
)
(
)
(
)
4,02,02,0111
=
+
=
=
+
=
=
=
XPXPZP
,
(
)
(
)
(
)
3,02,01,0224
=
+
=
=
+
=
=
=
XPXPZP .
Khi đó, ta có bng phân phi ca đại lưng ngu nhiên
Z
Z
0
1
4
(
)
i
zZP
=
3,0
4,0
3,0
T bng phân phi trên, ta có kì vng
( ) ( )
6,13,0.44,0.13,0.0
3
1
=++===
=i
ii
zZPzZE
Bài ging
42
Trong trường hp
(
)
XgZ
=
tng quát, ta th tính trc tiếp vng ca đại lượng
ngu nhiên
Z
như sau:
( ) ( ) ( )
=
==
1i
ii
xXPxgZE
Trong d trên, ta th tính vng ca đại lượng ngu nhiên
Z
(
)
(
)
(
)
(
)
(
)
(
)
6,12,0.22,0.13,0.02,0.11,0.2
22222
=++++=ZE
.
6.2. Đại lượng ngu nhiên liên tc.
Khi
X
đại lượng ngu nhiên liên tc, vn đề s phc tp hơn. Gi s đại lượng ngu
nhiên
X
m mt độ
(
)
xf
X
đã biết
(
)
XgY
=
. Ta s tìm hàm mt độ
(
)
xf
Y
ca
Y
.
Ta có:
( ) ( ) ( )( ) ( )
=<==<=
X
D
XY
duufxXgYPxYPxF
, trong đó
xugD
X
<
=
)(
Sau đó, ly đạo hàm
(
)
xF
Y
vế, ta đưc mt độ
(
)
xf
Y
ca đại lượng ngu nhiên
Y
.
Ví d 5.2.
Cho đại lượng ngu nhiên
X
có hàm mt độ xác sut là
(
)
xf
. Tìm hàm mt độ ca
a)
1
2
+
=
X
Z
.
b)
3
X
Y
=
.
Gii.
a) Áp dng công thc, ta có:
( ) ( ) ( )
=
<=<+=<=
2
1
2
1
12
x
F
x
XPxXPxZPxF
XZ
.
Ly đạo hàm, ta được
( ) ( )
[ ]
=
==
2
1
2
1
'
2
1
,
2
1
'
x
f
xx
FxFxf
XZZ
.
Vy hàm mt độ ca
Z
( )
=
2
1
2
1 x
fxf
Z
.
b)
(
)
(
)
(
)
(
)
(
)
33
3
xFxXPxXPxYPxF
XY
=<=<=<=
.
Ly đạo hàm, ta được hàm mt độ ca
Y
là:
( ) ( )
[ ]
(
)
[
]
(
)
(
)
3
2
333
3
1
.''.'
x
xfxxFxFxf
XYY
===
Ví d 5.3.
Cho đại lượng ngu nhiên
X
phân phi chun
(
)
2
;~
σµ
NX
, đặt
σ
µ
=
X
Y
.
Chng minh rng
Y
có phân phi chun
(
)
1;0~ NY
.
Gii.
Chương II. Đại lượng ngu nhiên. Hàm phân phi xác sut
43
( ) ( ) ( ) ( )
µσµσ
σ
µ
+=+<=
<
=<= xFxXPx
X
PxYPxF
XY
.
Ly đạo hàm, ta được
( ) ( )
[ ]
( )
[ ]
( )
( )( )
22
2
2
2
2
1
2
1
.''
x
x
XXYY
eexfxFxFxf
+
==+=+==
π
σ
πσ
σµσµσ
σ
µµσ
.
Vy
Y
có phân phi chun
(
)
1;0~ NY
.
Chương II. Đại lượng ngu nhiên. Hàm phân phi xác sut
45
6. Bài tp chương.
1.
M
t nhóm có
10
ng
ườ
i g
m có
6
nam và
4
n
. Ch
n ng
u nhiên ra
3
ng
ườ
i. G
i
X
là s
n
trong nhóm. L
p b
ng phân ph
i xác su
t c
a
X
và tính
(
)
XE
,
(
)
XD
(
)
X
mod
.
2.
Cho
Đ
LNN liên t
c
X
có hàm m
t
độ
( )
( )
[ ]
[ ]
=
2;0,0
2;0,2
4
3
x
xxx
xf
.
a)
V
đồ
th
c
a
(
)
xf
.
b)
Tính
(
)
5,1
>
XP
(
)
1,19,0
<
<
XP
.
3.
Cho
Đ
LNN liên t
c
X
có hàm m
t
độ
( )
[
]
[ ]
=
3;0,0
3;0,
2
x
xkx
xf
.
a)
Tìm h
ng s
k
.
b)
Tính
(
)
2
>
XP
.
c)
Tìm
(
)
XMed
.
4.
Cho hàm m
t
độ
c
a
Đ
LNN
X
( )
>>
=
0,0
0,0,
1
x
xe
xf
x
λ
λ
λ
a)
Tìm hàm phân ph
i c
a
X
và tính xác su
t
(
)
λ
<
XP
0 .
b)
Tính kì v
ng và ph
ươ
ng sai c
a
X
.
5.
M
t ng
ườ
i nuôi 100 con gà. Xác su
t
để
m
i con gà
đẻ
trong m
t ngày là 8,0
=
p
. G
i
X
là s
tr
ng thu
đượ
c trong m
t ngày.
a)
Tính xác su
t
để
thu
đượ
c ít nh
t 80 qu
tr
ng trong m
t ngày.
b)
Gi
s
, giá bán m
i qu
tr
ng gà 2000 VN
Đ
và chi phí cho m
i con là 200.1 VN
Đ
.
G
i
Y
là s
ti
n l
i trong m
t ngày. Tính ti
n l
i trung bình?
6.
M
t h
p
đự
ng 7 s
n ph
m x
u và 3 s
n ph
m t
t. Ch
n ng
u nhiên cùng lúc
2
s
n
ph
m. G
i
X
là s
s
n ph
m t
t trong hai s
n ph
m l
y ra.
a)
L
p b
ng phân ph
i xác su
t c
a
X
.
b)
Tính
(
)
XE
,
(
)
XD
(
)
XMod
.
7.
Cho
Đ
LNN
X
r
i r
c và có phân ph
i xác su
t nh
ư
sau
X
1 3 5 7 9
P
0,1 0,4 0,2 0,2 0,1
a)
Tính
(
)
73
XP
.
b)
Xác
đị
nh
(
)
XMed
,
(
)
XMod
,
(
)
XE
(
)
XD
Đáp s và hướng dn.
1.
Dùng các công th
c:
X
0
1
2
3
P
30
5
30
15
30
9
30
1
Bài ging
46
(
)
2,1
=
XE
,
(
)
56,0
=
XD
(
)
1mod
=
X
2.
b)
(
)
15625,05,1
>
XP
,
(
)
1495,01,19,0
<
<
XP
.
3.
a)
9
1
=k
, b)
( )
27
19
2 =>XP
, c)
( )
>
<
=
3,1
30,
27
0,0
3
x
x
x
x
xF
.
Median
m
nghi
m c
a ph
ươ
ng trình
2
1
27
3
=
x
hay
3
2
3
=x
. V
y
3
2
3
=m
.
4.
a)
( )
>
=
0,1
0,0
xe
x
xF
x
λ
,
(
)
1
10
=< eXP
λ
, b)
(
)
λ
=
XE
,
(
)
2
λ
=XD
.
C. Phương pháp ging dy.
- Thuyết trình, đàm thoi khơi động hot động t giác, tích cc ca sinh viên.
- S dng hình thc trc quan: bng, đồ th, kí hiu,…
- Yêu cu SV đọc bài ging trước khi lên lp.
- Kim tra, đánh giá vic làm bài tp ca SV.
- S dng phương tin dy hc hin đại như Mic, Projector.
- Gi
ng viên g
i bài gi
ng cho sinh viên
đọ
c tr
ướ
c. Gi
ng viên trình bày bài gi
ng trên l
p
theo ph
ươ
ng pháp thuy
ế
t trình h
i
đ
áp. Giao bài t
p cho sinh viên v
nhà làm. Gi
i thi
u m
t
s
tài li
u tham kh
o.
D. Tài liu tham kho
[1] Đu Thế Cp, Xác sut thng kê: thuyết các i tp (Chương 2), NXB
Giáo dc, 2006.
[2] Đinh Văn Gng, Bài tp xác sut và thng kê (Chương 2), NXB Giáo dc, 2007.
[3] PGS. TS. Phm Xuân Kiu, Giáo Trình c sut thng kê (Chương 2), NXB
Giáo dc, 2005.
[4] Đặng Công Hanh, Đng Ngc Dc, Giáo trình Lý thuyết xác sut Thng
toán (Chương 2), trường Đại hc Duy Tân,1996.
Chương III.
Các quy lut phân phi thường gp.
A. Mc tiêu.
- S dng hình thc trc quan: bng, đ th, kí hiu,…
- ng dng Excel cho vic tính c giá tr ca biến ngu nhiên phân phi chun,
phân phi Poisson, phân phi Student, phân phi chi bình phương.
- Yêu cu SV đọc bài ging trước khi lên lp.
- Kim tra, đánh giá vic làm bài tp ca SV.
- S
d
ng ph
ươ
ng ti
n d
y h
c hi
n
đạ
i nh
ư
Mic, Projector.
B. Ni dung.
1. Quy lut phân phi ri rc.
1.1. Phân phi nh thc.
Định nghĩa.
Gi
X
s ln biến c
A
xut hin trong dãy
n
phép th Bernoulli. Khi đó,
X
là
đại lượng ngu nhiên có phân phi nh thc.
Kí hiu
(
)
pnBX
;~
.
Công thc xác sut:
(
)
knkk
n
qpCkXP
==
, trong đó
pq
=
1
.
Các tính cht.
Cho
(
)
pnBX ;~
, ta có
a)
(
)
npXE
=
.
b)
(
)
npqXD
=
.
c)
pnpXqnp
+
mod
.
Chng minh
a) Gi
i
X
“s ln đi lượng c
A
xut hin trong phép th th
(trong y phép
th Bernoulli), ta có bng phân phi ca
i
X
là:
X
0
1
K
Bài ging
48
P
0
p
1
p
K
n
p
trong đó
(
)
knkk
nk
qpCkXPp
=== .
Suy ra
( )
=
=
=
===
n
k
knkk
n
n
k
knkk
n
n
k
k
qpCkqpCkpkXE
100
...
.
Ta có
( )
=
=+
n
k
kknk
n
n
xpCxp
0
. Đạo hàm hai vế theo
x
, ta đưc
( )
=
=+
n
k
kknk
n
n
xpkCxpn
1
1
1
hay
( )
=
=+
n
k
kknk
n
n
xpkCxxpn
1
1
.
Chn
q
x
=
, ta suy ra
=
=
n
k
kknk
n
xpkCnp
1
. Vy
(
)
npXE
=
(đpcm).
b) D dàng chng minh được
( )
( )
nppnnqpCkpkXE
n
k
knkk
n
n
k
k
+===
=
=
2
0
2
0
22
1..
.
Khi đó
(
)
(
)
(
)
(
)
(
)
(
)
npqnpnpnpnppnnXEXEXD ==+==
2
2
2
2
2
1
(đpcm).=
c) Do
(
)
{
}
n
pppXXP
;;;maxmod
10
K
=
=
. Theo Chương 2, ta có:
+ Nếu
q
np
nguyên thì
1mod
+
=
=
qnpXqnp
.
+ Nếu
q
np
không nguyên thì
[
]
[
]
1mod
+
=
<
qnpXqnp
.
Vy
pnpXqnp
+
mod
(đpcm).
Ví d 1.1.
Bn
5
viên đạn vào mc tiêu, xác sut trúng mc tiêu ca mi viên đạn
8,0
. Gi
X
đại lượng ngu nhiên ch s viên đạn trúng mc tiêu. Lp bng phân phi ca
X
.
Tính
(
)
XE
?
Gii
Ta có
X
có th nhn các giá tr
0
,
1
,
2
,
3
,
4
,
5
. Khi đó, ta có:
(
)
kkk
CkXP
==
5
5
2,08,0
,
5;0
=k
.
T đó, ta có bng phân phi
0
1
2
3
4
5
500
5
2,0.8,0C
411
5
2,0.8,0C
322
5
2,0.8,0C
233
5
2,0.8,0C
144
5
2,0.8,0C
055
5
2,0.8,0C
D thy kì vng
(
)
48,0.5
=
=
=
npXE
.
1.2. Phân phi siêu bi.
Định nghĩa.
Chương III. Các quy lut phân phi xác sut thường gp.
49
Gi
X
s ln chn đưc phn t có tính cht
A
trong
n
ln chn không lp t mt
tp hp
N
phn t, trong đó
M
phn t tính cht
A
. Khi đó,
X
được gi
đại lượng ngu nhiên có phân phi siêu bi.
Kí hiu:
(
)
nMNHX ;;~
.
Công thc xác sut:
( )
n
N
kn
MN
k
M
C
CC
kXP
==
.
.
Các tính cht.
Cho
(
)
nMNHX ;;~
. Ta có
a)
(
)
npXE
=
.
b)
( )
1
=
N
nN
npqXD
.
trong đó
N
M
p =
,
pq
=
1
.
Chng minh
Trước hết, ta chng minh công thc
1
.
0
=
=
n
k
n
N
kn
MN
k
M
C
CC
. Tht vy, ta có
(
)
(
)
(
)
NMNM
xxx +=++
111
hay
=
=
=
=
N
k
tt
N
MN
l
ll
MN
M
k
kk
M
xCxCxC
000
So sánh h s ca
n
x
hai vế, ta được
n
N
n
k
kn
MN
k
M
CCC =
=
0
.
(đpcm)
Ta có:
( ) ( )
( )
( )
( )
( ) ( )
( )
( ) ( )
=
=
=
=
=
=
=
=
=
====
n
k
N
nN
kn
MN
k
M
n
k
kn
MN
n
k
kn
MN
n
k
n
N
kn
MN
k
M
n
k
n
N
kn
MN
k
M
n
k
C
CC
np
nNn
N
C
kMk
M
np
nNn
N
nM
C
kMk
M
kN
np
nMC
CkNC
N
M
n
C
CkC
kXkPXE
1
1
1
1
11
000
.
!!1
!1
.
!!.1
!1
!!
!
.
.
!!.
!
..
Chú ý rng:
1
.
.
1
1
1
1
11
11
1
1
01
1
1
1
==
=
=
n
k
n
N
kn
MN
k
M
n
k
N
nN
kn
MN
k
M
C
CC
C
CC
, (trong đó
1
1
=
kk
,
1
1
=
MM
,
1
1
=
NN
1
1
=
nn
).
Vy ta có điu phi chng minh.
b) Ta có.
Tương t như câu a), ta d dàng chng minh được:
Bài ging
50
( )
( )
( ) ( ) ( )
( ) ( )
( )
( )
( )
11
11
1
00
0
22
+
=+
=
=+==
==
==
=
NN
NnMnMMn
n
N
M
NN
nnMM
kXkPkXPkk
kXPkXE
n
k
n
k
n
k
Khi đó
(
)
(
)
(
)
(
)
( )
11
...
1
2
2
=
=
+
=
=
N
nN
npq
N
nN
N
MN
N
M
n
N
Mn
N
NnMnM
N
Mn
XEXEXD
Ví d 1.2.
Mt hp
4
viên bi đỏ
3
viên bi xanh. Ly ngu nhiên
3
bi t hp. Gi
X
s
bi xanh ly được.
a) Tính xác sut ly được
2
bi xanh.
b) Lp bng phân phi xác ca
X
. T đó tính kì vng và phương sai.
Gii
a) Ta có
7
=
N
,
3
=
M
(
)
3;3;7~ HX
. Khi đó
( )
3
7
1
4
2
3
.
2
C
CC
XP ==
.
b) D thy
X
có th nhn các giá tr
0
,
1
,
2
,
3
. Khi đó, ta có bng phân phi xác sut
như sau:
X
0
1
2
3
P
3
7
3
4
0
3
.
C
CC
3
7
2
4
1
3
.
C
CC
3
7
1
4
2
3
.
C
CC
3
7
0
4
3
3
.
C
CC
Do
(
)
3;3;7~ HX
nên
( )
7
9
7
3
.3 ==XE
( )
49
24
17
37
7
3
1.
7
3
.3 =
=XD
.
1.3. Phân phi Poisson.
Định nghĩa.
Gi
X
s ln phn t nh cht
A
xut hin trong mt khong thi gian (hoc
trên mt min, mt ng) nào đó. Khi đó,
X
được gi đi lượng ngu nhiên có
phân phi Poisson vi tham s
λ
s trung bình ca s ln phn t tính cht
A
xy ra.
Kí hiu:
(
)
λ
PX ~
.
Công thc xác sut:
( )
!
k
e
kXP
k
λ
λ
==
.
Chương III. Các quy lut phân phi xác sut thường gp.
51
Các tính cht.
Cho
(
)
λ
PX ~
. Ta có
a)
(
)
λ
=
XE
.
b)
(
)
λ
=
XD
.
c)
[
]
(
)
[
]
λ
λ
Xmod1
.
Chng minh
a) Ta :
( ) ( )
( )
λλ
λ
λ
λ
λλλ
λ
==
====
+∞
=
+∞
=
+∞
=
ee
k
e
k
ke
kXkPXE
k
k
k
k
k
1
1
00
!1!
, (do
λ
λ
e
l
l
l
=
+∞
=
0
!
).
b) Ta
( )
( )
(
)
+∞
=
+∞
=
+∞
=
+∞
=
+
====
000
2
0
22
!!
1
!
k
k
k
k
k
k
k
k
ke
k
ekk
k
ek
kXPkXE
λλλ
λλλ
.
D thy
λ
λ
λ
=
+∞
=
0
!
k
k
k
ke
(
)
( )
2
2
2
2
0
!2!
1
λ
λ
λ
λ
λ
λ
=
=
+∞
=
+∞
=
k
k
k
k
k
e
k
ekk
nên
(
)
λλ
+=
22
XE
. Vy
(
)
(
)
(
)
(
)
λ
==
2
2
XEXEXD
(đpcm).
c) Ta
( )
!
k
e
kXP
k
λ
λ
==
( )
( )
!1
1
1
+
=+=
+
k
e
kXP
k
λ
λ
.
D thy
(
)
(
)
kXPkXP
=
+
=
1
khi và ch khi
1
λ
k
(
)
(
)
kXPkXP
=
<
+
=
1
khi và ch khi
1
>
λ
k
.
Do
Nk
nên
[
]
(
)
[
]
λ
λ
Xmod1
(đpcm)
Chú ý.
Lut phân phi Poisson có ý nghĩa thc tế rt ln và được ng dng rng rãi trong vic
kim tra cht lượng sn phm. Đặc bit gii quyết mt s bài toán sau đây
Ví d 1.3.
Ti mt CLB Bóng bàn, biết rng trung bình mi ngày
5
người đến tp luyn. Tính
xác sut để trong mt ngày mà ta xét.
a) Có
3
người đến tp luyn.
b) Có ít nht
4
ngưi đến tp luyn.
Gii
Gi
X
là s người đến tp luyn trong ngày. Ta có
(
)
5~ PX
. Khi đó
a)
( )
!
3
5
3
35
==
e
XP
.
b)
( ) ( )
=
=<=
3
0
9
!
9
1414
k
k
k
e
XPXP
.
Bài ging
52
Ví d 1.4.
Xét s khách hàng vào ca hàng mua ĐTDĐ trong mt tháng đi lưng ngu nhiên
tuân theo phân phi Poisson vi mt độ trung bình
9
khách hàng trong mt ngày.
a) Tìm xác sut để trong mt ngày
40
khách hàng.
b) Tìm xác sut để trong mt tun
100
khách hàng.
c) Tìm xác sut để trong mt ngày có hơn
40
khách hàng.
Gii
Gi
X
là s khách hàng vào ca hàng mua ĐTDĐ.
a) Ta có
(
)
9
=
XE
. Khi đó
( )
!
40
9
4
409
==
e
XP
.
b) S khách hàng trung bình vào ca hàng mua ĐTDĐ trong mt tun
367.9
=
.
(
)
36
=
YE
.
Khi đó, ta có
( )
!
100
36
100
10036
==
e
YP
.
c) Ta có
( ) ( )
=
==>
40
0
9
!
9
140140
k
k
k
e
XPXP
.
2. Quy lut phân phi liên tc.
2.1. Phân phi đều.
Định nghĩa.
Đại lượng ngu nhiên liên tc
X
được gi phân phi đều trên đon
[
]
ba;
nếu
hàm mt độ ca
X
( )
[ ]
[ ]
=
bax
bax
ab
xf
;,0
;,
1
.
Kí hiu:
(
)
baUX ;~
.
Các tính cht.
Cho
(
)
baUX ;~
. Ta có:
a)
( )
2
ab
XE
+
=
.
b)
( )
(
)
12
2
ab
XD
=
.
2.2. Phân phi mũ.
Định nghĩa.
Chương III. Các quy lut phân phi xác sut thường gp.
53
Đại lượng ngu nhiên liên tc
X
được gi có phân phi mũ tham s
λ
(
0
>
λ
)
hàm mt độ ca nó có dng
( )
<
=
0,0
0,
x
xe
xf
x
λ
λ
.
Kí hiu:
(
)
λ
EX ~
.
Các tính cht.
Cho
(
)
λ
EX ~
. Ta có:
a)
( )
λ
1
=XE
.
b)
( )
2
1
λ
=XD
.
Chng minh
a) Ta có:
( )
+∞
=
0
dxexXE
x
λ
λ
Đặt
xt
λ
=
. Khi đó
( )
(
)
(
)
λλλλ
11.121
0
=
Γ
=
Γ
==
+∞
duteXE
t
b) Ta có:
( )
+∞
=
0
22
dxexXE
x
λ
λ
.
Đặt
xu
λ
=
. Khi đó
( )
(
)
(
)
222
0
2
2
2
21.231
λλλλ
=
Γ
=
Γ
==
+∞
dtetXE
t
.
Suy ra
( )
(
)
( )( )
2
2
2
1
λ
== XEXEXD
.
Trong đó, Hàm Gamma được xác định như sau
( )
+∞
=Γ
0
1
dxxe
x
α
α
.
Các tính cht ca hàm Gamma:
a)
(
)
11
=
Γ
.
b)
(
)
!1 nn
=
+
Γ
,
Nn
.
c)
π
=
Γ
2
1
.
Ví d 2.1.
Tui th
X
(tính bng gi) ca mt thiết b có phân phi mũ
(
)
00125,0~ EX
.
Hãy tính và đưa ra ý nghĩa các giá tr sau:
a)
(
)
720
>
XP
.
b)
(
)
720600
<
<
XP
.
Bài ging
54
c)
(
)
XE
.
d)
(
)
800
<
XP
.
Gii
a) Ta có
(
)
(
)
(
)
(
)
(
)
9,0720.00125,0
1172017207201720
====<=> eeFXPXPXP
Ý nghĩa
(
)
720
>
XP
là xác sut để tui th ca thiết b ln hơn
720
gi
9,0
e
.
b)
(
)
(
)
(
)
(
)
(
)
9,075,0600.00125,0720.00125,0
11600720720600
===<< eeeeFFXP
.
Ý nghĩa
(
)
720600
<
<
XP
xác sut để tui th ca thiết b nm trong khong
(
)
720;600
9,075,0
ee
.
c)
( )
00125,0
1
=
XE
.
d)
( ) ( )
e
eFXP
1
11800800
800.00125,0
===<
.
2.3. Phân phi chun. Phân phi chun tc.
Định nghĩa.
Đại lượng ngu nhiên liên tc
X
được gi phân phi chun vi vng
µ
,
phương sai
2
σ
nếu hàm mt độ ca nó có dng
( )
( )
2
2
2
2
1
σ
µ
πσ
=
x
exf
.
Kí hiu
(
)
2
;~
σµ
NX
.
Chú ý.
Chương III. Các quy lut phân phi xác sut thường gp.
55
Nếu
0
=
µ
1
=
σ
thì
(
)
1;0~ NX
, ta nói
X
phân phi chun tc.
Tính cht.
Cho
(
)
2
;~
σµ
NX
. Ta có:
a)
(
)
µ
=
XE
.
b)
(
)
2
σ
=XD
.
Hàm Gauss.
Đó là hàm
( )
=Φ
x
t
dtex
0
2
2
2
1
π
, hay còn gi là tích phân Laplace, trong đó:
+
( )
2
2
2
1
x
exf
=
π
gi hàm mt độ Gauss.
+
( )
=
x
t
dtexF
2
2
2
1
π
gi là hàm phân phi xác sut Gauss.
D thy
(
)
xf
(
)
xF
cũng hàm mt đ hàm phân phi xác sut ca đại lượng
ngu nhiên
(
)
1;0~ NX
.
Nhn xét:
a)
(
)
(
)
xx
Φ
=
Φ
,
Rx
.
b)
( )
2
1
lim =Φ
+∞
x
x
, do
2
2
0
2
2
π
=
+∞
dte
t
.
c)
( )
2
2
2
1
x
exf
=
π
hàm s chn nên có đồ th nhn
Oy
làm trc đối xng.
Định lí.
Cho
(
)
xF
là hàm phân phi xác sut ca đại lượng ngu nhiên
(
)
1;0~ NX
. Ta có
a)
( ) ( )
xxF Φ+=
2
1
.
b)
(
)
(
)
(
)
(
)
(
)
α
β
α
β
β
α
Φ
Φ
=
=
<
<
FFXP
.
c)
(
)
(
)
αα
Φ=<
2XP
,
0
>
α
.
Chng minh.
a) D thy
( ) ( )
xdtedtedtexF
x
tt
x
t
Φ+=+==
2
1
2
1
2
1
2
1
0
2
0
22
222
πππ
b) Ta
Bài ging
56
( ) ( ) ( ) ( )
( ) ( ) ( ) ( )
αβ
βα
β
α
β
α
β
α
ΦΦ=+=
+==<<
00
0
0
dxxfdxxf
dxxfdxxfdxxfXP
c) Ta có
(
)
(
)
(
)
(
)
(
)
(
)
(
)
αααααααα
Φ=ΦΦ==<<=<
2FFXPXP
.
Tính cht.
Cho
(
)
2
;~
σµ
NX
. Ta
a)
( )
Φ+=
σ
µ
x
xF
2
1
.
b)
( )
Φ
Φ=
=<<
σ
µα
σ
µβ
σ
µα
σ
µβ
βα
FFXP
.
c)
( )
Φ=<
σ
α
αµ
2
XP
,
0
>
α
.
Tính cht.
Cho
(
)
1;0~
NZ
. Gi
α
z
là s tha mãn
(
)
α
α
=
>
zZP
(
10
α
). Ta có
a)
αα
zz
=
1
, trong đó
α
1
z
là s tha mãn
(
)
α
α
=
>
1
1
zZP
b)
(
)
α
α
2
=>
zZP
(vi
2
1
0
α
)
Chú ý.
Giá tr hàm
( )
=Φ
x
t
dtex
0
2
2
2
1
π
đưc cho trong Bng 2. Chng hn
(
)
475,096,1
=
Φ
. Ta
quy ước
(
)
5,0
=
Φ
m
vi mi
4
m
.
H qu. (Quy tc
k
-sigma).
Nếu
(
)
2
;~
σµ
NX
thì
(
)
(
)
kkXP
Φ=<
2
σµ
Vi
3
=
k
, ta quy tc
3
-sigma
(
)
(
)
9973,0323
=Φ=<
σµ
XP
. Quy tc này
nghĩa sai s gia
X
µ
không quá
σ
3
gn chc chn. Khi đó, vi xác sut
9973,0
giá tr ca đại lượng ngu nhiên
X
nm trong khong
(
)
σ
µ
σ
µ
3;3
+
.
Ví d 2.2.
Cho đại lượng ngu nhiên
Z
phân phi chun tc
(
)
1;0
N
. Tìm din tích phn nm
bên dưới đường cong chun tc này.
a) bên phi đường thng
84,1
=
z
.
b) gia hai đường thng
97,1
=
z
86,0
=
z
.
Chương III. Các quy lut phân phi xác sut thường gp.
57
Gii
a) Ta có din tích bng
( ) ( )
033,0467,0
2
1
84,1
2
1
84,1 ==Φ=>ZP
.
b) Ta có din tích bng
(
)
(
)
(
)
(
)
(
)
781,0476,0305,097,186,097,186,086,097,1
=
+
=
Φ
+
Φ
=
Φ
Φ
=
<
<
ZP
Ví d 2.3.
Cho đại lưng ngu nhiên
Z
phân phi chun tc
(
)
1;0~ NZ
. Da vào hình v sau,
hãy tìm giá tr
k
sao cho
a)
(
)
3015,0
=
>
kZP
.
b)
(
)
4197,018,0
=
<
<
ZkP
.
Gii
a) Da vào hình v, ta có:
( ) ( )
kkZP Φ==>
2
1
3015,0
.
Khi đó
(
)
1985,0
=
Φ
k
. T Bng 2, ta suy ra
52,0
=
k
.
b) Ta
(
)
(
)
(
)
(
)
(
)
kkZkP
Φ
Φ
=
Φ
Φ
=
=
<
<
18,018,04197,018,0
Khi đó,
(
)
(
)
4197,018,0
Φ
=
Φ
k
. T Bng 2, ta có
(
)
0714,018,0
=
Φ
.
Suy ra
(
)
(
)
(
)
37,237,24911,04197,00714,0
Φ
=
Φ
=
=
=
Φ
k
.
Bài ging
58
Vy
37,2
=
k
.
Ví d 2.4.
Cho đại lưng ngu nhiên ngu nhiên
X
phân phi chun
(
)
2
10;50~
NX
. Tìm xác
sut để
X
nhn các giá tr trong khong
(
)
62;45
.
Gii
Ta có xác sut cn tìm
( ) ( ) ( ) ( ) ( )
5,02,105,02,1
10
5045
10
5062
6245
Φ+Φ=ΦΦ=
Φ
Φ=<< XP
Da vào Bng 2, ta tính được
(
)
3849,02,1
=
Φ
,
(
)
1915,05,0
=
Φ
.
Vy
(
)
5764,01915,03849,06245
=
+
=
<
<
XP
.
Ví d 2.5.
Cho đại lượng ngu ngu nhiên có phân phi
(
)
2
5,0;1
N
. Hãy tìm các xác sut sau:
a)
(
)
213,15
<
XP
.
b)
(
)
64,01
<
XP
.
c)
(
)
1,2
<
XP
.
d)
(
)
3,2
>
XP
.
Gii
a) Ta có
( ) ( ) ( )
( ) ( ) ( ) ( )
664,6
2
1
664,14426,012426,0
12426,0
5,0
15
5,0
1213,1
213,15
=+Φ+ΦΦ+Φ=
ΦΦ=
Φ
Φ=< XP
b)
( )
( )
79946,039973,0.228,12
5,0
64,0
264,01
==Φ=
Φ=<XP
.
c)
( ) ( )
9861,04861,0
2
1
2,2
2
1
5,0
11,2
2
1
1,2
=+=Φ+=
Φ+=<XP
.
d)
( ) ( )
( )
00466,049534,0
2
1
6,2
2
1
5,0
13,2
2
1
13,213,2
==Φ=
Φ+==> XPXP
Ví d 2.6.
Đường kính ca mt loi chi tiết do mt máy sn xut phân phi chun, vng
mm
20
và có phương sai
(
)
2
2,0 mm
. Tính xác sut ly ngu nhiên mt chi tiết
Chương III. Các quy lut phân phi xác sut thường gp.
59
a) Có đường kính trong khong
mm
9,19
đến
mm
3,20
.
b) Có đường kính sai khác vi kì vng không quá
mm
3,0
.
Gii
Gi
X
đường kính ca mt chi tiết, ta có
(
)
(
)
2
2,0;20~ NX
. Khi đó
a) Ta
( )
( ) ( ) ( ) ( )
6247,01915,04332,05,05,15,05,1
2,0
209,19
2,0
203,20
3,209,19
=+=Φ+Φ=ΦΦ=
Φ
Φ=<< XP
b) Áp dng công thc, ta suy ra
( )
( )
8664,04332,0.25,12
2,0
3,0
23,0
==Φ=
Φ=<
µ
XP
.
Ví d 2.7.
Gi
X
ch s thông minh
IQ
(Intelligent Quota) ca hc sinh la tui 12-15. Gi
s
(
)
25;85~
NX
.
a) Cho biết ch s
IQ
trung bình ca hc sinh là bao nhiêu?
b) Tính xác sut chn được hc sinh rt thông minh (
90
X
).
c) Tính t l hc sinh có ch s
(
)
95;80
IQ
.
d) Gi
Y
s hc sinh có ch s
(
)
95;80
IQ
trong lp có
50
hc sinh. Hãy ch rõ
phân phi xác sut ca
Y
.
Gii
a) Ch s
IQ
trung bình ca hc sinh là
(
)
85
=
XE
.
b)
( )
1587,05413,011
2
1
5
8590
2
1
)90(
==Φ=
Φ=XP
.
c) T l hc sinh có ch s
(
)
95;80
IQ
(
)
9580
<
<
XP
. Khi đó
( ) ( ) ( ) ( ) ( )
82,08186,03413,04773,0
1212
5
8580
5
8595
9580
=+=
Φ+Φ=ΦΦ=
Φ
Φ=<< XP
d) Mt lp gm
50
hc sinh được chn t tp hp hc sinh vi t l
(
)
82,09580
<
<
=
XPp
được xem
50
phép th vi xác sut
82,0
=
p
. Do đó
Y
phân phi nh thc
(
)
82,0;50
B
, tc là:
(
)
mmm
CmYP
==
50
50
18,0.82,0
,
50;0
=m
.
Ví d 2.8.
Cho đi lượng
X
có phân phi chun
(
)
2
;5~
σ
NX
. Cho biết
(
)
2,09
=
>
XP
. Tính
2
σ
.
Gii
Bài ging
60
Ta có
( )
Φ=
Φ=>
σσ
4
2
159
2
1
9
XP
.
Khi đó
(
)
2,09
=
>
XP
suy ra
3,0
4
=
Φ
σ
hay
85,0
4
=
σ
. Vy
14,22
2
=
σ
.
2.4. Phân phi Chi bình phương.
Định nghĩa.
Đại lượng ngu nhiên
X
được gi phân phi Chi bình phương
(
)
2
χ
vi
bc
t do nếu hàm mt độ ca nó có dng:
( )
>
Γ
=
0,0
0,
2
2
.
2
1
22
x
x
n
xe
xf
n
nx
.
Kí hiu
(
)
nX
2
~
χ
.
Các tính cht.
+ Nếu dãy các đại lưng ngu nhiên độc lp
1
X
,
2
X
,
K
,
n
X
phân phi chun
tc thì
22
2
2
1 n
XXXX +++=
L
có phân phi Chi bình phương
(
)
2
χ
vi
n
bc t do.
+ Cho
(
)
nX
2
~
χ
. Ta có:
a)
(
)
nXE
=
.
b)
(
)
nXD 2
=
.
Chng minh
a) Ta có:
( ) ( )
+
+
+
Γ
=
Γ
==
0
22
2
0
2
2
1
2
2
2
1
2
2
dxex
n
dx
n
ex
xdxxxfXE
xn
nn
xn
Đặt:
2
x
t =
. Khi đó
( )
n
n
nn
n
n
dtet
n
XE
t
n
=
Γ
Γ
=
+Γ
Γ
=
Γ
=
+
2
2
.
2
21
2
.2.
2
1
.2.
2
1
0
2
.
b) Tương t, ta chng minh được
( )
( ) ( )
2
2
2
1
0
2
1
2
2
22
+=
Γ
==
+∞
+
+∞
nndxex
n
dxxfxXE
xn
n
.
Khi đó
(
)
(
)
(
)
nXEXEXD 2
22
==
(đpcm)
Chương III. Các quy lut phân phi xác sut thường gp.
61
2.5. Phân phi Student.
Định nghĩa.
Đại lưng ngu nhiên liên tc
X
được gi phân phi Student vi
bc t do
nếu hàm mt độ ca
X
có dng:
( )
2
1
2
1
2
2
1
.
1
+
+
Γ
+
Γ
=
n
n
x
n
n
n
xf
π
Kí hiu:
(
)
nTX ~
.
Các tính cht.
+
( )
2
1
2
1
2
2
1
.
1
+
+
Γ
+
Γ
=
n
n
x
n
n
n
xf
π
hàm s chn nên đồ th nhn
Oy
làm trc đối
xng.
+
(
)
1;0~ NX
,
(
)
nY
2
~
χ
X
,
Y
độc lp thì
n
Y
X
T =
có phân phi
(
)
nT
, vi
bc t
do.
+ Cho
(
)
nTX ~
. Ta có:
a)
(
)
0
=
XE
.
b)
( )
2
=
n
n
XD
.
2.6. Công thc tính gn đúng.
2.6.1. Phân ph
i siêu b
i và phân ph
i nh
th
c.
Định lí.
Cho
(
)
nMNHX ;;~
. Nếu
N
khá ln (
nN 10
>
)
N
M
p =
thì ta th coi
N
M
nBX ;~
.
Tc là ta có công thc tính gn đúng
knk
k
n
n
N
kn
MN
k
M
N
M
N
M
C
C
CC
1
.
.
2.6.2. Phân ph
i nh
th
c và phân ph
i Poisson.
Định lí.
Cho
(
)
pnBX ;~
. Nếu
p
khá (gn
0
, ta xem
1,0
<
p
) và khi
n
khá ln (
30
n
) thì
nk ;0=
, ta có th coi
(
)
npPX ~
.
Bài ging
62
Tc là ta có công thc gn đúng
( ) ( )
(
)
!
1
k
np
eppCkXP
k
np
kn
kk
n
==
,
nk ;0=
.
Nhn xét.
Cho
(
)
pnBX ;~
. Nếu
p
khá ln (gn
1
) và
n
khá ln t
nk ;0=
, ta có thng phân
phi Poisson để tính gn đúng.
Tht vy,
(
)
pnBX ;~
thì
(
)
pnBY
1;~
, trong đó
X
s ln biến c
A
xut hin
Y
là s ln biến c
A
xut hin.
Do
p
khá ln nên
p
1
khá bé. Do dó
(
)
(
)
pnPY
1~
.
Khi đó
( ) ( )
(
)
(
)
[
]
( )
!
1
1
kn
pne
knYPkXP
kn
pn
====
.
Ví d 2.9.
Mt ca hàng sn xut đĩa nhc, trung bình sn xut
1000
đĩa thì
1
đĩa hng. Tìm
xác sut để khi hãng đó sn xut
3000
đĩa thì có nhiu hơn
5
đĩa không b hng.
Gii
Xác sut để được đĩa không hng trong
1000
đĩa là
1000
999
=p
.
Gi
X
s đĩa không b hng. Ta
1000
999
;3000~ BX
, ta có
3000
=
n
1000
999
=p
khá ln.
Suy ra
(
)
λ
PY ~
vi
3
1000
1
.3000 ==
λ
.
Do
XY
=
3000
nên ta
( ) ( ) ( )
92,01008,01494,00498,055
5
0
=+++====>
=
L
k
kYPYPXP
.
2.6.3. Phân ph
i nh
th
c và phân ph
i chu
n.
Định lí.
Cho
(
)
pnBX ;~
. Nếu
p
không qgn
0
1
, khi đó
khá ln (
30
n
,
10
np
) thì
ta có th coi
(
)
npqnpNX ;~
.
Tc là ta có công thc gn đúng
( ) ( )
==
npq
npk
f
npq
qpCkXP
kn
kk
n
1
1
,
nk ;0=
,
trong đó:
( )
2
2
2
1
x
exf
=
π
.
Ví d 2.10.
Biến c A: mt anh B u mt gái” có xác sut
(
)
25,0
=
=
pAP
không đổi. Tìm
xác sut để khi anh B quen vi 243 người cô gái thì có đúng 70 ln biến c A xy ra.
Chương III. Các quy lut phân phi xác sut thường gp.
63
Gii.
Chú ý rng, anh B quen vi 243 người là mt phép th độc lp.
Gi X là s ln biến c A xy ra trong 243 phép th độc lp. Ta có
(
)
25,0;243~ BX
.
Do
30243
>
=
n
1025,0.243
>
=
np
nên ta xem
(
)
npqnpX ;~
vi
25,0.243
=
np
75,0.25,0.243
=
npq
.
Vy
( ) ( )
0231,037,1
75,6
1
75,0.25,0.243
25,0.24370
75,0.25,0.243
1
70 ==
= ffXP
.
3. Đại lượng ngu nhiên nhiu chiu.
3.1. Khái nim.
các phn đã hc, chúng ta đã xét các đại lượng ngu nhiên các giá tr th ca
chúng được biu din bng mt s. Các đi lượng ngu nhiên đó được gi đại lượng
ngu nhiên mt chiu. Ngoài các đại lượng ngu nhiên mt chiu, trong thc tế ta n
gp các đại lượng ngu nhiên các giá tr th ca nó được xác định bng
2
,
3
,
K
,
n
s. Nhng đại lượng ngu nhiên này đưc gi các đại lượng ngu nhiên
hai chiu, ba chiu,
K
,
chiu.
Xét đi lượng ngu nhiên hai chiu, hiu
(
)
YX ;
. Trong đó,
X
Y
được gi là
các thành phn ca đại lưng ngu nhiên hai chiu. Hai đại lượng ngu nhiên
X
Y
được xét đồng thi to nên h hai đại lượng ngu nhiên. Tương t như vy, đi lưng
ngu nhiên
chiu có th xem là h ca
đại lượng ngu nhiên.
Ví d 3.1.
Mt máy sn xut mt loi sn phm. Nếu kích thước ca sn phm được đo bng
chiu dài
X
chiu rng
Y
, thì ta đi lượng ngu nhiên hai chiu
(
)
YX ;
, còn nếu
tính thêm c chiu cao
Z
na thì ta có đại lượng ngu nhiên ba chiu
(
)
ZYX ;;
.
Trong thc tế, người ta cũng chia các đại lượng ngu nhiên nhiu chiu thành hai loi:
ri rc và liên tc.
Các đại lượng ngu nhiên nhiu chiu được gi ri rc nếu các thành phn ca nó là
đại lưng ngu nhiên ri rc.
Các đại lượng ngu nhiên nhiu chiu được gi liên tc nếu các thành phn ca
đại lượng ngu nhiên liên tc.
Sau đây, ta xét các đại lượng ngu nhiên hai chiu.
3.2. Quy lut phân phi xác sut ca đại lượng ngu nhiên hai chiu.
Đối vi đại lượng ngu nhiên hai chiu, ngưi ta cũng dùng bng phân phi xác sut,
hàm phân phi xác sut, hàm mt độ xác sut để thiết lp quy lut phân phi xác sut
ca chúng.
* Bng phân phi xác sut ca đại lượng ngu nhiên hai chiu.
Bng phân phi xác sut ca đại lượng ngu nhiên hai chiu
(
)
YX ;
ri rc là
Bài ging
64
Y
X
1
y
2
y
h
y
j
1
x
11
p
12
p
h
p
1
(
)
11
xp
2
x
21
p
22
p
h
p
2
(
)
21
xp
M
M
M
M
M
k
x
1
k
p
2
k
p
kh
p
(
)
k
xp
1
i
(
)
12
yp
(
)
22
yp
(
)
h
yp
2
1
trong đó
(
)
jiij
yYxXPp === ;
xác sut đồng thi để đại lượng
X
ly giá tr
i
x
;
ki ;1=
Y
ly giá tr
j
y
;
hj ;1=
. Bng này có th vô hn khi
k
,
h
nhn giá tr
+
.
Các tính cht.
a) 10
ij
p
b) 1
1 1
=
= =
k
i
h
j
ij
p .
3.3. Hàm phân phi ca đại lượng ngu nhiên hai chiu.
Xét đại lượng ngu nhiên hai chiu
(
)
YX ;
có th ri rc hoc liên tc. Xét
x
,
y
hai
s thc bt kì, khi đó biến c
(
)
yYxX
<
<
;
biến c để
X
nhn giá tr nh hơn
x
,
Y
nhn giá tr nh hơn
y
.
Hàm phân phi xác sut ca đại lượng ngu nhiên hai chiu
(
)
(
)
yYxXPyxF
<
<
=
;;
Các phân phi biên ca đại lượng ngu nhiên hai chiu ri rc
(
)
YX ;
là:
a) Phân phi xác sut ca
X
( )
=
==
k
j
iji
pxXP
1
.
b) Phân phi xác sut ca
Y
( )
=
==
h
i
iji
pyYP
1
.
Ví d 3.2.
Cho bng phân phi ca đại lượng ngu nhiên
2
chiu
(
)
YX ;
như sau
Y
X
1
2
3
1
10,0
25,0
10,0
2
15,0 05,0 35,0
Tìm bng phân phi ca các đại lượng
X
Y
sau đó tính
(
)
3;1,2F
?
Gii.
Ly tng hàng và tng ct tương ng, ta có các phân phi biên như sau
Chương III. Các quy lut phân phi xác sut thường gp.
65
X
1
2
(
)
i
xXP
=
45,0
55,0
Y
1
2
3
(
)
j
yYP =
25,0
30,0
45,0
Ta có:
( )
55,005,015,025,010,03;1,2
22211211
2
1,2
3
3
=+++=+++==
< <
pppppF
i j
x y
ij
.
4. Bài tp chương.
1.
Tung hai con xúc x
c
đồ
ng th
i. G
i
X
là t
ng s
ch
m xu
t hi
n trên hai con xúc x
c
đ
ó. L
p b
ng phân ph
i xác su
t c
a
X
.
2.
M
t
đổ
i tuy
n có 3 v
n
độ
ng viên. Xác su
t thi
đấ
u th
ng tr
n c
a t
ng v
n
độ
ng viên
l
n l
ượ
t là
4,0
;
3,0
;
6,0
. M
i v
n
độ
ng viên thi
đấ
u
độ
c l
p m
t tr
n v
i
độ
i b
n.
a)
Tìm phân ph
i xác su
t s
tr
n th
ng c
a
độ
i tuy
n.
b)
L
p hàm phân ph
i xác su
t s
tr
n th
ng c
a
độ
i tuy
n.
c)
Tìm xác su
t
độ
i tuy
n th
ng ít nh
t m
t tr
n.
3.
Trong m
t h
p có ch
a 3 bi
đỏ
4
bi
đ
en. L
y ng
u nhiên t
ng viên cho
đế
n khi l
y
đượ
c bi
đỏ
thì d
ng. G
i
X
là s
bi c
n l
y. L
p b
ng phân ph
i xác su
t c
a
X
.
4.
Trong m
t h
p có 3 bi
đỏ
4
bi
đ
en. L
y ng
u nhiên t
h
p ra
2
viên. N
ế
u
đượ
c
2
bi
đỏ
thì b
tr
l
i h
p
4
bi
đỏ
, n
ế
u
đượ
c
1
bi
đỏ
thì b
tr
l
i h
p
2
bi
đỏ
, n
ế
u có
2
viên
đề
u
đ
en thì thôi. G
i
X
là bi
ế
n ng
u nhiên ch
s
bi
đỏ
sau khi th
c hi
n phép th
.
L
p b
ng phân ph
i c
a
X
.
5.
M
t h
p
đự
ng
10
s
n ph
m t
t,
2
s
n ph
m x
u. L
y ng
u nhiên t
ng s
n ph
m cho
đế
n khi l
y ra
đượ
c s
n ph
m t
t. Tìm phân ph
i xác su
t s
s
n ph
m
đượ
c l
y ra.
6.
hai h
p bi
I
,
II
. H
p
I
8
bi xanh và
2
bi
đỏ
. H
p
II
7
bi xanh và
2
bi
đỏ
.
T
h
p
I
l
y ng
u nhiên
2
bi b
vào h
p
II
, sau
đ
ó t
h
p
II
l
y ra
2
bi.
a)
Tìm phân ph
i xác su
t s
bi xanh
đượ
c l
y ra.
b)
L
p hàm phân ph
i xác su
t s
bi xanh
đượ
c l
y ra.
7.
Cho
X
là bi
ế
n ng
u nhiên có phân ph
i xác su
t
X
1
2
3
4
5
6
7
P
a
a2
a2
a3
2
a
2
2a
aa +
2
7
a)
Xác
đị
nh
.
b)
Tính
(
)
5
XP
,
(
)
3
<
XP
.
c)
Tìm s
k
nh
nh
t sao cho
( )
2
1
kXP
.
8.
Bi
ế
n ng
u nhiên r
i r
c
X
có b
ng phân ph
i xác su
t
X
0 1 2 3 4
(
)
XP
0,05 0,2 0,3 0,3 0,15
a)
L
p hàm phân ph
i
(
)
xF
và v
đồ
th
c
a
(
)
xF
.
Bài ging
66
b)
Tìm
(
)
20
XP
,
(
)
31
<
XP
(
)
2
>
XP
.
9.
Trong các hàm sau
đ
ây, hàm nào là hàm m
t
độ
xác su
t:
a)
( )
[
]
[ ]
=
1;0,3
1;0,0
2
xx
x
xf
.
b)
( )
<
=
π
π
2
,
1
cos
1
2
,0
2
x
x
x
x
xf
Tìm
(
)
XE
,
(
)
XD
,
(
)
XMod
(
)
XMed
c
a bi
ế
n ng
u nhiên
X
t
ươ
ng
ng.
10.
Cho bi
ế
n ng
u nhiên
X
có hàm phân ph
i
( ) ( )
>
<
<
=
3,1
32,2
2,0
2
x
xx
x
xF
X
.
a)
Tìm hàm m
t
độ
(
)
xf
.
b)
Tính
[
]
6,11
<
<
XP
.
11.
Cho bi
ế
n ng
u nhiên
X
có hàm phân ph
i
( )
>
<+
=
3
1
,1
3
1
1,
4
3
4
3
1,0
x
xx
x
xF
X
.
a)
Tìm hàm m
t
độ
c
a bi
ế
n ng
u nhiên
X
.
b)
Tính
(
)
02
<
<
XP .
12.
Cho hàm s
( )
( )
<
<
>
=
21,2
10,
20,0
2
2
xxa
xax
xx
xf .
a)
Tìm
a
để
(
)
xf
là hàm m
t
độ
.
b)
Tìm hàm phân ph
i t
ươ
ng
ng.
13.
Cho hàm s
( )
=
2
;
2
,0
2
;
2
,cos
ππ
ππ
x
xxA
xf
.
a)
Tìm
A
để
(
)
xf
là hàm m
t
độ
c
a bi
ế
n ng
u nhiên nào
đ
ó.
b)
Tìm hàm phân ph
i
(
)
xF
X
. Tìm
<
36
ππ
XP
.
14.
Cho hàm
( )
<
=
0,
0,0
22
xeBx
x
xf
x
.
a)
Xác
đị
nh
để
(
)
xf
là hàm m
t
độ
c
a bi
ế
n ng
u nhiên nào
đ
ó.
b)
Tìm hàm phân ph
i
(
)
xF
X
.
Chương III. Các quy lut phân phi xác sut thường gp.
67
15.
Tìm
k
để
hàm
( )
<
>
=
0,0
)0(0,
2
x
xke
xf
x
θ
θ
là hàm m
t
độ
c
a bi
ế
n
X
nào
đ
ó. Tìm
hàm phân ph
i t
ươ
ng
ng.
16.
Cho bi
ế
n ng
u nhiên
X
có hàm phân ph
i
( )
>
<+
=
2,1
22,
2
arcsin
1
2
1
2,0
x
x
x
x
xF
π
a)
Tìm
(
)
11
<
<
XP
.
b)
Tìm hàm m
t
độ
(
)
xf
.
17.
Cho hàm m
t
độ
c
a bi
ế
n ng
u nhiên
X
là:
( )
>>
=
0,0
0,0,
1
x
xe
xf
x
λ
λ
λ
.
a)
Tìm hàm phân ph
i c
a bi
ế
n ng
u nhiên
X
và tính xác su
t
(
)
λ
<
XP 0
.
b)
Tính kì v
ng và ph
ươ
ng sai c
a
X
.
18.
Cho bi
ế
n ng
u nhiên
X
có b
ng phân ph
i xác su
t:
X
1
0
1
2
P
1,0
3,0
2,0
4,0
a)
L
p b
ng phân ph
i xác su
t c
a bi
ế
n ng
u nhiên 2008
2
+= XY .
b)
Tính
(
)
YE
,
(
)
YD
.
19.
Cho
(
)
8,0
=
AP . Tìm xác su
t khi th
c hi
n 100 phép th
thì
a)
S
l
n x
y ra bi
ế
n c
A l
n h
ơ
n 75.
b)
S
l
n x
y ra bi
ế
n c
A không quá 75.
c)
S
l
n x
y ra bi
ế
n c
A là n
m trong
đ
o
n
[
]
90;75
.
Đáp s và hướng dn.
1.
Ta có:
X
2
3
4
5
6
7
8
9
10
11
12
P
36
1
36
2
36
3
36
4
36
5
36
6
36
5
36
4
36
3
36
2
36
1
2.
G
i
X
là s
tr
n th
ng c
a
độ
i tuy
n.
a)
Ta có:
X
0
1
2
3
P
168,0
436,0
324,0
072,0
b)
( )
>
<
<
<
=
3,1
32,928,0
21,604,0
10,168,0
0,0
x
x
x
x
x
xf
Bài ging
68
c)
D
th
y:
(
)
(
)
832,068,011011
=
=
=
=
XPXP
.
3.
Vì trong h
p có
4
bi
đ
en nên ta ch
l
y nhi
u nh
t là 5 bi là
đượ
c bi
đỏ
.
B
ng phân ph
i
X
1
2
3
4
5
P
7
3
6
3
.
7
4
5
3
.
6
3
.
7
4
4
3
.
5
2
.
6
3
.
7
4
4
1
.
5
2
.
6
3
.
7
4
4.
Ta có
X
nh
n các giá tr
t
ươ
ng
ng là:
3
,
4
,
5
.
X
3
4
5
P
2
7
2
4
C
C
2
7
1
4
1
3
.
C
CC
2
7
2
3
C
C
5.
Ta có
X
1
2
3
P
5
4
45
8
45
1
6.
Ta có
a)
X
0
1
2
P
2475
82
2475
916
2475
1477
7.
a)
1910
2
=+ aa
,
0
a
1,0
=
a
, b)
2,0
, c)
3,0
8.
a)
( )
>
<
<
<
<
=
4,1
43,805,0
32,505,0
21205,0
10,05,0
0,0
x
x
x
x
x
x
xf
b)
(
)
(
)
(
)
05,01020
=
=
+
=
=
<
XPXPXP
,
(
)
(
)
(
)
65,03231
=
=
+
=
=
<
XPXPXP
,
(
)
(
)
45,0212
=
=
>
XPXP .
9.
C
hai hàm này là hàm m
t
độ
xác su
t.
10.
a)
( ) ( )
>
<
=
3,0
32,22
2,0
x
xx
x
xf
, b) 0
11.
a)
( )
>
<
=
3
1
,0
3
1
1,
4
3
1,0
x
x
x
xf , b)
( )
4
3
02 =<< XP
.
Chương III. Các quy lut phân phi xác sut thường gp.
69
12.
a)
2
3
=a .
13.
2
1
=A
.
14.
a)
( )
1=
+∞
dxxf
4
1
1
0
22
==
+∞
BdxeBx
x
.
15.
θ
2
1
=k .
16.
a)
( )
3
1
11 =<< XP
, b)
( )
>
<
=
2,0
22,
4
1
2,0
2
x
x
x
x
xf
π
.
17.
a)
( )
>>
=
0,0,1
0,0
λ
λ
xe
x
xF
x
, b)
(
)
λ
=
XE
,
(
)
2
λ
=XD
.
18.
b)
(
)
9,2009
=
YE
,
(
)
9,3
=
YD
.
C. Phương pháp ging dy.
- ng dng Excel cho vic tính c giá tr ca biến ngu nhiên phân phi chun,
phân phi Poisson, phân phi Student, phân phi chi bình phương.
- Gi
ng viên g
i bài gi
ng cho sinh viên
đọ
c tr
ướ
c. Gi
ng viên trình bày bài gi
ng trên l
p
theo ph
ươ
ng pháp thuy
ế
t trình h
i
đ
áp. Giao bài t
p cho sinh viên v
nhà làm. Gi
i thi
u m
t
s
tài li
u tham kh
o.
D. Tài liu tham kho
[1] Đậu Thế Cp, Xác sut thng kê: thuyết các bài tp, NXB Giáo dc,
2006.
[2] Đặng Công Hanh, Đặng Ngc Dc, Giáo trình thuyết xác sut Thng
toán, trường Đi hc Duy Tân,1996
[3] PGS. TS. Phm Xuân Kiu, Giáo Trình xác sut thng , NXB Giáo dc,
2005.
[4] Trn Văn Minh, PTh Vân Anh, Xác sut thng , NXB Giao thông vn ti,
2008.
[5] Trn Văn Minh, Phí Th Vân Anh, Hưng dn gii bài tp Xác sut thng ,
NXB Giao thông vn ti, 2008.
Chương IV. Lí thuyết mu
A. Mc tiêu.
- Gii thiu các khái nim: tng th, mu ngu nhiên, mu c th, kích thước mu.
- Xây dng các bng phân phi thc nghim và v biu đồ ca chúng.
- Xây dng hàm phân phi mu và gii thiu đa giác tn sut tích lu.
- Gii thiu c đặc trưng ca mu:trung bình mu, phương sai mu chưa hiu chnh
đã hiu chnh, đ lch chun mu chưa hiu chnh và đã hiu chnh, t l mu.
- Gi
i thi
u lu
t phân ph
i c
a các
đặ
c tr
ư
ng m
u.
B. Ni dung.
1. Tng th và mu.
1.1. M đầu.
Trong thc tế, ta thường phi nghiên cu mt tp hp các phn t theo mt hay
nhiu du hiu đặc trưng cho các phn t. Nhưng tp hp quá nhiu phn t thì
không th nghiên cu tt c các phn t, nếu m như vy s tn thi gian, công
sc, Do đó, người ta thường nghiên cu mt phn, đặc bit c phương pháp
chn mu.
Gi s ta cn nghiên cu mt tp hp gm
N
phn t, tp hp này gi tng th,
mi phn t ca tp hp này gi là mt th,
N
gi kích thước ca tng th.
Ta ly ngu nhiên
phn t, t tng th gi là mt mu. S
gi kích thước
mu. T nhng thông tin có được trên mu này ta suy ra kết lun ca tng th, do đó
phi ly mu như thế nào để đại din cho tng th. Trong mi ngành, mi lĩnh vc
có các phương pháp riêng mang tính đặc thù ca ngành, để sao cho vic ly mu đại
din trung thc cho tng th.
Ví d 1.1.
Ta xét bài toán sau: Để chiến lược cho chương trình dinh dưỡng quc gia nhm
tăng chiu cao ca người dân, người ta đi tìm chiu cao ca nhng ngưi trưởng
thành Vit Nam.
Khi đó, trong bài toán này:
a) Tp hp gm tt c nhng người trưởng thành Vit Nam, ta gi là tng th.
b) Mi người trong tng th, được gi là mt th.
c) Chiu cao ca ngưi trong tng thmt đại lưng ngu nhiên.
Bài ging
72
d) Do s người trưởng thành Vit Nam là rt ln, nên ta không th đo chiu cao tt
c được ch ra mt s người (chng hn 500 người) để đo chiu cao. Tp hp
500 ngưi này được gi là mt mu, s 500 được gi là kích thước mu.
Ta nói rng mt mu là ngu nhiên nếu trong phép ly mu đó, mi phn t ca tng
th đều được chn mt cách độc lp xác sut được chn như nhau. Ngoài
phương pháp ly mu ngu nhiên, ta còn có các phương pháp ly mu khác na như
chn mu vi xác sut không đều, chn mu theo nhóm tri, mu chùm v.v… Trong
bài ging này, chúng ta gii thiu cách ly mu đơn gin nht được s dng rng
rãi trong các lĩnh vc khác nhau.
Khi chn mu nếu phn t đã chn loi ra khi tng th mi chn phn t tiếp theo
thì gi là mu không hoàn li, nếu phn t đã chn tr li tng th mi chn phn t
tiếp theo thì gi là mu có hoàn li.
Khi kích thước ca tng th đủ ln thì có th coi 2 cách ly trên là như nhau.
1.2. Mu ngu nhiên, mu c th.
Tiến hành
quan sát độc lp v biến ngu nhiên
X
nào đó trên tng th.
Ta gi
i
X
là quan sát th
i
ca biến ngu nhiên
X
,
ni ;1=
.
Khi đó
(
)
n
XXX
;;;
21
K
đưc gi mu ngu nhiên, trong đó
1
X
,
2
X
,
K
,
n
X
độc
lp và có cùng phân phi xác sut vi
X
.
Ta gi
i
x
kết qu quan sát th
i
. Khi đó
(
)
n
xxx
;;;
21
K
giá tr quan sát được.
Đó giá tr c th ca mu ngu nhiên
(
)
n
XXX
;;;
21
K
nhn còn được gi mu
c th (hoc mu thc nghim).
Chú ý.
+ Ta ch xét các kết qu quan sát độc lp.
+ Khi xét thuyết, ta dùng mu ngu nhiên, còn khi làm toán thì ta dùng mu c
th.
Ví d 1.2.
Xét mt tng th mt hp gm
10
cây thước, trong đó
3
cây thước dài
10
cm,
5
cây thước dài
20
cm và
2
cây thước dài
30
cm.
Gi
X
biến ngu nhiên đặc trưng cho chiu dài (xét v lượng) ca cây thước.
Ta xét tng th v mt định lượng.
Khi đó,
X
có bng phân phi như sau:
X
10
cm
20
cm
25
cm
P
10
3
10
5
10
2
Ta thc hin vic ly ngu nhiên (có hoàn li)
5
cây thước. Khi đó
Gi
i
X
là chiu dài ca cây thưc được ly ra ln th
i
,
5;1=i
thì
i
X
có phân phi
i
X
10
cm
20
cm
30
cm
Chương IV. Lí thuyết mu.
73
P
10
3
10
5
10
2
Như vy,
1
X
,
2
X
,
K
,
5
X
5
biến ngu nhiên độc lp cùng phân phi vi
X
.
(
)
n
XXX
;;;
21
K
là mu ngu nhiên.
Ta thc hin
5
ln ly cây thước như sau:
(
)
n
xxx
;;;
21
K
mu c th trong
5
ln
quan sát, chng hn là:
cmxX
10
11
=
=
,
cmxX
10
22
=
=
,
cmxX
30
33
=
=
,
cmxX
20
44
=
=
,
cmxX
30
55
=
=
.
Vy
(
)
(
)
cmcmcmcmcmxxx
n
30;20;30;10;10;;;
21
=
K
.
Bây gi, ta xét tng th v mt định tính.
Nếu ta xem nhng cây thước nh hơn
20
cm là “không đạt yêu cu”.
Ly ngu nhiên
1
cây thước. Gi
X
là s cây thước “không đạt yêu cu”.
Ta có bng phân phi xác sut ca
X
X
0
1
P
10
3
10
7
Gi
i
X
là s cây thước “không đạt yêu cukhi ly cây thước th
i
,
5;1
=i
. Ta thy
các
i
X
có cùng phân phi vi
X
.
(
)
n
XXX
;;;
21
K
là mu ngu nhiên.
Ta xem chiu dài c th ca cây thước được ly ra. Khi đó, chng hn ta kết qu
sau
(
)
(
)
1;0;0;1;1;;;
21
=
n
xxx
K
được gi là mu c th.
1.3. Bng phân phi tn s.
1.3.1. Phân loi mu và bng phân phi tn s.
Gi s mu
(
)
n
xxx ;;
2;1
K
kích thước
, nếu các
i
x
,
ni ;1=
khác nhau khi đó mu
gi là mu đơn.
Nếu trong mu
k
giá tr khác nhau vi
1
x
xut hin
1
n
ln,
2
x
xut hin
2
n
ln,
…,
k
x
xut hin
k
n
ln thì
nnnn
k
=
+
+
+
L
21
i
n
gi tn s ca
i
x
,
ni
;1
=
.
Mu có các giá tr ging nhau gi là mu lp.
Nếu ta xếp
i
x
theo th t tăng dn
n
xxx
<
<
<
L
21
lp bng gm các
i
x
tn s
i
n
tương ng thì ta được mt bng
X
1
x
2
x
k
x
T
n s
1
n
2
n
k
n
gi là bng phân phi tn s.
Bài ging
74
Trong trường hp mu kích thước ln các giá tr ca
X
khác nhau không nhiu,
để tin li cho vic tính toán ta phân min giá tr ca
X
thành
k
khong (có th chia
đều hoc không đu)
[
)
10
;
aa
;
[
)
21
;
aa
;
K
;
[
)
kk
aa
;
1
nếu
i
n
giá tr ca mu
xut hin trong khong
[
)
10
;
aa
;
ki
;1
=
thì bng
X
[
)
10
;
aa
[
)
21
;aa
[
)
kk
aa
;
1
T
n s
1
n
2
n
k
n
gi là bng phân phi tn s phân lp.
Ta quy ước đầu mút bên trái ca mt khong thuc khong đó khi tính tn s ca
mi lp.
Ví d 1.3.
Thng cân nng
X
ca
370
tr sơ sinh, ta được bng phân phi tn s phân lp
sau
X
6,24,2
8,26,2
0,38,2
2,30,3
4,32,3
6,34,3
Tn s
6
44
76
100
95
49
Thông thường, người ta chia s liu thành t
5
đến
15
lp. Nếu s lp nhiu hơn có
th giúp phân tích tt hơn, nhưng s ci thin không nhiu, nếu s lp ít quá, thì các
thông tin có th b mt khi x lí.
Mt bng phân phi tn s phân lp có th đưa v bng phân phi lp bng phép ly
trung bình cng ca mi lp.
Ví d 1.4.
Xét Ví d 1.3, ta có:
X
5,2
7,2
9,2
1,3
3,3
5,3
Tn s
6
44
76
100
95
49
1.3.2. Bng phân phi tn sut.
T bng phân phi, nếu ta đặt
n
n
f
i
i
=
,
ki
;1
=
thì
i
f
được gi tn sut xut hin
ca
i
x
,
ki
;1
=
. Khi đó, bng
i
x
1
x
2
x
k
x
T
n su
t
1
f
2
f
k
f
được gi bng phân phi tn sut, nó rt ging vi bng phân phi xác sut ca
mt biến ngu nhiên ri rc.
Để đưc mt hình dung v phân phi mu, người ta thường dùng đồ th để biu
din bng phân phi tn sut.
1.3.3. Đa giác tn sut
Trên đồ th, ni các đim có ta độ
(
)
ii
fx
;
,
ki ;1=
ta được đường gp khúc gi đa
giác tn sut.
Chương IV. Lí thuyết mu.
75
1.3.4. Biu đồ tn s.
Nếu mu cho bng các khong phân lp, ta y dng mt biu đồ gm các hình ch
nht cnh nhau đáy bng độ i ca khong phân lp din tích bng tn s
ca lp và gi là biu đồ tn s.
Sau đây, là bng phân phi tn sut, đa giác tn sut và biu đồ tn s ca Ví d 1.3.
X
5,2
7,2
9,2
1,3
3,3
5,3
Tn s
6
44
76
100
95
49
Tn sut 0,162 0,119 0,205 0,270 0,257 0,132
0
0.05
0.1
0.15
0.2
0.25
0.3
2,5 2,7 2,9 3,1 3,3 3,5
x
Tn sut
Đa giác tn sut
0
20
40
60
80
100
120
1
Biu đồ tn s
Bài ging
76
1.4. Hàm phân phi mu.
Vi ĐLNN
X
, ta không th biết đưc hàm phân phi xác sut
(
)
xF
ca nó. Tuy
nhiên t mt mu thng
(
)
n
xxx
;;;
21
K
ca
X
da vào bng phân phi tn sut ta
có th thu được nhng thông tin quan trng v
(
)
xF
.
T bng phân phi tn sut, ta xây dng hàm
(
)
xF
n
như sau:
(
)
<
=
xx
in
i
fxF
. Hay
( )
>
<+++
<+
<
=
n
nnn
n
xx
xxxfff
xxxff
xxxf
xx
xF
,1
,
,
,
,0
1121
3221
211
1
L
KKK
(
)
xF
n
đưc gi là hàm phân phi thc nghim hoc phân phi mu.
Trên đồ th, ni các đim có ta độ
=
k
i
ii
fx
1
;
,
ki ;1=
ta được được đường gp khúc
gi là đa giác tn sut tích lũy.
Sau đây, là biu đ đa giác tn sut tích lũy d 1.1.
0
0.2
0.4
0.6
0.8
1
1.2
2.5 2.7 2.9 3.1 3.3 3.5 3.7 3.9
X
Y
Biu đồ đa giác tn sut tích lũy.
2. Các tham s đặc trưng ca mu
2.1. T l mu.
Ta gi hàm phân phi xác sut ca biến ngu nhiên này hàm phân phi mu;
vng, phương sai ca biến ngu nhiên là trung bình mu và phương sai mu. Khi đó,
ta có:
Chương IV. Lí thuyết mu.
77
Trung bình mu:
n
nxnxnx
fxfxfxx
kk
kk
+
+
+
=+++=
L
L
2211
2211
Phương sai mu:
(
)
(
)
(
)
( ) ( ) ( )
n
nxxnxxnxx
fxxfxxfxxs
kk
kk
2
2
2
21
2
1
2
2
2
21
2
1
2
ˆ
+++
=
+++=
L
L
Đặt:
n
nxnxnx
fxfxfxx
kk
kk
2
2
2
21
2
1
2
2
2
21
2
1
2
+++
=+++=
L
L
.
Theo tính cht phương sai, ta có
( )
(
)
n
xnxn
xxs
k
i
ii
2
1
2
2
22
ˆ
==
=
.
Phương sai mu hiu chnh:
(
)
(
)
(
)
1
ˆ
1
2
2
2
21
2
1
22
+++
=
=
n
nxxnxxnxx
s
n
n
s
kk
L
.
Tương t vi biến ngu nhiên, ta gi
2
ˆˆ
ss =
độ lch mu và
2
ss =
độ lch mu hiu chnh.
Để tính các đặc trưng này, ta thưng lp bng sau
i
x
i
n
ii
nx
ii
nx
2
1
x
1
n
11
nx
1
2
1
nx
2
x
2
n
22
nx
2
2
2
nx
k
x
k
n
kk
nx
kk
nx
2
xn
2
xn
T bng này, ta có
n
nx
x
k
i
ii
=
=
1
,
n
nx
x
k
i
ii
=
=
1
2
2
,
(
)
2
22
ˆ
xxs =
22
ˆ
1
s
n
n
s
=
.
Ví d 2.1.
Cho mu
i
x
5 10 15 20 25
i
n
15
25
30
20
10
a) Viết hàm phân phi mu.
b) Tính trung bình mu, phương sai mu, phương sai mu hiu chnh.
Gii.
Bài ging
78
a) Bng phân phi xác sut
X
5
10
15
20
25
T
n su
t
15,0
25,0
30,0
20,0
10,0
T
đ
ó, ta có hàm phân ph
i là:
( )
>
<
<
<
<
=
25,1
2520,90,0
2015,70,0
1510,45,0
105,15,0
5,0
x
x
x
x
x
x
xF
n
b) Ta có bng tính
i
x
i
n
ii
nx
ii
nx
2
5 15 75 375
10
25
250
2500
15
30
450
6750
20
20
400
8000
25 10 250 6250
100
1425
23875
T bng, ta có:
25,14
100
1425
==x ; 75,238
100
23875
2
==x ;
(
)
(
)
6875,3525,1475,238
ˆ
2
2
22
=== xxs
; 0480,366875,35.
99
100
ˆ
1
22
==
= s
n
n
s
.
Ví d 2.2.
Theo dõi s lượng bài tp mt nhóm gm
100
sinh viên gii trong mt tun, ta thu
được bng s liu sau:
T
ng s
bài t
p
41
44
45
46
48
52
54
S
sinh viên
hoàn thành
10
20
30
15
10
10
5
a) Tính trung nh mu, phương sai mu chưa hiu chnh, phương sai mu hiu
chnh.
b) Nhng sinh viên thc hin xong mt phép toán toán ln hơn
48
bài là nhng sinh
sinh viên tc độ làm bài tp nhanh. Tính t l sinh viên tc độ làm bài tp
nhanh?
c) Tính trung bình mu, phương sai mu hiu chnh ca nhng hc sinh viên tc
độ làm bài tp nhanh.
Gii.
a) Ta lp bng như sau:
i
x
i
n
ii
nx
ii
nx
2
Chương IV. Lí thuyết mu.
79
41
10
141
16810
44
20
880
38720
45
30
1350
60750
46
15
690
31740
48
10
480
23040
52
10
520
27040
54 5 270 14580
100
4600
212680
Da vào bng trên, ta có
S bài tp trung bình:
46
100
4600
==x
bài.
Phương sai mu chưa hiu chnh:
(
)
( )
8,1046
100
212680
ˆ
2
2
22
=== xxs
.
Phương sai mu hiu chnh là:
909,108,10.
99
100
ˆ
1
22
==
= s
n
n
s
.
b) T l mu là
25,0
100
51010
=
+
+
=f
.
c) Ta có bng
i
x
i
n
ii
nx
ii
nx
2
48
10
480
23040
52
10
520
27040
54 5 270 14580
25
1270
64660
Khi
đ
ó, ta có:
8,50
25
1270
==x
,
( )
(
)
68,50.2564660
1
1
2
2
=
=
n
s
.
2.2. S mt (Mode) ca mu.
S mt ca mu ngu nhiên
X
, hiu
(
)
XMod
, được xác định như sau: Nếu
mu được cho dưới bng phân phi tn s thì Mode là giá tr có tn s ln nht.
Đối vi trường hp mu được cho dưới dng bng phân phi tn s phân lp,
người ta định nghĩa khong Mode khong din tích ca hình ch nht dng
trên khong đó là ln nht.
Mode mt ch tiêu thường được chú ý trong các bài toán v kinh tế. Chng hn
mt nhà máy sn xut ô tô Civic mun mt s lượng ô đủ đáp ng nhu cu ca
người cn mua thì phi cý đến ô loi gì khách hàng thường hay hi mua
nht.
2.3. S trung v (Median) ca mu.
Trung v ca mt mu s liu, hiu bi
(
)
XMed
, mt s tính cht sau: S
các giá tr ca mu hơn hoc bng
(
)
XMed
thì bng s các giá tr ca mu ln
hơn hoc bng
(
)
XMed
.
Bài ging
80
Xét trường hp các giá tr ca mu là phân bit. Gi s các giá tr ca mu được sp
xếp theo th t tăng dn
n
xxx
<
<
<
L
21
.
Khi đó, d thy nếu
l thì ly
(
)
2
1+
=
n
xXMed
.
Nếu
n
chn thì ta ly
( )
+=
+1
22
2
1
nn
xxXMed
.
Trong trường hp giá tr
i
x
tn s
i
n
, gi
k
ch s nht sao cho
2
21
n
nnn
k
+++
L
. Lúc đó, ta định nghĩa
(
)
k
nXMed
=
.
Ví d 2.3.
Cho mu
T
ng s
bài t
p
41
44
45
46
48
52
54
S
sinh viên
hoàn thành
10 20 30 15 10 10 5
Ta có
100
=
n
.
10
1
=
n
,
20
2
=
n
,
30
3
=
n
,
15
4
=
n
,
10
5
=
n
,
10
6
=
n
,
5
7
=
n
.
D thy: s sinh viên hoàn thành ít hơn hoc bng
44
bài là
2
100
30
21
<=+ nn
và s sinh viên hoàn thành ít hơn hoc bng
45
bài tp
2
100
60
321
>=++ nnn
.
Vy
(
)
45
=
XMed
.
Trong trường hp mu được cho dưới dng bng phân phi tn s phân lp, ta định
nghĩa khái nim trung v như sau:
Gi s, ta
m
khong vi các đim chia
m
aaa
<
<
<
L
10
, vi
[
)
101
;aaC
=
,
[
)
212
;aaC
=
,
K
,
[
)
mmm
aaC ;
1
=
, trong đó khong
i
C
tn s
i
r
,
nnnn
m
=
+
+
+
L
21
.
Khong
k
C
được gi khong trung v nếu
k
s nht sao cho
2
21
n
nnn
k
+++
L
.
S trung v
(
)
XMed
s ti đó, đưng thng
m
x
=
chia đôi din tích ca biu
đồ tn s. Rõ ràng, s trung v luôn luôn nm trong khong trung v.
Ví d 2.4.
Tìm khong trung v và s trung v cho Ví d 1.3.
Gii
Ta
22610076446
2
370
12676446 =+++<<=++
nên khong trung v
(
)
2,3;0,3
.
Chương IV. Lí thuyết mu.
81
Để tính s trung v, ta nhn thy: chiu cao ca khong trung v này
500
2,0
100
=
.
Khi đó, ta chia hình ch nht dng trên khong này thành
2
phn din tích
phn bên trái là
59126185
=
.
Ta có
59.500
=
AM
hay
500
59
=AM
. Vy s trung v
( )
500
59
0,3 +=xMed
.
Hoc ta có th tính như sau:
Hình ch nht có din tích phn bên phi là
41185226
=
.
Ta có
41.500
=
BM
hay
500
41
=BM
.
Vy s trung v
( )
500
41
2,3 =xMed
Da vào bài toán này, ta có nhn xét sau.
Nhn xét.
Ta chng minh được
( )
( )
k
kk
k
i
i
k
n
aa
n
n
aXMed
1
1
2
=
=
s trung v, trong đó
[
)
kkk
aaC ;
1
=
khong trung v.
2.4. Các quy lut phân phi mu.
Người ta chng minh đưc các kết qu sau:
Định lí.
Cho
X
tuân theo phân phi chun
(
)
2
;
σµ
N
thì
a) Nếu biết
2
σ
thì
+
n
NX
2
;~
σ
µ
.
+
( )
1;0~. Nn
X
σ
µ
.
b) Nếu chưa biết
2
σ
thì
+
30
n
, ta có
( )
1~.
nTn
S
X
µ
.
A B
C
M
Bài ging
82
+
30
>
n
, ta có
( )
1;0~. Nn
S
X
µ
.
c) Nếu biết
µ
thì
( )
( )
n
X
n
i
i
2
2
1
2
~
χ
σ
µ
=
.
d) Nếu chưa biết
µ
thì
(
)
( )
1~
2
2
1
2
=
n
XX
n
i
i
χ
σ
.
Chú ý.
Nếu không biết phân phi ca biến ngu nhiên
X
nhưng vi kích thước mu
30
>
n
,
khi đó ta xem biến ngu nhiên
X
có phân phi chun
(
)
2
;
σµ
N
.
3. Bài tp chương.
1.
Tr
ng l
ượ
ng c
a
100
tr
em c
a m
t tr
ườ
ng m
u giáo ghi nh
n
đượ
c nh
ư
sau:
Tr
ng l
ượ
ng (kg) 41 44 45 46 48 52 54
S
tr
em 10 20 30 15 10 10 5
a) Hãy xác
đị
nh
x
là tr
ng l
ượ
ng trung bình c
a các sinh viên và
độ
l
ch m
u hi
u ch
nh
s
.
b) Hãy xác
đị
nh b
ng phân ph
i xác su
t.
2.
K
ế
t qu
thi h
c kì môn toán A1 c
a m
t kh
i sinh viên khóa
14
g
m
72
b
n nh
ư
sau:
Đ
i
m 3 5 6 8 9
S
sinh viên 40 15 5 10 2
a) Hãy l
p b
ng phân ph
i t
n s
c
a các k
ế
t qu
này.
b) L
p b
ng phân ph
i t
n su
t và v
đ
a giác t
n su
t.
3.
Th
ng kê cân n
ng c
a
400
tr
s
ơ
sinh, ta
đượ
c b
ng phân ph
i t
n s
phân l
p sau
X
6,24,2
8,26,2
0,38,2
2,30,3
4,32,3
4,32,3
8,36,3
Tn
s
6
44
76
100
95
30
49
a) L
p b
ng phân ph
i t
n s
không phân l
p.
b) L
p b
ng phân ph
i t
n su
t, v
đ
a giác t
n su
t và bi
u
đồ
t
n s
.
c) L
p hàm phân ph
i m
u và v
đ
a giác t
n su
t tích l
ũ
y.
d) Tìm
(
)
XMod
, xác
đị
nh kho
ng trung v
.
4.
Giá c
a m
t lo
i c
phi
ế
u bán trên th
tr
ườ
ng ch
ng khoáng trong
100
phiên giao d
ch
đượ
c cho
b
ng sau
Giá c
phi
ế
u
(1000
đ
)
1513
1715
1917
2119
2321
S
phiên giao
d
ch
5 18 42 27 8
Hãy tính các giá tr
x
, ph
ươ
ng sai m
u
2
ˆ
s
,
2
s
.
5.
Tu
i c
a
70
nhân viên trong m
t c
ơ
quan
đượ
c ghi l
i nh
ư
sau:
Kho
ng T
n s
D
ướ
i 20
7
20-30 13
30-40 26
40-50 15
50-60 6
Trên 60 3
Tìm kho
ng trung v
, s
trung v
, kho
ng Mod.
6. Để
nghiên c
u nhu c
u tiêu th
s
a h
p trong m
t khu v
c, ng
ườ
i ta ti
ế
n hành kh
o sát
800 gia
đ
ình. K
ế
t qu
đượ
c cho
b
ng d
ướ
i
đ
ây:
Nhu c
u (h
p/tháng) S
gia
đ
ình Nhu c
u (h
p/tháng) S
gia
đ
ình
30-34 35 55-59 142
35-39 48 60-64 94
40-44 83 65-70 50
45-49 159 70-74 10
50-54 189
b) Tìm s
h
p s
a tiêu th
trung bình.
Chương IV. Lí thuyết ước lượng
84
c) Tìm kho
ng trung v
, s
trung v
.
7.
Khi ki
m tra th
l
c m
t nhóm sinh viên, ta có k
ế
t qu
v
cân n
ng nh
ư
sau:
i
x
(kg)
5,475,42
5,525,47
5,575,52
5,625,57
5,675,62
S
sinh
viên
i
n
8
14
28
18
12
a) Tính
x
là cân n
ng trung bình cho các sinh viên và tính
độ
l
ch m
u.
b) L
p b
ng phân ph
i xác su
t m
u.
8. Đ
i
u tra n
ă
ng su
t lúa trên di
n tích
100
ha tr
ng lúa c
a m
t vùng, ta thu
đượ
c b
ng s
li
u sau.
N
ă
ng su
t (t
/ha) 41 44 45 46 48 52 54
S
ha có n
ă
ng su
t
t
ươ
ng
ng
10 20 30 15 10 10 5
a) Tính trung bình m
u, ph
ươ
ng sai m
u, ph
ươ
ng sai m
u hi
u ch
nh.
b) Nh
ng th
a ru
ng có n
ă
ng su
t t
48
t
tr
lên g
i là nh
ng th
a ru
ng có n
ă
ng su
t cao.
Tính t
l
th
a ru
ng có n
ă
ng su
t cao.
c) Tính trung bình m
u, ph
ươ
ng sai m
u hi
u ch
nh c
a nh
ng th
a ru
ng có n
ă
ng su
t cao.
C. Phương pháp ging dy.
- Phi hp phương pháp thuyết trình và vn đáp gii quyết vn đề.
- Đưa ví d c th đ nêu rõ ý nghĩa ca vic chn mu.
- T trc quan sinh động đến tư duy tru tượng. Áp dng mt s ví d thc tế.
- Yêu cu SV đọc bài ging trước khi lên lp.
- Kim tra, đánh giá vic làm bài tp ca SV.
- S dng phương tin dy hc hin đại như Mic.
D. Tài liu tham kho
[1] Đu Thế Cp, Xác sut thng kê: Lí thuyết và các bài tp, NXB Giáo dc, 2006.
[2] Đng Hùng Thng, Thng kê và ng dng, NXB Giáo dc, 2008.
[3] PGS. TS. Phm Xuân Kiu, Giáo Trình xác sut thng , NXB Giáo dc,
2005.
[4] Trn Văn Minh, Phí Th Vân Anh, Xác sut thng vi các tính toán trên
Excel, NXB Giao Thông Vn ti, 2008.
[5] Đng Công Hanh, Đng Ngc Dc, Giáo trình thuyết xác sut Thng
toán, trường Đại hc Duy Tân,1996
[6] Trn Văn Minh, Phí Th Vân Anh, Hướng dn gii bài tp Xác sut thngvi
các tính toán trên Excel, NXB Giao Thông Vn ti, 2008.
Chương V.
Lí thuyết ước lượng
A. Mc tiêu.
- Gii thiu các khái nim ước lượng: ước lưng đim, ước lượng khong.
- Gii thiu các loi ước lượng đim: ước lượng không chch, ước lưng hiu qu, ước
lượng vng.
- Gi
i thi
u các ph
ươ
ng pháp
ướ
c l
ượ
ng kho
ng cho: kì v
ng, ph
ươ
ng sai, t
l
.
B. Ni dung.
Xét mt tng th
gi s ta quan tâm đến biến lượng
X
đo lưng mt du hiu
nào đó ca tng th
. Khi đó
X
được coi đại lượng ngu nhiên. Phân phi xác
sut ca
X
thường rt khó nm bt, thông thưng ta gii hn vic xác định mt
s các tham s đặc trưng ca
X
như các giá tr:
(
)
XE
,
(
)
XD
,
(
)
XMed
,
(
)
XMod
,
Các tham s này không th xác định chính xác được (nếu không biết phân phi ca
X
), phi ưc lượng t các giá tr ca
X
trên mt mu chn ngu nhiên. Như vy,
bài toán ước lượng tham s được phát biu như sau:
Gi s
X
mt đại lượng ngu nhiên tham s đc trưng
θ
nào đó (chưa biết)
ta quan tâm. Vn đề đặt ra là: Căn c trên
n
giá tr
1
x
,
2
x
,
K
,
n
x
ca
X
đo được trên
mt mu kích thước
n
được ly t tng th
, cn tìm mt giá tr gn đúng
θ
ˆ
ca
θ
.
1. Ước lưng đim.
Định nghĩa.
Mt hàm
(
)
nn
xxxT ;;;
ˆ
21
K=
θ
ca
n
giá tr
1
x
,
2
x
,
K
,
n
x
được gi mt ước lượng
đim cho
θ
.
Như vy, mt ước lưng
n
T=
θ
ˆ
mt m ca
đại lượng ngu nhiên
1
X
,
2
X
,
K
,
n
X
nên cũng đại lượng ngu nhiên. nhiu hàm ước lượng
(
)
nn
xxxT ;;;
ˆ
21
K=
θ
ca tham s
θ
khác nhau. Tuy nhiên, vic la chn mt ước lượng nào “ttđược
da vào các tiêu chun dưới đây.
Định nghĩa.
Hàm ưc lượng
(
)
nn
xxxT
;;;
ˆ
21
K
=
θ
đưc gi là ước lượng không chch nếu
(
)
θθ
=
ˆ
E
.
Bài ging
86
T định nghĩa, ta thy nếu
(
)
nn
xxxT ;;;
ˆ
21
K=
θ
hàm ước lượng không chch ca
θ
thì
(
)
0
ˆ
=
θθ
E
.
Tính cht không chch có nghĩa là ước lượng
θ
ˆ
không có sai s h thng.
Định nghĩa.
Hàm ưc lượng
(
)
nn
xxxT ;;;
ˆ
21
K=
θ
được gi ước lượng vng nếu vi mi
0
>
ε
thì
(
)
1
ˆ
lim =<
+∞
εθθ
E
n
hay
(
)
1
ˆ
lim
=+<<
+∞
εθθεθ
E
n
.
Tính cht vng đảm bo cho ước lưng
θ
ˆ
gn
θ
tùy ý vi xác sut cao (gn
1
) khi
kích thước mu
đủ ln.
Định nghĩa.
Ước lưng
θ
ˆ
đưc gi mt ước lưng hiu qu ca
θ
nếu mt ước lưng
không chch và có phương sai
(
)
θ
ˆ
D
nh nht.
2. Ước lượng khong.
Ước lượng đim dù tt nht cũng ch cho ta mt giá tr trong tp hn nên ta không
biết được độ chính c cũng như xác sut để đim ước lượng độ chính xác, do đó
không đánh giá được sai lm khi dùng
θ
ˆ
thay cho
θ
.
Để khc phc hn chế đó, người ta đưa ra ước lượng khong tin cy cho tham s
θ
,
nghĩa da vào mt ưc lượng
θ
ˆ
, tìm mt khong
(
)
21
;
θ
θ
vi
1
θ
,
2
θ
hai ước
lượng đim ca tham s
θ
sao cho
(
)
α
γ
θ
θ
θ
=
=
<
<
1
21
P
.
Trong đó
α
γ
=
1
xác sut cho trước gi là độ tin cy ca khong ưc lượng, do đó
α
kh năng mc sai lm ca khong ước lượng còn
(
)
21
;
θ
θ
gi khong tin cy
12
θ
θ
đưc gi là độ dài ca khong tin cy.
Ý nghĩa ca khong tin cy ch th nói trong
%100
trường hp ly mu
khong
(
)
21
;
θ
θ
cha tham s
θ
chưa biết thì khng đnh
21
θ
θ
θ
<
<
th tin cy
mc
γ
.
Để ước lượng mt tham s
θ
, ta thc hin quy tc sau:
+ Chn mt hàm
(
)
θ
;;;;
21
n
XXXGG
K
=
sao cho phân phi ca
G
xác định hoàn toàn
(không cha tham s
θ
na).
+ Khi đó vi độ tin cy
α
γ
=
1
cho trước, ta m cp giá tr
0
1
>
α
,
0
2
>
α
sao cho
α
α
α
=
+
21
và tương ng vi chúng là các phân v tha mãn điu kin
(
)
1
1
α
α
=
>
gGP
(
)
21
1
2
α
α
=
>
gGP
(
(
)
21
2
α
α
=
<
gGP
)
Khi đó,
(
)
(
)
αααθ
αα
==<<
11;;;;
21211
12
gXXXGgP
n
K
.
+ Dùng các phép biến đổi tương đương, ta đưa bt đẳng thc trên v dng
(
)
α
θ
θ
θ
=
<
<
1
21
P
.
Chương V. Lí thuyết ước lượng
87
Nhn xét.
Trong Chương này, chúng ta ch xét trường hp
2
1
2
21
γ
α
αα
===
.
2.1. Ước lượng khong tin cy cho kì vng
Gi s đại lưng ngu nhiên
X
phân phi chun
(
)
2
;~
σµ
NX
vi tham s
µ
θ
=
chưa biết. T mt mu c th
(
)
n
xxx
;;;
21
K
ca
X
. Bài toán đặt ra tìm khong tin
cy cho
(
)
µ
=
XE
.
2.1.1. Đã biết phương sai
2
0
2
σσ
=
.
Ta chn hàm:
n
X
ZG
0
σ
µ
==
. Do
(
)
2
;~
σµ
NX
nên
(
)
1;0~
NZ
.
2
2
α
α
=
> zZP
2
1
2
1
α
α
=
>
zZP
. D thy
22
1
αα
zz
=
.
Khi đó
22
αα
zZz
<
<
hay
2
0
2
0
αα
σ
µ
σ
z
n
Xz
n
X +<<
.
Li có
α
αααα
=
>
<=
<<
11
2222
zZPzZPzZzP
.
Suy ra
α
αα
=
<<
1
22
zZzP
.
Vy
α
σ
µ
σ
αα
=
+<<
1
2
0
2
0
z
n
Xz
n
XP
.
Khong ước lượng tin cy cho kì vng
+
2
0
2
0
;
αα
σσ
z
n
Xz
n
X
, trong đó
2
α
z
được
xác đnh t công thc
22
1
2
1
2
γα
α
==
Φ z
vi
( )
=Φ
m
t
dtem
0
2
2
.
Đại lượng
2
0
α
σ
ε
z
n
=
được gi độ chính xác ca ước lượng, phn ánh độ lch
trung bình ca trung bình mu so vi kì vng lí thuyết vi độ tin cy
α
γ
=
1
.
Khi đó, đội khong tin cy
2
0
2
2
α
σ
ε
z
n
=
.
Ví d 2.1.
Gi s
X
trng lượng ca em nam la tui n
10
thuc 4 qun thành ph Hà
Ni. Ta mun biết
(
)
XE
, trng lượng trung nh ca các em nam la tui lên
10
thuc 4 qun thành ph Hà Ni.điu kin v tài chính, thi gian nên ta không th
ly trng lượng ca mi em trong khu vc này, ta chn ngu nhiên
50
em. Qua trng
Bài ging
88
lượng ca
50
em này, ta xác định đưc
32=
X
kg. Gi s phân phi ca
X
độ lch
tiêu chun
5,2
=
σ
kg (đã biết). Vy, vi độ tin cy
%95
=
γ
, ta th i trng
lượng trung bình
(
)
XE
nm trong khong nào? Xét trưng hp trên vi độ tin cy
%95
=
γ
.
Gii
Ta có
5,2
=
σ
đã biết.
Vi đ tin cy
%95
=
γ
, ta có
%5
=
α
. Khi đó
96,1
2
=
α
z
. Khi đó, khong ước lượng tin
cy cho kì vng
(
)
XE
+
2
0
2
0
;
αα
σσ
z
n
xz
n
x
hay
+ 96,1.
50
5,2
2,3;96,1.
50
5,2
2,3
.
Ví d 2.2.
Mt thy giáo mun ước lượng đim n Toán ca toàn b sinh viên Đại hc A. Gi
s đim môn Toán ca sinh viên tuân theo phân phi chun vi
3,0
0
=
σ
đim. Thy
giáo kim tra lp
100
sinh viên và biết được đim môn Toán ca
100
bn này
như sau:
Đ
i
m 3 5 6 8
S
sinh viên 20 30 40 10
a) Ước lượng đim trung bình môn Toán ca toàn sinh viên Đi hc A vi độ tin cy
%95
.
b) Vi độ chính xác ca ước lượng
25,0
. Hãy xác định độ tin cy.
Gii.
a) Da vào bng, ta có
3,5
100
530
==x
,
3,0
0
=
σ
,
100
=
n
,
%95
=
γ
suy ra
05,0
=
α
96,1
2
=
α
z
.
Khi đó, khong ước lượng đim trung bình
+
2
1
0
2
1
0
;
αα
σσ
z
n
xz
n
x
Hay
(
)
059,53;941,52
.
b) Ta có
25,0
2
0
==
α
σ
ε
z
n
. Suy ra
833,025,0
0
2
==
σ
α
n
z
.
Khi đó, t công thc
22
1
2
1
2
1
2
αα
α
==
Φ z
ta có
(
)
594,0833,02
Φ
=
γ
.
Vy đ tin cy là
%4,59
=
γ
.
2.1.2. Chưa biết phương sai
2
σ
.
Tương t như trên, ta thay
2
0
σ
bng phương sai mu hiu chnh
2
s
Chương V. Lí thuyết ước lượng
89
Ta chn hàm
n
S
X
TG
µ
==
.
Ta xét
2
trường hp sau:
TH1.
30
n
.
Khi
30
n
thì hàm
n
S
X
TG
µ
==
có phân phi Student vi
1
n
bc t do. Do
T
có hàm mt độ đối xng nên tương t câu a), ta được
2
;1
2
;1
αα
µ
+<<
nn
t
n
S
Xt
n
S
X
.
Khong ước lượng tin cy cho vng
+
2
;1
2
;1
;
αα
nn
t
n
S
Xt
n
S
X
, trong đó
2
;1
α
n
t
được xác định tng thc
α
α
=
>
2
;n
tTP
.
Chú ý:
2
;1
2
1;1
αα
=
nn
tt
.
TH 2.
30
>
n
.
Khi
30
>
n
thì hàm
n
S
X
TG
µ
==
phân phi tim cn chun
(
)
1;0
N
, vic tìm
khong ưc lượng vi độ tin cy
α
γ
=
1
được m như câu a) vi
2
0
σ
bng phương
sai mu hiu chnh
2
s
.
Vy khong ước lượng tin cy cho vng
+
22
;
αα
z
n
S
Xz
n
S
X
, trong đó
2
α
z
được xác định tng thc
22
1
2
1
2
γα
α
==
Φ z
vi
( )
=Φ
m
t
dtem
0
2
2
.
Ví d 2.3.
Cân ngu nhiên
25
bao xi măng, ta thu được bng kết qu sau:
i
x (kg)
47 48 49 50 51 52 53
i
n (s
bao)
2
3
4
6
5
3
2
Tìm khong tin cy
%95
cho trng lượng trung bình
µ
, biết trng lượng
X
ca bao xi
măng tuân theo phân phi chun
(
)
2
;
σµ
N
vi
σ
chưa biết.
Gii
Đây i toán ưc lượng khong tin cy cho giá tr trung bình vi phương sai chưa
biết.
Do chưa biết
σ
3025
<
=
n
nên ta
( )
1~
== nTn
S
X
TG
µ
.
Ta có trung bình
04,50=x
phương sai mu hiu chnh
153,36042,1307 ==
s
.
Bài ging
90
Ta có
064,2
2
05,0
;24
2
;1
=
=
tt
n
α
. Khi đó, khong tin cy cho kì vng là
+=
+
064,2.
5
153,36
04,50;064,2.
5
153,36
04,50;
2
;1
2
;1
αα
nn
t
n
s
xt
n
s
x
Ví d 2.4.
Mt nhà máy tiến hành mt nghiên cu xem trung bình mt công nhân hoàn thành bao
nhiêu sn phm trong mt tháng. Mt mu ngu nhiên gm
59
công nhân được chn
và kết qu như sau
14, 18, 22, 30, 36, 28, 42, 79, 36, 52, 15, 47, 95, 16, 27, 111, 37, 63, 127, 23, 31, 70,
27, 111, 30, 147, 72, 37, 25, 7, 33, 29, 35, 41, 48, 15, 29, 73, 26, 15, 26, 31, 57, 40, 18,
85, 28, 32, 22, 37, 60, 41, 35, 26, 20, 58, 33, 23, 35.
Hãy xây dng khong ưc lượng tin cy trung bình
µ
s sn phm mt công nhân
hoàn thành trong mt tháng vi độ tin cy
%95
=
γ
.
Gii.
T bng s liu trên, ta có
59
=
n
,
05,41=x
,
99,27
=
s
,
96,1
2
05,0
=
z
.
Do
3059
>
=
n
nên ta có khong tin cy cho
µ
+
22
;
αα
z
n
s
Xz
n
s
X
+= 96,1.
59
99,27
05,41;96,1.
59
99,27
05,41
.
2.2. Ước lượng khong tin cy cho phương sai.
Gi s biến ngu nhiên
X
phân phi chun
(
)
2
;~
σµ
NX
vi tham s
(
)
2
σθ
== XD
chưa biết. T mt mu c th
(
)
n
xxx ;;;
21
K
ca
X
. Bài toán đặt ra
tìm khong tin cy cho phương sai
2
σ
.
2.2.1. Đã biết k vng toán
(
)
0
µ
µ
=
=
XE
.
Ta chn hàm
( )
2
1
2
0
2
σ
µ
χ
=
==
n
i
i
X
G
, Ta có
2
χ
có phân phi
(
)
n
22
~
χχ
Khi đó, xét các phân v sau đây
2
2
2
;
2
α
χχ
α
=
>
n
P
2
1
2
2
1;
2
α
χχ
α
=
>
n
P
.
Ta có
2
2
;
22
2
1;
αα
χχχ
nn
<<
hay
( ) ( )
2
2
1;
1
2
0
2
2
2
;
1
2
0
αα
χ
µ
σ
χ
µ
==
<<
n
n
i
i
n
n
i
i
XX
.
Li có
αχχχ
αα
=
<<
1
2
2
;
22
2
1; nn
P
.
Chương V. Lí thuyết ước lượng
91
Vy
( ) ( )
α
χ
µ
σ
χ
µ
αα
=
<<
==
1
2
2
1;
1
2
0
2
2
2
;
1
2
0
n
n
i
i
n
n
i
i
XX
P
.
Khong ước lượng tin cy cho phương sai là
( ) ( )
==
2
2
1;
1
2
0
2
2
;
1
2
0
;
αα
χ
µ
χ
µ
n
n
i
i
n
n
i
i
XX
.
2.2.2. Chưa biết k vng toán
(
)
µ
=
XE
.
Ta chn hàm
(
)
2
1
2
2
σ
χ
=
==
n
i
i
XX
G
.
Ta có
2
χ
có phân phi
(
)
1~
22
n
χχ
. Thc hin tương t a), ta thu được
Vy
(
)
(
)
α
χ
σ
χ
αα
=
<<
=
=
1
2
2
1;1
1
2
2
2
2
;1
1
2
n
n
i
i
n
n
i
i
XXXX
P
.
Khong ước lượng tin cy cho phương sai là
(
)
(
)
=
=
2
2
1;1
1
2
2
2
;1
1
2
;
αα
χχ
n
n
i
i
n
n
i
i
XXXX
.
Ví d 2.5.
Mc hao phí nguyên liu cho mt đơn v sn phm là đại lượng ngu nhiên
X
có phân
phi chun
(
)
2
;
σµ
N
. Quan sát
28
sn phm, ta thu được kết qu sau
L
ượ
ng nguyên li
u hao phí
(kg)
19
5,19
20
5,20
S
s
n ph
m
5
6
14
3
Vi độ tin cy
%90
, hãy ước lượng phương sai ca
X
trong
2
trường hp sau
a) Biết
20
=
µ
kg.
b) Chưa biết
µ
.
Gii.
a) Đã biết
20
=
µ
kg. Ta
( )
25,7
4
1
2
=
=i
ii
Xn
µ
,
337,41
2
05,0;28
=
χ
,
928,16
2
95,0;28
=
χ
.
Khi đó, vi độ tin cy
%90
=
γ
, ta có khong ước lượng tin cy cho phương sai là
Bài ging
92
( ) ( )
==
2
2
1;
1
2
0
2
2
;
1
2
0
;
αα
χ
µ
χ
µ
n
n
i
i
n
n
i
i
xx
=
928,16
25,7
;
337,43
25,7
.
b) Ta có
(
)
25,7
4
1
2
=
=i
ii
xxn
,
113,40
2
05,0;128
=
χ
,
151,16
2
95,0;128
=
χ
. Khi đó, khong ước lượng tin
cy cho phương sai
(
)
(
)
=
=
2
2
1;1
1
2
2
2
;1
1
2
;
αα
χχ
n
n
i
i
n
n
i
i
xxxx
=
151,16
25,7
;
113,40
25,7
.
2.3. Ước lượng khong tin cy cho t l.
Nếu trong phép ly mu, ta ch quan m đến s xut hin hoc không xut hin tính
cht
A
nào đó. Gi
p
t l phn t tính cht
A
trong toàn b tng th,
p
chưa
biết. Gi s trong mt mu kích thước
n
k
phn t có tính cht
A
. Cơ s toán hc
cho vic xây dng khong tin cy cho t l
p
(chưa biết) là đnh lí sau đây:
Định lí.
Tn sut mu
n
k
f =
mt đại lượng ngu nhiên phân phi xp x chun vi
vng
(
)
pfE
=
và phương sai
( )
(
)
n
pp
fD
=
1
vi điu kin
( )
>
>
51
5
pn
np
.
Do ta không biết
p
nên ta không biết được
(
)
fD
. Tuy nhiên vi mt s điu kin, ta
th xp x
p
bi
f
. Nghĩa ta coi
( )
(
)
n
ff
fD
1
. Đ th dùng xp x này, ta
cn điu kin sau
( )
>
>
101
10
fn
nf
.
Khi đó, đại lượng ngu nhiên
(
)
(
)
( )
ff
npf
Df
pf
=
1
s phân phi xp x chun tc
(
)
1;0
N
.
Khi đó, để ước lượng khong tin cy cho t l vi độ tin cy
α
γ
=
1
Ta chn hàm:
( )
n
ff
pf
ZG
==
1
. Khi đó
(
)
1;0~
NZ
.
Khi đó,
22
αα
zZz
<
<
, hay
(
)
(
)
22
11
αα
z
n
ff
fpz
n
ff
f
+<<
.
T đẳng thc
α
αα
=
<< 1
22
zZzP
, sau khi biến đổi ta được
Chương V. Lí thuyết ước lượng
93
(
)
(
)
α
αα
=
+<<
1
11
22
z
n
ff
fpz
n
ff
fP
.
Khi đó, khong ước lượng tin cy cho t l
p
(
)
(
)
+
22
1
;
1
αα
z
n
ff
fz
n
ff
f
, trong đó
2
α
z
đưc xác định t công thc
222
1
2
γα
α
==
Φ
z
vi
( )
=Φ
m
t
dtem
0
2
2
.
Ví d 2.6.
Kim tra ngu nhiên
500
xe máy ca nhà máy A chuyên sn xut xe máy t
360
xe máy đạt cht lượng tt. Hãy ước lưng t l ti thiu xe máy đt cht lượng tt ca
nhà máy A vi độ tin cy
%95
.
Gii.
Đây là bài toán ưc lượng t l.
Ta có
7,0
500
360
==f
,
103507,0.500
>
=
=
nf
,
(
)
101501
>
=
fn
.
Khi đó
( )
500
3,0.7,0
7,0
1
p
n
ff
pf
ZG
=
==
,
95,0%95
=
=
γ
,
475,0
222
1
2
===
Φ
γα
α
z
nên
96,1
2
=
α
z
.
Vy khong ước lượng tin cy cho t l
(
)
(
)
+=
+
96,1.
500
3,0.7,0
7,0;96,1.
500
3,0.7,0
7,0
500
1
;
500
1
22
αα
z
ff
fz
ff
f
.
T l ti thiu xe máy đạt cht lượng là
66,0
=
p
.
Ví d 2.7.
Ti mt rng nguyên sinh, người ta đeo vòng cho
1000
con chim. Sau mt thi gian,
bt li
200
con thì
40
con đeo vòng. Th ước lượng s chim trong ng vi đ
tin cy
%99
.
Gii.
Đây là bài toán ưc lượng t l. Ta có:
2,0
200
40
==f
. Ta
10402,0.200
>
=
=
nf
(
)
10328,0.4001
>
=
=
fn
.
Khi đó,
90,0%90
=
=
γ
,
45,0
2
90,0
2
2
===
Φ
γ
α
z
nên
645,1
2
=
α
z
.
Khi đó, khong ước lượng tin cy cho t l là
Bài ging
94
(
)
(
)
+=
+
64,1.
200
1,0.9,0
2,0;64,1.
200
1,0.9,0
2,0
500
1
;
500
1
22
αα
z
ff
fz
ff
f
Hay
(
)
24,0;17,0
.
Khi đó, s chim trong vùng vi độ tin cy là
17,0
2000
;
24,0
2000
.
2.4. Ước lượng kích thước mu.
Vi độ tin cy
γ
đã cho, ta thy mi quan h gia kích thước mu
n
độ i
khong tin cy. Kích thước mu càng ln thì khong tin cy càng hp, nghĩa độ
chính xác ca ước lưng càng cao, sai s càng nh. Tuy nhiên, kích thước mu càng
ln thì đòi hi nhà nghiên cu càng nhiu thi gian, tin ca và công sc.
Vy bài toán đặt ra : Cn chn kích thước mu ti thiu bao nhiêu để đạt được
độ chính xác mong mun.
2.4.1. Trường hp ước lượng cho trung bình
µ
.
Gi s mun ước lượng
µ
vi sai s không quá
ε
cho trước vi độ tin cy
γ
.
Khi đó, vi xác sut
γ
, ta xét 2 trường hp sau
TH1: Nếu biết phương sai
2
σ
thì
n
zX
σ
µ
α
2
. Khi đó, ta cn bt đẳng thc
ε
σ
α
n
z
2
hay
2
2
ε
σ
α
z
n
, trong đó
2
α
z
được xác định t ng thc
222
1
2
γα
α
==
Φ
z
vi
( )
=Φ
z
t
dtez
0
2
2
.
Vy
n
là s nguyên dương nh nht tha mãn bt đẳng thc này.
TH2: Nếu không biết phương sai
2
σ
thì ta thay
σ
trong công thc
2
2
ε
σ
α
z
n
bi
s
.
Do
s
ph thuc
nên ta thưng ly mu có kích thước
30
>
m
để tính
x
s
. Vy
tha mãn công thc sau
2
2
ε
α
sz
n
vi điu kin vế phi ca công thc này ln hơn hoc bng
30
.
Ví d 2.8.
Biết rng độ lch tiêu chun chiu cao người ln
3
inch, ta mun xây dng mt
khong tin cy vi
%90
=
γ
cho chiu cao trung bình
µ
vi sai s không q
5,0
inch.
Hãy xác định kích thước mu ti thiu để đạt được yêu cu trên.
Chương V. Lí thuyết ước lượng
95
Gii
Ta biết phương sai
3
=
σ
,
5,0
=
ε
độ tin cy
%90
=
γ
.
Ta có
826,96
5,0
64,1.3
2
2
2
=
=
ε
σ
α
z
n
. Vy
97
=
n
.
2.4.2. Trường hp ước lượng cho t l
p
.
Gi s mun ước lượng
p
vi sai s không quá
ε
cho trước vi độ tin cy
γ
. Ta
s nguyên dương nh nht tha mãn
(
)
2
2
2
1
ε
α
ffz
n
, trong đó
2
α
z
được xác
định t công thc
222
1
2
γα
α
==
Φ z
vi
( )
=Φ
z
t
dtez
0
2
2
.
vi điu kin
( )
>
>
101
10
fn
nf
.
Ví d 2.9.
Phòng cnh sát giao thông mun ưc lượng t l xe ch quá ti trên đường vi độ tin
cy
%95
=
γ
sai s không vượt quá
05,0
thì cn phi kim tra bao nhiêu lượt xe
chy trên đường? Biết rng trong
100
xe đã kim tra thì
40
xe quá ti.
Gii
Đây là bài toán ưc lượng kích thc mu cho t l.
Ta có
4,0
100
40
==f
,
%95
=
γ
,
96,1
025,0
2
=
=
zz
α
,
05,0
=
ε
.
Khi đó, ta có:
(
)
( )
7936,368
05,0
4,01.4,0.96,1
1
2
2
2
2
2
=
=
ε
α
ffz
n
.
Do
là s t nhiên nên ta chn
369
=
n
.
Vây, s xe cn kim tra là
369
.
3. Bài tp chương.
1. Đ
o s
c b
n c
a m
t lo
i k
m công nghi
p, ng
ườ
i ta thu
đượ
c b
s
li
u sau
đ
ây
x
4500 4800 4900 5000 5125 5200 5375 6500
i
n
1 3 3 7 5 3 2 1
a)
Bi
ế
t r
ng s
c b
n c
a k
m có phân ph
i chu
n v
i
độ
l
ch
300
=
σ
. Hãy xây d
ng
kho
ng tin c
y cho s
c b
n trung bình c
a k
m v
i
độ
tin c
y %95
=
γ
.
b)
Tìm kho
ng tin c
y cho s
c b
n trung bình c
a k
m v
i gi
thi
ế
t là không bi
ế
t ph
ươ
ng
sai
2
σ
.
2.
Ki
m tra s
c kh
e c
a sinh viên m
t tr
ườ
ng
đạ
i h
c, ta thu
đượ
c chi
u cao c
a 500 sinh
viên nh
ư
sau.
Bài ging
96
Cao (cm)
S
sinh viên
150-154 20
154-158 60
158-162 120
162-164 140
164-168 80
168-172 60
172-174 20
Bi
ế
t
độ
l
ch tiêu chu
n c
a chi
u cao ng
ườ
i l
n là cm5 . Tìm kho
ng tin c
y cho chi
u cao
trung bình c
a sinh viên tr
ườ
ng
đạ
i h
c v
i
độ
tin c
y là
%95
.
3.
G
i
X
là m
c tiêu th
x
ă
ng c
a m
t lo
i ô tô (lít/
km100
). Ng
ườ
i ta ki
m tra
36
chi
ế
c
và thu
đượ
c k
ế
t qu
sau.
x
4,5 4,8 5,1 5,3 5,6 5,9 6,2 6,4
i
n
3 5 6 7 6 4 3 2
V
i
độ
tin c
y %95
=
γ
, hãy xác
đị
nh kho
ng
ướ
c l
ượ
ng cho m
c hao phí x
ă
ng trung bình
cho 100 km c
a lo
i ô tô này?
4.
Th
ng kê t
i m
t tr
m
đă
ng kí xe máy trong m
t tháng, trong 3600 xe m
i
đă
ng kí thì
240
xem Air Blade. Hãy
ướ
c l
ượ
ng t
l
ph
n tr
ă
m t
i
đ
a bán
đượ
c c
a lo
i xe Air
Blade trên th
tr
ườ
ng xe máy v
i
độ
tin c
y
%95
=
γ
.
5.
M
t nông dân mu
n
ướ
c l
ượ
ng t
l
n
y m
m cho m
t gi
ng lúa m
i. Khi ông ta
đ
i
u
tra 1000 h
t và th
y
đượ
c 640 h
t n
y m
m.
a)
V
i
độ
tin c
y 95%, hãy
ướ
c l
ượ
ng t
l
n
y m
m cho gi
ng lúa này.
b)
N
ế
u mu
n
ướ
c l
ượ
ng t
l
n
y m
m có sai s
không v
ượ
t quá 2% và
đạ
t
độ
tin c
y 95%
thì c
n gieo ít nh
t bao nhiêu h
t?
c)
V
i
độ
tin c
y 97%, hãy
ướ
c l
ượ
ng s
h
t gi
ng n
y m
m t
i thi
u khi gieo 10000 h
t.
6.
M
t kho hàng có
000.10
h
p s
a. Ng
ườ
i ta nghi ng
s
a b
h
ng, b
ng cách l
y ki
m
tra 250 h
p thì th
y có 5 h
p b
h
ư
.
a)
V
i
độ
tin c
y %95
=
γ
, hãy
ướ
c l
ượ
ng t
l
p
s
h
p s
a b
h
ư
trong kho.
b)
Ướ
c l
ượ
ng s
h
p s
a b
h
ư
trong kho v
i
độ
tin c
y
%95
=
γ
.
7.
Cho
X
là n
ă
ng su
t lúa
m
t khu v
c (
đơ
n v
tính t
/ha).
Đ
i
u tra
m
t s
th
a
ru
ng, ta có k
ế
t qu
sau
đ
ây:
X
30-35 35-40 40-45 45-50 50-55
(S
hecta)
6 10 28 40 16
a)
Hãy
ướ
c l
ượ
ng n
ă
ng su
t lúa trung bình c
a toàn vùng, v
i
độ
tin c
y
%95
=
γ
.
b)
Nh
ng th
a ru
ng
đạ
t n
ă
ng su
t t
45 t
/ha tr
lên là nh
ng th
a ru
ng
đạ
t n
ă
ng su
t
cao. Hãy
ướ
c l
ượ
ng t
l
nh
ng th
a ru
ng
đạ
t n
ă
ng su
t cao c
a vùng này, v
i
độ
tin
c
y %95
=
γ
.
c)
N
ế
u mu
n
ướ
c l
ượ
ng n
ă
ng su
t lúa trung bình c
a toàn vùng
đạ
t
đượ
c
độ
chính xác
4,1
=
ε
t
/ha thì
độ
tin c
y là bao nhiêu?
8.
Theo dõi chi
u cao c
a
144
y B
ch
Đ
àn tr
ng trên
đấ
t phèn sau
1
n
ă
m, ta
đượ
c k
ế
t
qu
sau
đ
ây.
X
(cm) 250-300 300-350 350-400 400-450 450-500 500-550 550-600
n
5 20 25 30 30 23 11
a)
Tính chi
u cao trung bình
độ
l
ch m
u c
a
X
.
b)
Hãy l
p kho
ng
ướ
c l
ượ
ng c
a chi
u cao trung bình lo
i cây B
ch
Đ
àn sau
1
n
ă
m v
i
độ
tin c
y %95
=
γ
.
Chương V. Lí thuyết ước lượng
97
Đáp s và hướng dn.
1.
a)
(
)
6,5200;4,4965 , b)
(
)
9,5227;1,4938
2.
30500
>
=
n
nên ta xem
(
)
σ
µ
;~
NX
.
(
)
16,163;28,162
.
3.
Ta có
3036
>
=
n
nên ta xem
(
)
σ
µ
;~
NX
và ch
ư
a bi
ế
t ph
ươ
ng sai
2
σ
.
4.
Kho
ng
ướ
c l
ượ
ng cho t
l
(
)
089,0;045,0 v
y, t
l
ph
n tr
ă
m t
i
đ
a là %9,8 .
5.
a)
(
)
6698,0;6102,0
b)
2213
=
n
, c)
6115
.
C. Phương pháp ging dy.
- Gii thiu ng dng ca ước lượng khong trong thc tế.
- Thuyết trình, vn đáp, và làm bài tp.
- S dng các bng ph lc cho vic tính các giá tr ca hàm phân phi chun, Poisson,
Student, chi bình phương.
- Yêu cu SV đọc bài ging trước khi lên lp.
- Kim tra, đánh giá vic làm bài tp ca SV.
Gi
ng viên g
i bài gi
ng cho sinh viên
đọ
c tr
ướ
c. Gi
ng viên trình bày bài gi
ng trên l
p theo
ph
ươ
ng pháp thuy
ế
t trình h
i
đ
áp. Giao bài t
p cho sinh viên v
nhà làm. Gi
i thi
u m
t s
tài
li
u tham kh
o.
D. Tài liu tham kho
[1] Đậu Thế Cp, Xác sut thng kê: thuyết các bài tp, NXB Giáo dc,
2006.
[2] Đặng Hùng Thng, Thng kê ng dng, NXB Giáo dc, 2008.
[3] PGS. TS. Phm Xuân Kiu, Giáo Trình xác sut thng , NXB Giáo dc,
2005.
[4] Trn Văn Minh, Phí Th Vân Anh, Xác sut thng vi các tính toán trên
Excel, NXB Giao Thông Vn ti, 2008.
[5] Đặng Công Hanh, Đặng Ngc Dc, Giáo trình thuyết xác sut Thng
toán, trường Đi hc Duy Tân,1996
[6] Trn Văn Minh, PTh Vân Anh, Hưng dn gii bài tp Xác sut thng
vi các tính toán trên Excel, NXB Giao Thông Vn ti, 2008.
Chương VI.
Kim định gi thiết thng kê.
A. Mc tiêu.
- Gii thiu v khái nim kim đnh gi thiết: cách đặt gi thiết
0
H
đối thiết
1
H
,
đưa ra các kh năng phm sai lm khi kim định.
- Nêu phương pháp chung khi thc hin mt bài toán kim định.
-
Đư
a ra ph
ươ
ng pháp ki
m
đị
nh cho: v
ng, ph
ươ
ng sai, t
l
trong t
ng tr
ườ
ng h
p c
th
.
B. Ni dung.
1. Các khái nim cơ bn
1.1. Đặt vn đề:
Trong chương này, chúng ta s gii quyết các bài toán dng như sau:
Gi s ta có hai gi thiết (hai kh năng) v mt vn đề nào đó, chng hn:
1. Có ý kiến cho rng tham s chưa biết
θ
ca phân phi nhn giá giá tr
0
θ
,
nhưng li có ý kiến cho rng
0
θ
θ
.
2. Có ý kiến cho rng sau khi áp dng phương pháp sn xut mi, t l sn phm
loi I ca nhà máy tăng lên (có nghĩa là phương pháp sn xut mi này có hiu
qu), nhưng có ý kiến cho rng t l sn phm loi I không thay đổi (có nghĩa là
phương pháp sn xut mi này không có hiu qu).
3. Có ý kiến cho rng Biến ngu nhiên đang xét tuân theo quy lut phân phi nh
thc, li có ý kiến không tán thành.
4. Có ý kiến cho rng kh năng làm vic ca con người ph thuc vào gii tính,
nhưng cũng có ý kiến cho rng không nh hưng.
Vn đề đặt ra là ta phi chn mt trong hai gi thiết được nêu ra. Nói cách khác, ta
chn gi thiết nào đ kh năng đúng cao hơn, kh năng sai thp hơn. Đ cho tin, ta
chn mt trong hai gi thiết gi thiết
0
H
còn gi thiết kia gi thiết đối (đối thiết)
1
H
. Khi đó
=
01
00
:
:
θθ
θθ
H
H
.
Bài gi
ng
100
Ví d 1.1.
Ông ch ca mt ca hàng buôn bán xe máy cho biết s xe máy n được trong mt
ngày ca ca hàng
30
xe máy. Đ kim tra li tuyên b này ca ông ch này
đúng hay sai, ta có th đặt:
=
30:
30:
1
0
θ
θ
H
H
.
Ví d 1.2.
Khi tìm hiu v chiu cao trung bình ca mt loi cây trong mt khu rng, vi chiu
cao
X
ca loi cây đó có phân phi chun
(
)
2
;
σµ
N
ta th đưa ra gi thiết
0
H
như
sau:
0
H
: “Chiu cao trung bình ca cây
20
=
µ
m”. Khi đó, các đối thiết ca
0
H
th là:
+ Đối thiết
1
H
: “Chiu cao trung bình ca cây là
20
µ
m”.
+ Đối thiết
1
H
: “Chiu cao trung bình ca cây là
20
<
µ
m”.
+ Đối thiết
1
H
: “Chiu cao trung bình ca cây là
20
>
µ
m”.
Để gii quyết bài toán này, thông tin duy nht chúng ta mu ngu nhiên
(
)
n
XXX ;;;
21
K
. Vn dng kết qu ca thuyết xác sut, ta tìm min
W
, sao cho khi
mu
(
)
WXXX
n
;;;
21
K
thì ta bác b gi thiết
0
H
, còn khi
(
)
WXXX
n
;;;
21
K
thì ta
chp nhn gi thiết
0
H
cho đến khi thông tin mi. Min
W
được gi là min bác
b.
Khi bác b hoc chp nhn gi thiết
0
H
, chúng ta có th mc hai loi sai lm sau:
Sai lm loi I: Bác b
0
H
nhưng thc tế
0
H
đúng.
Sai lm loi II: Chp nhn
0
H
nhưng thc tế
0
H
là sai.
Ta mong mun chn min
W
sao cho cc tiu c hai kh năng phm sai lm. Nhưng
khi mu c định thì mong mun này không th thc hin được, do đó thông thường
ta cho trước gii hn trên ca xác sut sai lm loi I.
hiu
α
,
α
thường rt nh
%1
=
α
,
%5
,
%10
,
K
ta s tìm min
α
W
sao cho kh
năng phm sai lm loi I không vượt quá
α
kh năng phm sai lm loi II đạt cc
tiu.
Tùy theo hoàn cnh c th, sai lm loi này có th tai hi hơn sai lm loi kia.
Ví d 1.3.
a) Để tránh sai lm cho đim mt hc sinh gii thp, thy giáo c cho đim cao mt
cách d dàng, khi đó kh năng mc phi sai lm cho đim mt hc sinh yếu kém cao
(đáng l phi cho đim thp) là tăng lên.
b) Để tránh sai lm cho vic bt nhm mt người vô ti (b oan), công an c th người
này cách d dàng, nhưng khi đó kh năng mc phi sai lm th nhm mt người ti
li tăng lên.
Có hai cách dùng để khng chế kh năng mc sai lm:
Cách th nht. Ta n định trước mc mc phi sai lm loi I sai lm loi II ri tính
toán tìm mt mu có kích thước ng vi hai mc sai lm này.
Cách th hai. Ta n định trước xác sut sai lm loi I (tc là trước mc ý nghĩa
α
)
chn min bác b
α
W
xác sut sai lm loi II nh nht.
Trong bài ging này, ta chn cách th hai.
Ch
ươ
ng VI. Ki
m
đị
nh gi
thi
ế
t th
ng kê
101
Cn chú ý rng: Bác b hoc chp nhn gi thiết tùy thuc vào giá tr thc nghim ca
tiêu chun
T
mc ý nghĩa
α
, kim định gi thuyết thng mt quy tc hành
động sao cho kh năng mc phi sai lm nh ( mc nào đó).
1.2. Phương pháp kim định gi thiết thng kê
Có th mô t phương pháp kim định gi thiết thng kê như sau:
Xut phát t yêu cu ca bài toán thc tế, ta đưa ra mt gi thiết
0
H
và gi thiết đi
1
H
ca nó.
Gi s rng
0
H
đúng, t đó tìm mt biến c xác sut đủ để th tin rng biến
c đó hu như không th xy ra trong mt phép th. Mun vy, t mu ngu nhiên
(
)
n
XXX ;;;
21
K
, ta chn thng
T
,
(
)
021
;;;;
θ
n
XXXfT
K
=
sao cho nếu
0
H
đúng thì
phân phi xác sut ca
T
xác định thng
T
gi tiêu chun kim định gi
thiết
0
H
.
Do quy lut phân phi xác sut
T
đã biết, nên vi
α
tùy ý, ta có th tìm được min
α
W
sao cho
(
)
α
α
=
WTP
.
Min
α
W
gi min bác b gi thiết
0
H
. Trong thc tế, ta thường ly
[
]
05,0;01,0
α
α
được gi là mc ý nghĩa ca kim định.
Thc hin mt phép th đối vi mu ngu nhiên
(
)
n
XXX ;;;
21
K
, ta được mu c th
(
)
n
xxx ;;;
21
K
. T mu này, ta tính được giá tr ca
T
,
(
)
021
;;;;
θ
n
xxxftT
K
=
=
.
Nếu
α
Wt
(tc là biến c
α
WT
xy ra) thì ta bác b
0
H
; chp nhn
1
H
.
Nếu
α
Wt
thì ta chp nhn
0
H
.
Chú ý.
Khi chp nhn
0
H
, không có nghĩa
0
H
đúng, mà ch nghĩa chưa cơ s bác b
0
H
, đành chp nhn
0
H
.
2. Kim định gi thiết v tham s.
2.1. Các loi kim định và phương pháp kim định gi thiết v các tham
s.
Khi nghiên cu mt đc nh hoc mt du hiu nào đó ca mt tng th ta xét mt
biến ngu nhiên
X
tác động lên tng th đó thường các du hiu ca tng th
được th hin qua các tham s đặc trưng ca
X
hay phân phi ca
X
, cho nên các gi
thiết v các tham s đặc trưng ca
X
cũng là các gi thiết thường gp.
Các loi kim định v tham s là:
1) Kim định hai phía đối vi tham s, tc kim định gi thiết
00
:
θ
θ
=
H
vi đi
thiết
01
:
θ
θ
H
vi
θ
tham s đặc trưng nào đó ca
X
chưa biết (thường
(
)
XE
hoc
(
)
XD
) và
0
θ
mt giá tr c th được đưa ra da vào s suy đoán nào đó.
2) Kim định phía phi đối vi tham s kim định gi thiết
00
:
θ
θ
=
H
vi đối thiết
01
:
θ
θ
>
H
.
3) Kim đnh phía trái đối vi tham s kim định gi thiết
00
:
θ
θ
=
H
vi đi thiết
01
:
θ
θ
<
H
.
Bài gi
ng
102
Để kim định gi thiết thng v các tham s như trên, người ta thường tiến hành
theo các bước như sau:
Bước 1: Lp mu ngu nhiên ca
X
(
)
n
XXX ;;;
21
K
chn mt hàm
(
)
021
;;;;
θ
n
XXXgG
K
=
gi tiêu chun kim đnh sao cho tìm được mt qui tc
kim định tt nht ca tham s
θ
. Sau đó tìm giá tr
(
)
021
;;;;
θ
n
xxxg
K
ng vi mt
mu thc nghim nào đó và gi là giá tr thc nghim.
Bước 2: Vi mc ý nghĩa
α
đã cho tìm min bác b
α
W
tương ng (min
α
W
ph
thuc vào các loi kim định).
Bước 3: Xét xem giá tr c th
(
)
021
;;;;
θ
n
xxxg
K
có thuc
α
W
hay không.
+ Nếu
(
)
α
θ
Wxxxg
n
021
;;;;
K
thì ta bác b gi thiết
0
H
tha nhn đi thiết
1
H
vi
mc ý nghĩa
α
(hay độ tin cy
α
γ
=
1
).
+ Nếu
(
)
α
θ
Wxxxg
n
021
;;;;
K
thì ta chưa có cơ s bác b gi thiết
0
H
tha nhn đối
thiết
1
H
vi mc ý nghĩa
α
.
Trong bài ging này, chúng ta ch kim định gi thiết v các tham s đặc trưng ca
biến ngu nhiên phân phi chun v t l ca tng th; các bước kim định được
trình bày mt cách ngn gn, ch nêu min bác b cho tng loi kim định.
2.2. Kim định gi thiết v trung bình ca ĐLNN X~N(µ; σ
2
).
Gi s trung bình ca tng th (cũng chính vng toán ca biến ngu nhiên
X
)
µ
chưa biết.
2.2.1. Trường hp đã biết
2
)(
σ
=XD
.
a) Kim định hai phía đối vi
µ
.
Cn kim đnh gi thiết:
00
:
µ
µ
=
H
vi đối thiết
01
:
µ
µ
H
(vi
0
µ
là mt giá tr nào
đó đã biết)
=
01
00
:
:
µµ
µµ
H
H
* Ta chn tiêu chun kim định
(
)
n
X
ZG
σ
µ
0
==
, biến ngu nhiên này phân
phi chun
(
)
1;0~
NZ
.
* Vi mc ý nghĩa
α
đã cho, ta xác định min bác b
α
W
như sau:
+∞
= ;;
22
ααα
zzW
, trong đó
2
α
z
được xác định t công thc
22
1
2
1
2
γα
α
==
Φ z
vi
( )
=Φ
m
t
dtem
0
2
2
.
* So sánh giá tr thc nghim
z
vi
2
α
z
.
+ Nếu
2
α
zz
>
(nghĩa
α
Wz
) thì ta bác b gi thiết
0
H
tha nhn đối thiết
1
H
vi mc ý nghĩa
α
.
Ch
ươ
ng VI. Ki
m
đị
nh gi
thi
ế
t th
ng kê
103
+ Nếu
2
α
zz
(nghĩa là
α
Wz
) thì ta chưa có cơ s bác b gi thiết
0
H
nên chp nhn
gi thiết
0
H
vi mc ý nghĩa
α
.
a) Kim định phía phi đối vi
µ
.
Cn kim định gi thiết:
00
:
µ
µ
=
H
vi đối thiết
01
:
µ
µ
>
H
>
01
00
:
:
µµ
µµ
H
H
* Ta chn tiêu chun kim định là
(
)
n
X
ZG
σ
µ
0
==
, biến ngu nhiên
Z
này
phân phi chun
(
)
1;0N
.
* Vi mc ý nghĩa
α
đã cho, ta xác định min bác b
α
W
như sau:
(
)
+∞
=
;
αα
zW
, trong đó
α
z
được xác đnh t công thc
( )
2
1
2
1
2
1
1 ===Φ
γαα
α
z
vi
( )
=Φ
m
t
dtem
0
2
2
.
* So sánh giá tr thc nghim
z
vi
α
z
.
+ Nếu
α
zz
>
(nghĩa
α
Wz
) thì ta bác b gi thiết
0
H
tha nhn đối thiết
1
H
vi
mc ý nghĩa
α
.
+ Nếu
α
zz
(nghĩa
α
Wz
) thì ta chưa cơ s bác b gi thiết
0
H
nên chp nhn
gi thiết
0
H
vi mc ý nghĩa
α
.
c) Kim định phía trái đối vi
µ
.
Cn kim định gi thiết:
00
:
µ
µ
=
H
vi đối thiết
01
:
µ
µ
<
H
<
01
00
:
:
µµ
µµ
H
H
* Ta chn tiêu chun kim định là
(
)
n
X
ZG
σ
µ
0
==
, biến ngu nhiên
Z
này
phân phi chun
(
)
1;0~
NZ
.
* Vi mc ý nghĩa
α
đã cho, ta xác định min bác b
α
W
như sau:
(
)
αα
=
1
; zW
, trong đó
α
1
z
được xác định t công thc
αα
zz
=
1
trong đó,
( )
2
1
2
1
2
1
1 ===Φ
γαα
α
z
vi
( )
=Φ
m
t
dtem
0
2
2
.
* So sánh giá tr thc nghim
z
vi
α
1
z
.
+ Nếu
α
zz
<
(nghĩa
α
Wz
) thì ta bác b gi thiết
0
H
tha nhn đi thiết
1
H
vi mc ý nghĩa
α
.
+ Nếu
α
zz
(nghĩa là
α
Wz
) thì ta chưa cơ s bác b gi thiết
0
H
nên chp
nhn gi thiết
0
H
vi mc ý nghĩa
α
.
Ví d 2.1.
Mt máy đóng gói t động quy định trng lưng trung bình
75
0
=
µ
g, độ lch
chun
15
=
σ
g. Sau mt thi gian sn xut, kim tra
80
gói, ta trng lượng trung
Bài gi
ng
104
bình mi gói
72
g. Cho kết lun v trng lượng gói trung bình sn xut vi mc
ý nghĩa
%5
=
α
đúng quy định không.
Gii
Trng lượng trung bình cho mi gói mì là
75
0
=
µ
g.
Trng lượng trung bình thc tế sn xut là
µ
chưa biết.
Ta đặt gi thuyết:
75:
00
=
=
µ
µ
H
; đối thiết
75:
01
=
µ
µ
H
.
Trong đó, ta biết:
15
=
σ
;
%5
=
α
.
Do đã biết
15
=
σ
nên ta chn tiêu chun kim định là
(
)
n
X
ZG
σ
µ
0
==
.
Đây là bài toán kim đnh hai phía.
Do
475,0025,0
2
1
2
05,0
==
Φ z
nên
96,1
2
05,0
=
z
Giá tr kim định:
79,180
15
7572
0
=
= n
x
z
σ
µ
.
Ta có
96,179,1 <
z
nên ta chưa cơ s đ bác b
0
H
nên chp nhn gi thuyết
0
H
,
tc là sn xut din ra bình thường.
Ví d 2.2.
Mt nhà máy cơ khí qui định chiu dài ca chi tiết máy A
20
cm. Ban giám đốc nhà
máy cho rng có mt t sn xut loi chi tiết máy đó không đúng chiu dài quy định.
Hãy kim định nhn định đó vi mc ý nghĩa
05,0
=
α
, biết rng sau khi chn ngu
nhiên
25
chi tiết máy ca t sn xut đó tkết qu đo đạc cho biết chiu dài trung
bình ca mu
5,20
cm chiu dài X ca loi chi tiết được sn xut ra t nhà máy
đó có phân phi chun
(
)
2
1;
µ
N
, vi
1
=
σ
đã biết.
Gii
Ta đặt gi thiết
20:
0
=
µ
H
cm và đối thiết
20:
1
µ
H
cm.
Ta có
25
=
n
,
5,20=x
cm,
1
=
σ
Do đã biết
1
=
σ
nên ta chn tiêu chun kim đnh
(
)
n
X
ZG
σ
µ
0
==
. Ta
(
)
1;0~
NZ
.
Ta có
5,225
1
205,20
=
=z
Đây là bài toán kim đnh hai phía
Vi mc ý nghĩa
05,0
=
α
, ta có
96,1
2
05,0
2
=
=
zz
α
.
Ta
96,15,2 >=z
nên ta bác b gi thiết
0
H
chp nhn gi thiết
1
H
vi mc ý
nghĩa
05,0
=
α
.
Vy kết lun ca giám đốc là đúng.
Ch
ươ
ng VI. Ki
m
đị
nh gi
thi
ế
t th
ng kê
105
2.1.2. Trường hp chưa biết
(
)
XD
.
Các bước kim định đối vi các loi gi thuyết này hoàn toàn tương t như trên nhưng
thay tiêu chun kim định
(
)
n
X
ZG
σ
µ
0
==
bi tiêu chun kim định
(
)
n
S
X
TG
0
µ
==
, biến ngu nhiên này phân phi Student
(
)
1
nT
, vi
1
n
bc
t do.
+ Trong trường hp
30
n
, ta thay
2
α
z
,
α
z
α
1
z
bi
2
;1
α
n
t
,
α
;1
n
t
α
1;1
n
t
.
+ Trong trường hp
30
>
n
, ta xem
(
)
n
S
X
ZG
0
µ
==
có phân phi chun
(
)
1;0N
.
Ví d 2.3.
Trng lượng ca mt loi sn phm do mt xí nghip sn xut theo qui định chung
6
kg. Sau mt thi gian sn xut, người ta tiến hành kim tra
121
sn phm do xí
nghip đó sn xut tính được s trung bình
975,5=x
kg độ lch tiêu chun điu
chnh là
4,2
=
s
.
Biết rng trng lượng
X
ca loi sn phm do nghip đó sn xut phân phi
chun
(
)
2
;
σµ
N
.
người cho rng trng lượng sn phm do xí nghip sn xut không đạt yêu cu
(không đúng vi qui định chung). y cho kết lun v li nhn xét đó vi mc ý
nghĩa
05,0
=
α
.
Gii
Ta đt gi thiết
6:
0
=
µ
H
kg và đối thiết
6:
1
µ
H
kg.
Ta có
975,5=x
,
121
=
n
,
4,2
=
s
.
Do
2
σ
chưa biết nên ta chn tiêu chun kim định
(
)
n
S
X
ZG
0
µ
==
.
Li có
30121
=
n
nên
(
)
1;0~ NZ
.
Ta có
(
)
1146,0121
4,2
6975,5
=z
.
Đây là bài toán kim định hai phía.
Vi mc ý nghĩa
05,0
=
α
, ta tính được
96,1
2
05,0
=
z
.
Ta
96,11146,0 <=
z
. Khi đó ta chưa cơ s để bác b
0
H
nên ta th kết lun
rng li nhn xét trên là không đúng vi
05,0
=
α
.
Ví d 2.4.
Tui th trung bình ca
1
loi bóng đèn do nhà máy A sn xut khi chưa ci tiến kĩ
thut sn xut
2000
gi, sau thi gian ci tiến kĩ thut người ta chn ngu nhiên
25
bóng đèn do nhà máy A sn xut cho thp kim tra th nghim kết qu cho biết
tui th trung bình ca mu thc nghim
2010=x
gi độ lch tiêu chun ca
mu
15
=
s
gi vi mc ý nghĩa
05,0
=
α
.
Bài gi
ng
106
Hãy cho biết kết lun v li nhn định “tui th bóng đèn tăng lên sau khi ci tiến
kĩ thut”. Biết rng biến ngu nhiên
X
ch tui th bóng đèn phân phi chun
(
)
2
;
σµ
N
.
Gii
Ta đặt gi thiết
2000:
0
=
µ
H
gi và đối thiết
2000:
1
>
µ
H
gi.
Ta có
25
=
n
,
15
=
s
.
Do
2
σ
chưa biết nên ta chn tiêu chun kim định là
(
)
n
S
X
ZG
0
µ
==
.
Li có
3025
<
=
n
nên
(
)
1~
nTZ
.
Đây là bài toán kim đnh phía phi.
Vi mc ý nghĩa
05,0
=
α
, ta tính được
064,2
05,0;125
=
t
.
Ta có
3
10
5.
15
20002010
=
=z
.
Ta có
064,2
3
10
>=z
Do đó, ta bác b gi thiết
0
H
và chp nhn đối thiết
1
H
tc là cho rng sau khi ci tiến
kĩ thut, tui th trung bình ca loi bong đèn tăng lên vi mc ý nghĩa
05,0
=
α
.
2.3. Kim định gi thiết v phương sai ca ĐLNN X~N(µ; σ
2
).
2.3.1. Trường hp đã biết
(
)
µ
=
XE
a) Kim định hai phía.
Cn kim định gi thiết:
2
0
2
0
:
σσ
=H
vi đối thiết
2
0
2
1
:
σσ
H
(vi
2
0
σ
mt giá tr
nào đó đã biết).
=
2
0
2
1
2
0
2
0
:
:
σσ
σσ
H
H
* Ta chn tiêu chun kim đnh
( )
2
0
1
2
2
σ
µ
χ
=
==
n
i
i
X
G
, biến ngu nhiên
2
χ
này
phân phi Chi bình phương vi
n
bc t do
(
)
n
22
~
χχ
.
* Vi mc ý nghĩa
α
đã cho, ta xác định min bác b
α
W
như sau:
+∞
=
;;
2
2
;
2
2
1;
ααα
χχ
nn
W
, trong đó
2
2
1;
α
χ
n
,
2
2
;
α
χ
n
được xác định da vào bng phân
phi Chi bình phương vi
bc t do.
* So sánh giá tr thc nghim
2
χ
vi
2
2
1;
α
χ
n
,
2
2
;
α
χ
n
.
+ Nếu
2
2
1;
2
α
χχ
<
n
hoc
2
2
;
2
α
χχ
n
>
(nghĩa là
α
Wz
) thì ta bác b gi thiết
0
H
tha
nhn đối thiết
1
H
vi mc ý nghĩa
α
.
Ch
ươ
ng VI. Ki
m
đị
nh gi
thi
ế
t th
ng kê
107
+ Nếu
2
2
;
22
2
1;
αα
χχχ
nn
(nghĩa
α
Wz
) thì ta chưa có cơ s bác b gi thiết
0
H
nên
chp nhn gi thiết
0
H
vi mc ý nghĩa
α
.
b) Kim định phía phi đối vi
2
σ
.
Cn kim định gi thiết:
2
0
2
0
:
σσ
=H
vi đi thiết
2
0
2
1
:
σσ
>H
.
>
2
0
2
1
2
0
2
0
:
:
σσ
σσ
H
H
Ta thc hin các bước tương t như trên nhưng min bác b ca kim đnh bên phi
(
)
+∞= ;
2
;
αα
χ
n
W
. So sánh giá tr thc nghim
2
χ
vi
2
;
α
χ
n
, ta đưa ra điu kết lun.
c) Kim định phía trái đối vi
2
σ
.
Cn kim định gi thiết:
2
0
2
0
:
σσ
=H
vi đi thiết
2
0
2
1
:
σσ
<H
.
<
2
0
2
1
2
0
2
0
:
:
σσ
σσ
H
H
Ta thc hin các bước tương t như trên nhưng min bác b ca kim định bên trái
(
)
2
1;
;
αα
χ
=
n
W
. So sánh giá tr thc nghim
2
χ
vi
2
1;
α
χ
n
, ta đưa ra điu kết lun.
2.3.2. Trường hp chưa biết
(
)
µ
=
XE
.
Trong thc tế, ta thường gp kim định các tham s ca biến ngu nhiên
(
)
2
;
σµ
N
vi
µ
σ
chưa biết. Các bước kim định ta thc hin tương t như trong trường hp đã
biết
(
)
µ
=
XE
, nhưng ta chn tiêu chun kim đnh
(
)
( )
2
0
2
2
0
1
2
2
1
σσ
χ
Sn
XX
G
n
i
i
=
==
=
, vi
S
độ lch mu hiu chnh, biến ngu nhiên
2
χ
này có phân phi Chi bình phương vi
1
n
bc t do
(
)
1~
22
n
χχ
.
Các cn ca min bác b
2
2
1;
α
χ
n
,
2
2
;
α
χ
n
,
2
1;
α
χ
n
,
2
;
α
χ
n
được thay bi
2
2
1;1
α
χ
n
,
2
2
;1
α
χ
n
,
2
1;1
α
χ
n
,
2
;1
α
χ
n
.
Ví d 2.5.
Kim tra s n định ca mt máy t đng v thi gian lp ghép mt b phn đin t.
Ngưi ta quan sát ngu nhiên
16
ln lp ghép ca máy t động đó và tính được độ lch
tiêu điu chnh ca mu thc nghim là
15=s
giây, gi s rng thi gian lp ghép
X
các b phn đin t ca máy t đng đó phân phi chun
(
)
2
;
σµ
N
máy được
xem n đnh nếu độ lch v thi gian không quá
5
giây. Hãy xét xem máy t động
trên hot động có n định không, biết rng mc ý nghĩa
05,0
=
α
?
Gii
Ta đt gi thiết
255:
22
0
==
σ
H
đối thiết
25:
2
1
>
σ
H
.
Do không biết
(
)
µ
=
XE
nên ta chn tiêu chun kim định
(
)
2
0
2
2
1
σ
χ
Sn
G
==
.
Bài gi
ng
108
Ta có
16
=
n
,
15=s
. Khi
(
)
9
25
225
25
15116
2
=
=
χ
.
Đây là bài toán kim đnh bên phi đối vi phương sai.
Vi mc ý nghĩa
05,0
=
α
, ta tính được
25
2
05,0;15
2
;1
==
χχ
α
n
Ta
259
<
nên ta chưa cơ s bác b gi thiết
0
H
nên chp nhn gi thiết
0
H
vi
mc ý nghĩa
05,0
=
α
.
2.4. Kim định gi thiết v t l các phn t có tính cht nào đó trong
tng th.
Cho mt tng th hai loi đối tưng, hiu
A
A
. Gi
p
t l s đối tượng
A
có trong tng th. Đi vi tham s
p
, ta có các loi kim định sau đây:
* Kim định hai phía.
Cn kim đnh gi thiết:
00
: ppH
=
vi đối thiết
01
: ppH
(vi
0
p
là mt giá tr nào
đó đã biết).
=
01
00
:
:
ppH
ppH
* Kim định phía phi.
Cn kim định gi thiết:
00
: ppH
=
vi đi thiết
01
: ppH
>
.
>
01
00
:
:
ppH
ppH
* Kim định phía trái.
Cn kim định gi thiết:
00
: ppH
=
vi đi thiết
01
: ppH
<
.
<
01
00
:
:
ppH
ppH
Như ta đã biết phn ước lượng t l ca tng th chương trước, nếu mu kích
thước
ln thì tn sut
=
=
n
i
i
X
n
X
1
1
bng s ln đối tưng
A
xut hin trong
ln
chn ngu nhiên mt đối tượng t tng th xp x biến ngu nhiên chun
(
)
(
)
pppN
1;
vi
n
khá ln.
Do đó, để kim định gi thiết v t l
p
ca tng th, ta chn tiêu chun kim định
( )
n
pp
pX
ZG
00
0
1
==
vi
khá ln, biến ngu nhiên
Z
này phân phi chun
(
)
1;0
N
.
Giá tr thc nghim
( ) ( )
n
pp
pf
n
pp
px
z
00
0
00
0
11
=
=
vi
fx =
t l đối tượng
A
trong mu vi
( )
>
>
101
10
0
0
pn
np
.
Vi mc ý nghĩa
α
khá ln tùy theo tng loi kim định, ta thc hin như sau:
* Đối vi kim định hai phía.
Ch
ươ
ng VI. Ki
m
đị
nh gi
thi
ế
t th
ng kê
109
Ta tìm
2
α
z
ri so sánh vi
z
+ Nếu
2
α
zz >
thì ta bác b gi thiết
0
H
và tha nhn đối thiết
1
H
vi mc ý nghĩa
α
.
+ Nếu
2
α
zz
thì ta chưa cơ s bác b gi thiết
0
H
nên chp nhn gi thiết
0
H
vi
mc ý nghĩa
α
.
* Đối vi kim định phía phi.
Ta tìm
α
z
ri so sánh vi
z
+ Nếu
α
zz
>
thì bác b gi thiết
0
H
và tha nhn đối thiết
1
H
vi mc ý nghĩa
α
.
+ Nếu
α
zz
thì ta chưa cơ s bác b gi thiết
0
H
nên chp nhn gi thiết
0
H
vi
mc ý nghĩa
α
.
* Đối vi kim định phía trái.
Ta tìm
α
1
z
ri so sánh vi
z
+ Nếu
αα
zzz
=
<
1
thì bác b gi thiết
0
H
và tha nhn đối thiết
1
H
vi mc ý nghĩa
α
.
+ Nếu
αα
zzz
=
1
thì ta chưa cơ s bác b gi thiết
0
H
nên chp nhn gi thiết
0
H
vi mc ý nghĩa
α
.
Ví d 2.6.
Mt nhà máy sn xut sn phm vi t l sn phm loi I lúc đầu
20,0
. Sau khi áp
dng phương pháp sn xut mi, kim tra
500
sn phm thy s sn phm loi I là
150
sn phm. Cho kết lun v phương pháp sn xut mi này vi mc ý nghĩa
%1
=
α
.
Gii
T l sn phm loi I lúc đầu
20,0
0
=
p
.
T l sn phm loi I khi áp dng phương pháp mi là
p
chưa biết.
Ta đt gi thiết:
2,0:
00
=
=
ppH
;
2,0:
01
=
ppH
.
Ta có
( )
>
>
101
10
0
0
pn
np
,
3,0
500
150
==f
;
500
=
n
;
Do
495,0005,0
2
1
2
01,0
==
Φ z
nên
58,2
995,0
2
01,0
=
=
zz
Giá tr kim định:
( )
n
pp
pf
z
n
00
0
1
=
Khi đó:
( )
59,5500
2,012,0
2,03,0
=z
Ta có
58,259,5
2
=>
α
zz
nên ta bác b gi thuyết
0
H
, tc là
0
pp
.
Khi đó, phương pháp sn xut mi đã làm thay đổi t l sn phm loi I.
Bài gi
ng
110
Ví d 2.7.
Mt công ti
A
sn xut bánh ko tuyên b rng
3
2
s tr em thích ăn bánh ca công ti.
Trong mt mu gm
100
tr em đưc hi,
55
em t ra thích bánh ca ng ti
A
.
Vi mc ý nghĩa
%5
, s liu nói trên chng t li tuyên b ca công ti hơi
quá đáng không?
Gii
Ta có gi thiết
3
2
:
00
== ppH
, đối thiết
3
2
:
0
<pH
.
Đây là kim định bên trái đối vi t l. Da vào các s liu ca mu, ta có:
( )
>
>
101
10
0
0
pn
np
,
55,0
100
55
==f
,
%5
=
α
nên
64,1
05,005,01
=
=
zz
Khi đó
( )
64,149,2100
3
2
1
3
2
3
2
55,0
1
00
0
<=
=
= n
pp
pf
z
.
Vy, li tuyên b ca công ti là quá s tht.
Ví d 2.8.
T l phế phm mt nhà máy lúc ban đầu
%10
. Sau mt thi gian, nhà máy ci
tiến kĩ thut sn xut, điu tra ngu nhiên
400
sn phm thy
30
phế phm. Vi
mc ý nghĩa
025,0
=
α
, hãy t xem vic ci tiến kĩ thut ca nhà máy m gim t
l phế phm ca nhà máy không?
Gii
Ta có gi thiết
1,0:
00
=
=
ppH
, đối thiết
1,0:
0
<
pH
.
400
=
n
,
075,0
400
30
==f
.
Đây là bài toán kim đnh bên trái đối vi t l.
Vi mc ý nghĩa
025,0
=
α
, ta có
96,1
025,0025,01
=
=
zz
.
Khi dó
( ) ( )
66,1400
1,011,0
1,0075,0
1
00
0
=
=
= n
pp
pf
z
Ta
025,0
zz >
ta chưa có cơ s c b gi thiết
0
H
nên chp nhn gi thiết
0
H
vi
mc ý nghĩa
025,0
=
α
. Tc vic ci tiến kĩ thut chưa làm t l phế phm gim
xung.
2.5. Kim định gi thiết v hai kì vng ca hai ĐLNN chun độc lp.
Gi s ta hai ĐLNN chun
(
)
2
11
;~
σµ
NX
mu ngu nhiên
(
)
n
XXX ;;;
21
K
(
)
2
22
;~
σµ
NY
có mu ngu nhiên là
(
)
m
YYY ;;;
21
K
.
2.5.1. Trường hp đã biết
2
1
σ
,
2
2
σ
.
a) Kim định hai phía ca
1
µ
.
Ch
ươ
ng VI. Ki
m
đị
nh gi
thi
ế
t th
ng kê
111
Cn kim định gi thiết:
210
:
µ
µ
=
H
vi đối thiết:
211
:
µ
µ
H
(vi
0
p
mt gtr
nào đó đã biết).
=
211
210
:
:
µµ
µµ
H
H
* Ta chn tiêu chun kim định
mn
YX
ZG
2
2
2
1
σσ
+
==
, biến ngu nhiên này phân
phi chun
(
)
1;0
N
.
* Vi mc ý nghĩa
α
đã cho, ta xác định min bác b
α
W
như sau:
+∞
= ;;
22
ααα
zzW
, trong đó
2
α
z
được xác định t công thc
22
1
2
1
2
γα
α
==
Φ z
vi
( )
=Φ
m
t
dtem
0
2
2
.
* So sánh giá tr thc nghim
z
vi
2
α
z
.
+ Nếu
2
α
zz >
(nghĩa
α
Wz
) thì ta bác b gi thiết
0
H
tha nhn đối thiết
1
H
vi mc ý nghĩa
α
.
+ Nếu
2
α
zz
(nghĩa là
α
Wz
) thì ta chưa có cơ s bác b gi thiết
0
H
nên chp nhn
gi thiết
0
H
vi mc ý nghĩa
α
.
b) Kim định phía phi ca
1
µ
.
Cn kim định gi thiết:
210
:
µ
µ
=
H
vi đối thiết:
211
:
µ
µ
>
H
.
>
211
210
:
:
µµ
µµ
H
H
* Ta chn tiêu chun kim định
mn
YX
ZG
2
2
2
1
σσ
+
==
, biến ngu nhiên này phân
phi chun
(
)
1;0
N
.
* Vi mc ý nghĩa
α
đã cho, ta xác định min bác b
α
W
như sau:
(
)
+∞
=
;
αα
zW
, trong đó
α
z
được xác đnh t công thc
( )
2
1
2
1
2
1
1 ===Φ
γαα
α
z
vi
( )
=Φ
w
t
dtew
0
2
2
.
* So sánh giá tr thc nghim
z
vi
α
z
.
+ Nếu
α
zz
>
(nghĩa
α
Wz
) thì ta bác b gi thiết
0
H
tha nhn đối thiết
1
H
vi
mc ý nghĩa
α
.
+ Nếu
α
zz
(nghĩa
α
Wz
) thì ta chưa cơ s bác b gi thiết
0
H
nên chp nhn
gi thiết
0
H
vi mc ý nghĩa
α
.
c) Kim định phía trái ca
1
µ
.
Bài gi
ng
112
Cn kim định gi thiết:
210
:
µ
µ
=
H
vi đi thiết:
211
:
µ
µ
<
H
.
<
211
210
:
:
µµ
µµ
H
H
* Ta chn tiêu chun kim định
mn
YX
ZG
2
2
2
1
σσ
+
==
, biến ngu nhiên này phân
phi chun
(
)
1;0N
.
* Vi mc ý nghĩa
α
đã cho, ta xác định min bác b
α
W
như sau:
(
)
αα
=
1
; zW
, trong đó
αα
zz
=
1
được xác định t công thc
( )
2
1
2
1
==Φ
γα
α
z
vi
( )
=Φ
w
t
dtew
0
2
2
.
* So sánh giá tr thc nghim
z
vi
α
1
z
.
+ Nếu
α
<
1
zz
(nghĩa là
α
Wz
) thì ta bác b gi thiết
0
H
tha nhn đi thiết
1
H
vi mc ý nghĩa
α
.
+ Nếu
α
1
zz
(nghĩa là
α
Wz
) thì ta chưa có cơ s bác b gi thiết
0
H
nên chp
nhn gi thiết
0
H
vi mc ý nghĩa
α
.
2.5.2. Trường hp chưa biết
2
1
σ
,
2
2
σ
và mu ln
30
>
+
nm
.
Các bước kim định ta thc hin tương t như trong trường hp đã biết
2
1
σ
2
2
σ
,
nhưng ta chn tiêu chun kim đnh
m
S
n
S
YX
ZG
2
2
2
1
+
==
, trong đó,
2
1
S
,
2
2
S
phương sai mu hiu chnh.
Ví d 2.9.
Người ta cân ngu nhiên
75
tr sơ sinh khu vc A
100
tr sơ sinh khu vc B,
kết qu cho theo bng sau đây
Các tham s
Khu vc
S tr được cân Trng lượng trung
bình
Phương sai điu
chnh
A
75
0,3=x
0,3
2
1
=s
B
100
2,3=y
0,5
2
2
=s
Có người cho rng trng lượng trung bình ca các tr sơ sinh hai khu vc trên là như
nhau. Hãy kim định li nhn xét đó vi mc ý nghĩa
05,0
=
α
. Gi s trng lượng
X
và trng lượng
Y
ca tr sơ sinh khu vc A và khu vc B đều có phân phi chun.
Gii
Gi
1
µ
vng toán ca biến ngu nhiên
X
(trng lượng trung bình ca toàn b tr
sơ sinh khu vc A)
2
µ
kì vng toán ca biến ngu nhiên
Y
(trng lượng trung
bình ca toàn b tr sơ sinh khu vc B).
Ch
ươ
ng VI. Ki
m
đị
nh gi
thi
ế
t th
ng kê
113
Ta đt gi thiết
210
:
µ
µ
=
H
đối thiết
211
:
µ
µ
H
.
Do chưa biết các phương sai
2
1
σ
,
2
2
σ
nên ta chn tiêu chun kim định
m
S
n
S
YX
ZG
2
2
2
1
+
==
.
Ta có
0,3=x
,
75
=
n
,
100
=
m
,
0,3=x
,
2,3=y
,
0,3
2
1
=s
0,5
2
2
=s
nên
3
2
3
10
.2,0
100
0,5
75
0,3
2,30,3
==
+
=z
Đây là bài toán kim định hai phía
Vi mc ý nghĩa
05,0
=
α
, ta có
96,1
2
05,0
2
=
=
zz
α
.
Ta
2
96,1
3
2
α
zz =<=
nên ta chưa cơ s bác b gi thiết
0
H
nên chp nhn gi
thiết
0
H
vi mc ý nghĩa
05,0
=
α
. Nghĩa cho rng trng lượng trung bình ca tr
sơ sinh hai khu vc là như nhau vi mc ý nghĩa
05,0
=
α
.
2.4.3. Trường hp
2
2
2
1
σσ
=
chưa biết và
302
+
mn
.
Các bước kim định ta thc hin tương t như trong trường hp đã biết
2
1
σ
2
2
σ
,
nhưng ta chn tiêu chun kim định
(
)
( ) ( )
[ ]
2
2
2
1
11
11
2
SmSn
mn
mnYX
TG
+
+
+
==
, biến
ngu nhiên này có phân phi Student
(
)
2
+
mnT
vi
2
+
mn
bc t do.
Trong đó,
2
1
S
,
2
2
S
phương sai mu hiu chnh và ta thay
2
α
z
,
α
z
α
1
z
bi
2
;2
α
+mn
t
,
α
;2+mn
t
,
α
+ 1;2mn
t
.
2.6. Kim định gi thiết thng kê v hai t l ca hai ĐLNN.
* Ta chn tiêu chun kim định là
( )
+
==
21
21
11
1
nn
FF
FF
ZG
, ĐLNN này phân
phi chun
(
)
1;0
N
, trong đó
21
2211
nn
FnFn
F
+
+
=
Giá tr thc nghim là
( )
+
=
21
21
11
1
nn
ff
ff
z
, trong đó
21
2211
nn
fnfn
f
+
+
=
.
a) Kim định hai phía ca
1
p
.
* Cn kim định gi thiết:
210
: ppH
=
vi đối thiết
211
:
ppH
(vi
0
p
mt giá tr
nào đó đã biết).
Bài gi
ng
114
=
211
210
:
:
ppH
ppH
* Vi mc ý nghĩa
α
đã cho, ta xác định min bác b
α
W
như sau:
+∞
= ;;
22
ααα
zzW
, trong đó
2
α
z
được xác định t công thc
22
1
2
1
2
γα
α
==
Φ z
vi
( )
=Φ
m
t
dtem
0
2
2
.
* So sánh giá tr thc nghim
z
vi
2
α
z
.
+ Nếu
2
α
zz >
(nghĩa
α
Wz
) thì ta bác b gi thiết
0
H
tha nhn đối thiết
1
H
vi mc ý nghĩa
α
.
+ Nếu
2
α
zz
(nghĩa là
α
Wz
) thì ta chưa có cơ s bác b gi thiết
0
H
nên chp nhn
gi thiết
0
H
vi mc ý nghĩa
α
.
b) Kim định phía phi ca
1
p
.
Cn kim định gi thiết:
210
: ppH
=
vi đi thiết
211
:
ppH
>
.
>
=
211
210
:
:
ppH
ppH
* Vi mc ý nghĩa
α
đã cho, ta xác định min bác b
α
W
như sau:
(
)
+∞
=
;
αα
zW
, trong đó
α
z
được xác định t công thc
( )
2
1
2
1
2
1
1 ===Φ
γαα
α
z
vi
( )
=Φ
w
t
dtew
0
2
2
.
* So sánh giá tr thc nghim
z
vi
α
z
.
+ Nếu
α
zz
>
(nghĩa
α
Wz
) thì ta bác b gi thiết
0
H
và tha nhn đối thiết
1
H
vi
mc ý nghĩa
α
.
+ Nếu
α
zz
(nghĩa
α
Wz
) thì ta chưa có cơ s bác b gi thiết
0
H
nên chp nhn
gi thiết
0
H
vi mc ý nghĩa
α
.
c) Kim định phía trái ca
1
p
.
Cn kim định gi thiết:
210
: ppH
vi đi thiết
211
: ppH
<
.
<
211
210
:
:
ppH
ppH
* Vi mc ý nghĩa
α
đã cho, ta xác định min bác b
α
W
như sau:
(
)
αα
=
1
; zW
, trong đó
α
1
z
được xác đnh t ng thc
( )
2
1
2
1
2
1
1
1
===Φ
γαα
α
z
vi
( )
=Φ
w
t
dtew
0
2
2
.
* So sánh giá tr thc nghim
z
vi
α
1
z
.
Ch
ươ
ng VI. Ki
m
đị
nh gi
thi
ế
t th
ng kê
115
+ Nếu
α
<
1
zz
(nghĩa
α
Wz
) thì ta bác b gi thiết
0
H
tha nhn đi thiết
1
H
vi mc ý nghĩa
α
.
+ Nếu
α
1
zz
(nghĩa là
α
Wz
) thì ta chưa cơ s bác b gi thiết
0
H
nên chp
nhn gi thiết
0
H
vi mc ý nghĩa
α
.
2.7. Kim định gi thiết thng kê v quy lut phân phi.
Ta đã biết rng khi
khá ln thì m phân phi thc nghim
(
)
xF
n
xp x hàm phân
phi
(
)
xF
ca biến ngu nhiên
X
, nhưng nhiu khi da vào s suy đoán, ta th
nhn biết được dng hàm phân phi
(
)
xF
ca biến ngu nhiên
X
.
Ta đt gi thiết
(
)
(
)
xFxFH
00
:
=
đối thiết
(
)
(
)
xFxFH
01
:
.
Để kim định gi thiết
0
H
, người ta dùng quy tc kim định Chi bình phương như
sau:
+ Ta lp mu ngu nhiên
(
)
n
XXX ;;;
21
K
ca
X
mu thc nghim
(
)
n
xxx ;;;
21
K
,
ta sp xếp mu thc nghim theo dng các bng phân phi không chia lp hoc chia
lp.
Xét xác sut:
+
(
)
ii
aXPp
=
=
hoc
(
)
iii
aXaPp
<
=
1
,
mi ;1=
nếu mu thc nghim ca
X
sp
xếp theo bng phân phi thc nghim không chia lp hoc chia lp và xác đnh các tn
sut
n
n
f
i
i
=
(vi
=
=
m
i
i
nn
1
) ca các gtr hoc ca các lp trong mu thc nghim, ta
xem lp
[
)
10
;aa
(
)
1
;
a
và lp
[
)
mm
aa ;
1
là lp
[
)
+∞
;
1m
a
.
X
1
x
2
x
m
x
i
n
1
n
2
n
m
n
X
[
)
(
)
110
;; aaa
=
[
)
21
;
aa
[
)
[
)
+∞
=
;;
11 mmm
aaa
i
n
1
n
2
n
k
n
Theo lut s ln Bernoulli, ta biết rng:
i
P
i
p
n
n
, khi
+∞n
,
mi ;1=
.
Biến ngu nhiên
(
)
n
np
n
np
npn
G
m
i
i
i
m
i
i
ii
=
==
== 1
2
1
2
2
χ
phân phi Chi bình phương
vi
1
rm
bc t do vi
khá ln,
m
s lượng c giá tr khác nhau hoc s lp
ng vi mu thc nghim cho theo bng phân phi thc nghim không chia lp v
r
s lượng các tham s chưa biết ca
(
)
xF
, các tham s này được ưc lượng bng
phương pháp hp lí cc đại.
Ta có quy tc kim định sau đây:
* Xác định giá tr thc nghim:
(
)
n
np
n
np
npn
m
i
i
i
m
i
i
ii
=
=
== 1
2
1
2
2
χ
.
* Vi mc ý nghĩa
α
, ta m s
2
,1
α
χ
rm
t bng phân phi Chi bình phương ri so
sánh vi
2
χ
.
Bài gi
ng
116
+ Nếu
22
;1
χχ
α
<
rm
thì bác b gi thiết
0
H
tha nhn đối thiết
1
H
vi mc ý nghĩa
α
.
+ Nếu
22
;1
χχ
α
rm
thì ta chưa cơ s bác b gi thiết
0
H
nên chp nhn gi thiết
0
H
vi mc ý nghĩa
α
.
Chú ý.
Các phân phi cơ bn cn kim định.
1. Nh thc:
(
)
pnBX ;~
.
+ Nếu
đã biết,
p
đã biết thì
0
=
r
.
+ Nếu
n
đã biết,
p
chưa biết thì
1
=
r
.
+ Nếu
chưa biết,
p
chưa biết thì
2
=
r
.
2. Poisson:
(
)
λ
PX ~
.
λ
chưa biết, ta thay
x=
λ
,
1
=
r
3. Chun:
(
)
2
;~
σµ
NX
Nếu
µ
,
σ
chưa biết, ta thay
x=
µ
,
s
=
σ
vi
( )
=
=
n
i
i
xx
n
s
1
2
1
1
phương sai mu
hiu chnh,
2
=
r
.
Ví d 2.10.
th cho rng s mt sp xut hin khi tung bn đồng tin đại lượng ngu nhiên
tuân theo quy lut phân phi xác sut nh thc
(
)
pnB ;
được không, nếu biết mc ý
nghĩa
01,0
=
α
.
Khi tung
100
ln thì người ta được kết qu sau đây
i
x
(s mt sp)
0
1
2
3
4
i
n
(s ln xut hin)
8
20
42
22
8
Gii
Gi
(
)
xF
hàm phân phi xác sut ca
X
(
)
xF
0
hàm phân phi ca biến ngu
nhiên có phân phi nh thc
(
)
pnB ;
. Ta
(
)
(
)
kn
kk
n
ppCkXP
==
1
.
Đặt gi thiết
(
)
(
)
xFxFH
00
:
=
đối thiết
(
)
(
)
xFxFH
00
:
.
Ta có
4
=
n
,
5,0
2
1
==p
.
Nếu
X
có phân phi nh thc
(
)
5,0;4
B
thì các xác sut
i
p
được xác định như sau
(
)
i
ii
i
Cp
+
=
4
41
5,015,0
;
4;0=i
Khi đó, ta có:
0625,0
1
=
p
,
25,0
2
=
p
,
375,0
3
=
p
,
25,0
3
=
p
,
0625,0
4
=
p
.
Để tính
(
)
=
=
m
i
i
ii
np
npn
1
2
2
χ
, ta lp bng sau đây
i
x
i
n
i
p
i
np
ii
npn
(
)
i
ii
np
npn
2
0
8
0625,0
25,6
75,1
49,0
Ch
ươ
ng VI. Ki
m
đị
nh gi
thi
ế
t th
ng kê
117
1
20
25,0
25
5
1
2
42
375,0
5,37
5,4
54,0
3
22
25,0
25
3
36,0
4
8
0625,0
25,6
75,1
49,0
Khi đó, ta có
(
)
88,2
5
1
2
2
=
=
=i
i
ii
np
npn
χ
.
Vi mc ý nghĩa
01,0
=
α
,
5
=
m
1
=
r
, ta có:
541,4
2
01,0;3
2
01,0;115
2
,1
===
χχχ
α
rm
.
Ta
22
;1
χχ
α
>
rm
nên ta chưa cơ s bác b gi thiết
0
H
nên chp nhn gi thiết
0
H
vi mc ý nghĩa
01,0
=
α
. Khi đó, ta cho rng s mt sp xut hin tuân theo phân
phi nh thc.
Ví d 2.11.
Gi
X
s ln khách đến bán ĐTDĐ mt ca hàng trong
96
ngày được cho theo
bng sau đây
X
(s ln khách đến)
0
1
2
3
4
i
n
(S ngày)
17
22
26
20
11
Vi mc ý nghĩa
01,0
=
α
, ta có xem
X
có phân phi Poisson được không?
Gii
Gi
(
)
xF
m phân phi xác sut ca
X
(
)
xF
0
hàm phân phi ca biến ngu
nhiên có phân phi Poisson
(
)
λ
P
. Ta có:
( )
!
2
i
i
x
e
xXP
λ
λ
==
Đặt gi thiết
(
)
(
)
xFxFH
00
:
=
đối thiết
(
)
(
)
xFxFH
00
:
.
Da vào bng, ta tính được
2
=
x
.
Nếu
X
có phân phi Poisson
(
)
2
P
thì các xác sut
i
p
được xác định như sau
!
2
2
i
x
i
x
e
p
i
=
;
5;1=i
Ta có:
1353,0
1
=
p
,
2707,0
2
=
p
,
2707,0
3
=
p
,
1804,0
4
=
p
,
0902,0
5
=
p
.
Đểnh
(
)
=
=
m
i
i
ii
np
npn
1
2
2
χ
, ta lp bng sau đây
i
x
i
n
i
p
i
np
ii
npn
(
)
i
ii
np
npn
2
0
17
1353,0
53,13
47,3
89,0
1
22
2707,0
07,27
07,5
95,0
2
26
2707,0
07,27
07,1
04,0
3
20
1804,0
04,18
96,1
21,0
4
11
0902,0
02,9
98,1
43,0
Khi đó, ta có
(
)
89,2
5
1
2
2
=
=
=i
i
ii
np
npn
χ
Bài gi
ng
118
Vi mc ý nghĩa
01,0
=
α
,
5
=
m
1
=
r
, ta có:
345,,11
2
01,0;3
2
01,0;115
2
,1
===
χχχ
α
rm
.
Ta
22
;1
χχ
α
>
rm
nên ta chưa cơ s bác b gi thiết
0
H
nên chp nhn gi thiết
0
H
vi mc ý nghĩa
01,0
=
α
. Khi đó, ta cho rng s ln
X
khách bán ĐTDĐ có phân
phi Poisson.
Ví d 2.12.
Đim trung bình hc tp ca
100
sinh viên được cho bng s liu sau đây
[
)
ii
aa ;
1
i
n
0-3
8
3-5
11
5-7
50
7-8
22
8-10
9
Vi mc ý nghĩa
05,0
=
α
, kim đnh gi thiết nói rng đim trung bình hc tp ca
sinh viên tuân theo lut phân phi chun.
Gii
Gi
(
)
xF
hàm phân phi xác sut ca
X
(
)
xF
0
hàm phân phi ca biến ngu
nhiên chun
(
)
2
;
σµ
N
,
( )
2
1
0
+
Φ=
σ
µ
x
xF
.
Đặt gi thiết
(
)
(
)
xFxFH
00
:
=
đối thiết
(
)
(
)
xFxFH
00
:
Da vào bng s liu, ta tính được:
02,6=x
,
22
84,1=s
Nếu
X
phân phi chun thì
(
)
2
84,1;02,6~ NX
các xác sut
i
p
được tính như
sau:
( ) ( )
051,0
2
1
84,1
02,63
33
1
=+
Φ==<= FXPp
( ) ( ) ( )
239,0
84,1
02,63
84,1
02,65
3553
2
=
Φ
Φ==<= FFXPp
( ) ( ) ( )
412,0
84,1
02,65
84,1
02,67
5775
3
=
Φ
Φ==<= FFXPp
( ) ( ) ( )
156,0
84,1
02,67
84,1
02,68
7887
4
=
Φ
Φ==<= FFXPp
( ) ( )
141,0
84,1
02,68
2
1
318
5
=
Φ=== FXPp
Để tính
(
)
=
=
m
i
i
ii
np
npn
1
2
2
χ
, ta lp bng sau đây
i
n
i
p
i
np
ii
npn
(
)
i
ii
np
npn
2
8 0,051 5,069 2,931 1,694
11 0,239 23,934 -12,934 6,99
Ch
ươ
ng VI. Ki
m
đị
nh gi
thi
ế
t th
ng kê
119
50 0,412 41,246 8,754 1,858
22 0,156 15,610 6,390 2,616
9 0,141 14,140 -5,140 1,869
Khi đó, ta có
(
)
026,15
5
1
2
2
=
=
=i
i
ii
np
npn
χ
.
Vi mc ý nghĩa
05,0
=
α
,
5
=
m
2
=
r
, ta có:
992,5
2
05,0;2
2
05,0;125
2
,1
===
χχχ
α
rm
.
+ Nếu
22
;1
χχ
α
<
rm
thì bác b gi thiết
0
H
tha nhn đối thiết
1
H
vi mc ý nghĩa
α
. Như vy, ta không th coi đim trung bình hc tp ca sinh viên phân phi
chun.
Ví d 2.13.
Kim tra ngu nhiên
130
cây trong mt khu rng và đo chiu cao ca chúng, kết
qu thu được cho theo bng sau đây:
[
)
ii
aa ;
1
30-36 36-42 42-48 48-54 54-60 60-66 66-72
i
n
2
8
35
43
22
15
5
người cho rng chiu cao
X
ca loi cây này phân phi chun
(
)
2
;
σµ
N
. Hãy
kim đnh li nhn định này vi mc ý nghĩa
05,0
=
α
.
Gii
Gi
(
)
xF
m phân phi xác sut ca
X
(
)
xF
0
hàm phân phi ca biến ngu
nhiên chun
(
)
2
;
σµ
N
,
( )
2
1
0
+
Φ=
σ
µ
x
xF
.
Đặt gi thiết
(
)
(
)
xFxFH
00
:
=
đối thiết
(
)
(
)
xFxFH
00
:
.
Da vào bng, ta tính được
5,51=x
68,7
=
s
.
Nếu
X
có phân phi chun thì
(
)
(
)
2
68,7;5,51~ NX
các xác sut
i
p
được tính như
sau:
( ) ( )
022,0
2
1
68,7
5,5136
3636
01
=+
Φ==<= FXPp
.
( ) ( ) ( )
087,0
68,7
5,5136
68,7
5,5142
36424236
002
=
Φ
Φ==<= FFXPp
( ) ( ) ( )
217,0
68,7
5,5142
68,7
5,5148
42484842
003
=
Φ
Φ==<= FFXPp
( ) ( ) ( )
303,0
68,7
5,5148
68,7
5,5154
48545448
004
=
Φ
Φ==<= FFXPp
( ) ( ) ( )
237,0
68,7
5,5154
68,7
5,5160
48546054
005
=
Φ
Φ==<= FFXPp
( ) ( ) ( )
104,0
68,7
5,5160
68,7
5,5166
48666660
006
=
Φ
Φ==<= FFXPp
( )
029,0
68,7
5,5166
2
1
66
7
=
Φ== XPp
Bài gi
ng
120
Để tính
(
)
=
=
m
i
i
ii
np
npn
1
2
2
χ
, ta lp bng sau đây
i
n
i
p
i
np
ii
npn
(
)
i
ii
np
npn
2
2
022,0
86,2
86,0
26,0
8
087,0
3,11
3,3
96,0
35
217,0
23,28
77,6
62,1
43
303,0
45,39
55,3
32,0
22
237,0
86,30
86.8
55,2
15
104,0
51,13
49,1
16,0
5
029,0
79,3
21,1
39,0
Khi đó, ta có
(
)
26,6
1
2
2
=
=
=
m
i
i
ii
np
npn
χ
.
Vi mc ý nghĩa
05,0
=
α
,
7
=
m
2
=
r
, ta có:
5,9
2
05,0;4
2
05,0;127
2
,1
===
χχχ
α
rm
.
Ta
22
;1
χχ
α
>
rm
nên ta chưa cơ s bác b gi thiết
0
H
nên chp nhn gi thiết
0
H
vi mc ý nghĩa
05,0
=
α
. Khi đó, ta cho rng chiu cao
X
ca loi cây trên
phân phi chun vi mc ý nghĩa
05,0
=
α
.
2.8. Kim định gi thiết thng kê v tính độc lp.
Gi s ta có mu ngu nhiên hai quan sát đồng thi v hai ĐLNN
X
Y
. T mu
kích thước
, ta có bng s liu sau đây
(
)
YX ;
1
y
2
y
K
h
y
Tng
1
x
11
n
12
n
K
h
n
1
1
n
2
x
21
n
22
n
K
h
n
2
2
n
K
K
K
K
K
K
k
x
1k
n
2k
n
K
kh
n
k
n
Tng
1
m
2
m
K
h
m
=
n
Trong đó,
=
=
h
j
iji
nn
1
,
ki ;1=
,
=
=
k
i
ijj
mm
1
,
hj ;1=
, và
=
k
i
h
j
ij
nn
Ta đặt gi thiết:
0
H
:
X
Y
độc lp.
Đối thiết:
1
H
:
X
Y
không độc độc lp.
Vi mc ý nghĩa
α
cho trưc. Hãy kim định gi thiết
0
H
.
Ch
ươ
ng VI. Ki
m
đị
nh gi
thi
ế
t th
ng kê
121
Biến ngu nhiên
==
= =
k
i
h
j
ji
ij
mn
n
nG
1 1
2
2
1
χ
phân phi Chi bình phương vi
(
)
(
)
11
hk
bc t do.
Ta có quy tc kim định sau đây:
* Xác định giá tr thc nghim:
=
= =
k
i
h
j
ji
ij
mn
n
n
1 1
2
2
1
χ
.
* Vi mc ý nghĩa
α
, ta tìm s
( )( )
2
;11
α
χ
hk
t bng phân phi Chi bình phương ri so
sánh vi
2
χ
.
+ Nếu
( )( )
22
;11
χχ
α
<
hk
thì bác b gi thiết
0
H
tha nhn đối thiết
1
H
vi mc ý
nghĩa
α
.
+ Nếu
( )( )
22
;11
χχ
α
hk
thì ta chưa cơ s bác b gi thiết
0
H
nên chp nhn gi thiết
0
H
vi mc ý nghĩa
α
.
Ví d 2.14.
Trong mt nhà máy dt may, mt nhà thng theo dõi
1000
công nhân m vic
trong mt năm và thy s ngày ngh vic ca h được cho trong bng sau
S ngày ngh vic trong năm
Nam
N
0-10 300 500
10-20 80 70
20 tr lên 20 30
Vi mc ý nghĩa
%5
=
α
, nhà thng này có th cho rng s ngày ngh ca công
nhân có ph thuc vào gii tính không?
Gii
Gi
X
là s ngày ngh ca công nhân,
Y
là gii tính ca công nhân.
Da vào bng s liu, ta có:
(
)
YX ;
1
y
2
y
Tng
1
x
300 500 800
2
x
80 70 150
3
x
20 30 50
Tng 400 600
1000
Đây là bài toán kim định gi thiết v tính độc lp ca
X
Y
.
Ta đt gi thiết: :
0
H
:
X
Y
độc lp.
Đối thiết:
1
H
:
X
Y
không đc độc lp.
Ta chn giá tr kim định:
=
= =
k
i
h
j
ji
ij
mn
n
n
1 1
2
2
1
χ
.
Khi đó, ta có:
Bài gi
ng
122
19,131
600.50
30
400.50
20
600.150
70
400.150
80
600.800
500
400.800
300
1000
222222
2
=
+++++=
χ
Vi mc ý nghĩa
%5
=
α
,
3
=
k
,
2
=
h
, ta có
( )( )
992,5
2
05,0;2
2
;11
==
χχ
α
hk
.
Ta
( )( )
22
;11
χχ
α
<
hk
thì bác b gi thiết
0
H
tha nhn đối thiết
1
H
vi mc ý
nghĩa
%5
. Vy s ngày ngh ca công nhân ph thuc vào gii tính.
3. Bài tp chương.
1.
Tr
ng l
ượ
ng
X
c
a s
n ph
m do m
t nhà máy s
n xu
t ra là m
t bi
ế
n ng
u nhiên có
phân ph
i chu
n
(
)
2
;
σµ
N
v
i
2
=
σ
kg và tr
ng l
ượ
ng trung bình là
20
=
µ
kg. Nghi
ng
nhà máy ho
t
độ
ng không bình th
ườ
ng làm thay
đổ
i tr
ng l
ượ
ng trung bình c
a s
n
ph
m, ng
ườ
i ta cân
100
s
n ph
m và k
ế
t qu
thu
đượ
c cho theo b
ng sau:
i
x : tr
ng l
ượ
ng s
n ph
m
18 19 20 21 22
i
n : s
s
n ph
m t
ươ
ng
ng
5 25 40 20 10
Hãy ki
m
đị
nh
đ
i
u nghi ng
trên v
i m
c ý ngh
ĩ
a 05,0
=
α
v
i gi
thi
ế
t: 20:
0
=
µ
H kg và
đố
i thi
ế
t 20:
0
µ
H .
2.
Theo k
thu
t quy
đị
nh thi
ế
t k
ế
quy
đị
nh chi
u dài trung bình c
a m
t chi ti
ế
t máy do
nhà máy A s
n xu
t là
20
cm. Sau m
t th
i gian s
n xu
t, có ý ki
ế
n cho r
ng nhà máy
A s
n xu
t lo
i chi ti
ế
t máy trên không
đạ
t yêu c
u.
Để
ki
m tra, ng
ườ
i tra ch
n ng
u
nhiên
64
chi ti
ế
t và
đ
o (phép
đ
o không có sai s
) k
ế
t qu
thu
đượ
c: chi
u dài trung bình
5,20=x cm và
độ
l
ch tiêu chu
n hi
u ch
nh là 1
=
s cm. Bi
ế
t r
ng chi
u dài lo
i chi
ti
ế
t trên là bi
ế
n ng
u nhiên chu
n
(
)
2
;
σµ
N . Hãy ki
m
đị
nh
đ
i
u nghi ng
trên v
i m
c
ý ngh
ĩ
a
05,0
=
α
v
i gi
thi
ế
t: 20:
0
=
µ
H cm và
đố
i thi
ế
t 20:
0
µ
H cm.
3.
M
t nhà th
ng kê theo dõi m
c thu nh
p c
a m
t s
ng
ườ
i
Công ty May và thu
đượ
c
s
li
u nh
ư
sau.
X (tr
ă
m ngàn) 8 11 13 15 17 19 21 23
n
(S
ng
ườ
i)
4 16 25 30 26 20 15 8
a)
Tính thu nh
p trung bình c
a
X
độ
l
ch chu
n
đ
i
u ch
nh
s
c
a thu nh
p
X
.
b)
V
i
độ
tin c
y
%95
=
γ
. Hãy
ướ
c l
ượ
ng thu nh
p trung bình c
a toàn b
công nhân
Công ty May.
c)
N
ế
u nhà th
ng kê cho r
ng thu nh
p m
i tháng
7,1
X
tri
u là cao. Hãy
ướ
c l
ượ
ng t
l
p
c
a nh
ng ng
ườ
i có thu nh
p cao trong Công ty v
i
độ
tin c
y
%99
=
γ
.
d)
N
ế
u ban giám
đố
c báo cáo r
ng thu nh
p trung bình là 6,1 tri
u. Nhà th
ng kê d
a vào
m
u k
ế
t qu
trên v
i m
c ý ngh
ĩ
a %5
=
α
. Nhà th
ng kê có th
tin c
y vào ý ki
ế
n
này không?
4. Đ
i
u tra doanh s
n hàng
X
(tri
u
đồ
ng/tháng) c
a các h
kinh doanh m
t lo
i hàng
n
ă
m nay, ta
đượ
c s
li
u sau
đ
ây
X
(tri
u/tháng) 11 11,5 12 12,5 13 13,5
S
h
10 15 20 30 15 10
a)
N
ế
u bi
ế
t r
ng nh
ng h
có doanh s
trên 12,5 tri
u / tháng là nh
ng h
có doanh s
cao.
Có bài báo công b
r
ng t
l
h
có doanh s
cao là %35 . Cho nh
n xét v
t
l
nh
ng
h
có doanh s
cao trong bài báo này v
i m
c ý ngh
ĩ
a
%5
.
Ch
ươ
ng VI. Ki
m
đị
nh gi
thi
ế
t th
ng kê
123
b)
N
ă
m tr
ướ
c, doanh s
n hàng c
a các h
y là 120 tri
u / n
ă
m (t
c là 10 tri
u /
tháng). Có th
cho r
ng doanh s
n hàng c
a các h
này n
ă
m nay t
ă
ng lên không v
i
m
c ý ngh
ĩ
a
%1
.
5.
M
t công ti kinh doanh xe
đạ
p
đ
i
n tuyên b
r
ng 60% khách hàng
ư
a thích s
n ph
m
c
a công ti.
Đ
i
u tra 400 khách hàng thì có 230 khách hàng
ư
a thích s
n ph
m c
a tông
ti này. V
i m
c ý ngh
ĩ
a
%5
=
α
, hãy xem t
l
trong tuyên b
c
a công ti có
đ
úng
không?
6.
Tr
ng l
ượ
ng c
a m
t gói b
t ng
t do m
t máy t
độ
ng
đ
óng theo thi
ế
t k
ế
là 500
gram/gói. Nghi ng
máy t
độ
ng
đ
óng gói này làm vi
c không bình th
ườ
ng làm cho
tr
ng l
ượ
ng c
a gói b
t ng
t có xu h
ướ
ng gi
m sút. Ng
ườ
i ta l
y ng
u nhiên 30 gói, cân
th
đượ
c tr
ng l
ượ
ng trung bình là 495 gram
độ
l
ch tiêu chu
n hi
u ch
nh 10
=
s
gram. V
i m
c ý ngh
ĩ
a
%5
=
α
, hãy cho k
ế
t lu
n v
nghi ng
này.
7.
Tr
ướ
c
đ
ây,
đị
nh m
c tiêu dùng
đ
i
n c
a m
t h
gia
đ
ình trong m
t tháng là 140 kW. Do
đờ
i s
ng nâng cao, ng
ườ
i ta theo dõi 100 h
gia
đ
ình và thu
đượ
c s
li
u sau
đ
ây
L
ượ
ng tiêu dùng
(kW)
100-120 120-140 140-160 160-180 180-200
S
h
gia
đ
ình 14 25 30 20 11
a)
V
i m
c ý ngh
ĩ
a
%5
=
α
, theo b
n có nên t
ă
ng
đị
nh m
c lên không?
b)
N
ế
u tr
ướ
c
đ
ây,
độ
bi
ế
n
độ
ng c
a m
c tiêu dùng
đ
i
n cho m
t h
gia
đ
ình là 400 (kW)
2
.
V
y, hi
n nay,
độ
bi
ế
n
độ
ng t
ă
ng không? Hãy cho k
ế
t lu
n v
i m
c ý ngh
ĩ
a
%5
=
α
.
8.
M
t
đạ
i lí xe máy
đ
ã kí h
p
đồ
ng v
i 2 nhà cung c
p A và B s
n xu
t th
linh ki
n
khung cho xe Dream II. D
a vào k
ế
t qu
th
khung,
đạ
i lí s
ch
n nhà cung c
p nào cho
đạ
i lí c
a mình. Nhà cung c
p A
đ
ã s
n xu
t th
đượ
c 10 cái v
i
độ
b
n trung bình là
4,8 tháng và
độ
l
ch tiêu chu
n là 1,1 tháng; nhà cung c
p B s
n xu
t th
13 cái có
độ
b
n trung bình là 4,3 tháng và
độ
l
ch tiêu chu
n là 0,9 tháng. Cho m
c ý ngh
ĩ
a
%10
=
α
, gi
s
độ
b
n c
a hai lo
i khung do các nhà cung c
p A và B s
n xu
t có
phân ph
i chu
n. N
ế
u bi
ế
t
độ
n
đị
nh v
độ
b
n (ph
ươ
ng sai) c
a hai lo
i khung là nh
ư
nhau, hãy xem tu
i th
trung bình c
a hai lo
i khung có khác nhau không?
9. Độ
l
ch tiêu chu
n c
a tr
ng l
ượ
ng X c
a m
t lo
i s
n ph
m là 0,1 kg. Nghi ng
độ
đồ
ng
đề
u c
a tr
ng l
ượ
ng s
n ph
m gi
m sút, ng
ườ
i ta cân th
25 s
n ph
m và thu
đượ
c
s
li
u sau
đ
ây
X (kg) 2,1 2,2 2,3 2,4 2,5
S
s
n ph
m 2 4 15 3 1
V
i m
c ý ngh
ĩ
a %5
=
α
, hãy cho bi
ế
t k
ế
t lu
n v
đ
i
u nghi ng
trên. Gi
thi
ế
t tr
ng l
ượ
ng
s
n ph
m có phân ph
i chu
n.
C. Phương pháp ging dy.
- Đưa ví d c th để thy rõ ng dng ca kim định.
- S dng các bng ph lc cho vic tính các giá tr ca hàm phân phi chun, Poisson,
Student, chi bình phương.
- Phi hp phương pháp thuyết trình và vn đáp gii quyết vn đề.
- Yêu cu SV đọc bài ging trước khi lên lp.
- Kim tra, đánh giá vic làm bài tp ca SV.
D. Tài liu tham kho
[1] Đu Thế Cp, Xác sut thng kê: Lí thuyết và các bài tp, NXB Giáo dc, 2006.
Bài gi
ng
124
[2] Đng Hùng Thng,
Th
ng kê và
ng d
ng
, NXB Giáo dc, 2008.
[3] PGS. TS. Phm Xuân Kiu,
Giáo Trình xác su
t th
ng
, NXB Giáo dc,
2005.
[4] Trn Văn Minh, Phí Th Vân Anh,
Xác su
t th
ng kê v
i các tính toán trên Excel
,
NXB Giao Thông Vn ti, 2008.
[5] Đng Công Hanh, Đặng Ngc Dc,
Giáo trình Lý thuy
ế
t xác su
t Th
ng
toán
, trường Đại hc Duy Tân,1996
[6] Trn Văn Minh, Phí Th Vân Anh,
H
ướ
ng d
n gi
i bài t
p Xác su
t th
ng v
i
các tính toán trên Excel
, NXB Giao Thông Vn ti, 2008.
Các b
ng s
125
Các bng s
Bng 1. Bng phân phi Poisson:
( )
!
k
e
kXP
k
λ
λ
==
,
(
)
0905,00,1.0,1
=
POISSON
,
( )
!
1
1.0
1
11.0
×
==
e
XP
(k;λ)
0.1
0.2
0.3
0.4
0.5
0.6
0.7
0.8
0.9
0
0.9048
0.8187
0.7408
0.6703
0.6065
0.5488
0.4966
0.4493
0.4066
1
0.0905
0.1637
0.2222
0.2681
0.3033
0.3293
0.3476
0.3595
0.3659
2
0.0045
0.0164
0.0333
0.0536
0.0758
0.0988
0.1217
0.1438
0.1647
3
0.0002
0.0011
0.0033
0.0072
0.0126
0.0198
0.0284
0.0383
0.0494
4
0.0000
0.0001
0.0003
0.0007
0.0016
0.0030
0.0050
0.0077
0.0111
5
0.0000
0.0000
0.0000
0.0001
0.0002
0.0004
0.0007
0.0012
0.0020
6
0.0000
0.0000
0.0000
0.0000
0.0000
0.0000
0.0001
0.0002
0.0003
(k;λ)
1
1.5
2
2.5
3
3.5
4
4.5
5
0
0.3679
0.2231
0.1353
0.0821
0.0498
0.0302
0.0183
0.0111
0.0067
1
0.3679
0.3347
0.2707
0.2052
0.1494
0.1057
0.0733
0.0500
0.0337
2
0.1839
0.2510
0.2707
0.2565
0.2240
0.1850
0.1465
0.1125
0.0842
3
0.0613
0.1255
0.1804
0.2138
0.2240
0.2158
0.1954
0.1687
0.1404
4
0.0153
0.0471
0.0902
0.1336
0.1680
0.1888
0.1954
0.1898
0.1755
5
0.0031
0.0141
0.0361
0.0668
0.1008
0.1322
0.1563
0.1708
0.1755
6
0.0005
0.0035
0.0120
0.0278
0.0504
0.0771
0.1042
0.1281
0.1462
Các b
ng s
126
Bng 2. Giá tr tích phân Laplace:
( )
=Φ
z
t
dtez
0
2
2
2
1
π
, Ví d:
(
)
47501,1,0,96.1
,NORMDIST
=
,
2
05.0
2
1
2
05.0
=
Φ z
.
z 0
1
2
3
4
5
6
7
8
9
0
0.0000
0.0040
0.0080
0.0120
0.0160
0.0199
0.0239
0.0279
0.0319
0.0359
0.1
0.0398
0.0438
0.0478
0.0517
0.0557
0.0596
0.0636
0.0675
0.0714
0.0753
0.2
0.0793
0.0832
0.0871
0.0910
0.0948
0.0987
0.1026
0.1064
0.1103
0.1141
0.3
0.1179
0.1217
0.1255
0.1293
0.1331
0.1368
0.1406
0.1443
0.1480
0.1517
0.4
0.1554
0.1591
0.1628
0.1664
0.1700
0.1736
0.1772
0.1808
0.1844
0.1879
0.5
0.1915
0.1950
0.1985
0.2019
0.2054
0.2088
0.2123
0.2157
0.2190
0.2224
0.6
0.2257
0.2291
0.2324
0.2357
0.2389
0.2422
0.2454
0.2486
0.2517
0.2549
0.7
0.2580
0.2611
0.2642
0.2673
0.2704
0.2734
0.2764
0.2794
0.2823
0.2852
0.8
0.2881
0.2910
0.2939
0.2967
0.2995
0.3023
0.3051
0.3078
0.3106
0.3133
0.9
0.3159
0.3186
0.3212
0.3238
0.3264
0.3289
0.3315
0.3340
0.3365
0.3389
1
0.3413
0.3438
0.3461
0.3485
0.3508
0.3531
0.3554
0.3577
0.3599
0.3621
1.1
0.3643
0.3665
0.3686
0.3708
0.3729
0.3749
0.3770
0.3790
0.3810
0.3830
1.2
0.3849
0.3869
0.3888
0.3907
0.3925
0.3944
0.3962
0.3980
0.3997
0.4015
1.3
0.4032
0.4049
0.4066
0.4082
0.4099
0.4115
0.4131
0.4147
0.4162
0.4177
1.4
0.4192
0.4207
0.4222
0.4236
0.4251
0.4265
0.4279
0.4292
0.4306
0.4319
1.5
0.4332
0.4345
0.4357
0.4370
0.4382
0.4394
0.4406
0.4418
0.4429
0.4441
1.6
0.4452
0.4463
0.4474
0.4484
0.4495
0.4505
0.4515
0.4525
0.4535
0.4545
1.7
0.4554
0.4564
0.4573
0.4582
0.4591
0.4599
0.4608
0.4616
0.4625
0.4633
1.8
0.4641
0.4649
0.4656
0.4664
0.4671
0.4678
0.4686
0.4693
0.4699
0.4706
1.9
0.4713
0.4719
0.4726
0.4732
0.4738
0.4744
0.4750
0.4756
0.4761
0.4767
2
0.4772
0.4778
0.4783
0.4788
0.4793
0.4798
0.4803
0.4808
0.4812
0.4817
2.1
0.4821
0.4826
0.4830
0.4834
0.4838
0.4842
0.4846
0.4850
0.4854
0.4857
2.2
0.4861
0.4864
0.4868
0.4871
0.4875
0.4878
0.4881
0.4884
0.4887
0.4890
2.3
0.4893
0.4896
0.4898
0.4901
0.4904
0.4906
0.4909
0.4911
0.4913
0.4916
2.4
0.4918
0.4920
0.4922
0.4925
0.4927
0.4929
0.4931
0.4932
0.4934
0.4936
2.5
0.4938
0.4940
0.4941
0.4943
0.4945
0.4946
0.4948
0.4949
0.4951
0.4952
2.6
0.4953
0.4955
0.4956
0.4957
0.4959
0.4960
0.4961
0.4962
0.4963
0.4964
2.7
0.4965
0.4966
0.4967
0.4968
0.4969
0.4970
0.4971
0.4972
0.4973
0.4974
2.8
0.4974
0.4975
0.4976
0.4977
0.4977
0.4978
0.4979
0.4979
0.4980
0.4981
2.9
0.4981
0.4982
0.4982
0.4983
0.4984
0.4984
0.4985
0.4985
0.4986
0.4986
3
0.4987
0.4987
0.4987
0.4988
0.4988
0.4989
0.4989
0.4989
0.4990
0.4990
Các b
ng s
127
Bng 3. Phân v α ca phân phi Student
α
α
=
>
2
;n
tTP
. Ví d:
(
)
0639.205.0,24
=
TINV
,
05.0
2
05.0
;24
=
> tTP
(n;α)
0.200
0.100
0.050
0.025
0.010
0.005
1
3.0777
6.3138
12.7062
25.4517
63.6567
127.3213
2
1.8856
2.9200
4.3027
6.2053
9.9248
14.0890
3
1.6377
2.3534
3.1824
4.1765
5.8409
7.4533
4
1.5332
2.1318
2.7764
3.4954
4.6041
5.5976
5
1.4759
2.0150
2.5706
3.1634
4.0321
4.7733
6
1.4398
1.9432
2.4469
2.9687
3.7074
4.3168
7
1.4149
1.8946
2.3646
2.8412
3.4995
4.0293
8
1.3968
1.8595
2.3060
2.7515
3.3554
3.8325
9
1.3830
1.8331
2.2622
2.6850
3.2498
3.6897
10
1.3722
1.8125
2.2281
2.6338
3.1693
3.5814
11
1.3634
1.7959
2.2010
2.5931
3.1058
3.4966
12
1.3562
1.7823
2.1788
2.5600
3.0545
3.4284
13
1.3502
1.7709
2.1604
2.5326
3.0123
3.3725
14
1.3450
1.7613
2.1448
2.5096
2.9768
3.3257
15
1.3406
1.7531
2.1314
2.4899
2.9467
3.2860
16
1.3368
1.7459
2.1199
2.4729
2.9208
3.2520
17
1.3334
1.7396
2.1098
2.4581
2.8982
3.2224
18
1.3304
1.7341
2.1009
2.4450
2.8784
3.1966
19
1.3277
1.7291
2.0930
2.4334
2.8609
3.1737
20
1.3253
1.7247
2.0860
2.4231
2.8453
3.1534
21
1.3232
1.7207
2.0796
2.4138
2.8314
3.1352
22
1.3212
1.7171
2.0739
2.4055
2.8188
3.1188
23
1.3195
1.7139
2.0687
2.3979
2.8073
3.1040
24
1.3178
1.7109
2.0639
2.3909
2.7969
3.0905
25
1.3163
1.7081
2.0595
2.3846
2.7874
3.0782
26
1.3150
1.7056
2.0555
2.3788
2.7787
3.0669
27
1.3137
1.7033
2.0518
2.3734
2.7707
3.0565
28
1.3125
1.7011
2.0484
2.3685
2.7633
3.0469
29
1.3114
1.6991
2.0452
2.3638
2.7564
3.0380
30
1.3104
1.6973
2.0423
2.3596
2.7500
3.0298
Các b
ng s
128
Bng 4. Phân v α ca phân phi Chi bình phương
(
)
αχχ
α
=>
2
;
2
n
P
. Ví d:
(
)
0863.1501.0,5
=
CHIINV
,
(
)
01.0
2
01.0;5
2
=>
χχ
P
.
(n;α)
0.010 0.025 0.050 0.950 0.975 0.990
1
6.6349
5.0239
3.8415
0.0039
0.0010
0.0002
2
9.2103
7.3778
5.9915
0.1026
0.0506
0.0201
3
11.3449
9.3484
7.8147
0.3518
0.2158
0.1148
4
13.2767
11.1433
9.4877
0.7107
0.4844
0.2971
5
15.0863
12.8325
11.0705
1.1455
0.8312
0.5543
6
16.8119
14.4494
12.5916
1.6354
1.2373
0.8721
7
18.4753
16.0128
14.0671
2.1673
1.6899
1.2390
8
20.0902
17.5345
15.5073
2.7326
2.1797
1.6465
9
21.6660
19.0228
16.9190
3.3251
2.7004
2.0879
10
23.2093
20.4832
18.3070
3.9403
3.2470
2.5582
11
24.7250
21.9200
19.6751
4.5748
3.8157
3.0535
12
26.2170
23.3367
21.0261
5.2260
4.4038
3.5706
13
27.6882
24.7356
22.3620
5.8919
5.0088
4.1069
14
29.1412
26.1189
23.6848
6.5706
5.6287
4.6604
15
30.5779
27.4884
24.9958
7.2609
6.2621
5.2293
16
31.9999
28.8454
26.2962
7.9616
6.9077
5.8122
17
33.4087
30.1910
27.5871
8.6718
7.5642
6.4078
18
34.8053
31.5264
28.8693
9.3905
8.2307
7.0149
19
36.1909
32.8523
30.1435
10.1170
8.9065
7.6327
20
37.5662
34.1696
31.4104
10.8508
9.5908
8.2604
21
38.9322
35.4789
32.6706
11.5913
10.2829
8.8972
22
40.2894
36.7807
33.9244
12.3380
10.9823
9.5425
23
41.6384
38.0756
35.1725
13.0905
11.6886
10.1957
24
42.9798
39.3641
36.4150
13.8484
12.4012
10.8564
25
44.3141
40.6465
37.6525
14.6114
13.1197
11.5240
26
45.6417
41.9232
38.8851
15.3792
13.8439
12.1981
27
46.9629
43.1945
40.1133
16.1514
14.5734
12.8785
28
48.2782
44.4608
41.3371
16.9279
15.3079
13.5647
29
49.5879
45.7223
42.5570
17.7084
16.0471
14.2565
30
50.8922
46.9792
43.7730
18.4927
16.7908
14.9535
| 1/136

Preview text:

lOMoARcPSD|36212343 ĐẠI HỌC DUY TÂN KHOA KHOA HỌC TỰ NHIÊN
BỘ MÔN XÁC SUẤT THỐNG KÊ BÀI GIẢNG
LÝ THUYẾT XÁC SUẤT VÀ THỐNG KÊ TOÁN (Lưu hành nội bộ) Đà Nẵng, năm 2018
Downloaded by Di?p DN - Chuyên Viên R&D (diepdn@bibabo.vn) Lời mở đầu
Trong khoa học cũng như trong đời sống hàng ngày, chúng ta rất thường gặp các
hiện tượng ngẫu nhiên (toán học gọi là biến cố ngẫu nhiên). Đó là các biến cố mà ta
không thể dự báo một cách chắc chắn rằng chúng xảy ra hay không xảy ra.
Lí thuyết xác suất là bộ môn toán học nghiên cứu nhằm tìm ra các quy luật chi phối
và đưa ra các phương pháp tính toán xác suất của các hiện tượng ngẫu nhiên. Ngày
nay lý thuyết xác suất đã trở thành một ngành toán học quan trọng cả về phương
diện lý thuyết và ứng dụng. Nó là công cụ không thể thiếu được mỗi khi ta nói đến
dự báo, bảo hiểm, mỗi khi cần đánh giá các cơ may, các nguy cơ rủi ro. Nhà toán
học Pháp Laplace ở thế kỷ 19 đã tiên đoán rằng: ‘Môn khoa học này hứa hẹn trở
thành một trong những đối tượng quan trọng nhất của tri thức nhân loại. Rất
nhiều những vấn đề quan trọng nhất của đời sống thực tế thuộc về những bài
toán của lý thuyết xác suất’.
Lí thuyết xác suất và thống kê toán học là môn học cơ bản được giảng dạy ở hầu hết các trường Đại học.
Ngoài tập bài giảng này ra, giảng viên khuyến khích sinh viên khi học môn học xác
suất và thống kê nên có ít nhất 1 tài liệu khác để đọc thêm, bất cứ cuốn sách nào về
xác suất thống kê có trên thị trường đều tốt. Nó sẽ bổ sung kiến thức cho bạn.
Trong quá trình soạn bài giảng này, giảng viên đã tham khảo nhiều ý kiến của các
đồng nghiệp, và giảng viên cũng cố gắng rất lớn trong quá trình biên soạn nhưng do
hạn chế về nhiều mặt nên không thể tránh được sai sót. Rất mong nhận được sự phê
bình và sự đóng góp ý kiến của các đồng nghiệp và các bạn sinh viên. Xin chân thành cảm ơn.
Biên soạn: Nguyễn Quang Thi Mục lục
Lời mở đầu ....................................................................................................... 3

Mục lục ............................................................................................................. v Chương I.
Các khái niệm cơ bản trong lí thuyết xác suất. ...................... 1
1. Nhắc lại một số công thức giải tích tổ hợp. ..........................................................1
1.1. Quy tắc cộng và quy tắc nhân........................................................................1
1.2. Hoán vị. ........................................................................................................2
1.3. Chỉnh hợp (chỉnh hợp không lặp). .................................................................2
1.4. Chỉnh hợp lặp................................................................................................2
1.5. Tổ hợp...........................................................................................................3
1.6. Công thức nhị thức Newton...........................................................................3
1.7. Bài tập...........................................................................................................3
2. Biến cố và các phép toán trên biến cố. .................................................................4
2.1. Phép thử và biến cố. ......................................................................................4
2.2. Các loại biến cố.............................................................................................4
2.3. Biến cố bằng nhau (biến cố tương đương). ....................................................5
2.4. Các phép toán trên biến cố. ...........................................................................5
2.5. Nhóm đầy đủ các biến cố. .............................................................................6
2.6. Bài tập...........................................................................................................6
3. Định nghĩa xác suất..............................................................................................7
3.1. Các định nghĩa xác suất.................................................................................7
3.2. Các định lí về xác suất...................................................................................9
3.3. Công thức xác suất đầy đủ. Công thức Bayes. .............................................13
3.4. Bài tập.........................................................................................................15
4. Dãy phép thử Bernoulli. Công thức Bernoulli. ...................................................15
4.1. Dãy phép thử Bernoulli. ..............................................................................15
4.2. Số có khả năng nhất. ...................................................................................16
5. Bài tập chương...................................................................................................19
Đáp số và hướng dẫn..........................................................................................21
Chương II. Đại lượng ngẫu nhiên. Hàm phân phối xác suất. ..................... 25
1. Khái niệm. Phân loại đại lượng ngẫu nhiên. .......................................................25
1.1. Đại lượng ngẫu nhiên rời rạc. ......................................................................26
1.2. Đại lượng ngẫu nhiên liên tục......................................................................26
1.3. Hàm phân phối của đại lượng ngẫu nhiên....................................................26
2. Đại lượng ngẫu nhiên rời rạc..............................................................................27
2.1. Bảng phân phối xác suất..............................................................................27
2.2. Hàm phân phối xác suất. .............................................................................28
2.3. Phép toán đại lượng ngẫu nhiên...................................................................31
3. Đại lượng ngẫu nhiên liên tục. ...........................................................................32
4. Các đặc trưng của đại lượng ngẫu nhiên.............................................................34
4.1. Kì vọng. ......................................................................................................34
4.2. Phương sai. .................................................................................................36
4.3. Mốt, trung vị và moment trung tâm. ............................................................37
5. Hàm của một đại lượng ngẫu nhiên....................................................................41
5.1. Đại lượng ngẫu nhiên rời rạc. ..................................................................... 41
6.2. Đại lượng ngẫu nhiên liên tục. .................................................................... 42
6. Bài tập chương. ................................................................................................. 45
Đáp số và hướng dẫn. ........................................................................................ 45
Chương III. Các quy luật phân phối thường gặp......................................... 47
1. Quy luật phân phối rời rạc. ................................................................................ 47
1.1. Phân phối nhị thức...................................................................................... 47
1.2. Phân phối siêu bội. ..................................................................................... 48
1.3. Phân phối Poisson....................................................................................... 50
2. Quy luật phân phối liên tục................................................................................ 52
2.1. Phân phối đều. ............................................................................................ 52
2.2. Phân phối mũ.............................................................................................. 52
2.3. Phân phối chuẩn. Phân phối chuẩn tắc. ....................................................... 54
2.4. Phân phối Chi bình phương. ....................................................................... 60
2.5. Phân phối Student....................................................................................... 61
2.6. Công thức tính gần đúng............................................................................. 61
3. Đại lượng ngẫu nhiên nhiều chiều. .................................................................... 63
3.1. Khái niệm. .................................................................................................. 63
3.2. Quy luật phân phối xác suất của đại lượng ngẫu nhiên hai chiều................. 63
3.3. Hàm phân phối của đại lượng ngẫu nhiên hai chiều. ................................... 64
4. Bài tập chương. ................................................................................................. 65
Đáp số và hướng dẫn. ........................................................................................ 67
Chương IV. Lí thuyết mẫu ............................................................................ 71
1. Tổng thể và mẫu................................................................................................ 71
1.1. Mở đầu. ...................................................................................................... 71
1.2. Mẫu ngẫu nhiên, mẫu cụ thể. ...................................................................... 72
1.3. Bảng phân phối tần số................................................................................. 73
1.4. Hàm phân phối mẫu.................................................................................... 76
2. Các tham số đặc trưng của mẫu ......................................................................... 76
2.1. Tỉ lệ mẫu. ................................................................................................... 76
2.2. Số mốt (Mode) của mẫu.............................................................................. 79
2.3. Số trung vị (Median) của mẫu..................................................................... 79
2.4. Các quy luật phân phối mẫu........................................................................ 81
3. Bài tập chương. ................................................................................................. 83
Chương V. Lí thuyết ước lượng .................................................................... 85
1. Ước lượng điểm. ............................................................................................... 85
2. Ước lượng khoảng............................................................................................. 86
2.1. Ước lượng khoảng tin cậy cho kì vọng ....................................................... 87
2.2. Ước lượng khoảng tin cậy cho phương sai.................................................. 90
2.3. Ước lượng khoảng tin cậy cho tỉ lệ. ............................................................ 92
2.4. Ước lượng kích thước mẫu. ........................................................................ 94
3. Bài tập chương. ................................................................................................. 95
Đáp số và hướng dẫn. ........................................................................................ 97
Chương VI. Kiểm định giả thiết thống kê.................................................... 99
1. Các khái niệm cơ bản ........................................................................................ 99
1.1. Đặt vấn đề: ................................................................................................. 99
1.2. Phương pháp kiểm định giả thiết thống kê ................................................ 101 vi
2. Kiểm định giả thiết về tham số......................................................................... 101
2.1. Các loại kiểm định và phương pháp kiểm định giả thiết về các tham số. ... 101
2.2. Kiểm định giả thiết về trung bình của ĐLNN X~N(µ; σ2). ........................ 102
2.3. Kiểm định giả thiết về phương sai của ĐLNN X~N(µ; σ2). ....................... 106
2.4. Kiểm định giả thiết về tỉ lệ các phần tử có tính chất nào đó trong tổng thể.108
2.5. Kiểm định giả thiết về hai kì vọng của hai ĐLNN chuẩn độc lập............... 110
2.6. Kiểm định giả thiết thống kê về hai tỉ lệ của hai ĐLNN. ........................... 113
2.7. Kiểm định giả thiết thống kê về quy luật phân phối................................... 115
2.8. Kiểm định giả thiết thống kê về tính độc lập. ............................................ 120
3. Bài tập chương................................................................................................. 122
Các bảng số................................................................................................... 125
Bảng 1. Bảng phân phối Poisson:......................................................................... 125
Bảng 2. Giá trị tích phân Laplace:........................................................................ 126
Bảng 3. Phân vị α của phân phối Student ............................................................. 127
Bảng 4. Phân vị α của phân phối Chi bình phương............................................... 128 Chương I.
Các khái niệm cơ bản trong lí thuyết xác suất.
A. Mục tiêu
- Ôn lại các kiến thức về Tập hợp và Giải tích tổ hợp như: tập hợp, các phép toán về tập hợp,
qui tắc nhân, hoán vị, chỉnh hợp, tổ hợp . . .
- Rèn luyện cách giải một số bài tập liên quan.
- Giới thiệu các khái niệm về phép thử, biến cố và phép toán giữa các biến cố.
- Nắm vững khái niệm về các biến cố xung và các biến cố độc lập.
- Xây dựng một số định nghĩa xác suất (định nghĩa cổ điển, định nghĩa theo hình học và định
nghĩa theo thống kê) và tìm công thức thể hiện định nghĩa đó.
- Nắm được các công thức cộng, công thức nhân xác suất.
- Hiểu được các công thức tính xác suất có điều kiện, công thức xác suất đầy đủ, công thức Bayes.
- Giới thiệu về dãy phép thử Bernoulli và công thức Bernoulli. B. Nội dung.
1. Nhắc lại một số công thức giải tích tổ hợp.
1.1. Quy tắc cộng và quy tắc nhân.
1.1.1. Quy tắc cộng.
Nếu một công việc được chia làm k trường hợp để thực hiện, trường hợp 1 có 1 n
cách thực hiện xong công việc, trường hợp 2 có n cách thực hiện xong công việc, 2
…, trường hợp k n cách thực hiện xong công việc và không có bất kì mỗi cách k
thực hiện nào ở các trường hợp nào lại trùng với một cách thực hiện ở các trường
hợp
khác, thì có n + n + L + n cách thực hiện xong công việc. 1 2 k
1.1.2. Quy tắc nhân.
Nếu một công việc được chia làm k giai đoạn, giai đoạn 1 có 1 n cách thực hiện
xong công việc, giai đoạn 2 có n cách thực hiện xong công việc, …, 2
giai đoạn k
n cách thực hiện xong công việc, thì có n n Ln cách thực hiện xong công k 2 3 k việc. Bài giảng
1.2. Hoán vị.
Một hoán vị từ n phần tử là một bộ có thể kể thứ tự gồm n phần tử khác nhau đã cho.
Số các hoán vị từ n phần tử kí hiệu là P . n
Công thức tính: P = ! n . n Ví dụ 1.1.
Có 4 sinh viên và 4 cái ghế được sắp xếp theo một hàng ngang. Sắp xếp mỗi sinh
viên ngồi một ghế. Có bao nhiều cách sắp xếp khác nhau?
Rõ ràng mỗi kiểu sắp xếp là một hoán vị của 4 phần tử. Số cách sắp xếp chỗ ngồi là P ! 4 . 4 =
1.3. Chỉnh hợp (chỉnh hợp không lặp).
Một chỉnh hợp chập k (1 ≤ k n ) từ n phần tử là một bộ có thể kể thứ tự gồm k
phần tử khác nhau lấy từ n phần tử đã cho
Số các chỉnh hợp chập k từ n phần tử kí hiệu là k A . n ! Công th n
ức tính: Ak = n − K − + = n (n ) 1 (n k ) 1 (n k)! Nhận xét.
Số các chỉnh hợp chập n của n phần tử bằng số các hoán vị của n phần tử, nghĩa là: n A = P . n n Ví dụ 1.2.
Có bao nhiêu số khác nhau gồm 3 chữ số phân biệt được thiết lập từ các chữ số 1, 2 , 3 , 4 , 5 ? Giải 3 ! 5
Một số gồm 3 chữ số phân biệt được thiết lập từ các chữ số bằng A = 60 . 5 = (5 − 3)!
1.4. Chỉnh hợp lặp.
Một chỉnh hợp lặp chập k ( k ≥ 1) từ n phần tử là một bộ có thể kể thứ tự gồm k
phần tử không nhất thiết khác nhau lấy từ n phần tử đã cho
Số các chỉnh hợp lặp chập k từ n phần tử kí hiệu là k A . n Công th k ức tính: k A = . n n Ví dụ 1.3. Giả sử A = { ; 1 } 3 ; 2
là tập hợp gồm 3 phần tử. Khi đó, các dãy 11, 21 hoặc 33 là
những chỉnh hợp lặp 2 từ 3 phần tử của A . Ta có thể liệt kê ra đây tất cả các chỉnh h 2
ợp lặp là: 11, 12 , 13 , 21, 22 , 23, 31, 32 , 33 . Và số chỉnh hợp đó là A3 = 32 = 9 . 2
Chương I. Các khái niệm cơ bản trong lí thuyết xác suất.
1.5. Tổ hợp.
Một tổ hợp chập k từ n phần tử là một tập con gồm k phần tử khác nhau đã cho.
Số các tổ hợp chập k từ n phần tử kí hiệu là k C . n n(n − ) 1 K(n k + ) 1 ! Công th n ức tính: C k = = n k! k ( ! n k )! Nhận xét: 0 C = 1 , nk k C
= C , với mọi k = ; 0 n . n n n Ví dụ 1.4.
Có bao nhiêu cách phân công 5 sinh viên đi lao động của một lớp gồm 45 sinh viên? Giải
Mỗi cách chọn ngẫu nhiên 5 người trong 50 sinh viên là một tổ hợp chập 5 của 50 .
Vậy số cách phân công khác nhau 5 sinh viên trong 50 sinh viên đi lao động là 5 50 C = 2118760 . 50 = ! 5 (50 − 5)! Ví dụ 1.5.
Có bao nhiêu cách phân công 50 sinh viên thành 3 nhóm I , II , III sao cho nhóm
I đúng 30 sinh viên. Giải Ta thấy có 30
C cách phân công 30 sinh viên vào nhóm 50
I . Số cách phân công
(50 − 30) sinh viên còn lại vào nhóm II III bằng số các chỉnh hợp lặp chập 20 của 2 , nghĩa là bằng 20
2 . Vậy, số cách phân công 50 sinh viên thành 3 nhóm I ,
II , III sao cho nhóm I có đúng 30 sinh viên là 30 20 C × 2 50
1.6. Công thức nhị thức Newton. n
Công thức: (a + b)n k n k k = Ca b n k =0 Nhận xét: a) (1 + x)n 0 1 n n
= C + C x + L + C x n n n n
b) (a b)n = ( ∑− )k k n k k 1 C a b n k =0 1.7. Bài tập. 1.
Tìm n từ các phương trình: a) 2 C = 45 , n 4 A b) n = 60 , 3 Cn 1 − 3 Bài giảng c) 8 12 C = C n n 2.
Trên mặt phẳng có 20 điểm (không có 3 điểm này cùng nằm trên một đường thẳng).
Qua mỗi cặp điểm, ta vẽ một đường thẳng. Hỏi có bao nhiêu đường thẳng như vậy. 3.
Từ thành phố A có 3 con đường đi đến thành phố B và từ B có 2 con đường đi tới
thành phố C . Hỏi có mấy cách đi từ A đến C (phải qua B )? 4.
Trên một đường tròn có 12 điểm. Có mấy cách vẽ dây cung có các mút là các điểm đã
cho. Có mấy tam giác nhận các điểm là đỉnh.
2. Biến cố và các phép toán trên biến cố.
2.1. Phép thử và biến cố.
Phép thử (phép thử ngẫu nhiên) là sự thực hiện một nhóm các điều kiện xác định và
có thể được lặp lại nhiều lần. Kết quả của nó, ta không đoán trước được.
Một kết quả của phép thử gọi là một biến cố. Ví dụ 2.1.
a) Để nghiên cứu hiện tượng ngẫu nhiên về sự xuất hiện sấp hay ngửa khi tung đồng
tiền, ta tiến hành phép thử: “tung một đồng tiền”. Kết quả nhận được sẽ là S (được
mặt sấp) hoặc N (được mặt ngửa). S N là những biến cố.
b) Chọn ngẫu nhiên một sinh viên trong lớp, ta được các biến cố, chẳng hạn: A :
“sinh viên đó là nữ”, B : “sinh viên đó là nam”, C : “sinh viên đó là sinh viên giỏi Toán”.
2.2. Các loại biến cố.
Biến cố không thể có (hay biến cố rỗng) là biến cố không bao giờ xảy ra khi phép
thử thực hiện. Kí hiệu: ∅ .
Biến cố ngẫu nhiên là biến cố có thể xảy ra hoặc không xảy ra tùy thuộc vào từng phép thử.
Biến cố sơ cấp là biến cố xảy ra khi và chỉ khi có một kết quả cụ thể trong số những
kết quả loại trừ nhau của phép thử. Kí hiệu là ω .
Biến cố chắc chắn là biến cố luôn luôn xảy ra khi phép thử thực hiện. Kí hiệu: Ω .
Biến cố chắc chắn gồm tất cả các biến cố sơ cấp. Ta thường coi đó là không gian
biến cố sơ cấp
. Ví dụ 2.2.
Trong Ví dụ 2.1. a) Nếu đồng tiền có hai mặt đều ngửa thì S là biến cố rỗng và N
là biến cố chắc chắn.
Trong Ví dụ 2.1. b) Nếu lớp học đó không có nam thì A là biến cố chắc chắn và B là biến cố rỗng. Ví dụ 2.3.
Gieo 1 một lần 1 con xúc xắc. Gọi B là biến cố “Mặt trên con xúc xắc của nó có i i chấm”, i = 6 ; 1 . Khi đó 4
Chương I. Các khái niệm cơ bản trong lí thuyết xác suất.
Không gian biến cố sơ cấp là Ω = {B , B , B , B , B , . Các 1 2 3 4 5 B6 } 1 B , B , K , 2 B là 6
những biến cố sơ cấp. Chú ý:
Mọi biến cố sơ cấp đều là biến cố ngẫu nhiên. Ngược lại, biến cố ngẫu nhiên nói
chung không là biến cố sơ cấp.
2.3. Biến cố bằng nhau (biến cố tương đương).
Biến cố A gọi là kéo theo biến cố B nếu A xảy ra thì B xảy ra. Kí hiệu: A B ⊂ .
Nếu đồng thời có A B ⊂ và B A
⊂ thì các biến cố A B gọi là bằng nhau. Kí hiệu: A = B .
2.4. Các phép toán trên biến cố.
Cho hai biến cố A B . Khi đó, ta gọi:
Tích của A B , hay A nhân B , là biến cố xảy ra khi A B đồng thời xảy ra. Kí hiệu: A B
. ( hoặc AB hoặc A B ).
Tổng của A B , hay A cộng B , là biến cố xảy ra khi A xảy ra hoặc B hoặc A B
. xảy ra. Kí hiệu: A + B (hoặc A B ).
Cho một biến cố A . Khi đó, ta gọi biến cố đối lập của biến cố A là biến cố xảy ra
nếu A không xảy ra và không xảy ra nếu A xảy ra. Kí hiệu: A . Tính chất.
Với các biến cố A , B , C tùy ý, ta có các tính chất sau:
1) A + B = B + A , AB = BA .
2) (A + B) + C = A + (B + C) , (AB)C = ( A BC) . 3) (
A B + C) = AB + AC , A + (BC) = (A + B)(A + C) . 4) Nếu A B
⊂ thì A + B = B , AB = A . 5) A = A .
6) A + A = Ω , AA = ∅ .
7) A + B = AB , AB = A + B (quy tắc đối ngẫu) 8) Với các biến cố 1 A , 2
A , K , A ta có n
A + A + L + A là biến cố xảy ra khi có ít nhất một biến cố = ). 1 2 A xảy ra ( i ; 1 n n i
A .A .L.A là biến cố xảy ra khi tất cả các = ). 1 2
A đều xảy ra ( i ; 1 n n i Ví dụ 2.4.
Bắn 3 mũi tên vào một tấm bia. Gọi A là biến cố “mũi tên thứ i trúng đích” i ( i = 3 ; 1 ). Hãy biểu diễn qua 1 A , 2 A , 3 A các biến cố:
A : Cả 3 mũi tên đều trúng đích.
B : Có đúng 1 mũi tên trúng đích.
C : Có ít nhất 1 mũi tên trúng đích.
D : Không có mũi tên nào trúng đích. 5 Bài giảng Giải Ta có: A = , , 1 A 2 A 3 A
B = A A2 A3 + 1 A A A3 + 1 A A2 1 2 3 A C = , 1 A + 2 A + 3 A D = . 1 A 2 A 3 A
2.5. Nhóm đầy đủ các biến cố.
Hai biến cố A B gọi là xung khắc nếu AB = ∅ . Các biến cố 1 A , 2
A , K , A gọi là đôi một xung khắc nếu hai biến cố khác nhau bất n
kì trong đó đều xung khắc, tức là A A = ∅ với mọi i j . i j Các biến cố 1 A , 2
A , K , A gọi là một nhóm đầy đủ các biến cố nếu chúng đôi một n
xung khắc và ít nhất một trong chúng xảy ra, tức là A + A + L + A = Ω , 1 2 A A = ∅ n i j
với mọi i j , và P(A với mọi i = ; 1 n . i ) > 0 Ví dụ 2.5.
a) Gieo một lần một con xúc xắc:
Đặt B là biến cố “mặt trên của con xúc xắc có i chấm”, i = 6 ; 1 . Dãy B , B , i 1 B , 2 3 B , 4 B , 5
B lập thành hệ đầy đủ các biến cố. Vì nó có tính chất: 6
B + B + K + B = Ω , với mọi , và P(B , với mọi i = 6 ; 1 . i ) > 0 1 2 6 B B = ∅ i j i j
b) Gieo một đồng tiền một lần:
Đặt A là biến cố “xuất hiện mặt sấp”, khi đó A là biến cố “xuất hiện mặt ngửa”. Ta
thấy rằng dãy A , A lập thành một hệ đầy đủ vì AA = ∅ và A + A = Ω . Chú ý.
Hai biến cố đối lập nhau thì xung khắc với nhau. Điều ngược lại nói chung là không đúng. 2.6. Bài tập. 1.
Xét phép thử: gieo con xúc xắc 2 lần. Mô tả không gian biến cố sơ cấp ứng với phép
thử trên. Tìm các biến cố : A “tổng số chấm chia hết cho 3 ”; B “trị tuyệt đối của hiệu
số chấm là số chẵn”. 2.
Kiểm tra theo thứ tự một lô hàng gồm N sản phẩm. Các sản phẩm đều thuộc một trong
2 loại: tốt hoặc xấu. Kí hiệu A ( k = ;
1 N ) là biến cố chỉ sản phẩm kiểm tra thứ k k
thuộc loại xấu. Viết bằng kí hiệu các biến cố dưới đây:
a) Cả N sản phẩm đều xấu.
b) Có ít nhất 1 sản phẩm xấu.
c) m sản phẩm kiểm tra đầu là tốt, các sản phẩm còn lại là xấu.
d) Các sản phẩm kiểm tra theo thứ tự chẵn là xấu, còn các sản phẩm kiểm tra theo thứ tự lẻ là tốt.
e) Không gian biến cố sơ cấp có bao nhiêu phần tử. 3.
Bắn 3 viên đạn vào một tấm bia. Gọi A là biến cố: “viên đạn thứ i trúng bia”, i = 3 ; 1 . i
B là biến cố: “có đúng 1 viên đạn trúng một tấm bia”, C là biến cố “có ít nhất 2 viên
đạn trúng bia” và D là biến cố “cả 3 viên đạn không trúng bia”. Hãy biểu diễn các biến
cố B , C , D , B + C qua các A A . i i 6
Chương I. Các khái niệm cơ bản trong lí thuyết xác suất. 4.
Bắn không hạn chế vào một mục tiêu cho đến khi có viên đạn trúng mục tiêu thì thôi
bắn. Giả sử mỗi lần bắn chỉ có 2 khả năng trúng bia (gọi là biến cố A ) hoặc chệch bia (biến cố A ).
a) Hãy mô tả không gian biến cố sơ cấp.
b) Hãy nêu một hệ đầy đủ các biến cố.
3. Định nghĩa xác suất.
3.1. Các định nghĩa xác suất.
3.1.1. Định nghĩa cổ điển.
Ta gọi các trường hợp đồng khả năng là các trường hợp mà khả năng xảy ra của chúng là ngang bằng nhau.
Ta gọi một trường hợp là thuận lợi cho biến cố A nếu trường hợp này xảy ra thì A xảy ra.
Giả sử có tất cả n(Ω) trường hợp đồng khả năng, trong số đó có n(A) trường hợp
thuận lợi cho biến cố A . Khi n A
đó, ta gọi xác suất của biến cố A P(A) ( ) = . n(Ω)
Như vậy, xác suất của biến cố là tỉ số về khả năng biến cố đó xuất hiện. Ví dụ 3.1.
Gieo một lần con xúc xắc cân đối và đồng chất. Tìm xác suất để
a) Mặt trên của nó có 1 chấm.
b) Mặt trên của nó có số chấm là số chẵn. Giải
a) Đặt B là biến cố “mặt trên của con xúc xắc có i chấm”, i = 6 ; 1 . i
Đặt A là biến cố “mặt trên của con xúc xắc có 1 chấm. Do con xúc xắc cân đối và
đồng chất nên khả năng xuất hiện các mặt 1 B , B , 2 3 B , B , 4 B , 5 B là như nhau và 6
n(Ω) = 6 và số khả năng thuận lợi cho A là 1. Vậy xác suất cúa biến cố A P(A) 1 = . 6
a) Đặt B là biến cố “mặt trên của con xúc xắc có số chấm là số chẵn”. Dễ thấy 3 1
B = {B ; B ;
và số khả năng thuận lợi cho . 1 2 3 B }
B là 3 . Vậy P(B) = = 6 2 Ví dụ 3.2.
Một lớp học gồm N sinh viên trong đó có M nam và N M nữ. Chọn ngẫu nhiên
s sinh viên. Tìm xác suất để trong s sinh viên được chọn thì có đúng k sinh viên nam Giải 7 Bài giảng
Số cách chọn s sinh viên trong N sinh viên là s C . N
Số cách chọn được k sinh viên nam trong M sinh viên là k C . M
Số cách chọn được s sinh viên trong lớp trong đó có k sinh viên nam và s k sinh viên nữ là k sk C × C . M N M k sk V C × C
ậy, xác suất cần tìm là P(A) M N M = . s CN
3.1.2. Định nghĩa hình học.
Giả sử tập hợp (vô hạn) các trường hợp đồng khả năng của một phép thử có thể biểu
thị bởi một miền Ω (chẳng hạn đoạn thẳng, mặt phẳng, không gian ba chiều v.v…)
còn tập hợp các kết quả thuận lợi cho cho biến cố A là một miền con S của Ω . Ta
lấy ngẫu nhiên một điểm trong miền Ω . Xác suất của biến cố A được xác định như sau:
P(A) = (độ đo của S )/(độ đo của Ω ).
Nếu miền Ω là đường cong hay đoạn thẳng thì “độ đo” của Ω là độ dài của nó.
Nếu miền Ω là hình phẳng hay mặt cong thì “độ đo” của Ω là diện tích của nó. Ví dụ 3.3.
Đường dây điện thoại ngầm nối một tổng đài đến một trạm dài 1 km . Tính xác suất
để dây đứt tại nơi cách tổng đài không quá 100 m biết rằng dây điện thoại đồng chất. Giải.
Do dây điện thoại là đồng chất nên khả năng nó bị đứt tại một điểm bất kì là như
nhau. Khi đó, tập hợp các trường hợp đồng khả năng có thể biểu thị bằng đoạn
thẳng nối tổng đài với trạm. Các trường hợp thuận lợi cho biến cố A “dây bị đứt tại
nơi cách tổng đài không quá 100 m ” là đoạn thẳng có độ dài 100 m . Khi đó P(A) 100 1 = = . 1000 10 Ví dụ 3.4.
Hai người bạn hẹn gặp nhau tại một địa điểm theo quy ước như sau:
Mỗi người độc lập đến điểm hẹn trong khoảng từ 7 giờ đến 8 giờ.
Mỗi người đến, nếu không gặp người kia thì đợi 30 phút hoặc đến 8 giờ không đợi nữa.
Tính xác suất hai người gặp nhau, nếu biết rằng mỗi người có thể đến chỗ hẹn trong
khoảng thời gian quy định một cách ngẫu nhiên và không tùy thuộc vào người kia đến lúc nào. Giải 8
Chương I. Các khái niệm cơ bản trong lí thuyết xác suất.
Gọi 7 + x , 7 + y là thời điểm hai người này đến điểm hẹn, 0 ≤ x, y ≤ 1. Các trường
hợp đồng khả năng tương ứng với các điểm (x; y) tạo thành hình vuông có cạnh
bằng 1, có diện tích (độ đo) bằng 1.
Các trường hợp thuận lợi cho biến cố A (hai người gặp nhau) tương ứng với các 1
điểm (x; y) thỏa mãn x y ≤ . 2 Dựa vào hình vẽ, ta có 3 2   3 Di 1 3 ện tích hình là 1 4 −   =
. Từ đó, ta có P(A) = =  2  4 1 4 Ví dụ 3.5.
Tìm xác suất để điểm M rơi vào hình tròn nội tiếp hình vuông có cạnh 2 cm. Giải
Hình tròn nội tiếp hình vuông có cạnh 2a có đường kính 2a .
Vậy diện tích hình tròn đó là 2 2 R π = a π
và diện tích hình vuông là 2
S = 2a × 2a = 4a . 2 Khi πa π
đó, xác suất phải tìm là P(A) = = . 4 2 a 4
3.1.3. Định nghĩa thống kê.
Giả sử trong n phép thử với điều kiện như nhau, biến cố A xuất hiện k lần. Khi đó ta g k ọi f =
tần suất xuất hiện biến cố A trong n phép thử. Khi n tăng lên n ( A) n
rất lớn, ta thấy rằng f
dao động quanh một số p cố định và tiến dần về số p n ( A)
đó. Ta gọi xác suất của biến cố A P(A) = p = lim f . n ( A) n→+∞
3.2. Các định lí về xác suất.
3.2.1. Định lí cộng xác suất. Định lí 3.1. Nếu 1 A , 2 A , K ,
A là các biến cố đôi
một xung khắc thì n
P(A + A + L + A = P A + L + P A . 1 2 n ) ( 1) ( n ) Định lí 3.2.
Với các biến cố tùy ý A B , ta có P(A + B) = P(A) + P(B)− P(AB) . Chứng minh
Do BA A nên A + BA = A . Từ đó
A + B = A + B(A + A) = A + BA + B A = A + B A.
Do A B A xung khắc nên P(A + B) = P(A)+ P(BA). Tương tự, ta có: 9 Bài giảng
B = BA + B A nên P(B) = P(BA) + P(B A) hay P(B A) = P(B) − P(AB).
Từ các điều kiện trên, ta suy ra: P(A + B) = P(A)+ P(B)− P(AB).
Áp dụng Định lí 3.2. và áp dụng nguyên lí quy nạp, ta có: Định lí 3.3. 1 P(A + A L +
+ A = ∑ P A − ∑ P A A + ∑ P A A A L −
+ −1 − P A A K A 1 2 n ) ( 1) ( i j ) ( i j k ) ( )n ( 1 2 n ) i 1 = i< j
i< j<k Ví dụ 3.6.
Trong số 50 sinh viên của lớp có 20 sinh viên giỏi Toán, 25 sinh viên giỏi Anh và
10 học sinh giỏi cả Toán và Anh. Chọn ngẫu nhiên một sinh viên của lớp. Tính xác
suất để sinh viên này giỏi Toán hoặc giỏi Anh. Giải
Gọi A B lần lượt là biến cố sinh viên được chọn giỏi Toán và giỏi Anh.
Khi đó A + B là biến cố sinh viên được chọn giỏi Toán hoặc giỏi Anh. Áp dụng
Định lí 3.2., ta có:
P(A + B) = P(A) + P(B) − P(AB) 20 25 10 7 = + − = 50 50 50 10 Ví dụ 3.7.
Xếp ngẫu nhiên n bức thư vào n phong bì đã ghi sẵn địa chỉ (mỗi phong bì chì có 1
thư). Tìm xác suất để có ít nhất 1 thư đến đúng địa chỉ. Giải
Đặt A là biến cố “bức thư thứ i đến đúng người nhận”, i = ;
1 n . Gọi A là biến cố i
“ít nhất 1 lá thư đến đúng địa chỉ”. Khi đó, ta có: A = A + A + L + A . Theo 1 2 Định lí n 3.3. ta suy ra 1
P(A + A + L + A P A P A A L 1 P A A K A 1 2 n ) = ∑
( 1) − ∑ ( i j )+ + (− )n− ( 1 2 n ) i=1 i< j = (− )k−1 1
P(A A LA i i i ) 1 2 k
1 i <i < < L i n 1 2 k n k !
Dễ thấy P(A A L A = vì các bức thư
i , K , i đến đúng địa chỉ, còn i i i ) ( ) 1 2 1 i , 2 k ! n k
lại n k khác có thể đến đúng người nhận hoặc không. − k n k ! 1 Ta có
P(A A LA = = . i i i ) ( ) C 1 2 n k
1≤i <i < < L i n ! n k! 1 2 k n 1 Vậy 1 P(A) = ∑ ( )k− −1 . 1 k! k = 10
Chương I. Các khái niệm cơ bản trong lí thuyết xác suất.
3.2.2. Xác suất có điều kiện. Định nghĩa.
Cho hai biến cố A B . Ta gọi xác suất của biến cố A khi biến cố B đã xảy ra
( P(B) > 0 ) là xác suất của A đối với điều kiện B . Kí hiệu: P(A/ B). Ng P AB
ười ta chứng minh được công thức P(A / B) ( ) =
, trong đó P(B) > 0 . P(B) Chứng minh
Ta chứng minh cho trường hợp phép thử có n trường hợp cùng khả năng. Giả sử
trong n trường hợp này có m trường hợp thuận lợi cho B k trường hợp thuận
lợi cho AB . Vì B đã xảy ra nên số trường hợp cùng khả năng lúc này là m , và số
trường hợp thuận lợi cho A trong đó chính là số trường hợp thuận lợi cho AB , tức kP AB
k . Vì vậy P(A / B) n ( ) = = . m P(B) n Chú ý.
Định nghĩa trên mang tính chất thuần túy toán học. Tuy nhiên trong trong thực tế, ta
có thể tính xác suất bằng trực giác.
3.2.3. Định lí nhân xác suất. Tính độc lập của các biến cố. Định lí 3.4.
Nếu các biến cố tùy ý A B cùng liên kết với một phép thử ( P(A), P(B) > 0 ), thì
ta có: P(AB) = P(A)P(B / A) = P(B)P(A / B) .
Áp dụng Định lí 3.4. và áp dụng nguyên lí quy nạp, ta có:
P(A A K A = P A .P A / A K P A / K 1 2 A A A n ) ( 1) ( 2 1) ( n 1 2 n 1 − )
Bây giờ ta đưa điều kiện để xác suất của tích bằng tích các xác suất.
Hai biến cố A B được gọi là độc lập nếu xác suất của biến cố này không phụ
thuộc vào sự xảy ra hay không xảy ra của biến cố kia, tức là:
P(A / B) = P(A) hoặc P(B / A) = P(B).
Chú ý rằng chỉ cần thỏa mãn một trong hai điều kiện này thì sẽ thỏa mãn điều kiện kia. Các biến cố 1 A , 2
A , K , A gọi là độc lập toàn thể nếu xác suất của mỗi biến cố n
trong đó không phụ thuộc vào sự xảy ra hay không xảy ra của một tổ hợp bất kì của các biến cố khác. Định lí 3.5.
a) Nếu A B độc lập thì P(AB) = P(A).P(B). b) Nếu các biến cố 1 A , 2 A , K , A độc lập toàn thể thì n
P(A A K A = P A .P A K P A . 1 2 n ) ( 1) ( 2 ) ( n ) 11 Bài giảng Tính chất
Nếu A B là hai biến cố độc lập thì các cặp biến cố
a) A B độc lập.
b) A B độc lập.
c) A B độc lập. Ví dụ 3.8.
Cho 3 hộp bi, mỗi hộp có 10 bi. Trong hộp thứ i i bi đỏ và 10 − i bi xanh ( i = 3 ;
1 ). Lấy ngẫu nhiên mỗi hộp ra 1 bi.
a) Tính xác suất cả 3 bi lấy ra đều đỏ.
b) Tính xác suất trong 3 bi lấy ra có 2 đỏ và 1 xanh.
c) Biết trong 3 bi lấy ra có 2 đỏ và 1 xanh. Tính xác suất bi lấy ra từ hộp thứ 2 có màu xanh. Giải
Gọi A là biến cố “lấy ra từ hộp thứ i bi đỏ” (i = 3 ; 1 ). Dễ thấy i 1 A , 2 A , 3 A độc lập 1 2 3 toàn thể và P( , P( , P( . 1 A ) 1 A ) 1 A ) = = = 10 10 10
a) Biến cố “cả 3 bi lấy ra đều đỏ” là 1 A . 2 A 3 A 1 2 3 6 Ta có P( . 1 A 2 A 3 A ) = P( 1 A ).P( 2 A ).P( 3 A ) = . . = 10 10 10 1000
b) Biến cố “trong 3 bi lấy ra có 2 đỏ và 1 xanh” là B = . 1 A 2 A 3 A + 1 A 2 A 3 A + 1 A 2 A 3 A
Do B là tổng của các biến cố đôi một xung khắc nên
P(B) = P( 1 A 2 A 3 A )+ P( 1 A 2 A 3 A )+ P( 1 A 2 A 3 A ) = P( 1 A )P( 2 A )P( 3 A )+ P( 1 A )P( 2 A )P( 3 A ) + P( 1 A )P( 2 A )P( 3 A ) 1 2 7 1 8 3 9 2 3 92 = . . + . . + . . = . 10 10 10 10 10 10 10 10 10 1000 c) Ta có: 2 A B =
. Khi đó xác suất bi lấy ra từ 2 A ( 1 A 2 A 3 A + 1 A 2 A 3 A + 1 A 2 A 3 A ) = 1 A 2 A 3 A 24 P 2 A B P( 1 A 2 A 3 A ) 1000 6
hộp thứ 2 có màu xanh là P(A / . 2 B) ( ) = = = = P(B) P(B) 92 23 1000 Ví dụ 3.9.
Một lô hàng gồm 10 sản phẩm, trong đó có 3 phế phẩm. Lấy ngẫu nhiên từng sản
phẩm ra kiểm tra đến khi gặp đủ 3 phế phẩm thì dừng lại.
a) Tính xác suất dừng lại ở lần kiểm tra thứ 3 .
b) Tính xác suất dừng lại ở lần kiểm tra thứ 4 . 12
Chương I. Các khái niệm cơ bản trong lí thuyết xác suất.
c) Biết rằng đã dừng lại ở lần kiểm tra thứ 4 , tính xác suất ở lần kiểm tra thứ 2 gặp phế phẩm. Giải
Gọi A là biến cố “kiểm tra lần thứ i gặp phế phẩm” ( i = 1 ; 1 0 ). i
a) Biến cố “dừng lại ở lần kiểm tra thứ 3 ” là 1 A . Ta có 2 A 3 A P( 1 A 2 A 3 A ) = P( 1 A ).P(A / 2 1 A ).P(A / 3 1 A 2 A ) 3 2 1 1 = . . = 10 9 8 120
b) Ta có biến cố “dừng lại ở lần kiểm tra thứ 4 ” là F = . 1 A 2 A 3 A 4 A + 1 A 2 A 3 A 4 A + 1 A 2 A 3 A 4 A Ta có P( . 1 A 2 A 3 A 4 A ) = P( 1 A ).P(A / 2 1 A ).P(A / 3 1 A 2 A ).P(A / 4 1 A 2 A 3 A ) 7 3 2 1 1 = . . . = 10 9 8 7 120 1
Tương tự, ta có: P( . 1 A 2 A 3 A 4 A ) = P( 1 A 2 A 3 A 4 A ) = 120 1 1
Do F là tổng của các biến cố đôi một xung khắc nhau nên P(F ) = 3. = . 120 40
c) Ta cần tính P(A / F . Thật vậy, ta có 2 ) 1 P( 2 A F ) P( 1 A 2 A 3 A 4 A )+ P( 1 A 2 A 3 A 4 A ) 2. P(A / . 2 F ) 120 2 = = = = P(F ) P(F ) 1 3 3.120
3.3. Công thức xác suất đầy đủ. Công thức Bayes. Cho 1 A , 2
A , K , A là một nhóm đầy đủ các biến cố liên kết với một phép thử. F n
biến cố bất kì liên kết với phép thử đó, hay F xảy ra khi một trong các biến cố 1 A , 2
A , K , A xảy ra. Khi đó, ta có Định lí sau đây n Định lí 3.6.
a) Với mọi biến cố F , ta luôn có P(F ) = P(A L . 1 ) P
. (F / A1)+ + P(A P . F / A n ) ( n )
Công thức này được gọi là công thức xác suất đầy đủ.
P(A .P F / A
P A .P F / A k ) ( k ) ( k ) ( k )
c) Với mỗi k ( k = ;
1 n ), ta có: P(A / F . k ) = = P(F ) ∑n P(A P . F / A i ) ( i ) i=1
Công thức này được gọi là công thức Bayes. Chứng minh
a) Ta có F = F
. = F(A + A + L + A = FA + FA + L + FA . 1 2 n ) 1 2 n Do F 1 A , F 2
A , K , FA đôi một xung khắc nên n 13 Bài giảng
P(F ) = P(FA L 1 ) + P(FA2 ) + + P(FAn ) = P(A L 1 ) P
. (F / A1)+ + P(A P . F / A n ) ( n ) P(A F . / k
) P(Ak )P(F Ak )
b) Dễ thấy rằng: P(A / F = =
và ta suy ra điều phải chứng k ) P(F ) P(F ) minh. Ví dụ 3.10.
Có 20 kiện hàng, mỗi kiện hàng có 10 sản phẩm. Trong số đố có 8 kiện hàng loại
I , mỗi kiện hàng có 1 phế phẩm; 7 kiện loại II , mỗi kiện có 3 phế phẩm; 5 kiện
loại III , mỗi kiện có 5 phế phẩm. Lấy ngẫu nhiên một kiện, rồi từ kiện đó lấy ra
ngẫu nhiên một sản phẩm.
a) Tính xác suất sản phẩm lấy ra là phế phẩm.
b) Biết sản phẩm được lấy là phế phẩm. Tính xác suất kiện được lấy là loại II . Giải
Gọi A là biến cố “lấy được sản phẩm loại i ”, i = I, II, III . Khi đó, i 1 A , 2 A , 3 A
nhóm đầy đủ các biến cố. Gọi F là biến cố “sản phẩm được lấy từ kiện là phế phẩm”.
a) Theo công thức xác suất đầy đủ, ta có
P(F ) = P( 1
A ).P(F / 1 A ) + P( 2
A ).P(F / 2 A ) + P( 3
A ).P(F / 3 A ) 8 1 7 3 5 5 = . + . + . = , 0 27 20 10 20 10 20 10
b) Theo công thức Bayes, ta có 21 P( 2 A F ) P( 2
A ).P(F / Ak ) P(A / . 2 F ) 200 7 = = = = P(F ) P(F ) 54 18 200 Ví dụ 3.11.
Có 5 bình đựng bi, trong đó có 2 bình loại 1: mỗi bình đựng 3 bi đen và 4 bi đỏ,
một bình loại 2: mỗi bình đựng 3 bi đen và 2 bi đỏ. Bình loại 3: mỗi bình đựng 4
bi đen và 3 bi đỏ. Chọn ngẫu nhiên một bình và từ bình đó, chọn ngẫu nhiên một bi.
a) Tính xác suất để bi lấy ra là bi đen.
b) Biết bi lấy ra là bi đen. Tính xác suất để bình lấy ra là bình loại 3. Giải
a) Gọi A là biến cố “bình chọn ra là bình loại i ”, F là biến cố “bi chọn ra là bi i đen”. Ta có 1 A , 2 A và 3
A là nhóm đầy đủ các biến cố. Khi đó 14
Chương I. Các khái niệm cơ bản trong lí thuyết xác suất.
P(F ) = P( 1
A )P(F / 1 A ) + P( 2
A )P(F / 2 A ) + P( 3
A )P(F / 3 A ) 2 3 1 3 2 4 = . + . + . = 5 , 0 2 5 7 5 5 5 7 2 4 P( 3
A ).P(F / 3 A ) . 5 7 16
b) Đây là xác suất có điều kiện P(A / . 3 F ) = = = P(F ) 5 , 0 2 35 3.4. Bài tập. 1.
Một lô hàng gồm có 150 sản phẩm có chứa 6% phế phẩm. Người ta dùng phương pháp
chọn mẫu để kiểm tra lô hang và quy ước rằng: Kiểm tra lần lượt 6 sản phẩm, nếu có ít
nhất 1 trong 6 sản phẩm đó là phế phẩm thì loại lô hàng. Tìm xác suất để chấp nhận lô hàng. 2.
Bắn liên tiếp vào một mục tiêu cho đến khi nào có 1 viên đạn đầu tiên trúng mục tiêu
thì ngừng bắn. Tìm xác suất sao cho phải bắn đến viên đạn thứ 6 biết rằng xác suất
trúng đích của mỗi viên đạn là ,
0 2 và các lần bắn là độc lập.
4. Dãy phép thử Bernoulli. Công thức Bernoulli.
4.1. Dãy phép thử Bernoulli.
Một dãy n phép thử gọi là một dãy n phép thử Bernoulli nếu thỏa mãn hai điều kiện sau đây:
- Dãy n phép thử đó là độc lập với nhau.
- Trong mỗi phép thử xác suất của biến cố A mà ta quan tâm có xác suất P(A) = p không đổi.
Xác suất p gọi là xác suất thành công, số lần A xuất hiện trong n phép thử gọi là
số lần thành công trong dãy n phép thử Bernoulli. Kí hiệu: P = ,
là xác suất để có k lần thành công. n (k ) Pn (k p) Định lí 4.1. P k, p C p q − = , k = ,
1 n , q = 1− p . n ( ) k k n k n Chứng minh
Kí hiệu A là biến cố “phép thử thứ i thành công”, i = ;
1 n . Gọi F là biến cố “có k i
lần thành công” thì F là tổng của k
C biến cố đôi một xung khắc có dạng n A A K A A
K A trong đó {i ;i ;K;i = ; 1 ; 2 K; . 1 2 n } { } n i i i 1 2 k ik i 1 + n
Do tính độc lập nên ta có:
P(A A K A A K A = P A P A KP A P A K P A p q − = i i i i i ) ( i ) ( i ) ( i ) ( i ) ( i ) k n k 1 2 k k +1 n 1 2 k k +1 n
Từ đó, ta suy ra: P k, p C p q − = (đpcm) n ( ) k k n k n Ví dụ 4.1.
Một lô hàng trong kho có 20% phế phẩm. 15 Bài giảng
a) Lấy ngẫu nhiên 5 sản phẩm. Tính xác suất trong 5 sản phẩm này. i) Có 2 phế phẩm.
ii) Có ít nhất 1 phế phẩm.
b) Cần lấy ít nhất bao nhiêu sản phẩm để xác suất có ít nhất một phế phẩm không nhỏ hơn 9 , 0 9 . Giải
a) Số phế phẩm trong 5 sản phẩm lấy ra là số lần thành công trong dãy 5 phép thử
Bernoulli với xác suất thành công là , 0 2 . i) Ta có . 5 P ( , 0 ; 2 2) 2 = C5 ( , 0 2)2 ( 8 , 0 )3 = . 0 2048 5 ii) Ta có P = ∑ . 5 P (k , 0 ; 2) = 1− 5 P ( 0 ; 0 .2) = 1 0 − C5 ( , 0 2)0 ( 8 , 0 )5 = , 0 67232 k 1 =
b) Gọi n là số sản phẩm cần lấy ra. Khi đó, xác suất có ít nhất một phế phẩm là n P = ∑ P k , 0 ; 2 = 1− P , 0 ; 0 2 = 1− 8 , 0 . n ( ) ( ) ( )n n k 1 = Ta c ln , 0 01
ần tìm n nhỏ nhất sao cho 1 − ( 8 , 0 )n ≥ 9 , 0 9 hay n ≥ = 20 6 , 4 . ln 8 , 0
Vậy, ít nhất phải lấy ra n = 21 sản phẩm.
4.2. Số có khả năng nhất.
Trong dãy n phép thử Bernoulli, số m có xác suất P(m) lớn nhất được gọi là số có khả năng nhất. Định lí 4.2.
Số có khả năng nhất bằng np q nếu np q nguyên; bằng [np q] hoặc bằng
[np q]+1 nếu np q không nguyên. Chứng minh Ta có P k, p C p q − = , P k p C p q . n ( + , 1 ) k 1 + k 1 + nk 1 − = n ( ) k k n k n n P + , 1 1 + 1 + − 1 − − n (k p) C k p k q n k n (n k) Khi p đó = = . P , − + 1 n (k p) C k p k q n k (k )q n Ta xét nhận xét sau:
(n k)p ≥1 hay (n k)p ≥ (k + )1q hay k np q . (k + ) 1 q (n k) và
p < 1 hay (n k)p < (k + )1q hay k > np q . (k + ) 1 q Khi đó, ta suy ra: Xác suất P ,
tăng khi k tăng từ 0 đến np q và nó giàm khi k tiếp tục tăng từ n (k p)
np q đến n . Vì k nhận giá trị nguyên nên ta có kết luận sau: 16
Chương I. Các khái niệm cơ bản trong lí thuyết xác suất.
- Nếu np q nguyên thì xác suất P ,
đạt giá trị lớn nhất tại hai giá trị của k n (k p)
k = np q k = np q 1 (chú ý rằng P , = , ). n (k p 0 ) Pn (k p 1 ) 0 1 +
- Nếu np q không nguyên thì xác suất P ,
đạt giá trị lớn nhất tại một giá trị n (k p)
của k k = [np q] 1, trong đó [np q] là kí hiệu phần nguyên của . 0 + np q Ví dụ 4.2.
Giả sử tỉ lệ người dân tham gia giao thông ở thành phố M có hiểu biết về luật giao
thông là 80% . Giả sử, ta chọn ngẫu nhiên 20 người tham gia giao thông trên
đường. Hãy tính xác suất trong các trường hợp sau:
a) Có 15 người hiểu biết luật giao thông.
b) Có 9 người không hiểu biết về luật giao thông.
c) Số người không hiểu biết về luật giao thông có khả năng nhất. Giải
Việc chọn ngẫu nhiên 20 người là dãy phép thử Bernoulli, với H là biến cố “người
được chọn hiểu biết luật giao thông” và p(H ) = 80% = 8 , 0
a) Gọi A là biến cố “có 15 người hiểu biết luật giao thông”. Khi đó, ta có:
P(A) = P 15 8 , 0 ; = C 8 , 0 . , 0 2 . 20 ( ) 15 20 ( )15 ( )5
b) Gọi B là biến cố “có 9 người không hiểu biết luật giao thông”. Khi đó, ta có:
P(B) = P 20 − 8 , 0 ; 9 = P , 0 ; 9 2 = C 8 , 0 . , 0 2 . 20 ( ) 20 ( ) 11 20 ( )11 ( )9
c) Áp dụng Định lí 3.8, ta có:
np q = 20. p(H )− (1 − p(H ) = 20. , 0 2 − (1− , 0 2) = , 3 2 không nguyên
Vậy, số người được chọn không hiểu biết luật giao thông là k .
0 = [np q] + 1 = 4 17 5. Bài tập chương. 1.
n sinh viên. Gọi A là biến cố sinh viên thứ k là nam. Hãy viết bằng kí hiệu các k biến cố sau:
a) Tất cả sinh viên là nam.
b) Có ít nhất 1 một sinh viên nữ.
c) Có đúng 1 một sinh viên nữ.
d) Có đúng 2 sinh viên là nữ. 2.
Chọn ngẫu nhiên 1 công nhân trong số các công nhân có mặt ở xí nghiệp. Gọi A là biến
cố xảy ra khi người công nhân được chọn là nam và B là biến cố người công nhân được
chọn ở khu tập thể; C là biến cố người công nhân được không hút thuốc là.
a) Hãy mô tả biến cố ABC .
b) Với điều kiện nào ta có A C B = A .
c) Khi nào thì ta có C = A . 3. Chứng minh rằng: a) r nr C = C , n n b) r r −1 r C = C + C , n+1 n n n c) r k r k C = ∑ C C , n nm m k =0 n d) 2 ∑ ( k C C , 2 = n ) n 2n k =0 n e) 2 n n C Ck C . 2n+k 2nk ∑( 2n ) k =0 4.
Cho các chữ số 0 , 1, 2 , 3 , 4 , 5 . Hỏi từ các chữ số này:
a) Lập được bao nhiêu số có 4 chữ số khác nhau trong đó nhất thiết phải có mặt chữ số 5 .
b) Lập được bao nhiêu số có 7 chữ số trong đó chữ số 5 có mặt đúng 3 lần còn các chữ
số khác có mặt không quá 1 lần? 5.
Các số 1, 2 , K, n lập thành một hang ngang. Hỏi có mấy cách sắp xếp sao cho:
a) Hai chữ số 1 và 2 đứng cạnh nhau.
b) Ba chữ số 1, 2 và 3 đứng cạnh nhau. 6.
Rút 2 lá bài từ bộ bài có 52 lá. Gọi A là biến cố “được 2 lá cơ”, B là biến cố “được
2 lá 10 ” và C là biến cố “được 2 lá đỏ”.
a) Các cặp biến cố sau, cặp nào xung khắc: A B , A C , B C .
b) Tính các xác suất: P(A + B) , P(B + C) và P(A + C).
c) Tính các xác suất: P(AB), P(BC) và P(AC). 7.
Một bàn dài gồm 2 dãy ghế đối diện nhau, mỗi dãy gồm 6 ghế. Người ta muốn sắp xếp
6 chỗ ngồi cho 6 sinh viên lớp A và 6 sinh viên lớp B vào bàn nói trên. Hỏi có bao
nhiêu cách sắp xếp trong mỗi trường hợp sau:
a) Bất cứ hai sinh viên nào ngồi cạnh nhau hoặc đối diện nhau thì khác lớp với nhau.
b) Bất cứ hai sinh viên nào ngồi đối diện nhau thì khác lớp với nhau. 8.
Có bao nhiêu cách sắp xếp 10 người ngồi thành ngang sao cho 2 hai người A , B ngồi
cạnh nhau và 2 người C , D không ngồi cạnh nhau. 9.
Có bao nhiêu người tham gia vào cuộc đấu cờ, nếu biết rằng cuộc đấu đó có tất cả 10
ván cờ và mỗi đấu thủ phải đấu mới mỗi đấu thủ khác một ván?
10. Gieo đồng thời 2 con xúc sắc. Tìm xác suất để:
a) Tổng số chấm xuất hiện trên 2 con xúc sắc là 7 .
b) Tổng số chấm xuất hiện trên 2 con xúc sắc là 8 . Bài giảng
c) Tổng số chấm xuất hiện trên 2 con xúc sắc hơn kém nhau 2 .
11. Bỏ ngẫu nhiên 5 lá thư vào 5 phong bì đã đề địa chỉ trước (mỗi phong bì chỉ chứa
đúng một lá thư). Tìm xác suất để:
a) Cả 5 lá thư đều đúng người nhận.
b) Lá thư thứ nhất đúng người nhận.
c) Lá thư thứ nhất và lá thư thứ hai đúng người nhận.
12. Xếp ngẫu nhiên 5 người lên 7 toa tàu được đánh số (mỗi toa tàu có thể chứa nhiều
người). Tìm xác suất các biến cố sau:
a) 5 người cùng lên một toa.
b) 5 người lên 5 toa đầu.
c) 5 người lên 5 toa khác nhau.
d) Hai người A B cùng lên toa đầu.
e) Hai người A B cùng lên một toa.
f) Hai người A B cùng lên một toa, ngoài ra không có ai khác lên toa này.
13. Ba khẩu súng độc lập cùng bắn vào một mục tiêu. Xác suất để khẩu thứ nhất bắn trúng là ,
0 7 , đề khẩu thứ hai bắn trúng là 8 ,
0 , để khẩu thứ ba bắn trúng là 5 , 0 . Mỗi khẩu bắn
một viên. Tính xác suất để: a) Có 1 khẩu bắn trúng. b) Có 2 khẩu bắn trúng.
c) Cả 3 khẩu bắn trật.
d) Ít nhất 1 khẩu bắn trúng.
e) Khẩu thứ nhất bắn trúng biết rằng đã có 2 hai khẩu bắn trúng.
14. Một hộp đựng 15 quả bóng bàn trong đó có 9 quả còn mới. Lần đầu người ta lấy ngẫu
nhiên 3 quả để thi đấu, sau đó lại trả vào hộp. Lần 2 lấy ngẫu nhiên 3 quả. Tìm xác
suất để 3 quả lấy ra lần sau đều mới.
15. Có hai hộp A B . Hộp A đựng 8 bi trắng và 2 bi đen. Hộp B đựng 9 bi trắng và 1
bi đen. Lấy ngẫu nhiên 2 bi từ hộp A bỏ sang hộp B rồi sau đó rút ngẫu nhiên 3 bi từ
hộp B . Tìm xác suất để trong 3 bi lấy từ hộp B có 2 bi trắng.
16. Một hộp chứa 5 tờ vé số, trong đó có đúng 1 tờ vé số trúng thưởng. 5 bạn Trường, Đại,
Học, Duy, Tân lần lượt rút ngẫu nhiên mỗi người 1 tờ vé số. Hỏi rút trước hay rút sau
có lợi hơn (xác suất được tờ vé số trúng thưởng cao hơn)? Hãy tổng quát bài toán này
cho n ( n ≥ 1) tờ vé số mà chỉ có đúng 1 tờ trúng thưởng.
17. Trong một lô hàng gồm có 100 sản phẩm, trong đó có 30 sản phẩm loại tốt, lấy ngẫu
nhiên lần lượt 4 sản phẩm không trả lại. Tìm xác suất để:
a) lần thứ 2 lấy được sản phẩm loại tốt.
b) lần thứ 3 lấy được sản phẩm loại tốt.
c) 2 lần đầu lấy được sản phẩm loại tốt.
18. Một số điện thoại có 7 số. Người gọi quên chữ số cuối cùng nhưng anh ta biết rằng số
đó khác 0 , và anh ta quay số đó một cách ngẫu nhiên. Tìm xác suất để anh ta thực hiện
được cuộc liên lạc mà không phải quay quá 3 lần.
19. Trong giờ bài tập, giáo viên cho một bài toán. Lớp có 30 sinh viên nhưng chỉ có 6 bạn
giải được bài toán này. Giáo viên gọi ngẫu nhiên một sinh viên cho đến khi có một sinh
viên giải được bài toán này. Tính xác suất giáo viên gọi đến sinh viên thứ 4 .
20. Một người bắn lần lượt 2 viên đạn vào một tấm bia. Xác suất trúng bia của viên đạn thứ nhất là 8 ,
0 và của viên đạn thứ hai là , 0 6 .
a) Tìm xác suất để có đúng 1 viên đạn trúng đích.
b) Biết rằng có 1 viên trúng đích. Tìm xác suất để đó là viên đạn thứ hai.
21. Một cửa hàng bán một loại sản phầm trong đó có 40% là do xưởng A sản xuất, còn lại
do xưởng B sản xuất. Tỉ lệ sản phẩm loại I do xưởng A sản xuất là 8 , 0 và của xưởng B sản xuất là 9 , 0 . 20
Chương I. Các khái niệm cơ bản trong lí thuyết xác suất.
a) Mua ngẫu nhiên một sản phẩm. Tìm xác suất để mua được sản phẩm loại I .
b) Mua một sản phẩm từ cửa hàng và thấy đó không phải là sản phẩm loại I . Hỏi sản
phẩm đó có khả năng do xưởng nào sản xuất nhiều hơn.
22. Bắn 3 viên đạn độc lập vào một mục tiêu. Xác suất trúng đích của mỗi viên tương ứng là 3 , 0 ; , 0 4 ; 5 ,
0 . Nếu chỉ 1 trúng thì mục tiểu bị phá hủy vơi xác suất là , 0 2 . Nếu ít
nhất 2 viên trúng thì mục tiêu chắc chắn bị phá hủy. Hãy tìm xác suất để mục tiêu bị
phá hủy khi bắn 3 viên trên.
Đáp số và hướng dẫn. 1. a) A A K A , b) K , 1 2 A + A + + A n 1 2 n
c) A A K A + A A K A K + + A A K A , 1 2 n 1 2 n 1 2 n
d) A A K A + A A A K A + K + A A K A A A 1 2 n 1 2 3 n 1 2 n−2 n 1 − n 2. a) A C
B là biến cố “người công nhân được chọn là nam và ở trong khu tập thể không hút thuốc”.
b) Khi A B , A C thì A C B = A . 3. Dùng các công thức: ! n n 2 C k =
, (a + b)nk n− = k k C a
b và (1 + x)n (1 + x)n = (1 + x) n . n
k!(n k )! n k =0 4. a) 204 , b) 3720 . 5. a) ( 2 n − ) 1 !, b) (n − 2)!. 2 2 2 6. C + C C
a) AB = ∅ , BC ≠ ∅ , AC ≠ ∅ , b) P(A + B) 13 4 =
, P(A + C) 26 = , 2 C 2 52 C52 2 2 C + C − 1
P(B + C ) = P(B) + P(C ) − P(BC ) 4 26 = , 2 C52 2 C 1
c) P(AB) = 0 , P(AC) 13 = , P(BC) = . 2 C 2 52 C52 7. a) ( )2 2. 6! , b) ( )2 6 2 . 6! . 8. 2 2.8!− 2 .6!. 9. 2
C = 10 ⇒ n = 5 . n 1 5 2 10. a) , b) , c) . 6 36 9 1 1 1.4! 1 1.1.3! 1 11. a) = , b) = , c) = . 5! 120 5! 5 5! 20 7 1 5! 5 3 7 1 74 1 3 3 7 6 . 6 12. A a) = , b) , c) 7 , d) = , e) = , f) = 5 4 7 7 5 7 5 7 5 2 7 7 75 7 5 4 7 7 35 13. a) , 0 22 , b) , 0 47 , c) , 0 03 , d) 9 , 0 7 , e) . 47
14. Gọi A là biến cố “cả 3 quả bóng lấy được lần sau đều mới”. Gọi B là biến cố “trong i 3
3 ” quả lấy ra thi đấu có i quả mới”, i = 3 ;
0 . Khi đó P(A) = ∑ P(B P A B hay i ) ( / i ) i=0 3 3 1 2 3 2 1 3 3 3 C C C .C C C . P(A) 6 9 9 6 8 9 C6 C7 C9 C6 = . + . + . + . 3 3 3 3 3 3 3 3 1 C 5 1 C 5 1 C 5 1 C 5 1 C 5 1 C 5 1 C 5 1 C 5 21 Bài giảng
15. Tương tự bài 14, ta được 2 2 1 1 1 2 1 2 2 1 C C .C C .C C .C C C . P(A) 2 9 3 8 2 10 2 8 11 1 C = . + . + . 2 3 2 3 2 3 1 C 0 1 C 2 1 C 0 1 C 2 1 C 0 1 C 2
16. Các xác suất trúng thưởng của 5 bạn Trường, Đại, Học, Duy, Tân là như nhau và đều 1 bằng . 5
17. Gọi A là biến cố “lần thứ i lấy được sản phẩm loại tốt”, i = 1 ; 1 00 . Khi đó i a) Chú ý rằng: P A
= P A .P A / A + P A .P A / . 2 A = nên ( 2 ) ( 1) ( 2 1) ( 1) ( 2 1A) 1 A 2 A + 1 A 2 A
b) A = A A A + A A A + A A A + A . . 1 2 3 1 2 3 1 2 3 1 2 A 3 A 2 A 29 c) Dễ thấy: P( . 1 A A ) 30 2 = = 2 A 330 100
18. Gọi A là biến cố “gọi đúng được số cuối cùng” và A là biến cố “gọi đúng được số i
cuối cùng ở lần thứ i ”, i = 3 ;
1 . Khi đó A = A + A .A + A .A . và chú ý rằng: 1 1 2 1 2 3 A
P(A) = P( 1 A ) + P( 1 A )P(A / 2 1 A ) + P A P A A P A A A 2 ( 1) ( / 2 1 ) ( / . 3 1 2 ) 1 8 1 8 7 1 1 = + . + . . = 9 9 8 9 8 7 3
19. Gọi A là biến cố “sinh viên được gọi lần thứ i giải được bài toán”, i = 3 ; 1 0 . i Ta có:
P(A) = P( 1 A 2 A 3 A 4 A ) = P( 1 A )P(A / 2 1 A )P(A / 3 1 A 2 A )P(A / 4 1 A 2 A 3 A ) 24 23 22 6 = . . . 30 29 28 27
20. Gọi A là biến cố “có đúng 1 viên trúng đích”. A là biến cố “viên đạn thứ i trúng i đích”, i = 2 ; 1 . a) A =
. Từ đó suy ra P(A) = , 0 44 . 1 A 2 A + 1 A 2 A
P A A .P A / A A 1 , 0 2 b) P(A A / 1 2 A) ( 1 2 ) ( 1 2 ) = = P(A) , 0 44
21. Gọi M là biến cố “Sản phẩm mua được loại I ”. N , Q lần lượt là biến cố “Sản phẩm
mua được do xưởng A sản xuất”, “Sản phẩm mua được do xưởng B sản xuất”.
a) Ta có: P(M ) = P(N ).P(M / N ) + P(Q).P(M / Q) = 40%. 8 , 0 + 60%. 9 , 0 . 8 6
b) Ta có: P(M / N ) =
P(M / Q) = . 14 14
22. Gọi A là biến cố “mục tiêu bị phá hủy”. B là biến cố “có i viên đạn bắn trúng mục i tiêu”, i = 3 ;
1 . C là biến cố “viên đạn thứ j bắn trúng mục tiêu”, j = 3 ; 1 . j 3
Ta có: P(A) = ∑ P(B P A B , i ) ( / i ) = , 0 438 i 1 = trong đó P(A / , P(A / B 2 ) = P(A / 3 B ) = 1 1 B ) = , 0 2
P(B = P C .P C .P C + P C .P C .P C + P C .P C . . Tương tự, ta 1 )
( 1) ( 2 ) ( 3) ( 1) ( 2) ( 3) ( 1) ( 2 )P(C3)
tính được P(B , P( . 3 B ) 2 )
C. Phương pháp giảng dạy.
- Vấn đáp và làm bài tập. 22
Chương I. Các khái niệm cơ bản trong lí thuyết xác suất.
- Đưa ra các ví dụ thường gặp trong thực tiễn để tạo động cơ và hướng đích tạo nên hứng thú học tập cho sinh viên.
- Kiểm tra, đánh giá việc làm bài tập của SV.
- Gợi mở từ trực quan sinh động đến tư duy trừu tượng giải quyết vấn đề.
- Phối hợp phương pháp thuyết trình và vấn đáp giải quyết vấn đề và làm bài tập.
- Yêu cầu SV đọc bài giảng trước khi lên lớp.
- Kiểm tra, đánh giá việc làm bài tập của SV.
- Sử dụng phương tiện dạy học hiện đại như Mic, Projector.
D. Tài liệu tham khảo
[1] Đậu Thế Cấp, Xác suất thống kê: Lí thuyết và các bài tập (Chương 1), NXB Giáo dục, 2006.
[2] Đinh Văn Gắng, Bài tập xác suất và thống kê (Chương 1), NXB Giáo dục, 2007.
[3] PGS. TS. Phạm Xuân Kiều, Giáo Trình xác suất và thống kê (Chương 1), NXB Giáo dục, 2005.
[4] Đặng Công Hanh, Đặng Ngọc Dục, Giáo trình Lý thuyết xác suất và Thống kê toán (Chương 1), trường Đại học Duy Tân, 1996. 23 Chương II.
Đại lượng ngẫu nhiên. Hàm phân phối xác suất.
A. Mục tiêu.
- Giới thiệu biến ngẫu nhiên và hàm phân phối xác suất.: biến ngẫu nhiên rời rạc cùng
với bảng phân phối xác suất của nó, biến ngẫu nhiên liên tục cùng với hàm mật độ của nó.
- Nắm các đặc trưng của biến ngẫu nhiên: kì vọng, phương sai, Mod, Med,… và hiểu được ý nghĩa của chúng. B. Nội dung.
1. Khái niệm. Phân loại đại lượng ngẫu nhiên. Định nghĩa.
Cho một phép thử và Ω là không gian các biến cố sơ cấp của nó. Một ánh xạ từ Ω 
 → R hay một quy tắc cho tương ứng mỗi kết quả của phép thử với mỗi một số
thực nào đó được gọi là một đại lượng ngẫu nhiên (biến ngẫu nhiên) liên kết với phép thử nào đó.
Ta thường kí hiệu đại lượng ngẫu nhiên bằng chữ in hoa X , Y , Z , K Giá trị của nó
được kí hiệu bằng chữ in thường x , y , z , K Ví dụ 1.1.
a) X là số con gái trong một lần sinh (1 con). X là đại lượng ngẫu nhiên. Giá trị của
nó có thể nhận là 0 , 1.
b) X là số viên đạn trúng đích khi bắn liên tiếp n viên đạn độc lập vào một mục tiêu.
Giá trị của nó có thể nhận là 0 , 1, K, n .
c) X là số sản phẩm tốt trong 10 sản phẩm được chọn ngẫu nhiên từ lô sản phẩm có
100 sản phẩm tốt và 50 phế phẩm. X cũng là đại lượng ngẫu nhiên. Giá trị của nó có
thể nhận là 0 , 1, K, 10.
d) X là số lần tung một đồng tiền cho đến khi được mặt ngửa thì dừng. Khi đó X
đại lượng ngẫu nhiên và giá trị của nó có thể nhận là 1, 2 , K, n , K
e) X là độ cao của một cây tại thời gian t nào đó. X là đại lượng ngẫu nhiên. Bài giảng
Trong ví dụ này, xét a): X là số con gái trong 1 lần sinh con. Ta thấy X thỏa mãn
định nghĩa đại lượng ngẫu nhiên ở trên. Thật vậy, ta có không gian đại lượng cố sơ cấp
là Ω = {T;G}, và X có thể nhận 2 giá trị 0 hoặc 1.
Với mỗi x R , ta sẽ chứng minh tập hợp {X < }
x là biến cố ngẫu nhiên. ∅, x ≤ 0 
Dễ dàng ta có: {X < } x = {
G}, 0 < x ≤ 1. Ba tập ∅ , {G} và Ω đều là biến cố ngẫu  Ω, x > 1 nhiên. Vậy {X < }
x là biến cố ngẫu nhiên.
Ta quan tâm nghiên cứu đến hai loại đại lượng: đại lượng ngẫu nhiên rời rạcđại
lượng ngẫu nhiên liên tục.

1.1. Đại lượng ngẫu nhiên rời rạc. Định nghĩa:
Đại lượng ngẫu nhiên rời rạc là đại lượng ngẫu nhiên mà các giá trị có thể nhận của nó
là tập hợp hữu hạn hoặc vô hạn đếm được. Trong Ví dụ 1.1. Các ví dụ a), b), c), d) đều
là đại lượng ngẫu nhiên rời rạc.
1.2. Đại lượng ngẫu nhiên liên tục. Định nghĩa.
Đại lượng ngẫu nhiên liên tục là đại lượng ngẫu nhiên mà các giá trị có thể nhận của
nó là lấp đầy khoảng (a;b) (hoặc đoạn [a;b]) nào đó, a có thể bằng − ∞ , b có thể bằng + ∞ .
1.3. Hàm phân phối của đại lượng ngẫu nhiên.
Ta nhận thấy tập hợp {X < }
x , x R thay đổi nếu x thay đổi. Do đó P { ( X < } x ) cũng
thay đổi, tức là xác suất này phụ thuộc vào x . Nó là hàm của x . Định nghĩa.
Cho X là đại lượng ngẫu nhiên. Ánh xạ F : R   [ → ] 1 ; 0 xác định bởi F : R   [ → ] 1 ; 0 x
 → F (x) = P(X < x)
được gọi là hàm phân phối xác suất của đại lượng ngẫu nhiên X . Ví dụ 1.2.
Tìm hàm phân phối của đại lượng ngẫu nhiên X chỉ số lần xuất hiện mặt sấp khi gieo
một đồng tiền cân đối và đồng chất. Giải
Không gian biến cố sơ cấp tương ứng với phép thử “gieo đồng tiền” là Ω = {S; N}. Vì
X có thể nhận 2 giá trị 0 hoặc 1. 26
Chương II. Đại lượng ngẫu nhiên. Hàm phân phối xác suất ∅, x ≤ 0  Vì vậy: {X < } x = {
S}, 0 < x ≤ 1 .  Ω, x > 1
Khi đó, hàm phân phối của đại lượng ngẫu nhiên X là:  , 0 x ≤ 0 P(∅), x ≤ 0   1
F (x) = P(X < x) = P {
( S}), 0 < x ≤ 1 hay F(x) =  , 0 < x ≤ 1  2 P(Ω), x > 1  ,1 x > 1
Các tính chất của hàm phân phối
a) Hàm phân phối F(x) là hàm không giảm.
b) P(a X < b) = F(b)− F(a).
c) P(X = a) = lim F(b)− F(a). b a+ →
d) F(− ∞) = 0 và F(+ ∞) = 1. Nhận xét.
Từ a) và d), ta có 0 ≤ F(x) ≤ 1. Tính chất a) và d) được gọi là tính chất đặc trưng của
hàm phân phối xác suất. Một hàm F(x) xác định trên R có tính chất a) và d) đều là
phân phối xác suất của một đại lượng ngẫu nhiên nào đó Ví dụ 1.3. 1 1
Cho hàm số F(x) = arctan x + . π 2
Chứng minh rằng F(x) là hàm phân phối xác suất của đại lượng ngẫu nhiên. Giải. 1 1 Do F'(x) = .
> 0 nên F (x) tăng. π 1 2 + x  1 1  1  π  1
Mặt khác lim F(x) = lim  arctan x +  = −  + = 0 x→−∞ x→−∞ π 2  π  2  2  1 1  1  π  1
và lim F(x) = lim  arctan x +  =   + = 1 x→+∞ x→+∞ π 2  π  2  2
nên F(x) là hàm phân phối xác suất của đại lượng ngẫu nhiên (đpcm).
2. Đại lượng ngẫu nhiên rời rạc
2.1. Bảng phân phối xác suất.
Giả sử X là đại lượng ngẫu nhiên rời rạc. Nó nhận các giá trị 1 x , x , K , 2 x , K có thể n
với các xác suất tương ứng là P(X = x p . i ) = ≥ 0 i Ta lập bảng sau đây 27 Bài giảng X 1 x x … 2 x n P(X = x p p i ) 1 p 2 n n
Với ∑ p = 1. Bảng này có thể vô hạn khi n nhận giá trị + ∞ . i i 1 =
Bảng trên được gọi là bảng phân phối xác suất của đại lượng ngẫu nhiên X .
2.2. Hàm phân phối xác suất.
Nếu ta sắp xếp các giá trị 1 x , x , K , 2
x , K theo thứ tự tăng dần, ví dụ n L L x < x < < x < thì hàm phân phối của 1 2
X có thể viết dưới dạng: n  , 0 x x1   p ,
x < x x 1 1 2  p + p ,
x < x x 1 2 2 3 F (x) =   K K K
p + p +L+ p , x < x x  1 2 n−1 n−1 n  ,1 x > xn
Nếu các giá trị ở vị trí bất kì thì ta có thể viết hàm phân phối dưới dạng: F(x) = ∑ p , i x < x i x R . Ví dụ 2.1.
Trong một lô hàng gồm có 4 sản phẩm tốt và 6 sản phẩm xấu. Lấy ngẫu nhiên 4 sản
phẩm. Gọi X số sản phẩm xấu lấy được.
Lập bảng phân phối xác suất của X . Viết hàm phân phối của X và tính xác suất P(0 ≤ X < ) 3 . Giải
Lấy ngẫu nhiên 4 sản phẩm thì với X là số sản phẩm tốt lấy được, ta có X có thể
nhận các giá trị là 0 , 1, 2 , 3 , 4 . 0 C . 4 C 1 1 C . 3 C 4 Ta có: P(X = 0) 6 4 = = , P(X = ) 1 6 4 = = , 4 C 210 4 C 35 10 10 2 C . 2 3 C . 1 C 8 4 C . 0 C 1 P( C X = 2) 3 6 4 = = , P(X = 3) 6 4 = = , P(X = 4) 6 4 = = . 4 C 7 4 C 21 4 C 14 10 10 10
Từ đó, ta có bảng phân phối xác suất như sau: X 0 1 2 3 4 P 1 4 3 8 1 210 35 7 21 14
Từ đó, ta có hàm phân phối xác suất là 28
Chương II. Đại lượng ngẫu nhiên. Hàm phân phối xác suất  , 0 x ≤ 0   1 , 0 < x ≤ 1 210  5  , 1 < x ≤ 2  F (x) 42 =  23 , 2 < x ≤ 3 42  13 , 3 < x ≤ 4 14  ,1 x > 4 23 23
Khi đó, ta có P(0 ≤ X < 3) = F(3)− F(0) = − 0 =
hoặc ta có thể tính như sau 42 42
P(0 ≤ X < 3) = P(X = 0) + P(X = ) 1 + P(X = ) 23 2 = 42 Ví dụ 2.2.
Bắn liên tiếp 3 viên đạn độc lập vào một mục tiêu. Xác suất trúng đích của mỗi viên đạn là 5 ,
0 . Gọi X là số viên đạn trúng đích trong 3 viên. Tìm hàm phân phối xác suất
của X . Viết hàm phân phối của X . Tính xác suất P(X ≥ ) 1 . Giải
Ta xem việc bắn 3 viên đạn độc lập vào một mục tiêu là tiến hành dãy 3 phép thử 1
Bernoulli. Xác suất bắn trúng đích của mỗi viên đạn là p = . 2 k 3−k Theo công th  1   1 
ức xác suất, ta có: P(X = k ) k
= C   1 −  , k = 3 ; 0 hay 3  2   2  3 P(X = k )  k 1 
= C   , k = 3 ;
0 , là phân phối xác suất của 3
X . Ta có thể viết dưới dạng  2  bảng sau: X 0 1 2 3 P 1 3 3 1 8 8 8 8
Hàm phân phối của X là 29 Bài giảng  , 0 x ≤ 0  1 , 0 < x ≤ 1 8  F (x) 5 =  , 1 < x ≤ 2 8 7  , 2 < x ≤ 3 8  ,1 x > 3 1 7
Xác suất P(X ≥ )
1 = 1− P(X < )
1 = 1− P(0) = 1− = . 8 8 Ví dụ 2.3.
Trong một lô hàng gồm có 10 máy vi tính mới thì có 3 chiếc bị lỗi, lấy ngẫu nhiên 4
máy trong 10 máy tính này. Gọi X là số máy tính bị lỗi trong 4 máy lấy ra. Hãy:
a) Lập bảng phân phối xác suất của X .
b) Khi lấy 4 máy thì có mấy máy bị lỗi là có khả năng xảy ra cao nhất.
c) Tìm xác suất khi lấy ra 4 máy sẽ có ít nhất một máy bị lỗi.
d) Nếu người nào đó lấy ngẫu nhiên ra 3 máy tính để kiểm tra thấy không có máy nào
bị lỗi thì sẽ chấp nhận cả lô hàng. Tìm xác suất người mua chấp nhận lô hàng và xác
suất người mua bác bỏ lô hàng. Giải. a) Ta có X ∈{ } 3 ; 2 ; 1 ; 0 0 4 C . 1 3 C .C 2 2 C .C 3 1 C .C P( C X = 0) 3 7 = , P(X = ) 3 7 1 = , P(X = 2) 3 7 = , P(X = 3) 3 7 = 4 4 4 4 1 C 0 1 C 0 1 C 0 1 C 0
Từ đó ta có bảng phân phối X 0 1 2 3 P(X = x 0 4 C .C 1 3 C .C 2 2 C . 3 1 C .C i ) 3 7 3 7 3 C7 3 7 4 4 4 4 1 C 0 1 C 0 1 C 0 1 C 0 1 C . 3 b) D C
ựa vào bảng xác suất, ta có P(X = ) 1 3 7 = = 5 ,
0 là cao nhất nên trong 4 máy 4 1 C 0
tính lấy ra thì bị 1 máy tính bị lỗi là có khả năng cao nhất. 0 C . 4 c) P( C X ≥ ) 1 = 1− P(0) = 1 3 7 − = 1 − 1 , 0 67 = 8 , 0 33. 4 1 C 0 3 C . 1 d) 3 C7 p = = ,
0 2917 là xác suất để người mua chấp nhận lô hàng. Xác suất để người 4 1 C 0
mua bác bỏ lô hàng là 1− p = 1− , 0 2917 = , 0 7083. 30
Chương II. Đại lượng ngẫu nhiên. Hàm phân phối xác suất
2.3. Phép toán đại lượng ngẫu nhiên.
Cho X Y là các đại lượng ngẫu nhiên có bảng phân phối xác suất X 1 x x … 2 x n P(X = x p p i ) 1 p 2 nY 1 y y … 2 y n P(Y = y q q i ) 1 q 2 n
Kí hiệu: p = P X = x ;Y = y để cho ĐLNN X nhận giá trị x và ĐLNN Y nhận giá ij ( i j ) i trị y . Giả sử
z , K , z là các giá trị khác nhau của tổng x + y , đặt j 1 z , 2 S i j + p = p . kij x + y = i j zk
Ta gọi tổng của X Y là đại lượng ngẫu nhiên X + Y có bảng phân phối xác suất là X + Y 1 z z … 2 z n
P(X + Y = z + p … + p i ) 1 p + 2 n Tương tự, giả sử , đặt ∗ p = p . k ∑ 1 z , z , K , 2
z là các giá trị khác nhau của tích x .y T i j ij x y = i j zk
Ta gọi tích của X Y là đại lượng ngẫu nhiên X Y
. (hoặc XY ) có bảng phân phối xác suất là X Y . 1 z z … 2 z n P(X Y . = z p … ∗ p i ) 1 p ∗ 2 n
Đại lượng ngẫu nhiên X Y gọi là độc lập nếu p = P X = x ;Y = y = p q . ij ( i j ) i j Ví dụ 2.4.
Cho X Y độc lập có bảng phân phối xác suất X 0 1 2 P(X = x , 0 2 3 , 0 5 , 0 i ) và Y −1 0 1 P(y = y , 0 4 3 , 0 3 , 0 i )
Tìm phân phối xác suất của X + Y , X Y . . Giải 31 Bài giảng Theo định nghĩa, ta có:
P(X + Y − 3 = 4 − ) = P(X = ; 0 Y = − )
1 = P(X = 0).P(Y = − ) 1 = , 0 2.0 4 . = , 0 08 .
P(X + Y − 3 = 3 − ) = P(X = ;
0 Y = 0) + P(X = ; 1 Y = − ) 1
= P(X = 0).P(Y = 0) + P(X = ) 1 .P(Y = − ) 1 = , 0 2. 3 , 0 + 3 , 0 . , 0 4 = 1 , 0 8 Tương tự,
P(X + Y − 3 = 2 − ) = P(X = ; 0 Y = ) 1 + P(X = ;
1 Y = 0) + P(X = ; 2 Y = − ) 1
P(X + Y − 3 = − ) 1 = P(X = ; 1 Y = ) 1 + P(X = ; 2 Y = 0)
P(X + Y − 3 = 0) = P(X = ; 2 Y = ) 1
Khi đó, ta có bảng phân phối của đại lượng X + Y − 3 là X + Y − 3 − 4 − 3 − 2 −1 0
P(X + Y − 3 = z , 0 08 1 , 0 8 3 , 0 5 , 0 24 1 , 0 5 i )
Tương tự, ta có bảng phân phối của XY X Y . − 2 −1 0 1 2 P(X Y . = z , 0 20 1 , 0 2 , 0 44 , 0 09 1 , 0 5 i )
3. Đại lượng ngẫu nhiên liên tục. Định nghĩa.
Đại lượng ngẫu nhiên liên tục X F (x) là hàm phân phối xác suất của nó. Nếu tồn x
tại hàm số f (x) xác định và không âm trên R sao cho F(x) = ∫ f (t)dt thì hàm số f (x) −∞
được gọi là hàm mật độ của X .
F (x) chính là diện tích giới hạn bởi đường cong của hàm mật độ f (x) và phần trục
hoành bên trái điểm x . Ví dụ 3.1. 2 x t 1 F (x) ∫ − =
e 2 dt được gọi là hàm phân phối chuẩn. Đó là diện tích giới hạn bởi 2π −∞ 2 1 x
đường cong f (x) 2 = e
và trục hoành bên trái x . 2π
Từ tính chất của hàm phân phối, ta suy ra tính chất của hàm mật độ là x
+ f (x) ≥ 0 , F(x) = ∫ f (t)dt . −∞ 32
Chương II. Đại lượng ngẫu nhiên. Hàm phân phối xác suất +∞
+ ∫ f (x)dx = 1 vì F(− ∞) = 0 và F(+ ∞) = 1. −∞ b
+ P(a X < b) = ∫ f (x)dx . Thật vậy a b a b
P(a X < b) = F (b) − F (a) = ∫ f (x)dx − ∫ f (x)dx = ∫ f (x)dx −∞ −∞ a Ví dụ 3.2. 1 1
Giả sử hàm phân phối của đại lượng ngẫu nhiên là F(x) = arctan x + . Tìm hàm mật π 2
độ của X và tính xác suất P(−1 ≤ X < ) 1 . Giải. 1
Ta có hàm mật độ f (x) = F'(x) = π ( 2 1+ x ) 1 π  π  1
và xác suất P(−1 ≤ X < ) 1 = F ( ) 1 − F(− ) 1 =  −  −  = . π  4  4  2 Định lí.
Nếu hàm phân phối F(x) của đại lượng ngẫu nhiên X liên tục tại x = a thì
P(x = a) = 0 . Chứng minh
Do P(a X < b) = F(b)− F(a) và liên tục tại a nên cho +
b a , ta có
P(x = a) = lim F (b) − F (a) = F (a) − F (a) = 0 . + ba Nhận xét.
Theo Định lí, nếu F(x) liên tục tại a b thì
P(a X < b) = P(a < X < b) = P(a < x b) = P(a X b). Ví dụ 3.3.  − Me x λ , x ; 0 λ 0 Gi > >
ả sử hàm mật độ của đại lượng ngẫu nhiên X f (x) =  .  , 0 x ≤ 0
Tìm M . Tìm hàm phân phối của X . Giải +∞
Theo tính chất của hàm mật độ, ta có: ∫ f (x)dx = 1. −∞ +∞ 0 +∞  1  +∞ D − xx M ễ thấy f ∫ (x) λ λ dx = 0dx ∫ + Me ∫ = M  − e  = .  λ  λ −∞ −∞ 0 0 Vậy M = λ . 33 Bài giảng
Ta có hàm phân phối F(x) được xác định như sau: x
+ Nếu x < 0 thì F(x) = ∫ f (x)dx = 0 . −∞ + Nếu x ≥ 0 thì x 0 x x
F (x) = ∫ f (x)dx = ∫ f (x)dx + ∫ f (x) −λx −λx
dx = 0 + ∫ λe dx = 1− e −∞ −∞ 0 0  , 0 x < 0 Vậy F(x) =  1 − − e λx , x ≥ 0
4. Các đặc trưng của đại lượng ngẫu nhiên. 4.1. Kì vọng. Định nghĩa.
Kì vọng của đại lượng ngẫu nhiên X , kí hiệu là: E(X ) xác định bởi:
+ Nếu X là đại lượng ngẫu nhiên rời rạc có bảng phân phối xác suất X 1 x x … 2 x n P(X = x p p i ) 1 p 2 n +∞
thì E(X ) = x p + x p +L + x p +L = . 1 1 2 2 x p n ni i i=1
Trong trường hợp có vô hạn x thì ta nói X có kì vọng và E(X ) là kì vọng của nó nếu n +∞
chuỗi ∑ x p hội tụ tuyệt đối. i i i=1
+ Nếu X là đại lượng ngẫu nhiên liên tục có hàm mật độ xác suất f (x) thì +∞
E(X ) = ∫ xf (x)dx . −∞
Ý nghĩa của kì vọng.
Kì vọng của đại lượng ngẫu nhiên là trung bình theo xác suất các giá trị có thể nhận
của đại lượng ngẫu nhiên đó. Tính chất
Với mọi đại lượng ngẫu nhiên X , Y , ta có:
a) E(C) = C với C là đại lượng ngẫu nhiên hằng số.
b) E(X + Y ) = E(X )+ E(Y ).
c) EX ) = λ.E(X ), λ là một số.
d) E(XY ) = E(X ) E
. (Y ) nếu X Y độc lập. 34
Chương II. Đại lượng ngẫu nhiên. Hàm phân phối xác suất Ví dụ 4.1.
Nghiên cứu về điểm thi môn Toán của 400 sinh viên một trường Đại học, ta được bảng số liệu như sau Điểm 2 3 4 6 7 8 Số sinh 10 60 160 100 40 30 viên
Gọi X là số điểm môn Toán của sinh viên một trường Đại học. a) Tính E(X )?
b) Tính tổng số điểm môn Toán của 400 sinh viên. Như vậy, điểm trung bình môn
Toán của một sinh viên là bao nhiêu? So sánh giá trị đó với E(X )? Giải
a) Ta lập bảng phân phối xác suất như sau X (Điểm) 2 3 4 6 7 8 P(X = x 1 6 16 10 4 3 i ) 40 40 40 40 40 40 1 6 16 10 4 3 1960
Khi đó, ta có E(X ) = 2. + 3. + 4. + 6. + 7. + 8. = . 40 40 40 40 40 40 400 b) Ta có tổng số điểm môn Toán của 400 sinh viên là 2.10 + 3.60 + 4 1 . 60 + 6.100 + 7 4 . 0 + 8.30 = 1960 . 1960
Suy ra điểm trung bình môn Toán của một sinh viên là . 400 D 1960
ễ thấy E(X ) =
. Khi đó, E(X ) là điểm trung bình môn Toán của sinh viên. 400
Như vậy, ta suy ra kì vọng của một đại lượng ngẫu nhiên X giá trị trung bình của
đại lượng ngẫu nhiên đó. Ví dụ 4.2.
Trong một cuộc thi vấn đáp, có hai hình thức thi như sau:
+ Hình thức thi thứ nhất là mỗi người phải trả lời 2 câu hỏi, mỗi câu trả lời đúng thì được 5 điểm.
+ Hình thức thi thứ hai là nếu trả lời đúng câu thứ nhất thì mới được trả lời câu thứ hai.
Câu thứ nhất trả lời đúng được 5 điểm, câu thứ hai trả lời đúng được 10 điểm.
Trong cả hai hình thức thi này, các câu trả lời sai đều không được điểm. Giả sử xác 3
suất trả lời đúng mỗi câu là và việc trả lời mỗi câu là độc lập với nhau. Theo bạn, 4
nên chọn hình thức nào để số điểm trung bình đạt được nhiều hơn. Giải. 35 Bài giảng 3
Gọi A là biến cố “trả lời đúng câu hỏi thứ i ”, i = ; 1 2 . Ta có: P( . 1 A ) = P( 2 A ) = i 4 Gọi X , 1
X là số điểm đạt được tương ứng với hai hình thức thi trên. Theo yêu cầu bài 2
toán, ta cần so sánh E(X E(X . 2 ) 1 )
Ta có bảng phân phối xác suất của X như sau 1 X 0 5 10 1 P(X = x 1 6 9 1 i ) 16 16 16
Khi đó, điểm trung bình trong hình thức thi thứ nhất là E(X . 1 ) = 7 5 ,
Ta có bảng phân phối xác suất của X là 2 X 0 5 15 2 P(X = x 1 3 9 2 i ) 4 16 16
Khi đó, điểm trung bình trong hình thức thi thứ hai là E(X . 2 ) = 3 , 9 75
Vậy, ta có E(X
nên chọn hình thức thi thứ hai.
1 ) < E( X 2 ) 4.2. Phương sai. Định nghĩa.
Cho X là một đại lượng ngẫu nhiên có kì vọng E(X ). Khi đó, ta gọi phương sai của
X là kì vọng của bình phương độ sai khác giữa X E(X ) , kí hiệu là D(X ) . Vậy 2
D(X ) = E(X E(X )) = E(X 2 )− E 2 (X )
Ý nghĩa của phương sai.
Phương sai là trung bình của bình phương sai số giữa X và EX . Như vậy, phương
sai càng nhỏ thì các giá trị của X càng tập trung quanh EX .

Do D(X ) ≥ 0 nên ta định nghĩa độ lệch chuẩn của đại lượng ngẫu nhiên X như sau Định nghĩa.
Độ lệch chuẩn của đại lượng ngẫu nhiên X là σ (X ) = D(X ) .
Độ lệch chuẩn được dùng thường xuyên hơn phương sai do có cùng đơn vị đo với đại
lượng ngẫu nhiên X . Tính chất.
Với mọi đại lượng ngẫu nhiên X , Y , ta có:
a) D(X ) ≥ 0 . D(X ) = 0 ⇔ X là đại lượng ngẫu nhiên hằng số.
b) D(C) = 0 với C là đại lượng ngẫu nhiên hằng số. c) DX ) 2
= λ D(X ), λ là một số. 36
Chương II. Đại lượng ngẫu nhiên. Hàm phân phối xác suất
d) D(X + λ) = D(X ), λ là một số.
e) D(X ) = E(X 2 )− E 2 (X ).
f) D(X + Y ) = D(X )+ D(Y ) nếu X Y độc lập. Ví dụ 4.3.
Điểm các môn Toán cao cấp 1 A , 2 A , 3
A , Xác suất thống kê (XSTK) và Kinh tế lượng
(KTL) của hai sinh viên An và Bình được cho theo bảng sau Môn TCC 1 A TCC 2 A TCC 3 A XSTK KTL Điểm của 7 6 8 9 5 An Điểm của 9 10 5 10 1 Bình
Gọi X , Y lần lượt là điểm môn Toán của bạn An và Bình.
a) Hãy tính E(X ), E(Y ) và so sánh E(X ), E(Y ).
b) Tính D(X ), D(Y ). So sánh các giá trị này. Giải a) Ta có 9 +10 + 5 +10 + 1 35 E(X ) 7 + 6 + 8 + 9 + 5 35 = = = 7 , E(Y ) = = = 7 . 5 5 5 5
Vậy E(X ) = E(Y ) . b) Ta có 2 2 2 2 2 E( 2 X ) 7 + 6 + 8 + 9 + 5 = = 51. 5
Khi đó D(X ) = E( 2 X ) 2
E (X ) = 55 − 7 2 = 6 . 2 2 2 2 2 E( 2 Y ) 9 +10 + 5 + 10 + 1 307 = = . 5 5 2 2 307
Khi đó D(Y ) = E(Y )− E (Y ) = − 72 = 1 ,
2 4 . Vậy D(Y ) > D(X ). 5
Ta thấy rằng An và Bình cùng có điểm trung bình các môn Toán, tuy nhiên An là “học đều” hơn Bình.
4.3. Mốt, trung vị và moment trung tâm. a) Mốt (mod). Định nghĩa.
Mốt là giá trị của đại lượng ngẫu nhiên X được kí hiệu là Mod(X ) mà tại đó hàm mật
độ f (x) đạt giá trị lớn nhất. 37 Bài giảng
Trường hợp X là đại lượng ngẫu nhiên rời rạc, Mod(X ) là giá trị của X mà tại đó xác
suất P(X = Mod(X )) là lớn nhất.
Mốt của X còn gọi là số có khả năng nhất. Chú ý.
a) Mốt có thể không tồn tại và khi nó tồn tại không nhất thiết là giá trị duy nhất.
b) Mốt không phải luôn luôn tồn tại, chẳng hạn khi tất cả các số liệu trong mẫu có số
lần xuất hiện bằng nhau. Ví dụ 4.4.
Cho đại lượng ngẫu nhiên X có bảng phân phối X 0 1 2 P(X = x 1 1 1 i ) 4 2 4 1
Ta có Mod(X ) = 1 vì P(X = ) 1 =
là xác suất lớn nhất. 2 Ví dụ 4.5.  , 0 x ≤ 0 
Cho X là đại lượng ngẫu nhiên có hàm mật độ f (x) =  2 x . Hãy xác định − x  4 e , x > 0 2 Mod (X ) . Giải Ta có:
+ f (x) = 0 , ∀x ≤ 0 . 2 x + − f (x) x 4 = e , ∀x > 0 . 2 2 x 2 2 2 x x 2 − − − 1 x 1  x  Ta có f '(x) = 4 e − 4 e = 4 e 1 −  . 2 4 2  2 
Khi đó f '(x) = 0 ⇔ x = − 2 hoặc x = 2 . Do x > 0 nên x = 2 . 1 2 D −
ựa vào bảng biến thiên, ta được f (x) ≤ f ( 2) 2 = e . 2
Vậy Mod(X ) = 2 . b) Phân vị.
Điểm x được gọi là phân v 0
với xác suất α của đại lượng ngẫu nhiên X nếu
P(X > x
(hoặc P(X < x ). 0 ) = α 0 ) = α
Hiển nhiên P(X x .
0 ) = 1 − P( X > x0 ) = 1 − α 38
Chương II. Đại lượng ngẫu nhiên. Hàm phân phối xác suất
Trong bài giảng này, chúng ta dùng phân vị P(X > x . 0 ) = α 1
Nếu α = thì điểm x này được gọi là trung vị của X . Khi đó, ta xác định như sau 2 0
c) Trung vị (median). Định nghĩa.
Cho X là một đại lượng ngẫu nhiên. Số m gọi là trung vị của X , kí hiệu Med(X ) nếu   1 P( 1
X < Med (X )) = F (Med (X )) ≤
P(X < Med (X )) ≤  2  2  ) 1 ( hoặc  (2)  1 P( 1
X Med (X )) = F (Med(X ))   ≥
P(X > Med(X )) ≥  2  2
* Nếu X là đại lượng ngẫu nhiên rời rạc Med(X ) là giá trị x sao cho kP( 1 X = L 1
x ) + P(X = x2 ) +
+ P(X = xk−1 ) ≤  2 
, trong đó x x ≤ L ≤ x . 1 2 kP( 1 X = L 1
x ) + P(X = x2 ) +
+ P(X = xk )  ≥  2 1
* Nếu X là đại lượng ngẫu nhiên liên tục thì Med(X ) thỏa F(Med(X )) = . 2 Ví dụ 4.6.  , 0 x ≤ 0 
Cho hàm phân phối của đại lượng ngẫu nhiên X F(x) = x, 0 < x ≤ 1.   , 1 x > 1 1 1
Ta có F(x) = suy ra Med(X ) = 2 2 Ví dụ 4.7.
Cho đại lượng ngẫu nhiên X có bảng phân phối X 0 1 2 P(X = x 1 1 1 i ) 4 2 4 1 1 3 1
Ta có P(X < ) 1 = ≤ và P(X ≤ ) 1 = >
nên Med(X ) =1. 4 2 4 2 Chú ý.
Theo định nghĩa trên thì X có thể có một hoặc nhiều trung vị. Nếu có 1 m , m cùng 2 thỏa ) 1 ( hoặc (2) và thì với m ; cũng là median của 1 m < m2
m bất kì thuộc [ 1 m2 ] X . Ví dụ 4.8.
Gọi X là số chấm xuất hiện khi gieo con xúc xắc. Khi đó X có bảng phân phối 39 Bài giảng X 1 2 3 4 5 6 P(X ) 1 1 1 1 1 1 6 6 6 6 6 6 1 1 1
Ta có P(X < 3) = P(X = )
1 + P(X = 2) = 2. = ≤ 6 3 2
P(X ≤ 3) = P(X = )
1 + P(X = 2) + P(X = 3) 1 1 1 = 3. = ≥ . 6 2 2 Suy ra m 3 . 1 = M 1 1
ặt khác P(X < 4) = P(X = )
1 + P(X = 2) + P(X = 3) = 3. = 6 2 1 2 1
P(X ≤ 4) = P(X = )
1 + P(X = 2) + P(X = 3) + P(X = 4) = 4. = ≥ . 6 3 2 Suy ra m 4 . 2 =
Khi đó, Med(X ) = m , m ∈[ 4 ;
3 ]. m = 3 hoặc m = 4 .
c) Moment trung tâm. Moment gốc. Định nghĩa.
Cho X là một đại lượng ngẫu nhiên có kì vọng E(X ) = a . Ta gọi moment trung tâm
cấp k của X là µ = µ X = E X a . k ( ) ( )k k
Ta gọi moment gốc cấp k là γ = E( k X . k )
Ta có γ = a . Theo công thức nhị thức Newton 1  nn µ = E X a E C a X C a E X n
( − )n = ∑ kn (− )k nk  = ∑ kn (− )k ( nk )  k=0  k =0 n = ∑ k C 1 γ γ n (− )k k nk 1 k =0 n Vậy µ = C 1 γ γ . nkn (− )k k nk 1 k =0 Ví dụ 4.9.
Đại lượng ngẫu nhiên X có bảng phân phối xác suất như sau: X 2 3 4 6 7 P(X ) 1 , 0 , 0 2 3 , 0 , 0 2 , 0 2
Tính E(X ), D(X ), σ (X ) , E( 3
X ), Med(X ), Mod(X ) , P( X EX < 2). Giải 40
Chương II. Đại lượng ngẫu nhiên. Hàm phân phối xác suất E(X ) = 2. 1 , 0 + 3. , 0 2 + 4 3 , 0 . + 6. , 0 2 + 7. , 0 2 = 4.6 . E( 2 X ) = 22. 1 , 0 + 32. , 0 2 + 42. 3 , 0 + 62. , 0 2 + 72 , 0 . 2 = 24 .
D(X ) = E( 2 X ) 2
E (X ) = 24 − 6 , 4 2 = 8 , 2 4 .
σ (X ) = D(X ) = 8 , 2 4 = 6 , 1 85 . E( 3 X ) = 2 . 3 1 , 0 + 33 , 0 . 2 + 43 3 , 0 . + 6 . 3 , 0 2 + 7 .3 , 0 2 = 137,2 . 1 1
Dễ thấy Med(X ) = 4 vì P(X < 4) = 3 ,
0 ≤ và P(X ≤ 4) = , 0 6 ≥ . 2 2
Mod (X ) = 4 vì max P(X = X .
i ) = P( X = 4) = 3 , 0
P( X EX < 2) = P( X − , 4 6 < 2) = P( 6 , 2 < X < 6 ,
6 ) = P(3) + P(4) + P(6) = , 0 2 + 3 , 0 + , 0 2 = 7 , 0
5. Hàm của một đại lượng ngẫu nhiên.
Nếu ta xác định Z = g(X ) là một hàm của đại lượng ngẫu nhiên X thì Z trở thành đại
lượng ngẫu nhiên mới. Vấn đề đặt ra là tìm cách xác định luật phân phối của Z qua
luật phân phối đã biết của X . Ở đây, ta chỉ xét các trường hợp đơn giản khi hàm g không quá phức tạp.
5.1. Đại lượng ngẫu nhiên rời rạc. Ví dụ 5.1.
Cho đại lượng ngẫu nhiên X có luật phân phối X − 2 −1 0 1 2 P(X = 1 , 0 , 0 2 3 , 0 , 0 2 , 0 2 1 x )
Xác định luật phân phối của đại lượng ngẫu nhiên 2
Z = X và tìm kì vọng của Z . Giải. Dễ dàng ta có
P(Z = 0) = P(X = 0) = 3 , 0 , P(Z = ) 1 = P(X = ) 1 + P(X = − ) 1 = , 0 2 + , 0 2 = , 0 4 ,
P(Z = 4) = P(X = 2) + P(X = 2 − ) = 1 , 0 + , 0 2 = 3 , 0 .
Khi đó, ta có bảng phân phối của đại lượng ngẫu nhiên Z Z 0 1 4 P(Z = z 3 , 0 , 0 4 3 , 0 i )
Từ bảng phân phối trên, ta có kì vọng 3
E(Z ) = ∑ z P Z z i ( = i ) = 0. 3 , 0 + 1 , 0 . 4 + 4. 3 , 0 = , 1 6 i 1 = 41 Bài giảng
Trong trường hợp Z = g(X ) tổng quát, ta có thể tính trực tiếp kì vọng của đại lượng
ngẫu nhiên Z như sau:
E(Z ) = ∑ g(x P X x i ) ( = i ) i=1
Trong ví dụ trên, ta có thể tính kì vọng của đại lượng ngẫu nhiên Z E(Z ) = (− 2)2 1 , 0 . + (− ) 1 2 , 0 . 2 + (0)2. 3 , 0 + ( ) 1 2. , 0 2 + (2)2. , 0 2 = 6 , 1 .
6.2. Đại lượng ngẫu nhiên liên tục.
Khi X là đại lượng ngẫu nhiên liên tục, vấn đề sẽ phức tạp hơn. Giả sử đại lượng ngẫu
nhiên X có hàm mật độ f
đã biết và Y = g(X ). Ta sẽ tìm hàm mật độ f của Y (x ) X (x ) Y . Ta có: F x P Y x P Y g X x f
u du , trong đó D = g u ( ) < x Y (
) = ( < ) = ( = ( ) < ) = ∫ X ( ) X DX
Sau đó, lấy đạo hàm F
vế, ta được mật độ f
của đại lượng ngẫu nhiên Y . Y (x ) Y (x ) Ví dụ 5.2.
Cho đại lượng ngẫu nhiên X có hàm mật độ xác suất là f (x) . Tìm hàm mật độ của a) Z = 2X + 1. b) 3 Y = X . Giải.
a) Áp dụng công thức, ta có: F . Z (x ) x 1 1
= P(Z < x) = P(2 x X + 1 < x)  −   −  = PX <  = F    2  X  2 
Lấy đạo hàm, ta được   x −1 f . Z (x ) x 1 1 1 = [FZ (x)]  −   −  ' x = F   ,   X  '= f     2   2  2  2  V 1  x −1
ậy hàm mật độ của Z f . Z (x ) = f   2  2  b) F = < = < = < = . Y (x ) P(Y x) P( 3 X x) P( 3 X
x ) FX (3 x )
Lấy đạo hàm, ta được hàm mật độ của Y là: f = = = Y (x )
[FY (x)]' [FX ( 1 3 x )]' (3
. x )' f (3 x ). 3 2 3 x Ví dụ 5.3. Cho X − µ
đại lượng ngẫu nhiên X có phân phối chuẩn X ~ N ( 2 µ;σ ), đặt Y = . σ
Chứng minh rằng Y có phân phối chuẩn Y ~ N( ) 1 ; 0 . Giải. 42
Chương II. Đại lượng ngẫu nhiên. Hàm phân phối xác suất F . Y (x ) X
= P(Y < x)  − µ  = P
< x  = P(X < σx + µ ) = FX x + µ)  σ 
Lấy đạo hàm, ta được ((σx+µ )−µ )2 2 x − − f
x = F x = F x σ + µ = f x σ + µ σ = e σ σ = e . Y (
) [ Y ( )]' [ X ( )]' X ( ) 1 1 2 2 2 . σ 2π 2π
Vậy Y có phân phối chuẩn Y ~ N( ) 1 ; 0 . 43
Chương II. Đại lượng ngẫu nhiên. Hàm phân phối xác suất 6. Bài tập chương. 1.
Một nhóm có 10 người gồm có 6 nam và 4 nữ. Chọn ngẫu nhiên ra 3 người. Gọi X
là số nữ ở trong nhóm. Lập bảng phân phối xác suất của X và tính E(X ) , D(X ) và mod(X ). 3
x(2 − x), x ∈ [ ; 0 2] 2.
Cho ĐLNN liên tục X có hàm mật độ f (x) = 4 .  ,0 x ∉ [ ; 0 2]
a) Vẽ đồ thị của f (x) .
b) Tính P(X > 5 , 1 ) và P( 9 , 0 < X < ) 1 , 1 .  2 kx , x ∈ [ ] 3 ; 0 3.
Cho ĐLNN liên tục X có hàm mật độ f (x) =  .  , 0 x ∉ [ ] 3 ; 0
a) Tìm hằng số k .
b) Tính P(X > 2).
c) Tìm Med(X ).  x − 1  e λ , x , 0 λ 0 4. > >
Cho hàm mật độ của ĐLNN X f (x) = λ   , 0 x ≤ 0
a) Tìm hàm phân phối của X và tính xác suất P(0 ≤ X < λ) .
b) Tính kì vọng và phương sai của X . 5.
Một người nuôi 100 con gà. Xác suất để mỗi con gà đẻ trong một ngày là p = 8 , 0 . Gọi
X là số trứng thu được trong một ngày.
a) Tính xác suất để thu được ít nhất 80 quả trứng trong một ngày.
b) Giả sử, giá bán mỗi quả trứng gà là 2000 VNĐ và chi phí cho mỗi con là 1.200 VNĐ.
Gọi Y là số tiền lời trong một ngày. Tính tiền lời trung bình? 6.
Một hộp đựng 7 sản phẩm xấu và 3 sản phẩm tốt. Chọn ngẫu nhiên cùng lúc 2 sản
phẩm. Gọi X là số sản phẩm tốt trong hai sản phẩm lấy ra.
a) Lập bảng phân phối xác suất của X .
b) Tính E(X ), D(X ) và Mod(X ) . 7.
Cho ĐLNN X rời rạc và có phân phối xác suất như sau X 1 3 5 7 9 P 0,1 0,4 0,2 0,2 0,1
a) Tính P(3 ≤ X ≤ 7).
b) Xác định Med(X ), Mod(X ), E(X ) và D(X )
Đáp số và hướng dẫn. 1. Dùng các công thức: X 0 1 2 3 P 5 15 9 1 30 30 30 30 45 Bài giảng E(X ) = , 1 2 , D(X ) = 5 , 0 6 và mod(X ) = 1 2. b) P(X > 5 , 1 ) ≈ 1 , 0 5625 , P( 9 , 0 < X < ) 1 , 1 ≈ 1 , 0 495.  , 0 x < 0  1 19  3 3. x
a) k = , b) P(X > 2) = , c) F(x) =  , 0 ≤ x ≤ 3 . 9 27 27  ,1 x > 3 3 x 1 3 3
Median m là nghiệm của phương trình = hay x = . Vậy m = . 27 2 3 2 3 2  , 0 x ≤ 0 4. a) F(x) =  − x
, P(0 ≤ X < λ) 1
= 1 − e , b) E(X ) = λ , D(X ) 2 = λ .  −
1 − e λ , x > 0
C. Phương pháp giảng dạy.
- Thuyết trình, đàm thoại khơi động hoạt động tự giác, tích cực của sinh viên.
- Sử dụng hình thức trực quan: bảng, đồ thị, kí hiệu,…
- Yêu cầu SV đọc bài giảng trước khi lên lớp.
- Kiểm tra, đánh giá việc làm bài tập của SV.
- Sử dụng phương tiện dạy học hiện đại như Mic, Projector.
- Giảng viên gửi bài giảng cho sinh viên đọc trước. Giảng viên trình bày bài giảng trên lớp
theo phương pháp thuyết trình hỏi đáp. Giao bài tập cho sinh viên về nhà làm. Giới thiệu một số tài liệu tham khảo.
D. Tài liệu tham khảo
[1] Đậu Thế Cấp, Xác suất thống kê: Lí thuyết và các bài tập (Chương 2), NXB Giáo dục, 2006.
[2] Đinh Văn Gắng, Bài tập xác suất và thống kê (Chương 2), NXB Giáo dục, 2007.
[3] PGS. TS. Phạm Xuân Kiều, Giáo Trình xác suất và thống kê (Chương 2), NXB Giáo dục, 2005.
[4] Đặng Công Hanh, Đặng Ngọc Dục, Giáo trình Lý thuyết xác suất và Thống kê
toán (Chương 2), trường Đại học Duy Tân,1996. 46 Chương III.
Các quy luật phân phối thường gặp.
A. Mục tiêu.
- Sử dụng hình thức trực quan: bảng, đồ thị, kí hiệu,…
- Ứng dụng Excel cho việc tính các giá trị của biến ngẫu nhiên có phân phối chuẩn,
phân phối Poisson, phân phối Student, phân phối chi bình phương.
- Yêu cầu SV đọc bài giảng trước khi lên lớp.
- Kiểm tra, đánh giá việc làm bài tập của SV.
- Sử dụng phương tiện dạy học hiện đại như Mic, Projector. B. Nội dung.
1. Quy luật phân phối rời rạc.
1.1. Phân phối nhị thức. Định nghĩa.
Gọi X là số lần biến cố A xuất hiện trong dãy n phép thử Bernoulli. Khi đó, X
đại lượng ngẫu nhiên có phân phối nhị thức.
Kí hiệu X ~ B(n; p).
Công thức xác suất: P(X = k) k k nk = C p q
, trong đó q = 1− p . n Các tính chất.
Cho X ~ B(n; p), ta có
a) E(X ) = np .
b) D(X ) = npq .
c) np q ≤ mod X np + p . Chứng minh
a) Gọi X là “số lần đại lượng cố A xuất hiện trong phép thử thứ n ” (trong dãy phép i
thử Bernoulli), ta có bảng phân phối của X là: i X 0 1 K n Bài giảng P p 0 1 p K p n
trong đó p = P X = k C p q − = . k ( ) k k n k n n n n
Suy ra E(X ) = ∑ k.p = k C . p q k C . p q . kk k nk = nk k nk n k =0 k =0 k =1 n
Ta có (p + x)n k n− = k k C p
x . Đạo hàm hai vế theo x , ta được n k =0 n n 1 1
n( p + x)n− = ∑ k nk k kC p
x 1 hay n( p + x)nx = ∑ k nk k kC p x . n n k =1 k =1 n
Chọn x = q , ta suy ra np k n− = k k kC p
x . Vậy E(X ) = np (đpcm). n k =1
b) Dễ dàng chứng minh được n n
E(X 2 ) = ∑ k 2.p = ∑ k 2 C k .
p k q nk = n − 2 1 + . k n (n )p np k =0 k =0 Khi đó 2 2 2 2
D(X ) = E(X )− (E(X )) = n(n − )
1 p + np − (np) = np np2 = npq (đpcm).=
c) Do P(X = mod X ) ma { x p ; p ;K =
; p . Theo Ch 0 1
ương 2, ta có: n }
+ Nếu np q nguyên thì np q = mod X = np q +1.
+ Nếu np q không nguyên thì [np q] < mod X = [np q]+1.
Vậy np q ≤ mod X np + p (đpcm). Ví dụ 1.1.
Bắn 5 viên đạn vào mục tiêu, xác suất trúng mục tiêu của mỗi viên đạn là 8 , 0 . Gọi X
là đại lượng ngẫu nhiên chỉ số viên đạn trúng mục tiêu. Lập bảng phân phối của X . Tính E(X )? Giải
Ta có X có thể nhận các giá trị 0 , 1, 2 , 3 , 4 , 5 . Khi đó, ta có: P(X = k ) k k 5−k = C 8 , 0 , 0 2 , k = 5 ; 0 . 5
Từ đó, ta có bảng phân phối 0 1 2 3 4 5 0 0 5 C 8 , 0 . , 0 2 1 1 4 C 8 , 0 , 0 . 2 2 2 3 C 8 , 0 , 0 . 2 3 3 2 C 8 , 0 . , 0 2 4 4 1 C 8 , 0 , 0 . 2 5 5 0 C 8 , 0 . , 0 2 5 5 5 5 5 5
Dễ thấy kì vọng E(X ) = np = 5 8 , 0 . = 4 .
1.2. Phân phối siêu bội. Định nghĩa. 48
Chương III. Các quy luật phân phối xác suất thường gặp.
Gọi X là số lần chọn được phần tử có tính chất A trong n lần chọn không lặp từ một
tập hợp có N phần tử, trong đó có M phần tử có tính chất A . Khi đó, X được gọi là
đại lượng ngẫu nhiên có phân phối siêu bội.
Kí hiệu: X ~ H (N; M ;n). k nk . Công th C C
ức xác suất: P(X = k ) M N M = . n CN Các tính chất.
Cho X ~ H (N; M ;n). Ta có
a) E(X ) = np . b) D(X ) N n = npq . N −1 trong M đó p = , q = 1− p . N Chứng minh n k C . nk Tr C
ước hết, ta chứng minh công thức ∑ M NM = 1. Thật vậy, ta có n k =0 CN M N M N
(1+ x)M (1+ x)NM = (1+ x)N hay ∑ k k C x C x C x Ml l = t t N MN k =0 l =0 k =0 n
So sánh hệ số của n
x hai vế, ta được k nk nC C . = C (đpcm) M N M N k =0 Ta có: n n k nk n k nk . . E(X ) kC C M kNC C
= ∑ kP(X = k ) = ∑ M N M = n nM N M n 0 0 C N k = k = 0 nMC N k = N M! M 1 ! n k − − ( ) nk kN C . C . n k! ( . M k) N M n ! (k − ) 1 ! (.M k ) N M ! = np∑ = npN! 1 ! 1 1 (N − ) k = k =
nM . n!(N n)! (n − ) 1 !(N n)! n k −1 nk C C .
= npM −1 NM N −1 k =1 CNn n k 1 − C . k n nk k 1 n 1 C C . 1 1 C Chú ý rằng: M 1 − N M M 1 N1 −M1 ∑ =
, (trong đó k = k 1, M = M 1, N − ∑ = 1 1 1 − 1 − 1 n k 1 = C C N n 1 k =0 N1 N = N 1 và n = n 1). 1 − 1 −
Vậy ta có điều phải chứng minh. b) Ta có.
Tương tự như câu a), ta dễ dàng chứng minh được: 49 Bài giảng n E( 2 X ) 2
= ∑ k P(X = k) k =0 n n = ∑ k(k − )
1 P(X = k ) + ∑ kP(X = k) k =0 k =0 M (M − ) 1 n(n − ) 1 M
Mn(nM M n + N ) = + n = N (N − ) 1 N N (N − ) 1 Khi đó
D(X ) = E( 2
X )− (E(X ))2
Mn nM M n + N Mn  =  −  N N − 1 N
M (N M ) N n N n = . n . . − − = npq N N N − 1 N − 1 Ví dụ 1.2.
Một hộp có 4 viên bi đỏ và 3 viên bi xanh. Lấy ngẫu nhiên 3 bi từ hộp. Gọi X là số bi xanh lấy được.
a) Tính xác suất lấy được 2 bi xanh.
b) Lập bảng phân phối xác của X . Từ đó tính kì vọng và phương sai. Giải 2 1 C . a) Ta có C
N = 7 , M = 3 và X ~ H ( 3 ; 3 ; 7
). Khi đó P(X = 2) 3 4 = . 3 C7
b) Dễ thấy X có thể nhận các giá trị 0 , 1, 2 , 3 . Khi đó, ta có bảng phân phối xác suất như sau: X 0 1 2 3 P 0 3 C . 1 2 C . 2 1 C . 3 0 C . 3 C4 3 C4 3 C4 3 C4 3 C 3 3 3 7 C7 C7 C7 3 9 3  3  7 − 3 24 Do X ~ H ( ) 3 ; 3 ; 7
nên E(X ) = 3. = và D(X ) = 3. .1−  = . 7 7 7  7  7 −1 49
1.3. Phân phối Poisson. Định nghĩa.
Gọi X là số lần phần tử có tính chất A xuất hiện trong một khoảng thời gian (hoặc
trên một miền, một vùng) nào đó. Khi đó, X được gọi là đại lượng ngẫu nhiên có
phân phối Poisson với tham số λ là số trung bình của số lần phần tử có tính chất A xảy ra.
Kí hiệu: X ~ P(λ) . −λ k Công th e λ
ức xác suất: P(X = k) = . k! 50
Chương III. Các quy luật phân phối xác suất thường gặp. Các tính chất.
Cho X ~ P(λ) . Ta có a) E(X ) = λ . b) D(X ) = λ .
c) [λ]−1 ≤ mod(X ) ≤ [λ]. Chứng minh +∞ +∞ −λ k +∞ k −1 a) Ta có: E(X ) ke
= ∑ kP(X = k ) λ −λ λ = ∑ = λe ∑ = λ −λ λ e e = λ , (do ! 1 k =0 ! k =0 k k =1 (k − ) +∞ λl λ ∑ = e ). ! l =0 l +∞ +∞ 2 −λ k +∞ −λ k +∞ − k e λ k k 1 λ k b) Ta có λ λ E( 2 X ) e ke = ∑ 2
k P(X = k ) ( − ) = ∑ = ∑ + ∑ . k k k k k ! k ! k ! =0 =0 =0 =0 +∞ −λ k
+∞ k(k − ) −λ k +∞ k −2 D ke λ 1 e λ −λ λ ễ thấy ∑ = λ và 2 2 ∑ = e λ ∑ = λ k k k k =0 ! k =2 ( − 2)! k =0 ! nên 2 2 E( 2
X ) = λ2 + λ . Vậy D(X ) = E(X )− (E(X )) = λ (đpcm). −λ k −λ k 1 + c) Ta có λ e λ P(X = ) e k =
P(X = k + ) 1 = . k! (k + ) 1 !
Dễ thấy P(X = k + )
1 ≥ P(X = k ) khi và chỉ khi k ≤ λ −1
P(X = k + )
1 < P(X = k ) khi và chỉ khi k > λ −1.
Do k N nên [λ]−1 ≤ mo ( d X ) ≤ [λ] (đpcm) Chú ý.
Luật phân phối Poisson có ý nghĩa thực tế rất lớn và được ứng dụng rộng rãi trong việc
kiểm tra chất lượng sản phẩm. Đặc biệt giải quyết một số bài toán sau đây Ví dụ 1.3.
Tại một CLB Bóng bàn, biết rằng trung bình mỗi ngày có 5 người đến tập luyện. Tính
xác suất để trong một ngày mà ta xét.
a) Có 3 người đến tập luyện.
b) Có ít nhất 4 người đến tập luyện. Giải
Gọi X là số người đến tập luyện trong ngày. Ta có X ~ P(5). Khi đó 5 − e 53 a) P(X = 3) = . ! 3 3 −9 e 9k
b) P(X ≥ 4) = 1− P(X < 4) = 1− ∑ . k k ! =0 51 Bài giảng Ví dụ 1.4.
Xét số khách hàng vào cửa hàng mua ĐTDĐ trong một tháng là đại lượng ngẫu nhiên
tuân theo phân phối Poisson với mật độ trung bình là 9 khách hàng trong một ngày.
a) Tìm xác suất để trong một ngày có 40 khách hàng.
b) Tìm xác suất để trong một tuần có 100 khách hàng.
c) Tìm xác suất để trong một ngày có hơn 40 khách hàng. Giải
Gọi X là số khách hàng vào cửa hàng mua ĐTDĐ. 9 − e 940
a) Ta có E(X ) = 9 . Khi đó P(X = 4) = . 4 ! 0
b) Số khách hàng trung bình vào cửa hàng mua ĐTDĐ trong một tuần là 9.7 = 36 . E(Y ) = 36 . 3 − 6 e 36100
Khi đó, ta có P(Y = 100) = . 10 ! 0 40 −9 e 9k
c) Ta có P(X > 40) = 1− P(X ≤ 40) = 1− ∑ . k k ! =0
2. Quy luật phân phối liên tục.
2.1. Phân phối đều. Định nghĩa.
Đại lượng ngẫu nhiên liên tục X được gọi là có phân phối đều trên đoạn [a;b] nếu  1  ,
x ∈ [a;b]
hàm mật độ của X f (x) = b a .  ,0
x ∉ [a;b]
Kí hiệu: X ~ U (a;b). Các tính chất.
Cho X ~ U (a;b). Ta có: a) + E(X ) b a = . 2 (b a)2 b) D(X ) = . 12 2.2. Phân phối mũ. Định nghĩa. 52
Chương III. Các quy luật phân phối xác suất thường gặp.
Đại lượng ngẫu nhiên liên tục X được gọi là có phân phối mũ tham số λ ( λ > 0 )  −
λe λx , x 0 hàm m ≥
ật độ của nó có dạng f (x) =  .  , 0 x < 0
Kí hiệu: X ~ E(λ). Các tính chất.
Cho X ~ E(λ). Ta có: 1 a) E(X ) = . λ b) D(X ) 1 = . 2 λ Chứng minh +∞
a) Ta có: E(X ) ∫ − =
xλe λx dx 0 1 +∞ −t Γ 2 1.Γ 1 1
Đặt t = λx . Khi đó E(X ) ( ) ( ) = ∫te du = = = λ λ λ λ 0 +∞ b) Ta có: E( 2 X ) ∫ 2 − =
x λe λx dx . 0 1 +∞ −t Γ 3 2.Γ 1 2
Đặt u = λx . Khi đó E( 2 X ) 2 ( ) ( ) = ∫t e dt = = = . 2 2 2 2 λ λ λ λ 0
Suy ra D(X ) = E( 2
X )− (E(X ))2 1 = . 2 λ +∞
Trong đó, Hàm Gamma được xác định như sau Γ(α ) = ∫ − α−1 e x x dx . 0
Các tính chất của hàm Gamma: a) Γ( ) 1 = 1 . b) Γ(n + ) 1 = ! n , n ∀ ∈ N .  1  c) Γ  = π .  2  Ví dụ 2.1.
Tuổi thọ X (tính bằng giờ) của một thiết bị có phân phối mũ X ~ E( 0 , 0 012 ) 5 .
Hãy tính và đưa ra ý nghĩa các giá trị sau:
a) P(X > 720).
b) P(600 < X < 720). 53 Bài giảng c) E(X ).
d) P(X < 800). Giải a) Ta có
P(X > 720) = 1 − P(X < 720) − P(X = 720) = 1 − F (720) = 1 − ( 0 − ,00125 7 . 20 1− e ) 0 − ,9 = e
Ý nghĩa P(X > 720) là xác suất để tuổi thọ của thiết bị lớn hơn 720 giờ là 0−,9 e .
b) P(600 < X < 720) = F(720)− F(600) = ( 0 − ,00125 7 . 20 1− e )− ( 0 − ,0012 . 5 600 1− e ) 0 − ,75 0 − ,9 = ee .
Ý nghĩa P(600 < X < 720) là xác suất để tuổi thọ của thiết bị nằm trong khoảng (600 7 ; 20) là 0 − ,75 0 − ,9 ee . c) E(X ) 1 = . 0 , 0 0125 −0,0012 . 5 800 1
d) P(X < 800) = F(800) = 1− e = 1 − . e
2.3. Phân phối chuẩn. Phân phối chuẩn tắc. Định nghĩa.
Đại lượng ngẫu nhiên liên tục X được gọi là có phân phối chuẩn với kì vọng µ , (x−µ )2 − ph 1 ương sai 2
σ nếu hàm mật độ của nó có dạng f (x) 2 2σ = e . σ 2π
Kí hiệu X ~ N ( 2 µ;σ ). Chú ý. 54
Chương III. Các quy luật phân phối xác suất thường gặp.
Nếu µ = 0 và σ = 1 thì X ~ N( ) 1 ;
0 , ta nói X phân phối chuẩn tắc. Tính chất. Cho X ~ N ( 2 µ;σ ). Ta có: a) E(X ) = µ . b) D(X ) 2 = σ . Hàm Gauss. 2 x t 1 Đó là hàm Φ(x) ∫ − =
e 2 dt , hay còn gọi là tích phân Laplace, trong đó: 2π 0 2 1 x + − f (x) 2 = e
gọi là hàm mật độ Gauss. 2π 2 x t 1 + F(x) ∫ − =
e 2 dt gọi là hàm phân phối xác suất Gauss. 2π −∞
Dễ thấy f (x) và F(x) cũng là hàm mật độhàm phân phối xác suất của đại lượng
ngẫu nhiên X ~ N( ) 1 ; 0 . Nhận xét:
a) Φ(− x) = −Φ(x), x ∀ ∈ R . 2 1 +∞ t − 2 b) π lim Φ(x) = , do 2 ∫e dt = . x→+∞ 2 2 0 2 c) − f (x) 1 x 2 = e
là hàm số chẵn nên có đồ thị nhận Oy làm trục đối xứng. 2π Định lí.
Cho F(x) là hàm phân phối xác suất của đại lượng ngẫu nhiên X ~ N( ) 1 ; 0 . Ta có 1
a) F(x) = + Φ(x). 2
b) P(α < X < β ) = F(β ) − F(α ) = Φ(β )− Φ(α ).
c) P( X < α ) = Φ 2 (α ), α > 0 . Chứng minh. 2 2 2 x t 0 t x t 1 1 1 1 a) D − −
ễ thấy F(x) = e 2 dt ∫ − = e 2 dt ∫ + e 2 dt ∫ = + Φ(x) 2π 2π 2π 2 −∞ −∞ 0 b) Ta có 55 Bài giảng β 0 β
P(α < X < β ) = ∫ f (x)dx = ∫ f (x)dx + ∫ f (x)dx α α 0 α β
= −∫ f (x)dx + ∫ f (x)dx = Φ(β )− Φ(α ) 0 0
c) Ta có P( X < α ) = P(−α < X < α ) = F(α )− F(−α ) = Φ(α )− Φ(−α ) = Φ 2 (α ). Tính chất. Cho X ~ N ( 2 µ;σ ). Ta có 1  x − µ 
a) F(x) = + Φ  . 2  σ   β − µ   α − µ   β − µ   α − µ 
b) P(α < X < β ) = F  − F  = Φ  − Φ  .  σ   σ   σ   σ   α 
c) P( X − µ < α ) = 2Φ  , α > 0.  σ  Tính chất. Cho Z ~ N( ) 1 ;
0 . Gọi z là số thỏa mãn P(Z > z = ( 0 ≤ α ≤ 1). Ta có α ) α α a) z = −z , trong đó P Z > z = 1 − α − 1 α α − 1 z là số thỏa mãn ( − 1 ) α α 1
b) P(Z > z ) = α 2 (với 0 ≤ α ≤ ) α 2 Chú ý. 2 x t 1 Giá trị hàm Φ(x) ∫ − =
e 2 dt được cho trong Bảng 2. Chẳng hạn Φ( 9 , 1 6) = , 0 475 . Ta 2π 0 quy ước Φ(m) = 5 ,
0 với mọi m ≥ 4 .
Hệ quả. (Quy tắc k -sigma). Nếu X ~ N ( 2
µ;σ ) thì P( X − µ < kσ ) = Φ 2 (k )
Với k = 3, ta có quy tắc 3 -sigma P( X − µ < 3σ ) = 2Φ( ) 3 = 9 , 0 973. Quy tắc này có
nghĩa là sai số giữa X và µ không quá σ
3 là gần chắc chắn. Khi đó, với xác suất 9 ,
0 973 giá trị của đại lượng ngẫu nhiên X nằm trong khoảng (µ − σ 3 ; µ + σ 3 ). Ví dụ 2.2.
Cho đại lượng ngẫu nhiên Z có phân phối chuẩn tắc N( ) 1 ;
0 . Tìm diện tích phần nằm
bên dưới đường cong chuẩn tắc này.
a) Ở bên phải đường thẳng z = 8 , 1 4.
b) Ở giữa hai đường thẳng z = − 9 , 1 7 và z = 8 , 0 6 . 56
Chương III. Các quy luật phân phối xác suất thường gặp. Giải 1 1
a) Ta có diện tích bằng P(Z > 8 , 1 4) = − Φ( 8 , 1 4) = − , 0 467 = , 0 033 . 2 2 b) Ta có diện tích bằng P(− 9 , 1 7 < Z < 8 , 0 6) = Φ( 8 , 0 6) − Φ(− 9 , 1 7) = Φ( 8 , 0 6) + Φ( 9 , 1 7) = 3 , 0 05 + , 0 476 = 7 , 0 81 Ví dụ 2.3.
Cho đại lượng ngẫu nhiên Z có phân phối chuẩn tắc Z ~ N( ) 1 ;
0 . Dựa vào hình vẽ sau,
hãy tìm giá trị k sao cho
a) P(Z > k) = 3 , 0 015.
b) P(k < Z < − 1 , 0 8) = , 0 4197 . Giải 1
a) Dựa vào hình vẽ, ta có: P(Z > k) = 3 , 0 015 = − Φ(k ). 2 Khi đó Φ(k) = 1 ,
0 985 . Từ Bảng 2, ta suy ra k = 5 , 0 2 .
b) Ta có P(k < Z < − 1 , 0 8) = , 0 4197 = Φ(− 1 , 0 ) 8 − Φ(k ) = −Φ( 1 , 0 8) − Φ(k ) Khi đó, Φ(k) = Φ( 1 , 0 8) − ,
0 4197 . Từ Bảng 2, ta có Φ( 1 , 0 8) = 0 , 0 714 . Suy ra Φ(k) = − , 0 0714 − , 0 4197 = − , 0 4911 = −Φ( 3 , 2 7) = Φ(− 3 , 2 7) . 57 Bài giảng Vậy k = 3 , 2 − 7 . Ví dụ 2.4.
Cho đại lượng ngẫu nhiên ngẫu nhiên X có phân phối chuẩn X ~ N ( 2 50 1 ; 0 ). Tìm xác
suất để X nhận các giá trị trong khoảng (45 6 ; 2). Giải
Ta có xác suất cần tìm là P(  −   −  45 < X < 62) 62 50 45 50 = Φ  − Φ  = Φ( , 1 2) − Φ(− , 0 05) = Φ( , 1 2) + Φ( 5 , 0 )  10   10 
Dựa vào Bảng 2, ta tính được Φ( , 1 2) = 3 , 0 849, Φ( 5 , 0 ) = 1 , 0 915 .
Vậy P(45 < X < 62) = 3 , 0 849 + 1 , 0 915 = 5 , 0 764 . Ví dụ 2.5.
Cho đại lượng ngẫu ngẫu nhiên có phân phối N ( 2 5 , 0 ; 1
). Hãy tìm các xác suất sau:
a) P(− 5 ≤ X < , 1 213).
b) P( X −1 < , 0 64). c) P(X < ) 1 , 2 . d) P(X > 3 , 2 ). Giải a) Ta có  −   − − 
P(− 5 ≤ X < , 1 213) , 1 213 1 5 1 = Φ  − Φ  = Φ( , 0 426) − Φ(−12)  5 , 0   5 , 0  = Φ( , 0 426) + Φ(12) ≈ Φ( , 0 426) + Φ(4) 1 ≈ 6 , 1 64 + = 6 , 6 64 2  6 , 0 4 
b) P( X −1 < , 0 64) = 2Φ  = 2Φ( , 1 28) = 2 3 , 0 . 9973 = , 0 79946 .  5 , 0  1  1 , 2 −1 1 1 c) P(X < ) 1 , 2 = + Φ  = + Φ( , 2 2) = + , 0 4861 = 9 , 0 861. 2  5 , 0  2 2 d)   −  P(X > 3 ,
2 ) = 1− P(X ≤ 3 , 2 ) 1 3 , 2 1 = 1 −  + Φ   2  5 , 0  1 = − Φ( ) 1 6 , 2 = − , 0 49534 = 0 , 0 0466 2 2 Ví dụ 2.6.
Đường kính của một loại chi tiết do một máy sản xuất có phân phối chuẩn, kì vọng
20mm và có phương sai ( , 0 mm)2 2
. Tính xác suất lấy ngẫu nhiên một chi tiết 58
Chương III. Các quy luật phân phối xác suất thường gặp.
a) Có đường kính trong khoảng 19 9
, mm đến 20 m 3 , m .
b) Có đường kính sai khác với kì vọng không quá m 3 , 0 m . Giải Gọi 2
X là đường kính của một chi tiết, ta có X ~ N (2 ; 0 ( , 0 2) ). Khi đó a) Ta có  −   −  P(19 9 , < X < 20 3 , ) 20 3 , 20 19 9 , 20 = Φ  − Φ   , 0 2   , 0 2  = Φ( 5 , 1 ) − Φ(− 5 , 0 ) = Φ( 5 , 1 ) + Φ( 5 , 0 ) = , 0 4332 + 1 , 0 915 = , 0 6247
b) Áp dụng công thức, ta suy ra P(   X − µ < 3 , 0 ) 3 , 0 = 2Φ  = 2Φ( 5 , 1 ) = 2 , 0 . 4332 = 8 , 0 664 .  , 0 2  Ví dụ 2.7.
Gọi X là chỉ số thông minh – IQ (Intelligent Quota) của học sinh lứa tuổi 12-15. Giả sử X ~ N (85 2 ; 5).
a) Cho biết chỉ số IQ trung bình của học sinh là bao nhiêu?
b) Tính xác suất chọn được học sinh rất thông minh ( X ≥ 90 ).
c) Tính tỉ lệ học sinh có chỉ số IQ ∈ (80 9 ; 5).
d) Gọi Y là số học sinh có chỉ số IQ ∈ (80 9
; 5) trong lớp có 50 học sinh. Hãy chỉ rõ
phân phối xác suất của Y . Giải
a) Chỉ số IQ trung bình của học sinh là E(X ) = 85 . 1  90 − 85  1
b) P(X ≥ 90) = − Φ  = − Φ( ) 1 = 1− 5 , 0 413 = 1 , 0 587 . 2  5  2
c) Tỉ lệ học sinh có chỉ số IQ ∈ (80 9
; 5) là P(80 < X < 95). Khi đó P(  −   −  80 < X < 95) 95 85 80 85 = Φ  − Φ  = Φ(2) − Φ(− ) 1 = Φ(2) + Φ( ) 1  5   5  = , 0 4773 + 3 , 0 413 = 8 , 0 186 ≈ 8 , 0 2
d) Một lớp gồm 50 học sinh được chọn từ tập hợp học sinh với tỉ lệ
p = P(80 < X < 95) ≈ 8 ,
0 2 được xem là 50 phép thử với xác suất p = 8 , 0 2 . Do đó Y
phân phối nhị thức B(50 8 , 0 ;
2), tức là: P(Y = m) m m 50−m = C 8 , 0 2 . 1 , 0 8 , m = 5 ; 0 0 . 50 Ví dụ 2.8.
Cho đại lượng X có phân phối chuẩn X ~ N ( 2 ;
5 σ ). Cho biết P(X > 9) = , 0 2 . Tính 2 σ . Giải 59 Bài giảng 1  9 − 5  1  4 
Ta có P(X > 9) = − Φ  = − Φ  . 2  σ  2  σ   4  4
Khi đó P(X > 9) = , 0 2 suy ra Φ  = 3 , 0 hay = 8 , 0 5 . Vậy 2 σ = 22 1 , 4 .  σ  σ
2.4. Phân phối Chi bình phương. Định nghĩa.
Đại lượng ngẫu nhiên X được gọi là có phân phối Chi bình phương ( 2 χ ) với n bậc  x n − −1  2 e . 2 x  , x > 0 t n
ự do nếu hàm mật độ của nó có dạng: f (x) =   n  2 2 . Γ    2    , 0 x ≤ 0 Kí hiệu X 2 ~ χ (n). Các tính chất.
+ Nếu dãy các đại lượng ngẫu nhiên độc lập X , 1 X , K , 2
X là có phân phối chuẩn n tắc thì 2 2 2 X = X L
có phân phối Chi bình phương ( 2 χ ) với 1 + X 2 + + X n bậc tự do. n + Cho X 2 ~ χ (n). Ta có:
a) E(X ) = n .
b) D(X ) = 2n . Chứng minh n x 1 +∞ +∞ − − +∞ 2 2 x e 1 n x
a) Ta có: E(X ) = ∫ xf (x)dx = ∫ x dx = n n ∫ − 2 2 x e dxn   n  −∞ 0 2 2 Γ  2 2 Γ  0  2   2  nn  Γ .   x +∞ n 1 1   2 2 −   t n
Đặt: t = . Khi đó E(X ) = 2 . . ∫t 2e dt = .2 Γ .  +1 = 2 = n . 2  n   n   2   n  Γ  0 Γ  Γ   2   2   2 
b) Tương tự, ta chứng minh được +∞ +∞ n x E( 2 X ) 2 + − = ∫ x f (x) 1 1 2 2 dx = . n
x e dx = n(n + 2)  n  −∞ 2 2 0 Γ   2 
Khi đó D(X ) = E(X 2 )− E 2 (X ) = 2n (đpcm) 60
Chương III. Các quy luật phân phối xác suất thường gặp.
2.5. Phân phối Student. Định nghĩa.
Đại lượng ngẫu nhiên liên tục X được gọi là có phân phối Student với n bậc tự do  n +1 n 1 + Γ  2 −    x  2 1 n 2
ếu hàm mật độ của X có dạng: f (x) = . 1  +  n π  n   n  Γ   2 
Kí hiệu: X ~ T (n). Các tính chất. n +1 n 1 + Γ  2 −    x  2 1 + f (x) 2 = . 1  + 
là hàm số chẵn nên có đồ thị nhận Oy làm trục đối n π  n   n  Γ   2  xứng. + X X ~ N ( ) 1 ; 0 , Y 2
~ χ (n) và X , Y độc lập thì T =
có phân phối T (n), với n bậc tự Y n do.
+ Cho X ~ T (n). Ta có: a) E(X ) = 0 . b) D( ) n X = . n − 2
2.6. Công thức tính gần đúng.
2.6.1. Phân phối siêu bội và phân phối nhị thức. Định lí. Cho M
X ~ H (N; M ; n) . Nếu N khá lớn ( N > 10n ) và p = thì ta có thể coi NM
X ~ Bn;  .  N k nk k nk .     T C C M M
ức là ta có công thức tính gần đúng M N M kC   1 −  . n n CN   N N
2.6.2. Phân phối nhị thức và phân phối Poisson. Định lí.
Cho X ~ B(n; p). Nếu p khá bé (gần 0 , ta xem p < 1 ,
0 ) và khi n khá lớn ( n ≥ 30 ) thì k = ;
0 n , ta có thể coi X ~ P(np) . 61 Bài giảng k k nk
np (np)k
Tức là ta có công thức gần đúng P(X = k) = C p − ≈ , k = ; 0 n . n (1 p) e k! Nhận xét.
Cho X ~ B(n; p). Nếu p khá lớn (gần 1) và n khá lớn thì k = ;
0 n , ta có thể dùng phân
phối Poisson để tính gần đúng.
Thật vậy, X ~ B(n; p) thì Y ~ B(n 1
; − p), trong đó X là số lần biến cố A xuất hiện và
Y là số lần biến cố A xuất hiện.
Do p khá lớn nên 1− p khá bé. Do dó Y ~ P(n(1− p)). −n(1− p ) e
[n(1− p)]nk
Khi đó P(X = k) = P(Y = n k) = . (n k)! Ví dụ 2.9.
Một cửa hàng sản xuất đĩa nhạc, trung bình sản xuất 1000 đĩa thì có 1 đĩa hỏng. Tìm
xác suất để khi hãng đó sản xuất 3000 đĩa thì có nhiều hơn 5 đĩa không bị hỏng. Giải 999
Xác suất để được đĩa không hỏng trong 1000 đĩa là p = . 1000  999  999
Gọi X là số đĩa không bị hỏng. Ta có X ~ B300 ; 0
 , ta có n = 3000 và p =  1000  1000 khá lớn. 1
Suy ra Y ~ P(λ) với λ = 3000. = 3 . 1000
Do Y = 3000 − X nên ta có 5
P(X > 5) = P(Y ≤ 5) = ∑ P(Y = k ) = 0 , 0 498 + 1 , 0 494 + L + 1 , 0 008 = 9 , 0 2 . k =0
2.6.3. Phân phối nhị thức và phân phối chuẩn. Định lí.
Cho X ~ B(n; p). Nếu p không quá gần 0 và 1, khi đó n khá lớn ( n ≥ 30 , np ≥ 10 ) thì
ta có thể coi X ~ N (np;npq).   1 T k k nk k np
ức là ta có công thức gần đúng P(X = k ) = C p 1 , k = ; 0 n , n ( − q)   ≈ f   npqnpq  2 trong 1 x − đó: f (x) 2 = e . 2π Ví dụ 2.10.
Biến cố A: “một anh B yêu một cô gái” có xác suất P(A) = p = , 0 25 không đổi. Tìm
xác suất để khi anh B quen với 243 người cô gái thì có đúng 70 lần biến cố A xảy ra. 62
Chương III. Các quy luật phân phối xác suất thường gặp. Giải.
Chú ý rằng, anh B quen với 243 người là một phép thử độc lập.
Gọi X là số lần biến cố A xảy ra trong 243 phép thử độc lập. Ta có X ~ B(243 , 0 ; 25) .
Do n = 243 > 30 và np = 243 , 0
. 25 > 10 nên ta xem X ~ (np;npq) với np = 243 , 0 . 25 và npq = 243 , 0 . 25 7 , 0 . 5.   V 1 70 − 243. , 0 25 1 ậy P(X = 70)   ≈ f = f ( 3 , 1 7) = 0 , 0 231. 243 , 0 . 25 7 , 0 . 5  243. , 0 25. , 0 75  7 , 6 5 
3. Đại lượng ngẫu nhiên nhiều chiều. 3.1. Khái niệm.
Ở các phần đã học, chúng ta đã xét các đại lượng ngẫu nhiên mà các giá trị có thể của
chúng được biểu diễn bằng một số. Các đại lượng ngẫu nhiên đó được gọi là đại lượng
ngẫu nhiên một chiều. Ngoài các đại lượng ngẫu nhiên một chiều, trong thực tế ta còn
gặp các đại lượng ngẫu nhiên mà các giá trị có thể có của nó được xác định bằng 2 ,
3 , K , n số. Những đại lượng ngẫu nhiên này được gọi là các đại lượng ngẫu nhiên
hai chiều, ba chiều, K, n chiều.
Xét đại lượng ngẫu nhiên hai chiều, kí hiệu là (X ;Y ). Trong đó, X Y được gọi là
các thành phần của đại lượng ngẫu nhiên hai chiều. Hai đại lượng ngẫu nhiên X Y
được xét đồng thời tạo nên hệ hai đại lượng ngẫu nhiên. Tương tự như vậy, đại lượng
ngẫu nhiên n chiều có thể xem là hệ của n đại lượng ngẫu nhiên. Ví dụ 3.1.
Một máy sản xuất một loại sản phẩm. Nếu kích thước của sản phẩm được đo bằng
chiều dài X và chiều rộng Y , thì ta có đại lượng ngẫu nhiên hai chiều (X ;Y ), còn nếu
tính thêm cả chiều cao Z nữa thì ta có đại lượng ngẫu nhiên ba chiều (X ;Y; Z ).
Trong thực tế, người ta cũng chia các đại lượng ngẫu nhiên nhiều chiều thành hai loại: rời rạc và liên tục.
Các đại lượng ngẫu nhiên nhiều chiều được gọi là rời rạc nếu các thành phần của nó là
đại lượng ngẫu nhiên rời rạc.
Các đại lượng ngẫu nhiên nhiều chiều được gọi là liên tục nếu các thành phần của nó
đại lượng ngẫu nhiên liên tục.
Sau đây, ta xét các đại lượng ngẫu nhiên hai chiều.
3.2. Quy luật phân phối xác suất của đại lượng ngẫu nhiên hai chiều.
Đối với đại lượng ngẫu nhiên hai chiều, người ta cũng dùng bảng phân phối xác suất,
hàm phân phối xác suất, hàm mật độ xác suất để thiết lập quy luật phân phối xác suất của chúng.
* Bảng phân phối xác suất của đại lượng ngẫu nhiên hai chiều.
Bảng phân phối xác suất của đại lượng ngẫu nhiên hai chiều (X ;Y ) rời rạc là 63 Bài giảng Y 1 y y … 2 y h X j 1 p ( 1 x ) 1 x 1 p 1 1 p … 2 p h 1 x 1 p (x2 ) 2 p 21 p … 22 p2h M M M … M M x p p p p 1 (xk ) k k1 k 2 khp p p 1 2 ( yh ) 2 ( y2 ) 2 ( 1 y ) i
trong đó p = P X = x ;Y = y là xác suất đồng thời để đại lượng X lấy giá trị x ; ij ( i j ) i i = ;
1 k Y lấy giá trị y ; j = ;
1 h . Bảng này có thể vô hạn khi k , h nhận giá trị + ∞ . j Các tính chất. a) 0 ≤ p ≤ 1 ij k h b) ∑ ∑ p = 1. ij i 1 = j 1 =
3.3. Hàm phân phối của đại lượng ngẫu nhiên hai chiều.
Xét đại lượng ngẫu nhiên hai chiều (X ;Y ) có thể rời rạc hoặc liên tục. Xét x , y là hai
số thực bất kì, khi đó biến cố (X < x;Y < y) là biến cố để X nhận giá trị nhỏ hơn x , và
Y nhận giá trị nhỏ hơn y .
Hàm phân phối xác suất của đại lượng ngẫu nhiên hai chiều là
F (x; y) = P(X < x;Y < y)
Các phân phối biên của đại lượng ngẫu nhiên hai chiều rời rạc (X ;Y ) là: k
a) Phân phối xác suất của X P(X = x p . i ) = ∑ ij j=1 h
b) Phân phối xác suất của Y P(Y = y p . i ) = ∑ ij i=1 Ví dụ 3.2.
Cho bảng phân phối của đại lượng ngẫu nhiên 2 chiều (X ;Y ) như sau Y 1 2 3 X 1 1 , 0 0 , 0 25 1 , 0 0 2 1 , 0 5 , 0 05 3 , 0 5
Tìm bảng phân phối của các đại lượng X Y sau đó tính F( ) 3 ; 1 , 2 ? Giải.
Lấy tổng hàng và tổng cột tương ứng, ta có các phân phối biên như sau 64
Chương III. Các quy luật phân phối xác suất thường gặp. X 1 2 P(X = x , 0 45 5 , 0 5 i ) và Y 1 2 3 P(Y = y , 0 25 3 , 0 0 , 0 45 j ) 2 3 Ta có: F( 3 ; 1 , 2
) = ∑ ∑ p = p + p + p + p = 1, 0 0 + , 0 25 + 1 , 0 5 + , 0 05 = 5 , 0 5 . ij 11 12 21 22 x <2 1 , y <3 i j 4. Bài tập chương. 1.
Tung hai con xúc xắc đồng thời. Gọi X là tổng số chấm xuất hiện trên hai con xúc xắc
đó. Lập bảng phân phối xác suất của X . 2.
Một đổi tuyển có 3 vận động viên. Xác suất thi đấu thắng trận của từng vận động viên lần lượt là , 0 4 ; 3 , 0 ; ,
0 6 . Mỗi vận động viên thi đấu độc lập một trận với đội bạn.
a) Tìm phân phối xác suất số trận thắng của đội tuyển.
b) Lập hàm phân phối xác suất số trận thắng của đội tuyển.
c) Tìm xác suất đội tuyển thắng ít nhất một trận. 3.
Trong một hộp có chứa 3 bi đỏ và 4 bi đen. Lấy ngẫu nhiên từng viên cho đến khi lấy
được bi đỏ thì dừng. Gọi X là số bi cần lấy. Lập bảng phân phối xác suất của X . 4.
Trong một hộp có 3 bi đỏ và 4 bi đen. Lấy ngẫu nhiên từ hộp ra 2 viên. Nếu được 2
bi đỏ thì bỏ trở lại hộp 4 bi đỏ, nếu được 1 bi đỏ thì bỏ trở lại hộp 2 bi đỏ, nếu có 2
viên đều đen thì thôi. Gọi X là biến ngẫu nhiên chỉ số bi đỏ sau khi thực hiện phép thử.
Lập bảng phân phối của X . 5.
Một hộp đựng 10 sản phẩm tốt, 2 sản phẩm xấu. Lấy ngẫu nhiên từng sản phẩm cho
đến khi lấy ra được sản phẩm tốt. Tìm phân phối xác suất số sản phẩm được lấy ra. 6.
Có hai hộp bi I , II . Hộp I có 8 bi xanh và 2 bi đỏ. Hộp II có 7 bi xanh và 2 bi đỏ.
Từ hộp I lấy ngẫu nhiên 2 bi bỏ vào hộp II , sau đó từ hộp II lấy ra 2 bi.
a) Tìm phân phối xác suất số bi xanh được lấy ra.
b) Lập hàm phân phối xác suất số bi xanh được lấy ra. 7.
Cho X là biến ngẫu nhiên có phân phối xác suất X 1 2 3 4 5 6 7 P a 2a 2a a 3 2 a 2 2a a 2 7 + a a) Xác định a .
b) Tính P(X ≥ 5), P(X < 3). 1
c) Tìm số k nhỏ nhất sao cho P(X k ) ≥ . 2 8.
Biến ngẫu nhiên rời rạc X có bảng phân phối xác suất X 0 1 2 3 4 P(X ) 0,05 0,2 0,3 0,3 0,15
a) Lập hàm phân phối F (x) và vẽ đồ thị của F(x). 65 Bài giảng
b) Tìm P(0 ≤ X ≤ 2), P(1 < X ≤ )
3 và P(X > 2). 9.
Trong các hàm sau đây, hàm nào là hàm mật độ xác suất:  , 0 x ∉ [ ] 1 ; 0 a) f (x) =  . 3 2 x , x ∈ [ ] 1 ; 0  2 , 0 x <  b) π f (x) =   1 1 2  cos , x ≥  2 x x π
Tìm E(X ), D(X ) , Mod(X ) và Med(X ) của biến ngẫu nhiên X tương ứng.  , 0 x < 2 
10. Cho biến ngẫu nhiên 2
X có hàm phân phối F . X (x ) = (
x − 2) , 2 < x ≤ 3   , 1 x > 3
a) Tìm hàm mật độ f (x) .
b) Tính P[1 < X < , 1 6].   , 0 x ≤ −1  3 3 1
11. Cho biến ngẫu nhiên X có hàm phân phối F . X (x ) =  x + , −1 < x ≤ 4 4 3  1  , 1 x >  3
a) Tìm hàm mật độ của biến ngẫu nhiên X .
b) Tính P(− 2 < X < 0).  , 0
x ≤ 0 ∨ x > 2 
12. Cho hàm số f (x) =  2 ax , 0 < x ≤ 1 .  2
a(2 − x) , 1 < x ≤ 2
a) Tìm a để f (x) là hàm mật độ.
b) Tìm hàm phân phối tương ứng.   π π 
Acos x, x ∈  − ;    2 2 
13. Cho hàm số f (x) =  .   π π  , 0 x ∉  − ;    2 2 
a) Tìm A để f (x) là hàm mật độ của biến ngẫu nhiên nào đó.  π π 
b) Tìm hàm phân phối F . Tìm P − ≤ X <  . X (x )  6 3   , 0 x < 0
14. Cho hàm f (x) =  .  2 −2 Bx e x , x ≥ 0
a) Xác định để f (x) là hàm mật độ của biến ngẫu nhiên nào đó.
b) Tìm hàm phân phối F . X (x ) 66
Chương III. Các quy luật phân phối xác suất thường gặp. x  − 15.
Tìm k để hàm f (x) = ke
, x ≥ 0 (θ > 0) là hàm mật độ của biến X nào đó. Tìm  , 0 x < 0
hàm phân phối tương ứng.
16. Cho biến ngẫu nhiên X có hàm phân phối  , 0 x ≤ −2   F (x) 1 1 x =  +
arcsin , − 2 < x ≤ 2 2 π 2  ,1 x > 2
a) Tìm P(−1 < X < ) 1 .
b) Tìm hàm mật độ f (x) .  x − 1  e λ , x , 0 λ 0 17. > >
Cho hàm mật độ của biến ngẫu nhiên X là: f (x) = λ .   , 0 x ≤ 0
a) Tìm hàm phân phối của biến ngẫu nhiên X và tính xác suất P(0 ≤ X < λ) .
b) Tính kì vọng và phương sai của X .
18. Cho biến ngẫu nhiên X có bảng phân phối xác suất: X −1 0 1 2 P 1 , 0 3 , 0 , 0 2 , 0 4
a) Lập bảng phân phối xác suất của biến ngẫu nhiên 2 Y = X + 2008 .
b) Tính E(Y ), D(Y ).
19. Cho P(A) = 8 ,
0 . Tìm xác suất khi thực hiện 100 phép thử thì
a) Số lần xảy ra biến cố A lớn hơn 75.
b) Số lần xảy ra biến cố A không quá 75.
c) Số lần xảy ra biến cố A là nằm trong đoạn [75 9 ; 0].
Đáp số và hướng dẫn. 1. Ta có: X 2 3 4 5 6 7 8 9 10 11 12 P 1 2 3 4 5 6 5 4 3 2 1 36 36 36 36 36 36 36 36 36 36 36 2.
Gọi X là số trận thắng của đội tuyển. a) Ta có: X 0 1 2 3 P 1 , 0 68 , 0 436 3 , 0 24 , 0 072 b)  , 0 x ≤ 0  1, 0 6 , 8 0 < x ≤ 1  f (x) =  , 0 60 , 4 1 < x ≤ 2  9, 0 2 , 8 2 x 3  < ≤  ,1 x > 3 67 Bài giảng
c) Dễ thấy: P(X ≥ )
1 = 1− P(X = 0) = 1− 01 6 , 8 = 8 , 0 32 . 3.
Vì trong hộp có 4 bi đen nên ta chỉ lấy nhiều nhất là 5 bi là được bi đỏ. Bảng phân phối X 1 2 3 4 5 P 3 4 3 4 3 3 4 3 2 3 4 3 2 1 . . . . . . . . . 7 7 6 7 6 5 7 6 5 4 7 6 5 4 4.
Ta có X nhận các giá trị tương ứng là: 3 , 4 , 5 . X 3 4 5 P 2 C 1 1 C . 2 4 C C 3 4 3 2 C 2 2 7 C7 C7 5. Ta có X 1 2 3 P 4 8 1 5 45 45 6. Ta có a) X 0 1 2 P 82 916 1477 2475 2475 2475 7. a) 10 2
a + 9a = 1, a ≥ 0 ⇒ a = 1 , 0 , b) , 0 2 , c) 3 , 0 8. a)  , 0 x ≤ 0   0 , 0 , 5 0 < x ≤ 1 , 0 205 1 x 2 f (x)  < ≤ =   5 , 0 0 , 5 2 < x ≤ 3  8 , 0 0 , 5 3 < x ≤ 4   ,1 x > 4
b) P(0 ≤ X < 2) = P(X = 0) + P(X = ) 1 = ,
0 05 , P(1 < X ≤ )
3 = P(X = 2) + P(X = ) 3 = , 0 65 ,
P(X > 2) = 1 − P(X ≤ 2) = , 0 45. 9.
Cả hai hàm này là hàm mật độ xác suất.  , 0 x < 2 
10. a) f (x) = 2(x − 2), 2 ≤ x ≤ 3 , b) 0   , 0 x > 3   , 0 x ≤ −1  3 1 3
11. a) f (x) =  , −1 < x ≤ , b) P(− 2 < X < 0) = . 4 3 4  1  , 0 x >  3 68
Chương III. Các quy luật phân phối xác suất thường gặp. 3 12. a) a = . 2 1 13. A = . 2 +∞ +∞ 2 −2x 1
14. a) ∫ f (x)dx = 1 ⇒ ∫ Bx e dx = 1 ⇔ B = . 4 −∞ 0 1 15. k = . θ 2  , 0 x ≤ −2  1  1
16. a) P(−1 < X < )
1 = , b) f (x) =  , − 2 < x ≤ 2 . 3 π 4 − 2 x   , 0 x > 2  , 0 x ≤ 0
17. a) F(x) =  x
, b) E(X ) = λ , D(X ) 2 = λ .  − 1 − λ e , x > , 0 λ > 0
18. b) E(Y ) = 2009 9 , , D(Y ) = 9 , 3 .
C. Phương pháp giảng dạy.
- Ứng dụng Excel cho việc tính các giá trị của biến ngẫu nhiên có phân phối chuẩn,
phân phối Poisson, phân phối Student, phân phối chi bình phương.
- Giảng viên gửi bài giảng cho sinh viên đọc trước. Giảng viên trình bày bài giảng trên lớp
theo phương pháp thuyết trình hỏi đáp. Giao bài tập cho sinh viên về nhà làm. Giới thiệu một số tài liệu tham khảo.
D. Tài liệu tham khảo
[1] Đậu Thế Cấp, Xác suất thống kê: Lí thuyết và các bài tập, NXB Giáo dục, 2006.
[2] Đặng Công Hanh, Đặng Ngọc Dục, Giáo trình Lý thuyết xác suất và Thống kê
toán, trường Đại học Duy Tân,1996
[3] PGS. TS. Phạm Xuân Kiều, Giáo Trình xác suất và thống kê, NXB Giáo dục, 2005.
[4] Trần Văn Minh, Phí Thị Vân Anh, Xác suất thống kê, NXB Giao thông vận tải, 2008.
[5] Trần Văn Minh, Phí Thị Vân Anh, Hướng dẫn giải bài tập Xác suất thống kê,
NXB Giao thông vận tải, 2008. 69
Chương IV. Lí thuyết mẫu A. Mục tiêu.
- Giới thiệu các khái niệm: tổng thể, mẫu ngẫu nhiên, mẫu cụ thể, kích thước mẫu.
- Xây dựng các bảng phân phối thực nghiệm và vẽ biểu đồ của chúng.
- Xây dựng hàm phân phối mẫu và giới thiệu đa giác tần suất tích luỹ.
- Giới thiệu các đặc trưng của mẫu:trung bình mẫu, phương sai mẫu chưa hiệu chỉnh
và đã hiệu chỉnh, độ lệch chuẩn mẫu chưa hiệu chỉnh và đã hiệu chỉnh, tỉ lệ mẫu.
- Giới thiệu luật phân phối của các đặc trưng mẫu. B. Nội dung.
1. Tổng thể và mẫu.
1.1. Mở đầu.
Trong thực tế, ta thường phải nghiên cứu một tập hợp các phần tử theo một hay
nhiều dấu hiệu đặc trưng cho các phần tử. Nhưng tập hợp có quá nhiều phần tử thì
không thể nghiên cứu tất cả các phần tử, vì nếu làm như vậy sẽ tốn thời gian, công
sức, … Do đó, người ta thường nghiên cứu một phần, đặc biệt các phương pháp chọn mẫu.
Giả sử ta cần nghiên cứu một tập hợp gồm N phần tử, tập hợp này gọi là tổng thể,
mỗi phần tử của tập hợp này gọi là một cá thể, N gọi là kích thước của tổng thể.
Ta lấy ngẫu nhiên n phần tử, từ tổng thể gọi là một mẫu. Số n gọi là kích thước
mẫu
. Từ những thông tin có được trên mẫu này ta suy ra kết luận của tổng thể, do đó
phải lấy mẫu như thế nào để đại diện cho tổng thể. Trong mỗi ngành, mỗi lĩnh vực
có các phương pháp riêng mang tính đặc thù của ngành, để sao cho việc lấy mẫu đại
diện trung thực cho tổng thể. Ví dụ 1.1.
Ta xét bài toán sau: Để có chiến lược cho chương trình dinh dưỡng quốc gia nhằm
tăng chiều cao của người dân, người ta đi tìm chiều cao của những người trưởng thành ở Việt Nam.
Khi đó, trong bài toán này:
a) Tập hợp gồm tất cả những người trưởng thành ở Việt Nam, ta gọi là tổng thể.
b) Mỗi người trong tổng thể, được gọi là một cá thể.
c) Chiều cao của người trong tổng thể là một đại lượng ngẫu nhiên. Bài giảng
d) Do số người trưởng thành ở Việt Nam là rất lớn, nên ta không thể đo chiều cao tất
cả được mà chỉ ra một số người (chẳng hạn 500 người) để đo chiều cao. Tập hợp
500 người này được gọi là một mẫu, số 500 được gọi là kích thước mẫu.
Ta nói rằng một mẫu là ngẫu nhiên nếu trong phép lấy mẫu đó, mỗi phần tử của tổng
thể đều được chọn một cách độc lập và có xác suất được chọn như nhau. Ngoài
phương pháp lấy mẫu ngẫu nhiên, ta còn có các phương pháp lấy mẫu khác nữa như
chọn mẫu với xác suất không đều, chọn mẫu theo nhóm trội, mẫu chùm v.v… Trong
bài giảng này, chúng ta giới thiệu cách lấy mẫu đơn giản nhất và được sử dụng rộng
rãi trong các lĩnh vực khác nhau.
Khi chọn mẫu nếu phần tử đã chọn loại ra khỏi tổng thể mới chọn phần tử tiếp theo
thì gọi là mẫu không hoàn lại, nếu phần tử đã chọn trả lại tổng thể mới chọn phần tử
tiếp theo thì gọi là mẫu có hoàn lại.
Khi kích thước của tổng thể đủ lớn thì có thể coi 2 cách lấy trên là như nhau.
1.2. Mẫu ngẫu nhiên, mẫu cụ thể.
Tiến hành n quan sát độc lập về biến ngẫu nhiên X nào đó trên tổng thể.
Ta gọi X là quan sát thứ i của biến ngẫu nhiên X , i = ; 1 n . i
Khi đó (X ; X ;K; X được gọi là mẫu ngẫu nhiên, trong đó 1 2
X , X , K , X độc n ) 1 2 n
lập và có cùng phân phối xác suất với X .
Ta gọi x là kết quả quan sát thứ i . Khi đó (x ; x ;K; x n giá trị quan sát được. 1 2 n ) i
Đó là giá trị cụ thể của mẫu ngẫu nhiên (X ; X ;K; X nhận còn được gọi là m 1 2 ẫu n )
cụ thể (hoặc mẫu thực nghiệm). Chú ý.
+ Ta chỉ xét các kết quả quan sát độc lập.
+ Khi xét lí thuyết, ta dùng mẫu ngẫu nhiên, còn khi làm toán thì ta dùng mẫu cụ thể. Ví dụ 1.2.
Xét một tổng thể là một hộp gồm có 10 cây thước, trong đó có 3 cây thước dài 10
cm, 5 cây thước dài 20 cm và 2 cây thước dài 30 cm.
Gọi X là biến ngẫu nhiên đặc trưng cho chiều dài (xét về lượng) của cây thước.
Ta xét tổng thể về mặt định lượng.
Khi đó, X có bảng phân phối như sau: X 10 cm 20 cm 25 cm P 3 5 2 10 10 10
Ta thực hiện việc lấy ngẫu nhiên (có hoàn lại) 5 cây thước. Khi đó Gọi X i = X
i là chiều dài của cây thước được lấy ra lần thứ i , 5 ;
1 thì i có phân phối X 10 cm 20 cm 30 cm i 72
Chương IV. Lí thuyết mẫu. P 3 5 2 10 10 10 Như vậy, X , 1 X , K , 2
X là 5 biến ngẫu nhiên độc lập có cùng phân phối với 5 X .
(X ; X ;K; X là mẫu ngẫu nhiên. 1 2 n )
Ta thực hiện 5 lần lấy cây thước như sau: (x ; x ;K; x là mẫu cụ thể trong 5 lần 1 2 n )
quan sát, chẳng hạn là: X = x = 10cm , = = 10 , = = 30 , 1 1 X x cm 2 2 X x cm 3 3
X = x = 2 c 0 m , = = 30 . 4 4 X x cm 5 5
Vậy (x ; x ;K; x = 10 1 ; 0 3 ; 0 ;20 3 ; 0 . 1 2 n ) ( cm cm cm cm cm)
Bây giờ, ta xét tổng thể về mặt định tính.
Nếu ta xem những cây thước nhỏ hơn 20 cm là “không đạt yêu cầu”.
Lấy ngẫu nhiên 1 cây thước. Gọi X là số cây thước “không đạt yêu cầu”.
Ta có bảng phân phối xác suất của X X 0 1 P 3 7 10 10
Gọi X là số cây thước “không đạt yêu cầu” khi lấy cây thước thứ i , i = 5 ; 1 . Ta thấy i
các X có cùng phân phối với X . (X ; X ;K; X là mẫu ngẫu nhiên. 1 2 n ) i
Ta xem chiều dài cụ thể của cây thước được lấy ra. Khi đó, chẳng hạn ta có kết quả sau (x ; x ;K;
được gọi là mẫu cụ thể. 1 2 x ) = n ( ) 1 ; 0 ; 0 ; 1 ; 1
1.3. Bảng phân phối tần số.
1.3.1. Phân loại mẫu và bảng phân phối tần số.
Giả sử mẫu (x x ;K; x có kích thước khác nhau khi đó mẫu ; 1 2
n , nếu các x , i = ; 1 n n ) i
gọi là mẫu đơn.
Nếu trong mẫu có k giá trị khác nhau với 1 x xuất hiện 1
n lần, x xuất hiện 2 n lần, 2
…, x xuất hiện n lần thì n + n + L + n = n n gọi là tần số của x , i = ; 1 n . k k 1 2 k i i
Mẫu có các giá trị giống nhau gọi là mẫu lặp.
Nếu ta xếp x theo thứ tự tăng dần x < x < L < x và lập bảng gồm các x và tần số i 1 2 n i
n tương ứng thì ta được một bảng i X 1 x x … 2 x k Tần số 1 n n … 2 n k
gọi là bảng phân phối tần số. 73 Bài giảng
Trong trường hợp mẫu có kích thước lớn các giá trị của X khác nhau không nhiều,
để tiện lợi cho việc tính toán ta phân miền giá trị của X thành k khoảng (có thể chia
đều hoặc không đều) [a ; ; [a ;
; K ; [a ;a và nếu có n giá trị của mẫu k 1 − k ) 1 a2 ) 0 1 a ) i
xuất hiện trong khoảng [a ; ; thì bảng 0 1 a ) i = ; 1 k X [a ; [a ; … [a ;a k 1 − k ) 1 a2 ) 0 1 a ) Tần số 1 n n … 2 n k
gọi là bảng phân phối tần số phân lớp.
Ta quy ước đầu mút bên trái của một khoảng thuộc khoảng đó khi tính tần số của mỗi lớp. Ví dụ 1.3.
Thống kê cân nặng X của 370 trẻ sơ sinh, ta được bảng phân phối tần số phân lớp sau 0 , 3 − , 3 , 3 2 − , 3 4 X , 2 4 − 6 , 2 6 , 2 − 8 , 2 8 , 2 − 0 , 3 2 , 3 4 − 6 , 3 Tần số 6 44 76 100 95 49
Thông thường, người ta chia số liệu thành từ 5 đến 15 lớp. Nếu số lớp nhiều hơn có
thể giúp phân tích tốt hơn, nhưng sự cải thiện không nhiều, nếu số lớp ít quá, thì các
thông tin có thể bị mất khi xử lí.
Một bảng phân phối tần số phân lớp có thể đưa về bảng phân phối lặp bằng phép lấy
trung bình cộng của mỗi lớp. Ví dụ 1.4.
Xét Ví dụ 1.3, ta có: X 5 , 2 7 , 2 9 , 2 1 , 3 3 , 3 5 , 3 Tần số 6 44 76 100 95 49
1.3.2. Bảng phân phối tần suất. T n
ừ bảng phân phối, nếu ta đặt f i = , i = ;
1 k thì f được gọi là tần suất xuất hiện i n i của x , i = ; 1 k . Khi đó, bảng i x x x i 1 x 2 k Tần suất f 1 f … 2 f k
được gọi là bảng phân phối tần suất, nó rất giống với bảng phân phối xác suất của
một biến ngẫu nhiên rời rạc.
Để có được một hình dung về phân phối mẫu, người ta thường dùng đồ thị để biểu
diễn bảng phân phối tần suất.
1.3.3. Đa giác tần suất
Trên đồ thị, nối các điểm có tọa độ (x ; f , i = ;
1 k ta được đường gấp khúc gọi là đa i i )
giác tần suất. 74
Chương IV. Lí thuyết mẫu.
1.3.4. Biểu đồ tần số.
Nếu mẫu cho bằng các khoảng phân lớp, ta xây dựng một biểu đồ gồm các hình chữ
nhật cạnh nhau có đáy bằng độ dài của khoảng phân lớp và có diện tích bằng tần số
của lớp và gọi là biểu đồ tần số.
Sau đây, là bảng phân phối tần suất, đa giác tần suất và biểu đồ tần số của Ví dụ 1.3. X 5 , 2 7 , 2 9 , 2 1 , 3 3 , 3 5 , 3 Tần số 6 44 76 100 95 49 Tần suất 0,162 0,119 0,205 0,270 0,257 0,132 0.3 0.25 0.2 t ấ u 0.1 s5 n ầ T0.1 0.05 0 2,5 2,7 2,9 3,1 3,3 3,5 x Đa giác tần suất 120 100 80 60 40 20 0 1 Biểu đồ tần số 75 Bài giảng
1.4. Hàm phân phối mẫu.
Với ĐLNN X , ta không thể biết được hàm phân phối xác suất F(x) của nó. Tuy
nhiên từ một mẫu thống kê (x ; x ;K; x của 1 2
X dựa vào bảng phân phối tần suất ta n )
có thể thu được những thông tin quan trọng về F(x).
Từ bảng phân phối tần suất, ta xây dựng hàm F như sau: F x f . Hay n ( ) = ∑ n (x ) i x <x i  , 0 x x1   f ,
x < x x 1 1 2  f + f ,
x < x x 1 2 2 3 F x n ( ) =  K K K
f + f +L+ f , x < x x  1 2 n−1 n−1 n  ,1 x > xn F
được gọi là hàm phân phối thực nghiệm hoặc phân phối mẫu. n (x )  k
Trên đồ thị, nối các điểm có tọa độ  x ; f , i = ;
1 k ta được được đường gấp khúc i ∑   i i=1 
gọi là đa giác tần suất tích lũy.
Sau đây, là biểu đồ đa giác tần suất tích lũy Ví dụ 1.1. 1.2 1 0.8 Y 0.6 0.4 0.2 0 2.5 2.7 2.9 3.1 3.3 3.5 3.7 3.9 X
Biểu đồ đa giác tần suất tích lũy.
2. Các tham số đặc trưng của mẫu
2.1. Tỉ lệ mẫu.
Ta gọi hàm phân phối xác suất của biến ngẫu nhiên này là hàm phân phối mẫu; kì
vọng, phương sai của biến ngẫu nhiên là trung bình mẫu và phương sai mẫu. Khi đó, ta có: 76
Chương IV. Lí thuyết mẫu.
Trung bình mẫu:
x n + x n + L + x n
x = x f + x f + L + x f 1 1 2 2 k k = 1 1 2 2 k k n
Phương sai mẫu: 2 2 2 s 2 ˆ = (x x L 1
) f1 + (x x 2
) f2 + + (x x k ) fk ( 2 2 2 x x L 1
) n1 + (x x 2
) n2 + + (x x k ) nk = n
x 2n + x 2n + L x 2n Đặt: x2 +
= x 2 f + x 2 f + L + x 2 f 1 1 2 2 k k . 1 1 2 2 = k k n k 2 ∑n x2 − n i i (x) Theo tính ch 2
ất phương sai, ta có s2 ˆ = x2 − (x) i 1 = = . n
Phương sai mẫu hiệu chỉnh: L 2 n 2 (x x)2 1 1
n + (x x)2 2 n2 + + (x x k )2n s = ˆs = k . n − 1 n − 1
Tương tự với biến ngẫu nhiên, ta gọi 2
ˆs = ˆs là độ lệch mẫu và 2 s =
s độ lệch mẫu hiệu chỉnh.
Để tính các đặc trưng này, ta thường lập bảng sau x n x n x 2n i i i i i i 2 1 x 1 n 1 x 1 n 1 x 1 n x 2 2 n 2 x 2 n2 x 2 n2 … … … … x n x n x2n k k k k k kn nx 2 nx kk x nx2n i i i i T n
ừ bảng này, ta có x i= = 1 , x2 i= = 1 , 2
ˆs = x − (x)2 2 và 2 2 s = ˆs . n n n −1 Ví dụ 2.1. Cho mẫu x 5 10 15 20 25 i n 15 25 30 20 10 i
a) Viết hàm phân phối mẫu.
b) Tính trung bình mẫu, phương sai mẫu, phương sai mẫu hiệu chỉnh. Giải. 77 Bài giảng
a) Bảng phân phối xác suất X 5 10 15 20 25 Tần suất 1 , 0 5 , 0 25 3 , 0 0 , 0 20 1 , 0 0
Từ đó, ta có hàm phân phối là:  , 0 x ≤ 5   1 , 0 , 5 5 < x ≤ 10  , 0 4 , 5 10 < x ≤ 15 F n (x ) =   7 , 0 , 0 15 < x ≤ 20  9 , 0 , 0 20 < x ≤ 25   ,1 x > 25 b) Ta có bảng tính x n x n x2n i i i i i i 5 15 75 375 10 25 250 2500 15 30 450 6750 20 20 400 8000 25 10 250 6250 ∑ 100 1425 23875 Từ bảng, ta có: 1425 2 23875 x = = 1 , 4 25; x = = 238 7 , 5 ; 100 100 2 n 2 100 ˆ2
s = x − (x)2 2 = 238 7 , 5 − (1 , 4 25)2 = 3 , 5 6875 ; s = ˆs = .35 6 , 875 = 36 0 , 480 . n − 1 99 Ví dụ 2.2.
Theo dõi số lượng bài tập ở một nhóm gồm 100 sinh viên giải trong một tuần, ta thu
được bảng số liệu sau: Tổng số 41 44 45 46 48 52 54 bài tập Số sinh viên 10 20 30 15 10 10 5 hoàn thành
a) Tính trung bình mẫu, phương sai mẫu chưa hiệu chỉnh, phương sai mẫu hiệu chỉnh.
b) Những sinh viên thực hiện xong một phép toán toán lớn hơn 48 bài là những sinh
sinh viên có tốc độ làm bài tập nhanh. Tính tỉ lệ sinh viên có tốc độ làm bài tập nhanh?
c) Tính trung bình mẫu, phương sai mẫu hiệu chỉnh của những học sinh viên có tốc độ làm bài tập nhanh. Giải. a) Ta lập bảng như sau: x n x n x 2n i i i i i i 78
Chương IV. Lí thuyết mẫu. 41 10 141 16810 44 20 880 38720 45 30 1350 60750 46 15 690 31740 48 10 480 23040 52 10 520 27040 54 5 270 14580 ∑ 100 4600 212680
Dựa vào bảng trên, ta có 4600
Số bài tập trung bình: x = = 46 bài. 100 2 2 2 212680
Phương sai mẫu chưa hiệu chỉnh: ˆs = x − (x) = − (46)2 = 10 8 , . 100 2 n 2 100
Phương sai mẫu hiệu chỉnh là: s = ˆs = .10 8 , = 10 9 , 09 . n −1 99 10 + 10 + 5
b) Tỉ lệ mẫu là f = = , 0 25 . 100 c) Ta có bảng x n x n x 2n i i i i i i 48 10 480 23040 52 10 520 27040 54 5 270 14580 ∑ 25 1270 64660 Khi đó, ta có: 1270 2 1 2 x = = 50 8 , , s = (64660 − 25 (.50 8,) )= 6. 25 n − 1
2.2. Số mốt (Mode) của mẫu.
Số mốt của mẫu ngẫu nhiên X , kí hiệu là Mod(X ), được xác định như sau: Nếu
mẫu được cho dưới bảng phân phối tần số thì Mode là giá trị có tần số lớn nhất.
Đối với trường hợp mẫu được cho dưới dạng bảng phân phối tần số phân lớp,
người ta định nghĩa khoảng Mode là khoảng có diện tích của hình chữ nhật dựng
trên khoảng đó là lớn nhất.
Mode là một chỉ tiêu thường được chú ý trong các bài toán về kinh tế. Chẳng hạn
một nhà máy sản xuất ô tô Civic muốn có một số lượng ô tô đủ đáp ứng nhu cầu của
người cần mua thì phải chú ý đến ô tô loại gì mà khách hàng thường hay hỏi mua nhất.
2.3. Số trung vị (Median) của mẫu.
Trung vị của một mẫu số liệu, kí hiệu bởi Med(X ), là một số có tính chất sau: Số
các giá trị của mẫu bé hơn hoặc bằng Med(X ) thì bằng số các giá trị của mẫu lớn
hơn hoặc bằng Med(X ). 79 Bài giảng
Xét trường hợp các giá trị của mẫu là phân biệt. Giả sử các giá trị của mẫu được sắp
xếp theo thứ tự tăng dần x < x < L < x . 1 2 n
Khi đó, dễ thấy nếu n lẻ thì lấy Med(X ) = x . n 1 + 2   1
Nếu n chẵn thì ta lấy Med(X )   =  x + x . n n  2  +1 2 2 
Trong trường hợp giá trị x có tần số n , gọi k là chỉ số bé nhất sao cho i i n L
. Lúc đó, ta định nghĩa Med(X ) . 1 n + n2 + + n ≥ = n k 2 k Ví dụ 2.3. Cho mẫu Tổng số 41 44 45 46 48 52 54 bài tập Số sinh viên 10 20 30 15 10 10 5 hoàn thành
Ta có n = 100. n 10 , n 20 , n 30 , n 15 , n 10 , n 10 , n 5. 1 = 2 = 3 = 4 = 5 = 6 = 7 = 100
Dễ thấy: số sinh viên hoàn thành ít hơn hoặc bằng 44 bài là n + n = 30 1 2 < 2 100
và số sinh viên hoàn thành ít hơn hoặc bằng 45 bài tập là n + n + n = 60 . 1 2 3 > 2
Vậy Med(X ) = 45 .
Trong trường hợp mẫu được cho dưới dạng bảng phân phối tần số phân lớp, ta định
nghĩa khái niệm trung vị như sau:
Giả sử, ta có m khoảng với các điểm chia là a < a < L < a , với C = a ; , 1 [ 0 1 a ) 0 1 m C = a ;
, K, C = a ;a , trong đó khoảng C có tần số là r , m [ m 1− m ) 2 [ 1 a2 ) i i
n + n + L + n = n . 1 2 m
Khoảng C được gọi là khoảng trung vị nếu k là số bé nhất sao cho k n L . 1 n + n2 + + n k 2
Số trung vị Med(X ) là số mà tại đó, đường thẳng x = m chia đôi diện tích của biểu
đồ tần số. Rõ ràng, số trung vị luôn luôn nằm trong khoảng trung vị. Ví dụ 2.4.
Tìm khoảng trung vị và số trung vị cho Ví dụ 1.3. Giải 370 Ta có 6 + 44 + 76 = 126 <
< 6 + 44 + 76 + 100 = 226 nên khoảng trung vị là 2 ( , 3 ; 0 , 3 2). 80
Chương IV. Lí thuyết mẫu. 100
Để tính số trung vị, ta nhận thấy: chiều cao của khoảng trung vị này là = 500 . , 0 2
Khi đó, ta chia hình chữ nhật dựng trên khoảng này là thành 2 phần có diện tích
phần bên trái là 185 −126 = 59 . 59 59
Ta có 500.AM = 59 hay AM =
. Vậy số trung vị Med(x) = 0 , 3 + . 500 500 C A B M
Hoặc ta có thể tính như sau:
Hình chữ nhật có diện tích phần bên phải là 226 −185 = 41. 41
Ta có 500.BM = 41 hay BM = . 500 41
Vậy số trung vị là Med(x) = , 3 2 − 500
Dựa vào bài toán này, ta có nhận xét sau. Nhận xét. kn
 ∑ n −  a a i ( k k 1 ) Ta ch   ứng minh được 1 = 2 − Med (X ) i = a
số trung vị, trong đó k nk C = a
; a khoảng trung vị. k [ k 1− k )
2.4. Các quy luật phân phối mẫu.
Người ta chứng minh được các kết quả sau: Định lí.
Cho X tuân theo phân phối chuẩn N( 2 µ;σ ) thì a) Nếu biết 2 σ thì  2 σ  + X ~ Nµ;  .  n
+ X − µ . n ~ N( ) 1 ; 0 . σ b) Nếu chưa biết 2 σ thì + X − µ n ≤ 30 , ta có
. n ~ T (n − ) 1 . S 81 Bài giảng + X − µ n > 30 , ta có . n ~ N ( ) 1 ; 0 . S n ∑( 2 X − µ i )
c) Nếu biết µ thì i 1= 2 ~ χ (n). 2 σ
n(X X i )2
d) Nếu chưa biết µ thì i 1= ~ 2 χ (n − ) 1 . 2 σ Chú ý.
Nếu không biết phân phối của biến ngẫu nhiên X nhưng với kích thước mẫu n > 30 ,
khi đó ta xem biến ngẫu nhiên X có phân phối chuẩn N( 2 µ;σ ). 82 3. Bài tập chương. 1.
Trọng lượng của 100 trẻ em của một trường mẫu giáo ghi nhận được như sau: Trọng lượng (kg) 41 44 45 46 48 52 54 Số trẻ em 10 20 30 15 10 10 5
a) Hãy xác định x là trọng lượng trung bình của các sinh viên và độ lệch mẫu hiệu chỉnh s .
b) Hãy xác định bảng phân phối xác suất. 2.
Kết quả thi học kì môn toán A1 của một khối sinh viên khóa 14 gồm 72 bạn như sau: Điểm 3 5 6 8 9 Số sinh viên 40 15 5 10 2
a) Hãy lập bảng phân phối tần số của các kết quả này.
b) Lập bảng phân phối tần suất và vẽ đa giác tần suất. 3.
Thống kê cân nặng của 400 trẻ sơ sinh, ta được bảng phân phối tần số phân lớp sau , 3 2 − , 3 X , 2 4 − 6 , 2 6 , 2 − 8 , 2 8 , 2 − 0 , 3 0 , 3 − , 3 2 , 3 2 − , 3 4 4 6 , 3 − 8 , 3 Tần số 6 44 76 100 95 30 49
a) Lập bảng phân phối tần số không phân lớp.
b) Lập bảng phân phối tần suất, vẽ đa giác tần suất và biểu đồ tần số.
c) Lập hàm phân phối mẫu và vẽ đa giác tần suất tích lũy.
d) Tìm Mod(X ) , xác định khoảng trung vị. 4.
Giá của một loại cổ phiếu bán trên thị trường chứng khoáng trong 100 phiên giao dịch được cho ở bảng sau Giá cổ phiếu 13 −15 15 −17 17 −19 19 − 21 21 − 23 (1000đ) Số phiên giao 5 18 42 27 8 dịch
Hãy tính các giá trị x , phương sai mẫu 2 ˆs , 2 s . 5.
Tuổi của 70 nhân viên trong một cơ quan được ghi lại như sau: Khoảng Tần số Dưới 20 7 20-30 13 30-40 26 40-50 15 50-60 6 Trên 60 3
Tìm khoảng trung vị, số trung vị, khoảng Mod. 6.
Để nghiên cứu nhu cầu tiêu thụ sữa hộp trong một khu vực, người ta tiến hành khảo sát
800 gia đình. Kết quả được cho ở bảng dưới đây: Nhu cầu (hộp/tháng) Số gia đình Nhu cầu (hộp/tháng) Số gia đình 30-34 35 55-59 142 35-39 48 60-64 94 40-44 83 65-70 50 45-49 159 70-74 10 50-54 189
b) Tìm số hộp sữa tiêu thụ trung bình.
Chương IV. Lí thuyết ước lượng
c) Tìm khoảng trung vị, số trung vị. 7.
Khi kiểm tra thể lực một nhóm sinh viên, ta có kết quả về cân nặng như sau: x 42 5 , − 47 5 , 47 5 , − 52 5 , 52 5 , − 57 5 , 57 5 , − 62 5 , 62 5 , − 67 5 , i (kg) Số 8 14 28 18 12 sinh viên n i
a) Tính x là cân nặng trung bình cho các sinh viên và tính độ lệch mẫu.
b) Lập bảng phân phối xác suất mẫu. 8.
Điều tra năng suất lúa trên diện tích 100 ha trồng lúa của một vùng, ta thu được bảng số liệu sau. Năng suất (tạ/ha) 41 44 45 46 48 52 54 Số ha có năng suất 10 20 30 15 10 10 5 tương ứng
a) Tính trung bình mẫu, phương sai mẫu, phương sai mẫu hiệu chỉnh.
b) Những thửa ruộng có năng suất từ 48 tạ trở lên gọi là những thửa ruộng có năng suất cao.
Tính tỉ lệ thửa ruộng có năng suất cao.
c) Tính trung bình mẫu, phương sai mẫu hiệu chỉnh của những thửa ruộng có năng suất cao.
C. Phương pháp giảng dạy.
- Phối hợp phương pháp thuyết trình và vấn đáp giải quyết vấn đề.
- Đưa ví dụ cụ thể để nêu rõ ý nghĩa của việc chọn mẫu.
- Từ trực quan sinh động đến tư duy trừu tượng. Áp dụng một số ví dụ thực tế.
- Yêu cầu SV đọc bài giảng trước khi lên lớp.
- Kiểm tra, đánh giá việc làm bài tập của SV.
- Sử dụng phương tiện dạy học hiện đại như Mic.
D. Tài liệu tham khảo
[1] Đậu Thế Cấp, Xác suất thống kê: Lí thuyết và các bài tập, NXB Giáo dục, 2006.
[2] Đặng Hùng Thắng, Thống kê và ứng dụng, NXB Giáo dục, 2008.
[3] PGS. TS. Phạm Xuân Kiều, Giáo Trình xác suất và thống kê, NXB Giáo dục, 2005.
[4] Trần Văn Minh, Phí Thị Vân Anh, Xác suất thống kê với các tính toán trên
Excel
, NXB Giao Thông Vận tải, 2008.
[5] Đặng Công Hanh, Đặng Ngọc Dục, Giáo trình Lý thuyết xác suất và Thống kê
toán
, trường Đại học Duy Tân,1996
[6] Trần Văn Minh, Phí Thị Vân Anh, Hướng dẫn giải bài tập Xác suất thống kê với
các tính toán trên Excel, NXB Giao Thông Vận tải, 2008. 84 Chương V.
Lí thuyết ước lượng
A. Mục tiêu.
- Giới thiệu các khái niệm ước lượng: ước lượng điểm, ước lượng khoảng.
- Giới thiệu các loại ước lượng điểm: ước lượng không chệch, ước lượng hiệu quả, ước lượng vững.
- Giới thiệu các phương pháp ước lượng khoảng cho: kì vọng, phương sai, tỉ lệ. B. Nội dung.
Xét một tổng thể Ω và giả sử ta quan tâm đến biến lượng X đo lường một dấu hiệu
nào đó của tổng thể Ω . Khi đó X được coi là đại lượng ngẫu nhiên. Phân phối xác
suất của X thường rất khó nắm bắt, và thông thường ta giới hạn ở việc xác định một
số các tham số đặc trưng của X như các giá trị: E(X ), D(X ), Med(X ), Mod(X ), …
Các tham số này không thể xác định chính xác được (nếu không biết phân phối của
X ), mà phải ước lượng từ các giá trị của X trên một mẫu chọn ngẫu nhiên. Như vậy,
bài toán ước lượng tham số được phát biểu như sau:
Giả sử X là một đại lượng ngẫu nhiên có tham số đặc trưng θ nào đó (chưa biết) mà
ta quan tâm. Vấn đề đặt ra là: Căn cứ trên n giá trị 1 x , x , K , 2
x của X đo được trên n
một mẫu kích thước n được lấy từ tổng thể Ω , cần tìm một giá trị gần đúng θˆ của θ .
1. Ước lượng điểm. Định nghĩa. Một hàm ˆ θ T x ; x ;K =
; x của n giá trị
x , K , x được gọi là một ước lượng n ( 1 2 n ) 1 x , 2 n
điểm cho θ .
Như vậy, một ước lượng θˆ = T là một hàm của n đại lượng ngẫu nhiên X , X , K, n 1 2
X nên nó cũng là đại lượng ngẫu nhiên. Có nhiều hàm ước lượng ˆ θ T x ; x ;K = ; x n ( 1 2 n ) n
của tham số θ khác nhau. Tuy nhiên, việc lựa chọn một ước lượng nào là “tốt” được
dựa vào các tiêu chuẩn dưới đây. Định nghĩa. Hàm ước lượng ˆ θ T x ; x ;K =
; x được gọi là ước lượng không chệch nếu (θˆ E )= θ . n ( 1 2 n ) Bài giảng
Từ định nghĩa, ta thấy nếu ˆ θ T x ; x ;K =
; x là hàm ước lượng không chệch của θ n ( 1 2 n )
thì E(ˆθ −θ )= 0 .
Tính chất không chệch có nghĩa là ước lượng θˆ không có sai số hệ thống. Định nghĩa. Hàm ước lượng ˆ θ T x ; x ;K =
; x được gọi là ước lượng vững nếu với mọi ε > 0 thì n ( 1 2 n ) E( ˆ lim θ − θ < ε )= 1 hay E( ˆ lim
θ − ε < θ < θ + ε ) = 1. n→+∞ n→+∞
Tính chất vững đảm bảo cho ước lượng θˆ gần θ tùy ý với xác suất cao (gần 1) khi
kích thước mẫu n đủ lớn. Định nghĩa.
Ước lượng θˆ được gọi là một ước lượng hiệu quả của θ nếu nó là một ước lượng (θˆ D )
không chệch và có phương sai nhỏ nhất.
2. Ước lượng khoảng.
Ước lượng điểm dù tốt nhất cũng chỉ cho ta một giá trị trong tập vô hạn nên ta không
biết được độ chính xác cũng như xác suất để điểm ước lượng có độ chính xác, do đó
không đánh giá được sai lầm khi dùng θˆ thay cho θ .
Để khắc phục hạn chế đó, người ta đưa ra ước lượng khoảng tin cậy cho tham số θ ,
nghĩa là dựa vào một ước lượng θˆ , tìm một khoảng (θ ; với 1 θ 2 ) θ , 1 θ là hai 2 ước
lượng điểm của tham số θ sao cho P(θ < θ < θ . 1 2 ) = γ = 1 − α
Trong đó γ = 1−α là xác suất cho trước gọi là độ tin cậy của khoảng ước lượng, do đó
α là khả năng mắc sai lầm của khoảng ước lượng còn (θ ; gọi là khoảng tin cậy 1 θ 2 ) và θ được gọi là 2 − θ1
độ dài của khoảng tin cậy.
Ý nghĩa của khoảng tin cậy là ở chỗ có thể nói trong 100% trường hợp lấy mẫu khoảng (θ ; chứa tham số có thể tin cậy ở 1 θ 2 )
θ chưa biết thì khẳng định θ1 < θ < θ2 mức γ .
Để ước lượng một tham số θ , ta thực hiện quy tắc sau:
+ Chọn một hàm G G(X ; X ;K =
; X ;θ sao cho phân phối của 1 2
G xác định hoàn toàn n )
(không chứa tham số θ nữa).
+ Khi đó với độ tin cậy γ = 1− α cho trước, ta tìm cặp giá trị α 0 , α 0 sao cho 1 > 2 >
α + α = α và tương ứng với chúng là 1 2
các phân vị thỏa mãn điều kiện
P(G > g ) = α và P(G > g
= 1 − α ( P(G < g = α ) 1 α − ) 1 α − ) α 1 1 2 2 2 2 Khi đó, P(g
< G X ; X ;K; X ; < g = 1 − − = 1 − . 1 α − ( θ 1 2 n ) α ) α α α 1 2 2 1
+ Dùng các phép biến đổi tương đương, ta đưa bất đẳng thức trên về dạng P(θ < θ < θ . 1 2 ) = 1 − α 86
Chương V. Lí thuyết ước lượng Nhận xét. α 1 Trong Ch − γ
ương này, chúng ta chỉ xét trường hợp α = α . 1 2 = = 2 2
2.1. Ước lượng khoảng tin cậy cho kì vọng
Giả sử đại lượng ngẫu nhiên X có phân phối chuẩn X ~ N ( 2
µ;σ ) với tham số θ = µ
chưa biết. Từ một mẫu cụ thể (x ; x ;K; x của 1 2
X . Bài toán đặt ra là tìm khoảng tin n )
cậy cho E(X ) = µ .
2.1.1. Đã biết phương sai 2 2 σ = σ . 0 Ta ch X − µ
ọn hàm: G = Z =
n . Do X ~ N ( 2
µ;σ ) nên Z ~ N ( ) 1 ; 0 . σ 0   α     α P Z > z =    và P Z > z = 1 − . Dễ thấy z = −z . α  2  α  α α  1− 2 1− 2   2  2 2 Khi σ σ
đó − z < Z < z hay 0 0 X z < µ < X + z . α α α α 2 2 n 2 n 2       Lại có       P z < Z < z
= 1 − P Z < −zP Z > z = 1 − α  . α α   α   α   2 2   2   2    Suy ra   P z < Z < z = 1 − α  . α α   2 2    V σ σ ậy  P X − 0 z < µ 0  < X + z = 1 − α  . α α   n 2 n 2    Kho σ σ
ảng ước lượng tin cậy cho kì vọng là  0 0   X z ; X + z
, trong đó z được α α  α  n 2 n 2  2   2 α 1 m t xác γ định từ công thức   Φ z = 1 − − = 2  với Φ(m) ∫ − = e dt . α  2 2 2  2  0 σ Đại lượng 0 ε =
z được gọi là độ chính xác của ước lượng, nó phản ánh độ lệch α n 2
trung bình của trung bình mẫu so với kì vọng lí thuyết với độ tin cậy γ = 1− α . 2 Khi σ
đó, độ dài khoảng tin cậy là 0 2ε = z . α n 2 Ví dụ 2.1.
Giả sử X là trọng lượng của em nam ở lứa tuổi lên 10 thuộc 4 quận ở thành phố Hà
Nội. Ta muốn biết E(X ), trọng lượng trung bình của các em nam ở lứa tuổi lên 10
thuộc 4 quận ở thành phố Hà Nội. Vì điều kiện về tài chính, thời gian nên ta không thể
lấy trọng lượng của mọi em trong khu vực này, ta chọn ngẫu nhiên 50 em. Qua trọng 87 Bài giảng
lượng của 50 em này, ta xác định được X = 32 kg. Giả sử phân phối của X có độ lệch tiêu chuẩn σ = 5 ,
2 kg (đã biết). Vậy, với độ tin cậy là γ = 95% , ta có thể nói trọng
lượng trung bình E(X ) nằm trong khoảng nào? Xét trường hợp trên với độ tin cậy γ = 95% . Giải Ta có σ = 5 , 2 đã biết.
Với độ tin cậy γ = 95% , ta có α = 5% . Khi đó z = 9 ,
1 6 . Khi đó, khoảng ước lượng tin α 2    5 , 2 5 , 2  c σ σ
ậy cho kì vọng E(X ) là  0 0   x z ; x + z hay , 3 2 − . 9 , 1 6 , 3 ; 2 + 9 , 1 . 6 . α α     n  50 50  2 n 2  Ví dụ 2.2.
Một thầy giáo muốn ước lượng điểm môn Toán của toàn bộ sinh viên Đại học A. Giả
sử điểm môn Toán của sinh viên tuân theo phân phối chuẩn với σ 3 , 0 điểm. Thầy 0 =
giáo kiểm tra lớp có 100 sinh viên và biết được điểm môn Toán của 100 bạn này là như sau: Điểm 3 5 6 8 Số sinh viên 20 30 40 10
a) Ước lượng điểm trung bình môn Toán của toàn sinh viên Đại học A với độ tin cậy là 95% .
b) Với độ chính xác của ước lượng là ,
0 25 . Hãy xác định độ tin cậy. Giải. 530
a) Dựa vào bảng, ta có x = = 3 , 5 , σ 3 ,
0 , n = 100 , γ = 95% suy ra α = 0 , 0 5 và 100 0 = z = 9 , 1 6 . α 2   Khi σ σ
đó, khoảng ước lượng điểm trung bình là  0 0   x z ; x + z α α   − 1 − 1 n 2 n 2  Hay (52 9 , 41 5 ; , 3 059). b) Ta có σ 0 n ε = z = , 0 25. Suy ra z = , 0 25 = 8 , 0 33 . α α n σ 2 2 0   Khi α 1 1 α đó, từ công thức   Φ z = 1 − − = −  ta có γ = 2Φ( 8 , 0 3 ) 3 ≈ 5 , 0 94 . α  2 2 2 2  2 
Vậy độ tin cậy là γ = 5 , 9 4% .
2.1.2. Chưa biết phương sai 2 σ .
Tương tự như trên, ta thay 2 σ bằng ph 0
ương sai mẫu hiệu chỉnh 2 s 88
Chương V. Lí thuyết ước lượng Ta ch X − µ
ọn hàm G = T = n . S
Ta xét 2 trường hợp sau: TH1. n ≤ 30 . Khi X − µ
n ≤ 30 thì hàm G = T =
n có phân phối Student với n −1 bậc tự do. Do T S có hàm m S S
ật độ đối xứng nên tương tự câu a), ta được X t < µ < X + t . α α n− ; 1 n− ; 1 n 2 n 2   Kho S S
ảng ước lượng tin cậy cho kì vọng là    X t ; X + t , trong đó t α α  α  n− ; 1 n− ; 1 n n− ; 1 2 n 2  2  
được xác định từ công thức   P T > t = α  . α   n; 2  Chú ý: t = −t . α α n− 1 ; 1 − n− ; 1 2 2 TH 2. n > 30 . Khi X − µ
n > 30 thì hàm G = T =
n có phân phối tiệm cận chuẩn N ( ) 1 ; 0 , và việc tìm S
khoảng ước lượng với độ tin cậy γ = 1− α được làm như câu a) với 2 σ bằng phương 0 sai mẫu hiệu chỉnh 2 s .   V S S
ậy khoảng ước lượng tin cậy cho kì vọng là    X z ; X + z , trong đó z α α  α  n 2 n 2  2   2 α 1 γ m t
được xác định từ công thức   Φ z = 1 − − = 2  với Φ(m) ∫ − = e dt . α  2 2 2  2  0 Ví dụ 2.3.
Cân ngẫu nhiên 25 bao xi măng, ta thu được bảng kết quả sau: x (kg) 47 48 49 50 51 52 53 i n (số bao) 2 3 4 6 5 3 2 i
Tìm khoảng tin cậy 95% cho trọng lượng trung bình µ , biết trọng lượng X của bao xi
măng tuân theo phân phối chuẩn N( 2
µ;σ ) với σ chưa biết. Giải
Đây là bài toán ước lượng khoảng tin cậy cho giá trị trung bình với phương sai chưa biết. Do ch X − µ
ưa biết σ và n = 25 < 30 nên ta có G = T =
n ~ T (n − ) 1 . S
Ta có trung bình x = 5 ,
0 04 phương sai mẫu hiệu chỉnh s = 130 , 7 042 = 36 1 , 53. 89 Bài giảng Ta có t = t = ,
2 064 . Khi đó, khoảng tin cậy cho kì vọng là α 0,05 n− ; 1 24; 2 2   s s  36 1 , 53 36 1 , 53    x t ; x + t = 50 0 , 4 − . , 2 064 5 ; 0 0 , 4 + . , 2 064  α α   n− ; 1 n− ; 1 n 2 n 2   5 5  Ví dụ 2.4.
Một nhà máy tiến hành một nghiên cứu xem trung bình một công nhân hoàn thành bao
nhiêu sản phẩm trong một tháng. Một mẫu ngẫu nhiên gồm 59 công nhân được chọn và kết quả như sau
14, 18, 22, 30, 36, 28, 42, 79, 36, 52, 15, 47, 95, 16, 27, 111, 37, 63, 127, 23, 31, 70,
27, 111, 30, 147, 72, 37, 25, 7, 33, 29, 35, 41, 48, 15, 29, 73, 26, 15, 26, 31, 57, 40, 18,
85, 28, 32, 22, 37, 60, 41, 35, 26, 20, 58, 33, 23, 35.
Hãy xây dựng khoảng ước lượng tin cậy trung bình µ số sản phẩm mà một công nhân
hoàn thành trong một tháng với độ tin cậy γ = 95% . Giải.
Từ bảng số liệu trên, ta có n = 59 , x = 41 0 , 5 , s = 27 9 , 9 , z = 9 , 1 6 . 0,05 2
Do n = 59 > 30 nên ta có khoảng tin cậy cho µ là    27 9 , 9 27 9 , 9   s s   X z ; X + z = 41 0 , 5 − . 9 , 1 6 4 ; 1 0 , 5 + 9 , 1 . 6 . α α     n  59 59  2 n 2 
2.2. Ước lượng khoảng tin cậy cho phương sai.
Giả sử biến ngẫu nhiên X có phân phối chuẩn X ~ N ( 2 µ;σ ) với tham số θ = D(X ) 2
= σ chưa biết. Từ một mẫu cụ thể (x ; x ;K; x của 1 2
X . Bài toán đặt ra là n ) 2
tìm khoảng tin cậy cho phương sai σ .
2.2.1. Đã biết kỳ vọng toán E(X ) = µ = µ . 0 ∑n(X − µ i )2 0 Ta chọn hàm 2 i 1 G = χ = = , Ta có 2 χ có phân phối 2 2 χ ~ χ (n) 2 σ     Khi α α
đó, xét các phân vị sau đây 2 2  P χ > χ  = 2 2    và P χ > χ = 1 − . α   α  n; 2  n 1 ; − 2 2   2  ∑n n (X − µ X i )2 ∑ 0 ( − µ i )2 0 Ta có 2 2 2 χ < χ < χ hay i 1= 2 i 1 < σ = < . α α 2 2 n 1 ; − n; χ χ 2 2 α α n; n 1 ; − 2 2   Lại có  P χ 2
< χ 2 < χ 2  = 1 − α  . α α   n − 1 ; n; 2 2  90
Chương V. Lí thuyết ước lượng n n 2 2   ∑(X − µ X i 0 ) ∑( − µ i 0 )  Vậy  i=1 2 i=1  P < σ < = 1 − α .  χ 2 χ 2  α α  n; n − 1 ;   2 2   n n 2 2   ∑(X − µ X i 0 ) ∑( − µ i 0 ) 
Khoảng ước lượng tin cậy cho phương sai là  i=1 i=1  .  ; 2 2  χ χ  α α n; n − 1 ;   2 2 
2.2.2. Chưa biết kỳ vọng toán E(X ) = µ .
n(X X i )2 Ta chọn hàm 2 i 1 G = χ = = . 2 σ Ta có 2 χ có phân phối 2 χ ~ 2 χ (n − )
1 . Thực hiện tương tự a), ta thu được  n 2 n 2  ∑(X X X X i ) ∑( − i )  Vậy  i=1 2 i=1  P < σ < = 1 − α .  χ 2 χ 2  α α  n− ; 1 n− − 1 ; 1   2 2   n 2 n 2  ∑(X X X X i ) ∑( − i ) 
Khoảng ước lượng tin cậy cho phương sai là  i=1 i=1  .  ; 2 2  χ χ  α α n− ; 1 n− − 1 ; 1   2 2  Ví dụ 2.5.
Mức hao phí nguyên liệu cho một đơn vị sản phầm là đại lượng ngẫu nhiên X có phân phối chuẩn N( 2
µ;σ ). Quan sát 28 sản phẩm, ta thu được kết quả sau
Lượng nguyên liệu hao phí 19 19 5 , 20 20 5 , (kg) Số sản phẩm 5 6 14 3
Với độ tin cậy 90% , hãy ước lượng phương sai của X trong 2 trường hợp sau a) Biết µ = 20 kg. b) Chưa biết µ . Giải. 4
a) Đã biết µ = 20 kg. Ta có ∑ n X µ , 2 χ = 41 3 , 37 , 2 χ = 16 9 , 28 . i ( − i )2 = 7,25 28;0,05 28;0 ,95 i 1 =
Khi đó, với độ tin cậy γ = 90% , ta có khoảng ước lượng tin cậy cho phương sai là 91 Bài giảng n n 2 2   ∑(x − µ x i 0 ) ∑( − µ i 0 )    7,25 7,25  i=1 i=1  =  ;  .  ; 2 2  χ χ  43 3 , 37 16 9 , 28   α α n; n − 1 ;   2 2  b) Ta có 4 ∑n x x , 2 χ = 40 1 , 13, 2 χ = 16 1
, 51. Khi đó, khoảng ước lượng tin i ( − i )2 = 7,25 28− ; 1 0,05 28 1 − ; 0,95 i 1 =  n 2 n 2  ∑ (x x x x i ) ∑( − i )   7,25 7,25 
cậy cho phương sai là  i=1 i=1  =  ;  .  ; 2 2  χ χ  40 1 , 13 16 1 , 51  α α n− ; 1 n− − 1 ; 1   2 2 
2.3. Ước lượng khoảng tin cậy cho tỉ lệ.
Nếu trong phép lấy mẫu, ta chỉ quan tâm đến sự xuất hiện hoặc không xuất hiện tính
chất A nào đó. Gọi p là tỉ lệ phần tử có tính chất A trong toàn bộ tổng thể, p chưa
biết. Giả sử trong một mẫu kích thước n k phần tử có tính chất A . Cơ sở toán học
cho việc xây dựng khoảng tin cậy cho tỉ lệ p (chưa biết) là định lí sau đây: Định lí. T k ần suất mẫu f =
là một đại lượng ngẫu nhiên có phân phối xấp xỉ chuẩn với kì n 1 np > 5 v p p
ọng E( f ) = p và phương sai D( f ) ( ) = với điều kiện  . n
n(1 − p) > 5
Do ta không biết p nên ta không biết được D( f ). Tuy nhiên với một số điều kiện, ta có th f 1 − f
ể xấp xỉ p bởi f . Nghĩa là ta coi D( f ) ( ) ≈
. Để có thể dùng xấp xỉ này, ta nnf > 10 cần điều kiện sau  .
n(1 − f ) > 10
( f p) ( f p) Khi n
đó, đại lượng ngẫu nhiên =
sẽ có phân phối xấp xỉ chuẩn tắc Df f (1 − f ) N ( ) 1 ; 0 .
Khi đó, để ước lượng khoảng tin cậy cho tỉ lệ với độ tin cậy γ = 1− α Ta ch f p
ọn hàm: G = Z =
n . Khi đó Z ~ N ( ) 1 ; 0 . f (1 − f ) f (1 − f ) f (1 − f )
Khi đó, − z < Z < z , hay f z < p < f + z . α α α α n n 2 2 2 2   Từ đẳng thức   P z < Z < z = 1 − α 
, sau khi biến đổi ta được α α   2 2  92
Chương V. Lí thuyết ước lượng f (1 − f ) f (1 − f )    P f z < p < f + z = 1 − α  . α α   n n 2 2 
Khi đó, khoảng ước lượng tin cậy cho tỉ lệ p là  f (1 − f ) f (1 − f )     f z ; f + z
, trong đó z được xác định từ công thức α α  α  n n 2 2  2   2 1 α γ m t   Φ z = − = 2  với Φ(m) ∫ − = e dt . α  2 2 2  2  0 Ví dụ 2.6.
Kiểm tra ngẫu nhiên 500 xe máy của nhà máy A chuyên sản xuất xe máy thì có 360
xe máy đạt chất lượng tốt. Hãy ước lượng tỉ lệ tối thiểu xe máy đạt chất lượng tốt của
nhà máy A với độ tin cậy 95% . Giải.
Đây là bài toán ước lượng tỉ lệ. 360 f = = 7 , 0 Ta có 500 , nf = 500 7 , 0 .
= 350 > 10 , n(1− f ) = 150 > 10. f p 7 , 0 Khi − p đó G = Z = n = 500 , γ = 95% = 9 , 0 5 , f (1 − f ) 7 , 0 . 3 , 0   1   α γ Φ z = − = = , 0 475  nên z = 9 , 1 6 . α  2 2 2 α  2  2
Vậy khoảng ước lượng tin cậy cho tỉ lệ là  f (1 − f ) f (1 − f )       , 0 7 3 , 0 . 7 , 0 . 3 , 0  f z ; f + z =  . α α   7 , 0 − . 9 , 1 6 7 , 0 ; + 9 , 1 . 6  500 500 2 2   500 500 
Tỉ lệ tối thiểu xe máy đạt chất lượng là p = , 0 66 . Ví dụ 2.7.
Tại một rừng nguyên sinh, người ta đeo vòng cho 1000 con chim. Sau một thời gian,
bắt lại 200 con thì có 40 con có đeo vòng. Thử ước lượng số chim trong vùng với độ tin cậy 99% . Giải.
Đây là bài toán ước lượng tỉ lệ. Ta có: 40 f = = , 0 2. Ta có nf = 200 , 0
. 2 = 40 > 10 và n(1− f ) = 400 8 , 0 . = 32 > 10 . 200   Khi γ 9 , 0 0 đó, γ = 90% = 9 , 0 0 ,   Φ z = = = , 0 45  nên z = 6 , 1 45. α  2 2 α  2  2
Khi đó, khoảng ước lượng tin cậy cho tỉ lệ là 93 Bài giảng f (1 − f ) f (1 − f )       9 , 0 1 , 0 . 9 , 0 . 1 , 0  f z ; f + z =  α α   , 0 2 − , 1 . 64 , 0 ; 2 + , 1 . 64  500 500 2 2   200 200  Hay ( 1 , 0 7 , 0 ; 24).  2000 2000 
Khi đó, số chim trong vùng với độ tin cậy là  ;  .  , 0 24 1 , 0 7 
2.4. Ước lượng kích thước mẫu.
Với độ tin cậy γ đã cho, ta thấy có mối quan hệ giữa kích thước mẫu n và độ dài
khoảng tin cậy. Kích thước mẫu càng lớn thì khoảng tin cậy càng hẹp, nghĩa là độ
chính xác của ước lượng càng cao, sai số càng nhỏ. Tuy nhiên, kích thước mẫu càng
lớn thì đòi hỏi nhà nghiên cứu càng nhiều thời gian, tiền của và công sức.
Vậy bài toán đặt ra là: Cần chọn kích thước mẫu tối thiểu là bao nhiêu để đạt được
độ chính xác mong muốn
.
2.4.1. Trường hợp ước lượng cho trung bình µ .
Giả sử muốn có ước lượng µ với sai số không quá ε cho trước với độ tin cậy γ .
Khi đó, với xác suất γ , ta xét 2 trường hợp sau TH1: N σ ếu biết phương sai 2
σ thì X − µ ≤ z
. Khi đó, ta cần có bất đẳng thức α n 2 2  σz  α σ   z ≤ ε hay 2 n ≥ 
 , trong đó z được xác định từ công thức α α n ε 2     2   2 1 α γ z t   Φ z = − = 2  với Φ(z) ∫ − = e dt . α  2 2 2  2  0
Vậy n là số nguyên dương nhỏ nhất thỏa mãn bất đẳng thức này. 2  σz   α 
TH2: Nếu không biết phương sai 2
σ thì ta thay σ trong công thức 2 n ≥   bởi s .  ε   
Do s phụ thuộc n nên ta thường lấy mẫu có kích thước m > 30 để tính x s . Vậy n thỏa mãn công thức sau 2  sz   α  2 n ≥ 
 với điều kiện vế phải của công thức này lớn hơn hoặc bằng 30 .  ε    Ví dụ 2.8.
Biết rằng độ lệch tiêu chuẩn chiều cao người lớn là 3 inch, ta muốn xây dựng một
khoảng tin cậy với γ = 90% cho chiều cao trung bình µ với sai số không quá 5 , 0 inch.
Hãy xác định kích thước mẫu tối thiểu để đạt được yêu cầu trên. 94
Chương V. Lí thuyết ước lượng Giải
Ta biết phương sai σ = 3, ε = 5 ,
0 và độ tin cậy γ = 90% . 2  σz   α   3. 6 , 1 4 2  Ta có 2 n ≥   =   = 96 8 , 26 . Vậy n = 97 .  ε   5 , 0   
2.4.2. Trường hợp ước lượng cho tỉ lệ p .
Giả sử muốn có ước lượng p với sai số không quá ε cho trước với độ tin cậy γ . Ta 2 z ff α (1 )
n là số nguyên dương nhỏ nhất thỏa mãn 2 n
, trong đó z được xác 2 ε α 2   2 1 α γ z t định từ công thức   Φ z = − = 2  với Φ(z) ∫ − = e dt . α  2 2 2  2  0 nf > 10 với điều kiện  .
n(1 − f ) > 10 Ví dụ 2.9.
Phòng cảnh sát giao thông muốn ước lượng tỉ lệ xe chở quá tải trên đường với độ tin
cậy γ = 95% và sai số không vượt quá ,
0 05 thì cần phải kiểm tra bao nhiêu lượt xe
chạy trên đường? Biết rằng trong 100 xe đã kiểm tra thì có 40 xe quá tải. Giải
Đây là bài toán ước lượng kích thức mẫu cho tỉ lệ. Ta có 40 f = = ,
0 4 , γ = 95% , z = z = 9 , 1 6 , ε = , 0 05 . 100 α 0,025 2 2 z f f α (1− ) 9 , 1 62. , 0 4 2 (.1− , 0 4) Khi đó, ta có: n ≥ = = 368 7 , 936 . 2 ε , 0 052
Do n là số tự nhiên nên ta chọn n = 369 .
Vây, số xe cần kiểm tra là 369 . 3. Bài tập chương. 1.
Đo sức bền của một loại kềm công nghiệp, người ta thu được bộ số liệu sau đây x 4500 4800 4900 5000 5125 5200 5375 6500 n 1 3 3 7 5 3 2 1 i
a) Biết rằng sức bền của kềm có phân phối chuẩn với độ lệch σ = 300 . Hãy xây dựng
khoảng tin cậy cho sức bền trung bình của kềm với độ tin cậy γ = 95% .
b) Tìm khoảng tin cậy cho sức bền trung bình của kềm với giả thiết là không biết phương sai 2 σ . 2.
Kiểm tra sức khỏe của sinh viên một trường đại học, ta thu được chiều cao của 500 sinh viên như sau. 95 Bài giảng Cao (cm) Số sinh viên 150-154 20 154-158 60 158-162 120 162-164 140 164-168 80 168-172 60 172-174 20
Biết độ lệch tiêu chuẩn của chiều cao người lớn là c
5 m . Tìm khoảng tin cậy cho chiều cao
trung bình của sinh viên trường đại học với độ tin cậy là 95% . 3.
Gọi X là mức tiêu thụ xăng của một loại ô tô (lít/ 100km ). Người ta kiểm tra 36 chiếc
và thu được kết quả sau. x 4,5 4,8 5,1 5,3 5,6 5,9 6,2 6,4 n 3 5 6 7 6 4 3 2 i
Với độ tin cậy γ = 95% , hãy xác định khoảng ước lượng cho mức hao phí xăng trung bình
cho 100 km của loại ô tô này? 4.
Thống kê tại một trạm đăng kí xe máy trong một tháng, trong 3600 xe mới đăng kí thì
có 240 xem Air Blade. Hãy ước lượng tỉ lệ phần trăm tối đa bán được của loại xe Air
Blade trên thị trường xe máy với độ tin cậy γ = 95% . 5.
Một nông dân muốn ước lượng tỉ lệ nảy mầm cho một giống lúa mới. Khi ông ta điều
tra 1000 hạt và thấy được 640 hạt nảy mầm.
a) Với độ tin cậy 95%, hãy ước lượng tỉ lệ nảy mầm cho giống lúa này.
b) Nếu muốn ước lượng tỉ lệ nảy mầm có sai số không vượt quá 2% và đạt độ tin cậy 95%
thì cần gieo ít nhất bao nhiêu hạt?
c) Với độ tin cậy 97%, hãy ước lượng số hạt giống nảy mầm tối thiểu khi gieo 10000 hạt. 6. Một kho hàng có 10 0
. 00 hộp sữa. Người ta nghi ngờ sữa bị hỏng, bằng cách lấy kiểm
tra 250 hộp thì thấy có 5 hộp bị hư.
a) Với độ tin cậy γ = 95% , hãy ước lượng tỉ lệ p số hộp sữa bị hư trong kho.
b) Ước lượng số hộp sữa bị hư trong kho với độ tin cậy γ = 95% . 7.
Cho X là năng suất lúa ở một khu vực (đơn vị tính tạ/ha). Điều tra ở một số thửa
ruộng, ta có kết quả sau đây: X 30-35 35-40 40-45 45-50 50-55 n (Số hecta) 6 10 28 40 16
a) Hãy ước lượng năng suất lúa trung bình của toàn vùng, với độ tin cậy γ = 95% .
b) Những thửa ruộng đạt năng suất từ 45 tạ/ha trở lên là những thửa ruộng đạt năng suất
cao. Hãy ước lượng tỉ lệ những thửa ruộng đạt năng suất cao của vùng này, với độ tin cậy γ = 95% .
c) Nếu muốn ước lượng năng suất lúa trung bình của toàn vùng đạt được độ chính xác ε = ,
1 4 tạ/ha thì độ tin cậy là bao nhiêu? 8.
Theo dõi chiều cao của 144 cây Bạch Đàn trồng trên đất phèn sau 1 năm, ta được kết quả sau đây. X (cm) 250-300 300-350 350-400 400-450 450-500 500-550 550-600 n 5 20 25 30 30 23 11
a) Tính chiều cao trung bình và độ lệch mẫu của X .
b) Hãy lập khoảng ước lượng của chiều cao trung bình loại cây Bạch Đàn sau 1 năm với độ tin cậy γ = 95% . 96
Chương V. Lí thuyết ước lượng
Đáp số và hướng dẫn. 1. a) (496 , 5 5 ; 4 20 , 0 6), b) (4938 5 ; 1 , 227 9 , ) 2.
n = 500 > 30 nên ta xem X ~ N (µ;σ ) . (16 , 2 28 1 ; 631 , 6). 3.
Ta có n = 36 > 30 nên ta xem X ~ N(µ;σ ) và chưa biết phương sai 2 σ . 4.
Khoảng ước lượng cho tỉ lệ là ( 0 , 0 45 0 , 0 ;
89) vậy, tỉ lệ phần trăm tối đa là 9 , 8 % . 5. a) ( 6 , 0 102 6 , 0 ; 69 )
8 b) n = 2213, c) 6115.
C. Phương pháp giảng dạy.
- Giới thiệu ứng dụng của ước lượng khoảng trong thực tế.
- Thuyết trình, vấn đáp, và làm bài tập.
- Sử dụng các bảng phụ lục cho việc tính các giá trị của hàm phân phối chuẩn, Poisson, Student, chi bình phương.
- Yêu cầu SV đọc bài giảng trước khi lên lớp.
- Kiểm tra, đánh giá việc làm bài tập của SV.
Giảng viên gửi bài giảng cho sinh viên đọc trước. Giảng viên trình bày bài giảng trên lớp theo
phương pháp thuyết trình hỏi đáp. Giao bài tập cho sinh viên về nhà làm. Giới thiệu một số tài liệu tham khảo.
D. Tài liệu tham khảo
[1] Đậu Thế Cấp, Xác suất thống kê: Lí thuyết và các bài tập, NXB Giáo dục, 2006.
[2] Đặng Hùng Thắng, Thống kê và ứng dụng, NXB Giáo dục, 2008.
[3] PGS. TS. Phạm Xuân Kiều, Giáo Trình xác suất và thống kê, NXB Giáo dục, 2005.
[4] Trần Văn Minh, Phí Thị Vân Anh, Xác suất thống kê với các tính toán trên
Excel, NXB Giao Thông Vận tải, 2008.
[5] Đặng Công Hanh, Đặng Ngọc Dục, Giáo trình Lý thuyết xác suất và Thống kê
toán, trường Đại học Duy Tân,1996
[6] Trần Văn Minh, Phí Thị Vân Anh, Hướng dẫn giải bài tập Xác suất thống kê
với các tính toán trên Excel, NXB Giao Thông Vận tải, 2008. 97 Chương VI.
Kiểm định giả thiết thống kê.
A. Mục tiêu.
- Giới thiệu về khái niệm kiểm định giả thiết: cách đặt giả thiết H và đối thiết 0 H , 1
đưa ra các khả năng phạm sai lầm khi kiểm định.
- Nêu phương pháp chung khi thực hiện một bài toán kiểm định.
- Đưa ra phương pháp kiểm định cho: kì vọng, phương sai, tỉ lệ… trong từng trường hợp cụ thể. B. Nội dung.
1. Các khái niệm cơ bản
1.1. Đặt vấn đề:
Trong chương này, chúng ta sẽ giải quyết các bài toán dạng như sau:
Giả sử ta có hai giả thiết (hai khả năng) về một vấn đề nào đó, chẳng hạn:
1. Có ý kiến cho rằng tham số chưa biết θ của phân phối nhận giá giá trị θ , 0
nhưng lại có ý kiến cho rằng θ ≠ θ . 0
2. Có ý kiến cho rằng sau khi áp dụng phương pháp sản xuất mới, tỉ lệ sản phẩm
loại I của nhà máy tăng lên (có nghĩa là phương pháp sản xuất mới này có hiệu
quả), nhưng có ý kiến cho rằng tỉ lệ sản phẩm loại I không thay đổi (có nghĩa là
phương pháp sản xuất mới này không có hiệu quả).
3. Có ý kiến cho rằng Biến ngẫu nhiên đang xét tuân theo quy luật phân phối nhị
thức, lại có ý kiến không tán thành.
4. Có ý kiến cho rằng khả năng làm việc của con người phụ thuộc vào giới tính,
nhưng cũng có ý kiến cho rằng không ảnh hưởng.
Vấn đề đặt ra là ta phải chọn một trong hai giả thiết được nêu ra. Nói cách khác, ta
chọn giả thiết nào để khả năng đúng cao hơn, khả năng sai thấp hơn. Để cho tiện, ta
chọn một trong hai giả thiết là giả thiết H còn giả thiết kia là gi 0
ả thiết đối (đối thiết) H :θ = θ 0 0 H . Khi đó . 1  H :θ ≠ θ 1 0 Bài giảng Ví dụ 1.1.
Ông chủ của một cửa hàng buôn bán xe máy cho biết số xe máy bán được trong một
ngày của cửa hàng là 30 xe máy. Để kiểm tra lời tuyên bố này của ông chủ này là H :θ = 30
đúng hay sai, ta có thể đặt:  0 . H :θ ≠ 30 1 Ví dụ 1.2.
Khi tìm hiểu về chiều cao trung bình của một loại cây ở trong một khu rừng, với chiều
cao X của loại cây đó có phân phối chuẩn N ( 2
µ;σ ) ta có thể đưa ra giả thiết H như 0
sau: H : “Chiều cao trung bình của cây là µ = 20 m”. Khi đó, các đối thiết của 0 H có 0 thể là:
+ Đối thiết H : “Chiều cao trung bình của cây là µ ≠ 20 m”. 1
+ Đối thiết H : “Chiều cao trung bình của cây là µ < 20 m”. 1
+ Đối thiết H : “Chiều cao trung bình của cây là µ > 20 m”. 1
Để giải quyết bài toán này, thông tin duy nhất chúng ta có là mẫu ngẫu nhiên
(X ; X ;K; X . Vận dụng kết quả của lí thuyết xác suất, ta tìm miền 1 2 W , sao cho khi n )
mẫu (X ; X ;K; X ∈ thì ta bác bỏ giả thiết ; ;K; thì ta 1 2
H , còn khi (X X X ∉ 1 2 n ) W n ) W 0
chấp nhận giả thiết H cho đến khi có thông tin mới. Miền 0
W được gọi là miền bác bỏ.
Khi bác bỏ hoặc chấp nhận giả thiết H , chúng ta có thể mắc hai lo 0
ại sai lầm sau:
Sai lầm loại I: Bác bỏ H nhưng thực tế là 0 H là đúng. 0
Sai lầm loại II: Chấp nhận H nhưng thực tế là 0 H là sai. 0
Ta mong muốn chọn miền W sao cho cực tiểu cả hai khả năng phạm sai lầm. Nhưng
khi có mẫu cố định thì mong muốn này không thể thực hiện được, do đó thông thường
ta cho trước giới hạn trên của xác suất sai lầm loại I.
Kí hiệu α , α thường rất nhỏ α = 1% , 5% , 10% , K ta sẽ tìm miền W sao cho khả α
năng phạm sai lầm loại I không vượt quá α và khả năng phạm sai lầm loại II đạt cực tiểu.
Tùy theo hoàn cảnh cụ thể, sai lầm loại này có thể tai hại hơn sai lầm loại kia. Ví dụ 1.3.
a) Để tránh sai lầm cho điểm một học sinh giỏi thấp, thầy giáo cứ cho điểm cao một
cách dễ dàng, khi đó khả năng mắc phải sai lầm cho điểm một học sinh yếu kém cao
(đáng lẽ phải cho điểm thấp) là tăng lên.
b) Để tránh sai lầm cho việc bắt nhầm một người vô tội (bị oan), công an cứ thả người
này cách dễ dàng, nhưng khi đó khả năng mắc phải sai lầm thả nhầm một người có tội lại tăng lên.
Có hai cách dùng để khống chế khả năng mắc sai lầm:
Cách thứ nhất. Ta ấn định trước mức mắc phải sai lầm loại I và sai lầm loại II rồi tính
toán tìm một mẫu có kích thước ứng với hai mức sai lầm này.
Cách thứ hai. Ta ấn định trước xác suất sai lầm loại I (tức là có trước mức ý nghĩa α )
chọn miền bác bỏ W có xác suất sai lầm loại II nhỏ nhất. α
Trong bài giảng này, ta chọn cách thứ hai. 100
Chương VI. Kiểm định giả thiết thống kê
Cần chú ý rằng: Bác bỏ hoặc chấp nhận giả thiết tùy thuộc vào giá trị thực nghiệm của
tiêu chuẩn T và mức ý nghĩa α , kiểm định giả thuyết thống kê là một quy tắc hành
động sao cho khả năng mắc phải sai lầm nhỏ (ở mức nào đó).
1.2. Phương pháp kiểm định giả thiết thống kê
Có thể mô tả phương pháp kiểm định giả thiết thống kê như sau:
Xuất phát từ yêu cầu của bài toán thực tế, ta đưa ra một giả thiết H và giả thiết đối 0 H của nó. 1
Giả sử rằng H đúng, từ đó tìm một biến cố có xác suất đủ bé để có thể tin rằng biến 0
cố đó hầu như không thể xảy ra trong một phép thử. Muốn vậy, từ mẫu ngẫu nhiên
(X ; X ;K; X , ta chọn thống kê T f X ; X ;K = ; X ;θ sao cho nếu 1 2 T , ( 1 2 H đúng thì n 0 ) n ) 0
phân phối xác suất của T là xác định và thống kê T gọi là tiêu chuẩn kiểm định giả thiết H . 0
Do quy luật phân phối xác suất T đã biết, nên với α bé tùy ý, ta có thể tìm được miền
W sao cho P(T W = . α ) α α
Miền W gọi là miền bác bỏ giả thiết H . Trong thực tế, ta thường lấy α ∈[ , 0 01 , 0 ; 0 ] 5 α 0
và α được gọi là mức ý nghĩa của kiểm định.
Thực hiện một phép thử đối với mẫu ngẫu nhiên (X ; X ;K; X , ta được mẫu cụ thể 1 2 n )
(x ; x ;K; x . Từ mẫu này, ta tính được giá trị của T = t f x ; x ;K = ; x ;θ . 1 2 T , ( 1 2 n 0 ) n )
Nếu t W (tức là biến cố T W xảy ra) thì ta bác bỏ H ; chấp nhận H . α α 0 1
Nếu t W thì ta chấp nhận H . α 0 Chú ý.
Khi chấp nhận H , không có nghĩa 0
H đúng, mà chỉ có nghĩa là chưa có cơ sở bác bỏ 0
H , đành chấp nhận 0 H . 0
2. Kiểm định giả thiết về tham số.
2.1. Các loại kiểm định và phương pháp kiểm định giả thiết về các tham số.
Khi nghiên cứu một đặc tính hoặc một dấu hiệu nào đó của một tổng thể ta xét một
biến ngẫu nhiên X tác động lên tổng thể đó và thường là các dấu hiệu của tổng thể
được thể hiện qua các tham số đặc trưng của X hay phân phối của X , cho nên các giả
thiết về các tham số đặc trưng của X cũng là các giả thiết thường gặp.
Các loại kiểm định về tham số là:
1) Kiểm định hai phía đối với tham số, tức là kiểm định giả thiết H :θ = θ với đối 0 0
thiết H :θ ≠ θ với 1 0
θ là tham số đặc trưng nào đó của X chưa biết (thường là E(X )
hoặc là D(X )) và θ là một giá trị cụ thể được đưa ra dựa vào sự suy đoán nào đó. 0
2) Kiểm định phía phải đối với tham số là kiểm định giả thiết H :θ = θ với đối thiết 0 0 H : θ > θ . 1 0
3) Kiểm định phía trái đối với tham số là kiểm định giả thiết H :θ = θ với đối thiết 0 0 H : θ < θ . 1 0 101 Bài giảng
Để kiểm định giả thiết thống kê về các tham số như trên, người ta thường tiến hành theo các bước như sau:
Bước 1: Lập mẫu ngẫu nhiên của X là (X ; X ;K; X và chọn một hàm 1 2 n ) G
g(X ; X ;K =
; X ;θ và gọi là tiêu chu 1 2
ẩn kiểm định sao cho tìm được một qui tắc n 0 )
kiểm định tốt nhất của tham số θ . Sau đó tìm giá trị g(x ; x ;K; x ;θ ứng với một 1 2 n 0 )
mẫu thực nghiệm nào đó và gọi là giá trị thực nghiệm.
Bước 2: Với mức ý nghĩa α đã cho tìm miền bác bỏ W tương ứng (miền W phụ α α
thuộc vào các loại kiểm định).
Bước 3: Xét xem giá trị cụ thể g(x ; x ;K; x ;θ có thuộc 1 2 W hay không. n 0 ) α
+ Nếu g(x ; x ;K; x ;θ ∈
thì ta bác bỏ giả thiết 1 2
H và thừa nhận đối thiết H với n 0 ) Wα 0 1
mức ý nghĩa α (hay độ tin cậy γ = 1− α ).
+ Nếu g(x ; x ;K; x ;θ ∉
thì ta chưa có cơ sở bác bỏ giả thiết 1 2
H và thừa nhận đối n 0 ) Wα 0
thiết H với mức ý nghĩa 1 α .
Trong bài giảng này, chúng ta chỉ kiểm định giả thiết về các tham số đặc trưng của
biến ngẫu nhiên có phân phối chuẩn và về tỉ lệ của tổng thể; các bước kiểm định được
trình bày một cách ngắn gọn, chỉ nêu miền bác bỏ cho từng loại kiểm định.
2.2. Kiểm định giả thiết về trung bình của ĐLNN X~N(µ; σ2).
Giả sử trung bình của tổng thể (cũng chính là kì vọng toán của biến ngẫu nhiên X ) là µ chưa biết.
2.2.1. Trường hợp đã biết 2
D( X ) = σ .
a) Kiểm định hai phía đối với µ .
Cần kiểm định giả thiết: H : µ = µ với đối thiết H : µ ≠ µ (với µ là một giá trị nào 0 0 1 0 0 đó đã biết) H : µ = µ  0 0 H : µ ≠ µ 1 0 (X − µ0 )
* Ta chọn tiêu chuẩn kiểm định là G = Z =
n , biến ngẫu nhiên này có phân σ
phối chuẩn Z ~ N( ) 1 ; 0 .
* Với mức ý nghĩa α đã cho, ta xác định miền bác bỏ W như sau: α         W = − ∞;−zz
, trong đó z được xác định từ công thức α  α   ;+∞ α  α  2   2  2   2 α 1 γ m t   Φ z = 1 − − = 2  với Φ(m) ∫ − = e dt . α  2 2 2  2  0
* So sánh giá trị thực nghiệm z với z . α 2
+ Nếu z > z (nghĩa là z W ) thì ta bác bỏ giả thiết H và thừa nhận đối thiết H α α 0 1 2 với mức ý nghĩa α . 102
Chương VI. Kiểm định giả thiết thống kê
+ Nếu z z (nghĩa là z W ) thì ta chưa có cơ sở bác bỏ giả thiết H nên chấp nhận α α 0 2
giả thiết H với mức ý nghĩa α . 0
a) Kiểm định phía phải đối với µ .
Cần kiểm định giả thiết: H : µ = µ với đối thiết H : µ > µ 0 0 1 0 H : µ ≤ µ  0 0 H : µ > µ 1 0 (X − µ0 )
* Ta chọn tiêu chuẩn kiểm định là G = Z =
n , biến ngẫu nhiên Z này có σ
phân phối chuẩn N ( ) 1 ; 0 .
* Với mức ý nghĩa α đã cho, ta xác định miền bác bỏ W như sau: α 1 1 1 W = z
, trong đó z được xác định từ công thức Φ(z α ) = 1 − α − = − α = γ − α ( ;+∞ α ) α 2 2 2 2 m t với Φ(m) ∫ − = e 2 dt . 0
* So sánh giá trị thực nghiệm z với z . α
+ Nếu z > z (nghĩa là z W ) thì ta bác bỏ giả thiết H và thừa nhận đối thiết H với α α 0 1 mức ý nghĩa α .
+ Nếu z z (nghĩa là z W ) thì ta chưa có cơ sở bác bỏ giả thiết H nên chấp nhận α α 0
giả thiết H với mức ý nghĩa α . 0
c) Kiểm định phía trái đối với µ .
Cần kiểm định giả thiết: H : µ = µ với đối thiết H : µ < µ 0 0 1 0 H : µ ≥ µ  0 0 H : µ < 1 µ0 (X − µ0 )
* Ta chọn tiêu chuẩn kiểm định là G = Z =
n , biến ngẫu nhiên Z này có σ
phân phối chuẩn Z ~ N ( ) 1 ; 0 .
* Với mức ý nghĩa α đã cho, ta xác định miền bác bỏ W như sau: α W = − ∞; z , trong đó z = −z trong đó, α ( 1 α − ) α − 1 z
được xác định từ công thức α − 1 α 2 m t Φ(z với Φ(m) ∫ − = e 2 dt . α ) 1 1 1 = 1 − α − = − α = γ − 2 2 2 0
* So sánh giá trị thực nghiệm z với α− 1 z .
+ Nếu z < −z (nghĩa là z W ) thì ta bác bỏ giả thiết H và thừa nhận đối thiết H α α 0 1 với mức ý nghĩa α .
+ Nếu z ≥ −z (nghĩa là z W ) thì ta chưa có cơ sở bác bỏ giả thiết H nên chấp α α 0
nhận giả thiết H với mức ý nghĩa α . 0 Ví dụ 2.1.
Một máy đóng mì gói tự động quy định trọng lượng trung bình là µ 75 g, độ lệch 0 =
chuẩn σ = 15 g. Sau một thời gian sản xuất, kiểm tra 80 gói, ta có trọng lượng trung 103 Bài giảng
bình mỗi gói là 72 g. Cho kết luận về trọng lượng mì gói trung bình sản xuất với mức
ý nghĩa α = 5% có đúng quy định không. Giải
Trọng lượng trung bình cho mỗi gói mì là µ 75 g. 0 =
Trọng lượng trung bình thực tế sản xuất là µ chưa biết.
Ta đặt giả thuyết: H : µ = µ = 75; đối thiết H : µ ≠ µ = 75. 0 0 1 0
Trong đó, ta biết: σ = 15; α = 5% . (X − µ0 )
Do đã biết σ = 15 nên ta chọn tiêu chuẩn kiểm định là G = Z = n . σ
Đây là bài toán kiểm định hai phía.   Do 1   Φ z = − 0 , 0 25 = , 0 475  nên z = 9 , 1 6 0,05  2 0,05  2  2 Giá tr x − µ0 72 − 75 ị kiểm định: z = n = 80 ≈ − , 1 79 . σ 15 Ta có z ≈ 7 , 1 9 < 9 ,
1 6 nên ta chưa có cơ sở để bác bỏ H nên chấp nhận giả thuyết 0 H , 0
tức là sản xuất diễn ra bình thường. Ví dụ 2.2.
Một nhà máy cơ khí qui định chiều dài của chi tiết máy A là 20 cm. Ban giám đốc nhà
máy cho rằng có một tổ sản xuất loại chi tiết máy đó không đúng chiều dài quy định.
Hãy kiểm định nhận định đó với mức ý nghĩa α = 0 ,
0 5 , biết rằng sau khi chọn ngẫu
nhiên 25 chi tiết máy của tổ sản xuất đó thì kết quả đo đạc cho biết chiều dài trung bình của mẫu là 20 5
, cm và chiều dài X của loại chi tiết được sản xuất ra từ nhà máy
đó có phân phối chuẩn N ( 2 µ 1
; ), với σ = 1 đã biết. Giải
Ta đặt giả thiết H : µ 20 cm và đối thiết H : µ 20 cm. 0 = 1 ≠
Ta có n = 25, x = 20 5 , cm, σ = 1 (X − µ0 )
Do đã biết σ = 1 nên ta chọn tiêu chuẩn kiểm định là G = Z = n . Ta có σ Z ~ N ( ) 1 ; 0 . 20 5 , − 20 Ta có z = 25 = 5 , 2 1
Đây là bài toán kiểm định hai phía Với mức ý nghĩa α = 0 ,
0 5 , ta có z = z = 9 , 1 6 . α 0,05 2 2 Ta có z = 5 , 2 > 9 ,
1 6 nên ta bác bỏ giả thiết H và chấp nhận giả thiết 0 H với mức ý 1 nghĩa α = 0 , 0 5 .
Vậy kết luận của giám đốc là đúng. 104
Chương VI. Kiểm định giả thiết thống kê
2.1.2. Trường hợp chưa biết D(X ) .
Các bước kiểm định đối với các loại giả thuyết này hoàn toàn tương tự như trên nhưng (X − µ0 )
thay tiêu chuẩn kiểm định G = Z =
n bởi tiêu chuẩn kiểm định σ (X − µ0 ) G = T =
n , biến ngẫu nhiên này có phân phối Student T (n − ) 1 , với n −1 bậc S tự do.
+ Trong trường hợp n ≤ 30 , ta thay z , z t , tt . α α α − 1 z bởi α n α ; 1 − n− 1 ; 1 α − n− ; 1 2 2 (X − µ0 )
+ Trong trường hợp n > 30 , ta xem G = Z =
n có phân phối chuẩn N ( ) 1 ; 0 . S Ví dụ 2.3.
Trọng lượng của một loại sản phẩm do một xí nghiệp sản xuất theo qui định chung là
6 kg. Sau một thời gian sản xuất, người ta tiến hành kiểm tra 121 sản phẩm do xí
nghiệp đó sản xuất và tính được số trung bình x = 9 ,
5 75 kg và độ lệch tiêu chuẩn điều chỉnh là s = , 2 4 .
Biết rằng trọng lượng X của loại sản phầm do xí nghiệp đó sản xuất có phân phối chuẩn N( 2 µ;σ ).
Có người cho rằng trọng lượng sản phẩm do xí nghiệp sản xuất không đạt yêu cầu
(không đúng với qui định chung). Hãy cho kết luận về lời nhận xét đó với mức ý nghĩa α = 0 , 0 5 . Giải
Ta đặt giả thiết H : µ 6 kg và đối thiết H : µ 6 kg. 0 = 1 ≠ Ta có x = 9 ,
5 75 , n = 121, s = , 2 4 . (X − µ0 ) Do 2
σ chưa biết nên ta chọn tiêu chuẩn kiểm định là G = Z = n . S
Lại có n = 121 ≥ 30 nên Z ~ N( ) 1 ; 0 . ( 9 , 5 75 − 6) Ta có z = 121 ≈ − 1 , 0 146 . , 2 4
Đây là bài toán kiểm định hai phía. Với mức ý nghĩa α = 0 ,
0 5 , ta tính được z = 9 , 1 6 . 0,05 2 Ta có z = 1 , 0 146 < 9 ,
1 6 . Khi đó ta chưa có cơ sở để bác bỏ H nên ta có thể kết luận 0
rằng lời nhận xét trên là không đúng với α = 0 , 0 5 . Ví dụ 2.4.
Tuổi thọ trung bình của 1 loại bóng đèn do nhà máy A sản xuất khi chưa cải tiến kĩ
thuật sản xuất là 2000 giờ, sau thời gian cải tiến kĩ thuật người ta chọn ngẫu nhiên 25
bóng đèn do nhà máy A sản xuất và cho thắp kiểm tra thử nghiệm và kết quả cho biết
tuổi thọ trung bình của mẫu thực nghiệm là x = 2010 giờ và độ lệch tiêu chuẩn của
mẫu là s = 15 giờ với mức ý nghĩa α = 0 , 0 5 . 105 Bài giảng
Hãy cho biết kết luận về lời nhận định “tuổi thọ bóng đèn có tăng lên sau khi cải tiến
kĩ thuật”. Biết rằng biến ngẫu nhiên X chỉ tuổi thọ bóng đèn có phân phối chuẩn N ( 2 µ;σ ). Giải
Ta đặt giả thiết H : µ 2000 giờ và đối thiết H : µ 2000 giờ. 0 = 1 >
Ta có n = 25, s = 15. (X − µ0 ) Do 2
σ chưa biết nên ta chọn tiêu chuẩn kiểm định là G = Z = n . S
Lại có n = 25 < 30 nên Z ~ T (n − ) 1 .
Đây là bài toán kiểm định phía phải. Với mức ý nghĩa α = 0 ,
0 5 , ta tính được t = 0 , 2 64 . 25− ; 1 0,05 Ta có 2010 − 2000 10 z = .5 = . 15 3 10 Ta có z = > , 2 064 3
Do đó, ta bác bỏ giả thiết H và chấp nhận đối thiết 0
H tức là cho rằng sau khi cải tiến 1
kĩ thuật, tuổi thọ trung bình của loại bong đèn dó có tăng lên với mức ý nghĩa α = 0 , 0 5 .
2.3. Kiểm định giả thiết về phương sai của ĐLNN X~N(µ; σ2).
2.3.1. Trường hợp đã biết E(X ) = µ
a) Kiểm định hai phía.
Cần kiểm định giả thiết: 2 2 H : σ = σ với đối thiết 2 2 H : σ ≠ σ (với 2 σ là một giá trị 0 0 1 0 0 nào đó đã biết).  2 H : σ = 2 σ  0 0  2 H : σ ≠ 2 1 σ 0 ∑n(X − µ i )2
* Ta chọn tiêu chuẩn kiểm định là 2 i 1 G = χ = = , biến ngẫu nhiên 2 χ này có 2 σ 0
phân phối Chi bình phương với n bậc tự do 2 2 χ ~ χ (n) .
* Với mức ý nghĩa α đã cho, ta xác định miền bác bỏ W như sau: α      2   2  W = − ∞; χ ∪ χ , trong đó 2 χ , 2 χ
được xác định dựa vào bảng phân α  α   + ; ∞ α  α α  n − 1 ; n n 1 ; − n; 2   ; 2  2 2
phối Chi bình phương với n bậc tự do.
* So sánh giá trị thực nghiệm 2 χ với 2 χ , 2 χ . α α n 1 ; − n; 2 2 + Nếu 2 2 χ < χ hoặc 2 2 χ > χ
(nghĩa là z W ) thì ta bác bỏ giả thiết H và thừa α α α 0 n 1 ; − n; 2 2
nhận đối thiết H với mức ý nghĩa α . 1 106
Chương VI. Kiểm định giả thiết thống kê + Nếu 2 2 2 χ ≤ χ ≤ χ
(nghĩa là z W ) thì ta chưa có cơ sở bác bỏ giả thiết H nên α α α 0 n 1 ; − n; 2 2
chấp nhận giả thiết H với mức ý nghĩa α . 0
b) Kiểm định phía phải đối với 2 σ .
Cần kiểm định giả thiết: 2 2 H : σ = σ với đối thiết 2 2 H : σ > σ . 0 0 1 0  2 H : σ ≤ 2 σ  0 0  2 H : σ > 2 1 σ 0
Ta thực hiện các bước tương tự như trên nhưng miền bác bỏ của kiểm định bên phảiW = χ
. So sánh giá trị thực nghiệm 2 χ với 2 χ
, ta đưa ra điều kết luận. α ( 2 ;+∞ n;α ) n
c) Kiểm định phía trái đối với 2 σ .
Cần kiểm định giả thiết: 2 2 H : σ = σ với đối thiết 2 2 H : σ < σ . 0 0 1 0  2 H : σ ≥ 2 σ  0 0  2 H : σ < 2 1 σ 0
Ta thực hiện các bước tương tự như trên nhưng miền bác bỏ của kiểm định bên tráiW = − ∞ χ
. So sánh giá trị thực nghiệm 2 χ với 2 χ
, ta đưa ra điều kết luận. α ( 2 ; n 1; α − ) n 1 ; α −
2.3.2. Trường hợp chưa biết E(X ) = µ .
Trong thực tế, ta thường gặp kiểm định các tham số của biến ngẫu nhiên N( 2 µ;σ ) với
µ và σ chưa biết. Các bước kiểm định ta thực hiện tương tự như trong trường hợp đã biết E(X ) = µ , nhưng ta chọn tiêu chuẩn kiểm định là n ∑(X X i )2 2 n −1 i 1 = ( ) 2 S G = χ = =
, với S là độ lệch mẫu hiệu chỉnh, biến ngẫu nhiên 2 2 σ 0 σ 0 2
χ này có phân phối Chi bình phương với n − 1 bậc tự do 2 χ ~ 2 χ (n − ) 1 .
Các cận của miền bác bỏ là 2 χ , 2 χ , 2 χ , 2 χ được thay bởi 2 χ , 2 χ , α α n 1 ; α − n;α α α n 1 ; − n; n− 1 ; 1 − n− ; 1 2 2 2 2 2 χ , 2 χ . n− 1 ; 1 α − n− ; 1 α Ví dụ 2.5.
Kiểm tra sự ổn định của một máy tự động về thời gian lắp ghép một bộ phận điện tử.
Người ta quan sát ngẫu nhiên 16 lần lắp ghép của máy tự động đó và tính được độ lệch
tiêu điều chỉnh của mẫu thực nghiệm là s = 15 giây, giả sử rằng thời gian lắp ghép X
các bộ phận điện tử của máy tự động đó có phân phối chuẩn N ( 2 µ;σ ) và máy được
xem là ổn định nếu độ lệch về thời gian không quá 5 giây. Hãy xét xem máy tự động
trên hoạt động có ổn định không, biết rằng mức ý nghĩa α = 0 , 0 5 ? Giải
Ta đặt giả thiết H : 2 σ = 52
25 và đối thiết H : 2 σ 25 . 0 = 1 > 2 (n − ) 2 1 Do không bi S
ết E(X ) = µ nên ta chọn tiêu chuẩn kiểm định là G = χ = . 2 σ 0 107 Bài giảng 2 (16 − ) 1 15 225
Ta có n = 16 , s = 15 . Khi dó χ = = ≈ 9 . 25 25
Đây là bài toán kiểm định bên phải đối với phương sai. Với mức ý nghĩa α = 0 , 0 5 , ta tính được 2 2 χ = χ = 25 n− ; 1 α 15;0,05
Ta có 9 < 25 nên ta chưa có cơ sở bác bỏ giả thiết H nên chấp nhận giả thiết 0 H với 0 mức ý nghĩa α = 0 , 0 5 .
2.4. Kiểm định giả thiết về tỉ lệ các phần tử có tính chất nào đó trong tổng thể.
Cho một tổng thể có hai loại đối tượng, kí hiệu là A A . Gọi p là tỉ lệ số đối tượng
A có trong tổng thể. Đối với tham số p , ta có các loại kiểm định sau đây: * Kiểm định hai phía.
Cần kiểm định giả thiết: H :
với đối thiết H : (với 0 p = p0 1 p p0
p là một giá trị nào 0 đó đã biết).
H : p = p  0 0 H : p ≠ 1 p0
* Kiểm định phía phải.
Cần kiểm định giả thiết: H :
với đối thiết H : . 0 p = p0 1 p > p0
H : p p  0 0 H : p > 1 p0 * Kiểm định phía trái.
Cần kiểm định giả thiết: H :
với đối thiết H : . 0 p = p0 1 p < p0
H : p p  0 0 H : p < 1 p0
Như ta đã biết ở phần ước lượng tỉ lệ của tổng thể ở chương trước, nếu mẫu có kích n 1
thước n lớn thì tần suất X = ∑ X bằng số lần đối tượng A xuất hiện trong n lần i n i=1
chọn ngẫu nhiên một đối tượng từ tổng thể xấp xỉ biến ngẫu nhiên chuẩn
N (p; p(1 − p)) với n khá lớn.
Do đó, để kiểm định giả thiết về tỉ lệ p của tổng thể, ta chọn tiêu chuẩn kiểm định X p G = Z 0 =
n với n khá lớn, biến ngẫu nhiên Z này có phân phối chuẩn p 1 0 ( − p0 ) N ( ) 1 ; 0 . Giá tr x p f p
ị thực nghiệm là z 0 = n 0 =
n với x = f là tỉ lệ đối tượng p 1 − 1 0 ( p0 ) p0 ( − p0 ) np > 10 0
A trong mẫu với  .
n(1 − p0 ) > 10
Với mức ý nghĩa α và n khá lớn tùy theo từng loại kiểm định, ta thực hiện như sau:
* Đối với kiểm định hai phía. 108
Chương VI. Kiểm định giả thiết thống kê
Ta tìm z rồi so sánh với z α 2
+ Nếu z > z thì ta bác bỏ giả thiết H và thừa nhận đối thiết H với mức ý nghĩa α . α 0 1 2
+ Nếu z z thì ta chưa có cơ sở bác bỏ giả thiết H nên chấp nhận giả thiết H với α 0 0 2 mức ý nghĩa α .
* Đối với kiểm định phía phải.
Ta tìm z rồi so sánh với z α
+ Nếu z > z thì bác bỏ giả thiết H và thừa nhận đối thiết H với mức ý nghĩa α . α 0 1
+ Nếu z z thì ta chưa có cơ sở bác bỏ giả thiết H nên chấp nhận giả thiết H với α 0 0 mức ý nghĩa α .
* Đối với kiểm định phía trái. Ta tìm α− 1 z
rồi so sánh với z + Nếu z < z
= −z thì bác bỏ giả thiết H và thừa nhận đối thiết H với mức ý nghĩa α − 1 α 0 1 α . + Nếu z z
= −z thì ta chưa có cơ sở bác bỏ giả thiết H nên chấp nhận giả thiết α − 1 α 0
H với mức ý nghĩa α . 0 Ví dụ 2.6.
Một nhà máy sản xuất sản phẩm với tỉ lệ sản phẩm loại I lúc đầu là , 0 20 . Sau khi áp
dụng phương pháp sản xuất mới, kiểm tra 500 sản phẩm thấy số sản phầm loại I là 150
sản phẩm. Cho kết luận về phương pháp sản xuất mới này với mức ý nghĩa α = 1% . Giải
Tỉ lệ sản phẩm loại I lúc đầu là p , 0 20 . 0 =
Tỉ lệ sản phẩm loại I khi áp dụng phương pháp mới là p chưa biết.
Ta đặt giả thiết: H : p = p = ,
0 2 ; H : p p = , 0 2 . 0 0 1 0 np > 10 150 Ta có  0 , f = = 3 , 0 ; n = 500 ; n(1 − p 500 0 ) > 10   Do 1   Φ z = − 0 , 0 05 = , 0 495  nên z = z = 5 , 2 8 0,01  2 0,01 0,995  2  2 Giá tr f p ị kiểm định: z n 0 = n p 1 0 ( − p0 ) 3 , 0 − , 0 2 Khi đó: z = 500 ≈ 5 , 5 9 , 0 2(1− , 0 2) Ta có z ≈ 5 , 5 9 > z = 5 ,
2 8 nên ta bác bỏ giả thuyết H , tức là p p . α 0 0 2
Khi đó, phương pháp sản xuất mới đã làm thay đổi tỉ lệ sản phẩm loại I. 109 Bài giảng Ví dụ 2.7. M 2
ột công ti A sản xuất bánh kẹo tuyên bố rằng số trẻ em thích ăn bánh của công ti. 3
Trong một mẫu gồm 100 trẻ em được hỏi, có 55 em tỏ ra thích bánh của công ti A .
Với mức ý nghĩa 5% , số liệu nói trên có chứng tỏ là lời tuyên bố của công ti là hơi
quá đáng không? Giải 2 2
Ta có giả thiết H : , đối thiết H : . 0 p = p0 = p 3 0 < 3
Đây là kiểm định bên trái đối với tỉ lệ. Dựa vào các số liệu của mẫu, ta có: np > 10 55  0 , f = = 5 , 0 5 , α = 5% nên z = −z = − 6 , 1 4 1−0,05 0,05 n(1 − p 100 0 ) > 10 2 5 , 0 5 − Khi f p đó 0 3 z = n = 100 = − , 2 49 < − , 1 64 . p0 (1 − p0 ) 2  2  1  −  3  3 
Vậy, lời tuyên bố của công ti là quá sự thật. Ví dụ 2.8.
Tỉ lệ phế phẩm ở một nhà máy lúc ban đầu là 10% . Sau một thời gian, nhà máy cải
tiến kĩ thuật sản xuất, điều tra ngẫu nhiên 400 sản phẩm thấy có 30 phế phẩm. Với mức ý nghĩa α = 0 ,
0 25 , hãy xét xem việc cải tiến kĩ thuật của nhà máy có làm giảm tỉ
lệ phế phẩm của nhà máy không? Giải
Ta có giả thiết H : p = p = 1 ,
0 , đối thiết H : p 1 , 0 . 0 0 0 < 30 n = 400 , f = = 0 , 0 75 . 400
Đây là bài toán kiểm định bên trái đối với tỉ lệ. Với mức ý nghĩa α = 0 , 0 25 , ta có z = −z = − 9 , 1 6 . 1−0,025 0,025 Khi dó f p0 0 , 0 75 − 1 , 0 z = n = 400 = − , 1 66 p0 (1 − p0 ) ( 1 , 0 1− ) 1 , 0 Ta có z > z
ta chưa có cơ sở bác bỏ giả thiết 0,025
H nên chấp nhận giả thiết 0 H với 0 mức ý nghĩa α = ,
0 025 . Tức là việc cải tiến kĩ thuật chưa làm tỉ lệ phế phẩm giảm xuống.
2.5. Kiểm định giả thiết về hai kì vọng của hai ĐLNN chuẩn độc lập.
Giả sử ta có hai ĐLNN chuẩn X ~ N ( 2 µ ;
có mẫu ngẫu nhiên là (X ; X ;K; X và 1 2 n ) 1 σ 1 ) Y ~ N ( 2 µ ;
có mẫu ngẫu nhiên là (Y ;Y ;K;Y . 1 2 m ) 2 σ 2 )
2.5.1. Trường hợp đã biết 2 σ1 , 2 σ 2 .
a) Kiểm định hai phía của µ . 1 110
Chương VI. Kiểm định giả thiết thống kê
Cần kiểm định giả thiết: H : µ = µ với đối thiết: H : µ ≠ µ (với 0 1 2 1 1 2
p là một giá trị 0 nào đó đã biết). H : µ = µ  0 1 2 H : µ ≠ µ 1 1 2 * Ta ch X Y
ọn tiêu chuẩn kiểm định là G = Z =
, biến ngẫu nhiên này có phân 2 2 σ σ 1 2 + n m
phối chuẩn N ( ) 1 ; 0 .
* Với mức ý nghĩa α đã cho, ta xác định miền bác bỏ W như sau: α         W = − ∞ − ; zz
, trong đó z được xác định từ công thức α  α   ;+∞ α  α  2   2  2   2 α 1 γ m t   Φ z = 1 − − = 2  với Φ(m) ∫ − = e dt . α  2 2 2  2  0
* So sánh giá trị thực nghiệm z với z . α 2
+ Nếu z > z (nghĩa là z W ) thì ta bác bỏ giả thiết H và thừa nhận đối thiết H α α 0 1 2 với mức ý nghĩa α .
+ Nếu z z (nghĩa là z W ) thì ta chưa có cơ sở bác bỏ giả thiết H nên chấp nhận α α 0 2
giả thiết H với mức ý nghĩa α . 0
b) Kiểm định phía phải của µ . 1
Cần kiểm định giả thiết: H : µ = µ với đối thiết: H : . 0 1 2 1 µ1 > µ2 H : µ ≤ µ  0 1 2 H : µ > µ 1 1 2 * Ta ch X Y
ọn tiêu chuẩn kiểm định là G = Z =
, biến ngẫu nhiên này có phân 2 2 σ σ 1 2 + n m
phối chuẩn N ( ) 1 ; 0 .
* Với mức ý nghĩa α đã cho, ta xác định miền bác bỏ W như sau: α 1 1 1 W = z
, trong đó z được xác định từ công thức Φ(z α ) = 1 − α − = − α = γ − α ( ;+∞ α ) α 2 2 2 2 w t với Φ(w) ∫ − = e 2 dt . 0
* So sánh giá trị thực nghiệm z với z . α
+ Nếu z > z (nghĩa là z W ) thì ta bác bỏ giả thiết H và thừa nhận đối thiết H với α α 0 1 mức ý nghĩa α .
+ Nếu z z (nghĩa là z W ) thì ta chưa có cơ sở bác bỏ giả thiết H nên chấp nhận α α 0
giả thiết H với mức ý nghĩa α . 0
c) Kiểm định phía trái của µ . 1 111 Bài giảng
Cần kiểm định giả thiết: H : µ = µ với đối thiết: H : µ < µ . 0 1 2 1 1 2 H : µ ≥ µ  0 1 2 H : µ < µ 1 1 2 * Ta ch X Y
ọn tiêu chuẩn kiểm định là G = Z =
, biến ngẫu nhiên này có phân 2 2 σ σ 1 2 + n m
phối chuẩn N ( ) 1 ; 0 .
* Với mức ý nghĩa α đã cho, ta xác định miền bác bỏ W như sau: α 1 1 W = − ∞; z , trong đó z
= −z được xác định từ công thức Φ(z α ) = − α = γ − α ( 1 α − ) α − 1 α 2 2 2 w t với Φ(w) ∫ − = e 2 dt . 0
* So sánh giá trị thực nghiệm z với α− 1 z . + Nếu z < z (nghĩa là
) thì ta bác bỏ giả thiết 1 z W
H và thừa nhận đối thiết H α − α 0 1 với mức ý nghĩa α . + Nếu z z (nghĩa là
) thì ta chưa có cơ sở bác bỏ giả thiết 1 z W H nên chấp α − α 0
nhận giả thiết H với mức ý nghĩa α . 0
2.5.2. Trường hợp chưa biết 2 σ1 , 2
σ 2 và mẫu lớn m + n > 30 .
Các bước kiểm định ta thực hiện tương tự như trong trường hợp đã biết 2 σ và 2 1 σ , 2 nh X Y
ưng ta chọn tiêu chuẩn kiểm định là G = Z = , trong đó, 2 S , 2 1 S là 2 S 2 S 2 1 2 + n m
phương sai mẫu hiệu chỉnh. Ví dụ 2.9.
Người ta cân ngẫu nhiên 75 trẻ sơ sinh ở khu vực A và 100 trẻ sơ sinh ở khu vực B,
kết quả cho theo bảng sau đây Các tham số Số trẻ được cân
Trọng lượng trung Phương sai điều bình chỉnh Khu vực A 75 2 x = 0 , 3 s , 3 0 1 = B 100 2 y = , 3 2 s 0 , 5 2 =
Có người cho rằng trọng lượng trung bình của các trẻ sơ sinh ở hai khu vực trên là như
nhau. Hãy kiểm định lời nhận xét đó với mức ý nghĩa α = 0 ,
0 5 . Giả sử trọng lượng X
và trọng lượng Y của trẻ sơ sinh ở khu vực A và khu vực B đều có phân phối chuẩn. Giải
Gọi µ là kì vọng toán của biến ngẫu nhiên 1
X (trọng lượng trung bình của toàn bộ trẻ
sơ sinh ở khu vực A) và µ là kì vọng toán của biến ngẫu nhiên 2
Y (trọng lượng trung
bình của toàn bộ trẻ sơ sinh ở khu vực B). 112
Chương VI. Kiểm định giả thiết thống kê
Ta đặt giả thiết H : µ = µ và đối thiết H : µ ≠ µ . 0 1 2 1 1 2
Do chưa biết các phương sai 2 σ , 2 1
σ nên ta chọn tiêu chuẩn kiểm định 2 X Y G = Z = . S 2 S 2 1 2 + n m Ta có x = 0 ,
3 , n = 75 , m = 100 , x = , 3 0 , y = , 3 2 , 2 s , 3 0 và 2 s 0 , 5 nên 1 = 2 = 0 , 3 − , 3 2 10 2 z = = − , 0 2. = − 0 , 3 0 , 5 3 3 + 75 100
Đây là bài toán kiểm định hai phía Với mức ý nghĩa α = 0 ,
0 5 , ta có z = z = 9 , 1 6 . α 0,05 2 2 2 Ta có z = < 9 ,
1 6 = z nên ta chưa có cơ sở bác bỏ giả thiết H nên chấp nhận giả 3 α 0 2
thiết H với mức ý nghĩa α = 0 ,
0 5 . Nghĩa là cho rằng trọng lượng trung bình của trẻ 0
sơ sinh ở hai khu vực là như nhau với mức ý nghĩa α = 0 , 0 5 .
2.4.3. Trường hợp 2 2 σ ch n + m − 2 . 1 = σ 2 ưa biết và ≤ 30
Các bước kiểm định ta thực hiện tương tự như trong trường hợp đã biết 2 σ và 2 1 σ , 2
(X Y) n + m − 2
nhưng ta chọn tiêu chuẩn kiểm định là G = T = , biến  1 1   +  ([n − ) 2 1 S + m −1 1 ( ) 2 S2 ]  n m
ngẫu nhiên này có phân phối Student T (n + m − 2) với n + m − 2 bậc tự do. Trong đó, 2 S , 2 1
S là phương sai mẫu hiệu chỉnh và ta thay 2 z , z t , α α α − 1 z bởi α n+m−2; 2 2 t , t . n+m−2 α ; n+m−2 1 ; α −
2.6. Kiểm định giả thiết thống kê về hai tỉ lệ của hai ĐLNN. * Ta ch F F
ọn tiêu chuẩn kiểm định là G = Z = 1 2
, ĐLNN này có phân F ( 1 1 1− F )   +   1 n n2  n F + n F
phối chuẩn N ( ) 1 ; 0 , trong đó 1 1 2 2 F = 1 n + n2 Giá tr f f n f + n f
ị thực nghiệm là z = 1 2 , trong đó 1 1 2 2 f = . 1 n + n2 f ( 1 1 1− f )   +   1 n n2 
a) Kiểm định hai phía của 1 p .
* Cần kiểm định giả thiết: H :
với đối thiết H : (với 0 1 p = p2 1 1 p p2
p là một giá trị 0 nào đó đã biết). 113 Bài giảng
H : p = p  0 1 2 H : p ≠ 1 1 p2
* Với mức ý nghĩa α đã cho, ta xác định miền bác bỏ W như sau: α         W = − ∞;−zz
, trong đó z được xác định từ công thức α  α   ;+∞ α  α  2   2  2   2 α 1 γ m t   Φ z = 1 − − = 2  với Φ(m) ∫ − = e dt . α  2 2 2  2  0
* So sánh giá trị thực nghiệm z với z . α 2
+ Nếu z > z (nghĩa là z W ) thì ta bác bỏ giả thiết H và thừa nhận đối thiết H α α 0 1 2 với mức ý nghĩa α .
+ Nếu z z (nghĩa là z W ) thì ta chưa có cơ sở bác bỏ giả thiết H nên chấp nhận α α 0 2
giả thiết H với mức ý nghĩa α . 0
b) Kiểm định phía phải của 1 p .
Cần kiểm định giả thiết: H :
với đối thiết H : . 0 1 p = p2 1 1 p > p2
H : p = p  0 1 2 H : p > 1 1 p2
* Với mức ý nghĩa α đã cho, ta xác định miền bác bỏ W như sau: α 1 1 1 W = z
, trong đó z được xác định từ công thức Φ(z α ) = 1 − α − = − α = γ − α ( ;+∞ α ) α 2 2 2 2 w t với Φ(w) ∫ − = e 2 dt . 0
* So sánh giá trị thực nghiệm z với z . α
+ Nếu z > z (nghĩa là z W ) thì ta bác bỏ giả thiết H và thừa nhận đối thiết H với α α 0 1 mức ý nghĩa α .
+ Nếu z z (nghĩa là z W ) thì ta chưa có cơ sở bác bỏ giả thiết H nên chấp nhận α α 0
giả thiết H với mức ý nghĩa α . 0
c) Kiểm định phía trái của 1 p .
Cần kiểm định giả thiết: H :
với đối thiết H : . 0 1 p p2 1 1 p < p2
H : p p  0 1 2 H : p < 1 1 p2
* Với mức ý nghĩa α đã cho, ta xác định miền bác bỏ W như sau: α W = − ∞; z , trong đó α ( 1 α − ) α − 1 z được xác định từ công thức 2 w t Φ(z α α γ với Φ(w) ∫ − = e 2 dt . −α ) 1 1 1 = 1 1 − − = − = − 2 2 2 0
* So sánh giá trị thực nghiệm z với α− 1 z . 114
Chương VI. Kiểm định giả thiết thống kê + Nếu z < z (nghĩa là
) thì ta bác bỏ giả thiết 1 z W
H và thừa nhận đối thiết H α − α 0 1 với mức ý nghĩa α . + Nếu z z (nghĩa là
) thì ta chưa có cơ sở bác bỏ giả thiết 1 z W H nên chấp α − α 0
nhận giả thiết H với mức ý nghĩa α . 0
2.7. Kiểm định giả thiết thống kê về quy luật phân phối.
Ta đã biết rằng khi n khá lớn thì hàm phân phối thực nghiệm F xấp xỉ hàm phân n (x )
phối F(x) của biến ngẫu nhiên X , nhưng nhiều khi dựa vào sự suy đoán, ta có thể
nhận biết được dạng hàm phân phối F(x) của biến ngẫu nhiên X .
Ta đặt giả thiết H : F
và đối thiết H : F . 1
(x) ≠ F0 (x) 0
(x) = F0 (x)
Để kiểm định giả thiết H , người ta dùng quy tắc kiểm định Chi bình ph 0 ương như sau:
+ Ta lập mẫu ngẫu nhiên (X ; X ;K; X của ; ;K; , 1 2
X và mẫu thực nghiệm là (x x x 1 2 n ) n )
ta sắp xếp mẫu thực nghiệm theo dạng các bảng phân phối không chia lớp hoặc chia lớp. Xét xác suất:
+ p = P X = a hoặc p = P a X < a , i = ;
1 m nếu mẫu thực nghiệm của X sắp i ( i−1 i ) i ( i )
xếp theo bảng phân phối thực nghiệm không chia lớp hoặc chia lớp và xác định các tần m su n ất f i =
(với n = ∑ n ) của các giá trị hoặc của các lớp trong mẫu thực nghiệm, ta i n i i =1 xem lớp [a ; là (− ; ∞
và lớp [a ;a là lớp [a + ; ∞ . m−1 ) m 1 − m ) 1 a ) 0 1 a ) X 1 x x … 2 x m n n n i 1 n 2 m X [a ;a = − ; [a ; … [a ;a a ; m−1 m ) = [ +∞ m−1 ) 1 a2 ) 0 1 ) ( ∞ 1 a ) n n n i 1 n 2 k Theo lu n
ật số lớn Bernoulli, ta biết rằng: i P
 → p , khi n   → +∞ , i = ; 1 m . i n m 2 ( 2 n np 2 i i ) m Bi n ến ngẫu nhiên G i = χ = ∑ = ∑
n phân phối Chi bình phương i=1 npi i=1 npi
với m r −1 bậc tự do với n khá lớn, m là số lượng các giá trị khác nhau hoặc số lớp
ứng với mẫu thực nghiệm cho theo bảng phân phối thực nghiệm không chia lớp vả r
là số lượng các tham số chưa biết của F(x), các tham số này được ước lượng bằng
phương pháp hợp lí cực đại.
Ta có quy tắc kiểm định sau đây: m 2 ( 2 n np 2 i i ) m * Xác n
định giá trị thực nghiệm: i χ = ∑ = ∑ − n . i=1 npi i=1 npi
* Với mức ý nghĩa α , ta tìm số 2 χ
từ bảng phân phối Chi bình phương rồi so mr − , 1 α sánh với 2 χ . 115 Bài giảng + Nếu 2 2 χ
< χ thì bác bỏ giả thiết H và thừa nhận đối thiết H với mức ý nghĩa mr − ; 1 α 0 1 α . + Nếu 2 2 χ
≥ χ thì ta chưa có cơ sở bác bỏ giả thiết H nên chấp nhận giả thiết mr − ; 1 α 0
H với mức ý nghĩa α . 0 Chú ý.
Các phân phối cơ bản cần kiểm định.
1. Nhị thức: X ~ B(n; p).
+ Nếu n đã biết, p đã biết thì r = 0 .
+ Nếu n đã biết, p chưa biết thì r = 1.
+ Nếu n chưa biết, p chưa biết thì r = 2 .
2. Poisson: X ~ P(λ) . λ chưa biết, ta thay λ = x , r = 1
3. Chuẩn: X ~ N ( 2 µ;σ ) n 1 N 2
ếu µ , σ chưa biết, ta thay µ = x , σ = s với s =
∑(x x là phương sai mẫu i ) n −1 i=1
hiệu chỉnh, r = 2 . Ví dụ 2.10.
Có thể cho rằng số mặt sấp xuất hiện khi tung bốn đồng tiền là đại lượng ngẫu nhiên
tuân theo quy luật phân phối xác suất nhị thức B(n; p) được không, nếu biết mức ý nghĩa α = 0 , 0 1.
Khi tung 100 lần thì người ta được kết quả sau đây x (số mặt sấp) 0 1 2 3 4 i
n (số lần xuất hiện) 8 20 42 22 8 i Giải
Gọi F(x) là hàm phân phối xác suất của X F
là hàm phân phối của biến ngẫu 0 (x)
nhiên có phân phối nhị thức B(n; p). Ta có P(X = k) k k = C p 1 p − − . n ( )n k
Đặt giả thiết H : F
và đối thiết H : F . 0
(x) ≠ F0(x) 0
(x) = F0 (x) 1
Ta có n = 4 , p = = 5 , 0 . 2
Nếu X có phân phối nhị thức B( 5 , 0 ; 4
) thì các xác suất p được xác định như sau i i i p = 4 C − 5 , 0 1− 5 , 0 ; i = 4 ; 0 i+1 4 ( ) i Khi đó, ta có: p 0 , 0 625 , p , 0 25 , p 3 , 0 75 , p , 0 25 , p , 0 0625 . 1 = 2 = 3 = 3 = 4 = m 2 ( 2 n np i i ) Để tính χ = ∑ , ta lập bảng sau đây 1 np i= i x n p np n np ( 2 n np i i ) i i i i i i npi 0 8 , 0 0625 , 6 25 , 1 75 , 0 49 116
Chương VI. Kiểm định giả thiết thống kê 1 20 , 0 25 25 − 5 1 2 42 3 , 0 75 37 5 , 5 , 4 5 , 0 4 3 22 , 0 25 25 − 3 3 , 0 6 4 8 , 0 0625 , 6 25 , 1 75 , 0 49 5 2 (n np i i )2 Khi đó, ta có χ = ∑ = 8 , 2 8 . np i 1 = i Với mức ý nghĩa α = 0 ,
0 1, m = 5 và r = 1, ta có: 2 2 2 χ = χ = χ = 5 , 4 41. mr − , 1 α 5 1 − − ; 1 0,01 ; 3 0,01 Ta có 2 2 χ
> χ nên ta chưa có cơ sở bác bỏ giả thiết H nên chấp nhận giả thiết mr − ; 1 α 0
H với mức ý nghĩa α = 0 ,
0 1. Khi đó, ta cho rằng số mặt sấp xuất hiện tuân theo phân 0 phối nhị thức. Ví dụ 2.11.
Gọi X là số lần khách đến bán ĐTDĐ ở một cửa hàng trong 96 ngày được cho theo bảng sau đây
X (số lần khách đến) 0 1 2 3 4 n (Số ngày) 17 22 26 20 11 i Với mức ý nghĩa α = 0 ,
0 1, ta có xem X có phân phối Poisson được không? Giải
Gọi F(x) là hàm phân phối xác suất của X F
là hàm phân phối của biến ngẫu 0 (x) −λ 2 nhiên có phân ph e λ
ối Poisson P(λ). Ta có: P(X = x = i ) x ! i
Đặt giả thiết H : F
và đối thiết H : F . 0
(x) ≠ F0(x) 0
(x) = F0 (x)
Dựa vào bảng, ta tính được x = 2 .
Nếu X có phân phối Poisson P(2) thì các xác suất p được xác định như sau i 2 e 2 xi p = ; i = 5 ; 1 i x ! i Ta có: p 1 , 0 353 , p , 0 2707 , p , 0 2707 , p 1 , 0 804 , p 0 , 0 902 . 1 = 2 = 3 = 4 = 5 = m 2 ( 2 n np i i ) Để tính χ = ∑ , ta lập bảng sau đây 1 np i= i x n p np n np ( 2 n np i i ) i i i i i i npi 0 17 1 , 0 353 13 5 , 3 , 3 47 8 , 0 9 1 22 , 0 2707 27 0 , 7 − 0 , 5 7 9 , 0 5 2 26 , 0 2707 27 0 , 7 − , 1 07 , 0 04 3 20 1 , 0 804 18 0 , 4 9 , 1 6 , 0 21 4 11 , 0 0902 0 , 9 2 9 , 1 8 , 0 43 5 2 (n np i i )2 Khi đó, ta có χ = ∑ = 8 , 2 9 np i 1 = i 117 Bài giảng Với mức ý nghĩa α = 0 ,
0 1, m = 5 và r = 1, ta có: 2 2 2 χ = χ = χ = 1 , 1 3 , 45 . mr − , 1 α 5 1 − − ; 1 0,01 ; 3 0,01 Ta có 2 2 χ
> χ nên ta chưa có cơ sở bác bỏ giả thiết H nên chấp nhận giả thiết mr − ; 1 α 0
H với mức ý nghĩa α = 0 ,
0 1. Khi đó, ta cho rằng số lần 0
X khách bán ĐTDĐ có phân phối Poisson. Ví dụ 2.12.
Điểm trung bình học tập của 100 sinh viên được cho ở bảng số liệu sau đây
[a ;a n i 1 − i ) i 0-3 8 3-5 11 5-7 50 7-8 22 8-10 9 Với mức ý nghĩa α = 0 ,
0 5 , kiểm định giả thiết nói rằng điểm trung bình học tập của
sinh viên tuân theo luật phân phối chuẩn. Giải
Gọi F(x) là hàm phân phối xác suất của X F
là hàm phân phối của biến ngẫu 0 (x) nhiên chu  x − µ  1 ẩn N ( 2
µ;σ ), F(x = Φ  . 0 ) +  σ  2
Đặt giả thiết H : F
và đối thiết H : F 0
(x) ≠ F0(x) 0
(x) = F0 (x)
Dựa vào bảng số liệu, ta tính được: x = 0 , 6 2 , 2 2 s = 8 , 1 4
Nếu X có phân phối chuẩn thì X ~ N( 2 , 6 02 8 , 1
; 4 ) và các xác suất p được tính như i sau:  − 
p = P(X < 3 = F = Φ  1 ) (3) 3 , 6 02 1 + = 0 , 0 51  8 , 1 4  2  −   −  p = PX < = FF = Φ  − Φ  2 (3 5) (5) (3) 5 , 6 02 3 0 , 6 2 = ,0239  8 , 1 4   8 , 1 4   −   −  p = PX < = FF = Φ  − Φ  3 (5 7) (7) (5) 7 , 6 02 5 , 6 02 = ,0412  8 , 1 4   8 , 1 4   −   −  p = PX < = FF = Φ  − Φ  4 (7 8) (8) (7) 8 0 , 6 2 7 , 6 02 = 1, 0 56  8 , 1 4   8 , 1 4   − 
p = P(X ≥ 8 = − F = − Φ  5 ) 1 (3) 1 8 0 , 6 2 = 1, 0 41 2  8 , 1 4  m 2 ( 2 n np i i ) Để tính χ = ∑ , ta lập bảng sau đây 1 np i= i n p np n np ( 2 n np i i ) i i i i i npi 8 0,051 5,069 2,931 1,694 11 0,239 23,934 -12,934 6,99 118
Chương VI. Kiểm định giả thiết thống kê 50 0,412 41,246 8,754 1,858 22 0,156 15,610 6,390 2,616 9 0,141 14,140 -5,140 1,869 5 2 (n np i i )2 Khi đó, ta có χ = ∑ = 15 0 , 26 . np i 1 = i Với mức ý nghĩa α = 0 ,
0 5 , m = 5 và r = 2 , ta có: 2 2 2 χ = χ = χ = 9 , 5 92 . mr − , 1 α 5−2− ; 1 0,05 2;0,05 + Nếu 2 2 χ
< χ thì bác bỏ giả thiết H và thừa nhận đối thiết H với mức ý nghĩa mr − ; 1 α 0 1
α . Như vậy, ta không thể coi điểm trung bình học tập của sinh viên có phân phối chuẩn. Ví dụ 2.13.
Kiểm tra ngẫu nhiên 130 cây có trong một khu rừng và đo chiều cao của chúng, kết
quả thu được cho theo bảng sau đây: [a ;a 30-36 36-42 42-48 48-54 54-60 60-66 66-72 i 1 − i ) n 2 8 35 43 22 15 5 i
Có người cho rằng chiều cao X của loại cây này có phân phối chuẩn N( 2 µ;σ ). Hãy
kiểm định lời nhận định này với mức ý nghĩa α = 0 , 0 5 . Giải
Gọi F(x) là hàm phân phối xác suất của X F
là hàm phân phối của biến ngẫu 0 (x)  x − µ  1 nhiên chuẩn N( 2
µ;σ ), F(x = Φ  . 0 ) +  σ  2
Đặt giả thiết H : F
và đối thiết H : F . 0
(x) ≠ F0(x) 0
(x) = F0 (x)
Dựa vào bảng, ta tính được x = 51 5 , và s = 7 6 , 8 . Nếu 2
X có phân phối chuẩn thì X ~ N (51 ; 5 , ( ,
7 68) ) và các xác suất p được tính như i sau:  −  p = P X < = F = Φ  . 1 ( 36) 0 (36) 36 51 5 , 1 + = , 0 022  7 6 , 8  2  −   −  p = PX < = FF = Φ  − Φ  2 (36 42) 0 (42) 0 (36) 42 51 5 , 36 51 5 , = , 0 087  7 6 , 8   7 6 , 8   −   −  p = PX < = FF = Φ  − Φ  3 (42 48) 0 (48) 0 (42) 48 51 5 , 42 51 5 , = , 0 217  7,68   7,68   −   −  p = PX < = FF = Φ  − Φ  4 (48 54) 0 (54) 0 (48) 54 51 5 , 48 51 5 , = 3 , 0 03  7 6 , 8   7,68   −   −  p = PX < = FF = Φ  − Φ  5 (54 60) 0 (54) 0 (48) 60 51 5 , 54 51 5 , = , 0 237  7,68   7 6 , 8   −   −  p = PX < = FF = Φ  − Φ  6 (60 66) 0 (66) 0 (48) 66 51 5 , 60 51 5 , = 1 , 0 04  7,68   7 6 , 8   − 
p = P(X ≥ 66 = − Φ  7 ) 1 66 51 5 , = , 0 029 2  7 6 , 8  119 Bài giảng m 2 ( 2 n np i i ) Để tính χ = ∑ , ta lập bảng sau đây 1 np i= i n p np n np ( 2 n np i i ) i i i i i npi 2 , 0 022 8 , 2 6 − 8 , 0 6 , 0 26 8 , 0 087 11 3 , − 3 , 3 9 , 0 6 35 , 0 217 2 , 8 23 7 , 6 7 , 1 62 43 3 , 0 03 3 , 9 45 5 , 3 5 3 , 0 2 22 , 0 237 30 8 , 6 − 8.86 5 , 2 5 15 1 , 0 04 13 5 , 1 , 1 49 1 , 0 6 5 , 0 029 7 , 3 9 , 1 21 3 , 0 9 m 2 (n np i i )2 Khi đó, ta có χ = ∑ = , 6 26 . np i 1 = i Với mức ý nghĩa α = 0 ,
0 5 , m = 7 và r = 2 , ta có: 2 2 2 χ = χ = χ = 5 , 9 . mr − , 1 α 7−2− ; 1 0,05 4;0,05 Ta có 2 2 χ
> χ nên ta chưa có cơ sở bác bỏ giả thiết H nên chấp nhận giả thiết mr − ; 1 α 0
H với mức ý nghĩa α = 0 ,
0 5 . Khi đó, ta cho rằng chiều cao 0
X của loại cây trên có
phân phối chuẩn với mức ý nghĩa α = 0 , 0 5 .
2.8. Kiểm định giả thiết thống kê về tính độc lập.
Giả sử ta có mẫu ngẫu nhiên hai quan sát đồng thời về hai ĐLNN X Y . Từ mẫu có
kích thước n , ta có bảng số liệu sau đây (X ;Y ) 1 y y K 2 y Tổng h x 1 1 n 1 1 n K 2 n1h 1 n x 2 n 21 n K 22 n2 n h 2 K K K K K K x n n K n n k k1 k 2 kh k Tổng 1 m m K 2 m ∑= n h Trong đó, h n = n , i = ; 1 k , iij j =1 k k h m = m , j = ;
1 h , và n = ∑∑ n jij ij i=1 i j
Ta đặt giả thiết: H : 0
X Y độc lập. Đối thiết: H : 1
X Y không độc độc lập.
Với mức ý nghĩa α cho trước. Hãy kiểm định giả thiết H . 0 120
Chương VI. Kiểm định giả thiết thống kê  2 k h n  Biến ngẫu nhiên 2  ij
G = χ = n∑ ∑
−1 có phân phối Chi bình phương với  1 1 n m i= j= i j  (k − ) 1 (h − ) 1 bậc tự do.
Ta có quy tắc kiểm định sau đây:  2 k h n
* Xác định giá trị thực nghiệm: 2  ij  χ = n∑∑ −1 .  1 1 n m i= j = i j
* Với mức ý nghĩa α , ta tìm số 2 χ t (
ừ bảng phân phối Chi bình phương rồi so k 1 − )(h 1 − );α sánh với 2 χ . + Nếu 2 2 χ thì bác b ( < χ
ỏ giả thiết H và thừa nhận đối thiết H với mức ý k 1 − )(h 1 − );α 0 1 nghĩa α . + Nếu 2 2 χ thì ta ch ( ≥ χ
ưa có cơ sở bác bỏ giả thiết H nên chấp nhận giả thiết k 1 − )(h 1 − );α 0
H với mức ý nghĩa α . 0 Ví dụ 2.14.
Trong một nhà máy dệt may, một nhà thống kê theo dõi 1000 công nhân làm việc
trong một năm và thấy số ngày nghỉ việc của họ được cho trong bảng sau
Số ngày nghỉ việc trong năm Nam Nữ 0-10 300 500 10-20 80 70 20 trở lên 20 30
Với mức ý nghĩa α = 5% , nhà thống kê này có thể cho rằng số ngày nghỉ của công
nhân có phụ thuộc vào giới tính không? Giải
Gọi X là số ngày nghỉ của công nhân, Y là giới tính của công nhân.
Dựa vào bảng số liệu, ta có: (X ;Y ) 1 y y Tổng 2 1 x 300 500 800 x 80 70 150 2 x 20 30 50 3 Tổng 400 600 1000
Đây là bài toán kiểm định giả thiết về tính độc lập của X Y .
Ta đặt giả thiết: : H : 0
X Y độc lập. Đối thiết: H : 1
X Y không độc độc lập.  2 k h n
Ta chọn giá trị kiểm định: 2  ij  χ = n∑∑ −1 .  1 1 n m i= j = i j  Khi đó, ta có: 121 Bài giảng  3002  2 5002 802 702 202 302 χ = 1000 + + + + + −1 = 13 1 , 9   800 4 . 00 800 6 . 00 150.400 150 6 . 00 50.400 50.600  
Với mức ý nghĩa α = 5% , k = 3, h = 2 , ta có 2 2 χ = χ = 9 , 5 92 . (k 1 − )(h 1 − );α 2;0,05 Ta có 2 2 χ thì bác b ( < χ
ỏ giả thiết H và thừa nhận đối thiết H với mức ý k 1 − )(h 1 − );α 0 1
nghĩa 5% . Vậy số ngày nghỉ của công nhân phụ thuộc vào giới tính. 3. Bài tập chương. 1.
Trọng lượng X của sản phẩm do một nhà máy sản xuất ra là một biến ngẫu nhiên có
phân phối chuẩn N ( 2
µ;σ ) với σ = 2 kg và trọng lượng trung bình là µ = 20 kg. Nghi
ngờ nhà máy hoạt động không bình thường làm thay đổi trọng lượng trung bình của sản
phẩm, người ta cân 100 sản phẩm và kết quả thu được cho theo bảng sau:
x : trọng lượng sản phẩm 18 19 20 21 22 i
n : số sản phẩm tương ứng 5 25 40 20 10 i
Hãy kiểm định điều nghi ngờ trên với mức ý nghĩa α = 0 ,
0 5 với giả thiết: H : µ 20 kg và 0 = đối thiết H : µ 20 . 0 ≠ 2.
Theo kỹ thuật quy định thiết kế quy định chiều dài trung bình của một chi tiết máy do
nhà máy A sản xuất là 20 cm. Sau một thời gian sản xuất, có ý kiến cho rằng nhà máy
A sản xuất loại chi tiết máy trên không đạt yêu cầu. Để kiểm tra, người tra chọn ngẫu
nhiên 64 chi tiết và đo (phép đo không có sai số) kết quả thu được: chiều dài trung bình x = 20 5
, cm và độ lệch tiêu chuẩn hiệu chỉnh là s = 1 cm. Biết rằng chiều dài loại chi
tiết trên là biến ngẫu nhiên chuẩn N ( 2
µ;σ ). Hãy kiểm định điều nghi ngờ trên với mức ý nghĩa α = 0 ,
0 5 với giả thiết: H : µ
20 cm và đối thiết H : µ 20 cm. 0 = 0 ≠ 3.
Một nhà thống kê theo dõi mức thu nhập của một số người ở Công ty May và thu được số liệu như sau. X (trăm ngàn) 8 11 13 15 17 19 21 23 n (Số người) 4 16 25 30 26 20 15 8
a) Tính thu nhập trung bình của X và độ lệch chuẩn điều chỉnh s của thu nhập X .
b) Với độ tin cậy γ = 95% . Hãy ước lượng thu nhập trung bình của toàn bộ công nhân ở Công ty May.
c) Nếu nhà thống kê cho rằng thu nhập mỗi tháng X ≥ 7 ,
1 triệu là cao. Hãy ước lượng tỉ lệ
p của những người có thu nhập cao trong Công ty với độ tin cậy γ = 99% .
d) Nếu ban giám đốc báo cáo rằng thu nhập trung bình là 6 ,
1 triệu. Nhà thống kê dựa vào
mẫu kết quả ở trên với mức ý nghĩa α = 5% . Nhà thống kê có thể tin cậy vào ý kiến này không? 4.
Điều tra doanh số bán hàng X (triệu đồng/tháng) của các hộ kinh doanh một loại hàng
năm nay, ta được số liệu sau đây X (triệu/tháng) 11 11,5 12 12,5 13 13,5 Số hộ 10 15 20 30 15 10
a) Nếu biết rằng những hộ có doanh số trên 12,5 triệu / tháng là những hộ có doanh số cao.
Có bài báo công bố rằng tỉ lệ hộ có doanh số cao là 35% . Cho nhận xét về tỉ lệ những
hộ có doanh số cao trong bài báo này với mức ý nghĩa 5% . 122
Chương VI. Kiểm định giả thiết thống kê
b) Năm trước, doanh số bán hàng của các hộ này là 120 triệu / năm (tức là 10 triệu /
tháng). Có thể cho rằng doanh số bán hàng của các hộ này năm nay tăng lên không với mức ý nghĩa 1% . 5.
Một công ti kinh doanh xe đạp điện tuyên bố rằng 60% khách hàng ưa thích sản phẩm
của công ti. Điều tra 400 khách hàng thì có 230 khách hàng ưa thích sản phẩm của tông
ti này. Với mức ý nghĩa α = 5% , hãy xem tỉ lệ trong tuyên bố của công ti có đúng không? 6.
Trọng lượng của một gói bột ngọt do một máy tự động đóng theo thiết kế là 500
gram/gói. Nghi ngờ máy tự động đóng gói này làm việc không bình thường làm cho
trọng lượng của gói bột ngọt có xu hướng giảm sút. Người ta lấy ngẫu nhiên 30 gói, cân
thử và được trọng lượng trung bình là 495 gram và độ lệch tiêu chuẩn hiệu chỉnh s = 10
gram. Với mức ý nghĩa α = 5% , hãy cho kết luận về nghi ngờ này. 7.
Trước đây, định mức tiêu dùng điện của một hộ gia đình trong một tháng là 140 kW. Do
đời sống nâng cao, người ta theo dõi 100 hộ gia đình và thu được số liệu sau đây Lượng tiêu dùng 100-120 120-140 140-160 160-180 180-200 (kW) Số hộ gia đình 14 25 30 20 11
a) Với mức ý nghĩa α = 5% , theo bạn có nên tăng định mức lên không?
b) Nếu trước đây, độ biến động của mức tiêu dùng điện cho một hộ gia đình là 400 (kW)2 .
Vậy, hiện nay, độ biến động tăng không? Hãy cho kết luận với mức ý nghĩa α = 5% . 8.
Một đại lí xe máy đã kí hợp đồng với 2 nhà cung cấp A và B sản xuất thử linh kiện
khung cho xe Dream II. Dựa vào kết quả thử khung, đại lí sẽ chọn nhà cung cấp nào cho
đại lí của mình. Nhà cung cấp A đã sản xuất thử được 10 cái với độ bền trung bình là
4,8 tháng và độ lệch tiêu chuẩn là 1,1 tháng; nhà cung cấp B sản xuất thử 13 cái có độ
bền trung bình là 4,3 tháng và độ lệch tiêu chuẩn là 0,9 tháng. Cho mức ý nghĩa
α = 10% , giả sử độ bền của hai loại khung do các nhà cung cấp A và B sản xuất có
phân phối chuẩn. Nếu biết độ ổn định về độ bền (phương sai) của hai loại khung là như
nhau, hãy xem tuổi thọ trung bình của hai loại khung có khác nhau không? 9.
Độ lệch tiêu chuẩn của trọng lượng X của một loại sản phẩm là 0,1 kg. Nghi ngờ độ
đồng đều của trọng lượng sản phẩm giảm sút, người ta cân thử 25 sản phẩm và thu được số liệu sau đây X (kg) 2,1 2,2 2,3 2,4 2,5 Số sản phầm 2 4 15 3 1
Với mức ý nghĩa α = 5% , hãy cho biết kết luận về điều nghi ngờ trên. Giả thiết trọng lượng
sản phẩm có phân phối chuẩn.
C. Phương pháp giảng dạy.
- Đưa ví dụ cụ thể để thấy rõ ứng dụng của kiểm định.
- Sử dụng các bảng phụ lục cho việc tính các giá trị của hàm phân phối chuẩn, Poisson, Student, chi bình phương.
- Phối hợp phương pháp thuyết trình và vấn đáp giải quyết vấn đề.
- Yêu cầu SV đọc bài giảng trước khi lên lớp.
- Kiểm tra, đánh giá việc làm bài tập của SV.
D. Tài liệu tham khảo
[1] Đậu Thế Cấp, Xác suất thống kê: Lí thuyết và các bài tập, NXB Giáo dục, 2006. 123 Bài giảng
[2] Đặng Hùng Thắng, Thống kê và ứng dụng, NXB Giáo dục, 2008.
[3] PGS. TS. Phạm Xuân Kiều, Giáo Trình xác suất và thống kê, NXB Giáo dục, 2005.
[4] Trần Văn Minh, Phí Thị Vân Anh, Xác suất thống kê với các tính toán trên Excel,
NXB Giao Thông Vận tải, 2008.
[5] Đặng Công Hanh, Đặng Ngọc Dục, Giáo trình Lý thuyết xác suất và Thống kê
toán
, trường Đại học Duy Tân,1996
[6] Trần Văn Minh, Phí Thị Vân Anh, Hướng dẫn giải bài tập Xác suất thống kê với
các tính toán trên Excel
, NXB Giao Thông Vận tải, 2008. 124 Các bảng số Các bảng số
Bảng 1. Bảng phân phối Poisson: −λ k λ − . 0 1 e × . 0 11 P(X = ) e k = , POISSON( 0 , 1 . 0 , 1 ) = 0 , 0 905 , P(X = ) 1 = k! ! 1 0.1 0.2 0.3 0.4 0.5 0.6 0.7 0.8 0.9 (k;λ)
0 0.9048 0.8187 0.7408 0.6703 0.6065 0.5488 0.4966 0.4493 0.4066
1 0.0905 0.1637 0.2222 0.2681 0.3033 0.3293 0.3476 0.3595 0.3659
2 0.0045 0.0164 0.0333 0.0536 0.0758 0.0988 0.1217 0.1438 0.1647
3 0.0002 0.0011 0.0033 0.0072 0.0126 0.0198 0.0284 0.0383 0.0494
4 0.0000 0.0001 0.0003 0.0007 0.0016 0.0030 0.0050 0.0077 0.0111
5 0.0000 0.0000 0.0000 0.0001 0.0002 0.0004 0.0007 0.0012 0.0020
6 0.0000 0.0000 0.0000 0.0000 0.0000 0.0000 0.0001 0.0002 0.0003 (k;λ) 1 1.5 2 2.5 3 3.5 4 4.5 5 0 0.3679 0.2231 0.1353 0.0821 0.0498 0.0302 0.0183 0.0111 0.0067 1 0.3679 0.3347 0.2707 0.2052 0.1494 0.1057 0.0733 0.0500 0.0337 2 0.1839 0.2510 0.2707 0.2565 0.2240 0.1850 0.1465 0.1125 0.0842 3 0.0613 0.1255 0.1804 0.2138 0.2240 0.2158 0.1954 0.1687 0.1404 4 0.0153 0.0471 0.0902 0.1336 0.1680 0.1888 0.1954 0.1898 0.1755 5 0.0031 0.0141 0.0361 0.0668 0.1008 0.1322 0.1563 0.1708 0.1755 6 0.0005 0.0035 0.0120 0.0278 0.0504 0.0771 0.1042 0.1281 0.1462 125 Các bảng số
Bảng 2. Giá trị tích phân Laplace: 2 z t 1   1 0 0 . 5 Φ(z) ∫ − =
e 2 dt , Ví dụ: NORMDIST ( . 1 9 , 6 ) 1 , 1 , 0 = 0 4 , 75 ,   Φ z = − . 2  .005  π 2 2 0  2  z 0 1 2 3 4 5 6 7 8 9
0 0.0000 0.0040 0.0080 0.0120 0.0160 0.0199 0.0239 0.0279 0.0319 0.0359
0.1 0.0398 0.0438 0.0478 0.0517 0.0557 0.0596 0.0636 0.0675 0.0714 0.0753
0.2 0.0793 0.0832 0.0871 0.0910 0.0948 0.0987 0.1026 0.1064 0.1103 0.1141
0.3 0.1179 0.1217 0.1255 0.1293 0.1331 0.1368 0.1406 0.1443 0.1480 0.1517
0.4 0.1554 0.1591 0.1628 0.1664 0.1700 0.1736 0.1772 0.1808 0.1844 0.1879
0.5 0.1915 0.1950 0.1985 0.2019 0.2054 0.2088 0.2123 0.2157 0.2190 0.2224
0.6 0.2257 0.2291 0.2324 0.2357 0.2389 0.2422 0.2454 0.2486 0.2517 0.2549
0.7 0.2580 0.2611 0.2642 0.2673 0.2704 0.2734 0.2764 0.2794 0.2823 0.2852
0.8 0.2881 0.2910 0.2939 0.2967 0.2995 0.3023 0.3051 0.3078 0.3106 0.3133
0.9 0.3159 0.3186 0.3212 0.3238 0.3264 0.3289 0.3315 0.3340 0.3365 0.3389
1 0.3413 0.3438 0.3461 0.3485 0.3508 0.3531 0.3554 0.3577 0.3599 0.3621
1.1 0.3643 0.3665 0.3686 0.3708 0.3729 0.3749 0.3770 0.3790 0.3810 0.3830
1.2 0.3849 0.3869 0.3888 0.3907 0.3925 0.3944 0.3962 0.3980 0.3997 0.4015
1.3 0.4032 0.4049 0.4066 0.4082 0.4099 0.4115 0.4131 0.4147 0.4162 0.4177
1.4 0.4192 0.4207 0.4222 0.4236 0.4251 0.4265 0.4279 0.4292 0.4306 0.4319
1.5 0.4332 0.4345 0.4357 0.4370 0.4382 0.4394 0.4406 0.4418 0.4429 0.4441
1.6 0.4452 0.4463 0.4474 0.4484 0.4495 0.4505 0.4515 0.4525 0.4535 0.4545
1.7 0.4554 0.4564 0.4573 0.4582 0.4591 0.4599 0.4608 0.4616 0.4625 0.4633
1.8 0.4641 0.4649 0.4656 0.4664 0.4671 0.4678 0.4686 0.4693 0.4699 0.4706
1.9 0.4713 0.4719 0.4726 0.4732 0.4738 0.4744 0.4750 0.4756 0.4761 0.4767
2 0.4772 0.4778 0.4783 0.4788 0.4793 0.4798 0.4803 0.4808 0.4812 0.4817
2.1 0.4821 0.4826 0.4830 0.4834 0.4838 0.4842 0.4846 0.4850 0.4854 0.4857
2.2 0.4861 0.4864 0.4868 0.4871 0.4875 0.4878 0.4881 0.4884 0.4887 0.4890
2.3 0.4893 0.4896 0.4898 0.4901 0.4904 0.4906 0.4909 0.4911 0.4913 0.4916
2.4 0.4918 0.4920 0.4922 0.4925 0.4927 0.4929 0.4931 0.4932 0.4934 0.4936
2.5 0.4938 0.4940 0.4941 0.4943 0.4945 0.4946 0.4948 0.4949 0.4951 0.4952
2.6 0.4953 0.4955 0.4956 0.4957 0.4959 0.4960 0.4961 0.4962 0.4963 0.4964
2.7 0.4965 0.4966 0.4967 0.4968 0.4969 0.4970 0.4971 0.4972 0.4973 0.4974
2.8 0.4974 0.4975 0.4976 0.4977 0.4977 0.4978 0.4979 0.4979 0.4980 0.4981
2.9 0.4981 0.4982 0.4982 0.4983 0.4984 0.4984 0.4985 0.4985 0.4986 0.4986
3 0.4987 0.4987 0.4987 0.4988 0.4988 0.4989 0.4989 0.4989 0.4990 0.4990 126 Các bảng số
Bảng 3. Phân vị α của phân phối Student       P T > t = α    . Ví dụ: TINV (24 . 0 , 0 ) 5 = 0 .
2 639 , P T > t = 0.05 α   . 0 05   n; 24; 2   2  (n;α) 0.200 0.100 0.050 0.025 0.010 0.005 1 3.0777 6.3138 12.7062 25.4517 63.6567 127.3213 2 1.8856 2.9200 4.3027 6.2053 9.9248 14.0890 3 1.6377 2.3534 3.1824 4.1765 5.8409 7.4533 4 1.5332 2.1318 2.7764 3.4954 4.6041 5.5976 5 1.4759 2.0150 2.5706 3.1634 4.0321 4.7733 6 1.4398 1.9432 2.4469 2.9687 3.7074 4.3168 7 1.4149 1.8946 2.3646 2.8412 3.4995 4.0293 8 1.3968 1.8595 2.3060 2.7515 3.3554 3.8325 9 1.3830 1.8331 2.2622 2.6850 3.2498 3.6897 10 1.3722 1.8125 2.2281 2.6338 3.1693 3.5814 11 1.3634 1.7959 2.2010 2.5931 3.1058 3.4966 12 1.3562 1.7823 2.1788 2.5600 3.0545 3.4284 13 1.3502 1.7709 2.1604 2.5326 3.0123 3.3725 14 1.3450 1.7613 2.1448 2.5096 2.9768 3.3257 15 1.3406 1.7531 2.1314 2.4899 2.9467 3.2860 16 1.3368 1.7459 2.1199 2.4729 2.9208 3.2520 17 1.3334 1.7396 2.1098 2.4581 2.8982 3.2224 18 1.3304 1.7341 2.1009 2.4450 2.8784 3.1966 19 1.3277 1.7291 2.0930 2.4334 2.8609 3.1737 20 1.3253 1.7247 2.0860 2.4231 2.8453 3.1534 21 1.3232 1.7207 2.0796 2.4138 2.8314 3.1352 22 1.3212 1.7171 2.0739 2.4055 2.8188 3.1188 23 1.3195 1.7139 2.0687 2.3979 2.8073 3.1040 24 1.3178 1.7109 2.0639 2.3909 2.7969 3.0905 25 1.3163 1.7081 2.0595 2.3846 2.7874 3.0782 26 1.3150 1.7056 2.0555 2.3788 2.7787 3.0669 27 1.3137 1.7033 2.0518 2.3734 2.7707 3.0565 28 1.3125 1.7011 2.0484 2.3685 2.7633 3.0469 29 1.3114 1.6991 2.0452 2.3638 2.7564 3.0380 30 1.3104 1.6973 2.0423 2.3596 2.7500 3.0298 127 Các bảng số
Bảng 4. Phân vị α của phân phối Chi bình phương P(χ 2 > χ 2 = . Ví dụ: CHIINV ( , 5 0 . 0 ) 1 = 15 0 . 863 , P( 2 2 χ > χ . 5;0 0 . 1 ) = 0 . 0 1 n α ; ) α (n;α) 0.010 0.025 0.050 0.950 0.975 0.990 1 6.6349 5.0239 3.8415 0.0039 0.0010 0.0002 2 9.2103 7.3778 5.9915 0.1026 0.0506 0.0201 3 11.3449 9.3484 7.8147 0.3518 0.2158 0.1148 4 13.2767 11.1433 9.4877 0.7107 0.4844 0.2971 5 15.0863 12.8325 11.0705 1.1455 0.8312 0.5543 6 16.8119 14.4494 12.5916 1.6354 1.2373 0.8721 7 18.4753 16.0128 14.0671 2.1673 1.6899 1.2390 8 20.0902 17.5345 15.5073 2.7326 2.1797 1.6465 9 21.6660 19.0228 16.9190 3.3251 2.7004 2.0879 10 23.2093 20.4832 18.3070 3.9403 3.2470 2.5582 11 24.7250 21.9200 19.6751 4.5748 3.8157 3.0535 12 26.2170 23.3367 21.0261 5.2260 4.4038 3.5706 13 27.6882 24.7356 22.3620 5.8919 5.0088 4.1069 14 29.1412 26.1189 23.6848 6.5706 5.6287 4.6604 15 30.5779 27.4884 24.9958 7.2609 6.2621 5.2293 16 31.9999 28.8454 26.2962 7.9616 6.9077 5.8122 17 33.4087 30.1910 27.5871 8.6718 7.5642 6.4078 18 34.8053 31.5264 28.8693 9.3905 8.2307 7.0149 19 36.1909 32.8523 30.1435 10.1170 8.9065 7.6327 20 37.5662 34.1696 31.4104 10.8508 9.5908 8.2604 21 38.9322 35.4789 32.6706 11.5913 10.2829 8.8972 22 40.2894 36.7807 33.9244 12.3380 10.9823 9.5425 23 41.6384 38.0756 35.1725 13.0905 11.6886 10.1957 24 42.9798 39.3641 36.4150 13.8484 12.4012 10.8564 25 44.3141 40.6465 37.6525 14.6114 13.1197 11.5240 26 45.6417 41.9232 38.8851 15.3792 13.8439 12.1981 27 46.9629 43.1945 40.1133 16.1514 14.5734 12.8785 28 48.2782 44.4608 41.3371 16.9279 15.3079 13.5647 29 49.5879 45.7223 42.5570 17.7084 16.0471 14.2565 30 50.8922 46.9792 43.7730 18.4927 16.7908 14.9535 128
Document Outline

  • Trang phụ bìa
  • Mục lục